Oxford Assess and Progress Clinical Dentistry

Download as pdf or txt
Download as pdf or txt
You are on page 1of 361

OXFORD ASSESS AND PROGRESS

Series Editors
Katharine Boursicot
Director, Health Professional Assessment Consultancy (HPAC)
Honorary Reader in Medical Education St George’s,
University of London
David Sales
Consultant in Medical Assessment

https://t.me/DentalBooksWorld
OXFORD ASSESS AND PROGRESS
Also available and forthcoming titles in the Oxford Assess
and Progress series

Clinical Dentistry
Nicholas Longridge, Peter Clarke, Raheel Aftab, and Tariq Ali

Clinical Medicine, Third Edition


Dan Furmedge and Rudy Sinharay

Clinical Specialties, Third Edition


Luci Etheridge and Alex Bonner

Clinical Surgery
Neil Borley, Frank Smith, Paul McGovern, Bernadette Pereira, and
Oliver Old

Emergency Medicine
Pawan Gupta

Medical Sciences
Jade Chow and John Patterson

Psychiatry
Gil Myers and Melissa Gardner

Situational Judgement Test, Third Edition


David Metcalfe and Harveer Dev

https://t.me/DentalBooksWorld
OXFORD ASSESS AND PROGRESS

Clinical Dentistry
Nicholas Longridge BSc (Hons), BDS (Hons),
MFDS RCSEd
Academic Clinical Fellow/​Specialty Registrar in Endodontics
Liverpool University Dental Hospital, United Kingdom

Peter Clarke BDS (Hons), MFDS RCSP (Glasg)


Specialty Registrar in Restorative Dentistry
University Dental Hospital of Manchester, United Kingdom

Raheel Aftab BDS, MJDF RCSEng, PgCert Primary Dental


General Dental Practitioner, Educational Supervisor
Kent, United Kingdom

Tariq Ali MB ChB, MRCS, DOHNS RCSEng, MRCEM, BDS,


MJDF RCSEng
Associate Lecturer, Faculty of Medicine, University of Queensland
Registrar Oral and Maxillofacial Surgery
Queensland, Australia

1
https://t.me/DentalBooksWorld
1
Great Clarendon Street, Oxford, OX2 6DP,
United Kingdom
Oxford University Press is a department of the University of Oxford.
It furthers the University’s objective of excellence in research, scholarship,
and education by publishing worldwide. Oxford is a registered trade mark of
Oxford University Press in the UK and in certain other countries
© Oxford University Press 209
The moral rights of the authors have been asserted
First Edition published in 209
Impression: 
All rights reserved. No part of this publication may be reproduced, stored in
a retrieval system, or transmitted, in any form or by any means, without the
prior permission in writing of Oxford University Press, or as expressly permitted
by law, by licence or under terms agreed with the appropriate reprographics
rights organization. Enquiries concerning reproduction outside the scope of the
above should be sent to the Rights Department, Oxford University Press, at the
address above
You must not circulate this work in any other form
and you must impose this same condition on any acquirer
Published in the United States of America by Oxford University Press
98 Madison Avenue, New York, NY 006, United States of America
British Library Cataloguing in Publication Data
Data available
Library of Congress Control Number: 20993586
ISBN 978–​0–​9–​88257–​3
Printed and bound in China by
C&C Offset Printing Co., Ltd.
Oxford University Press makes no representation, express or implied, that the
drug dosages in this book are correct. Readers must therefore always check
the product information and clinical procedures with the most up-​to-​date
published product information and data sheets provided by the manufacturers
and the most recent codes of conduct and safety regulations. The authors and
the publishers do not accept responsibility or legal liability for any errors in the
text or for the misuse or misapplication of material in this work. Except where
otherwise stated, drug dosages and recommendations are for the non-​pregnant
adult who is not breast-​feeding
Links to third party websites are provided by Oxford in good faith and
for information only. Oxford disclaims any responsibility for the materials
contained in any third party website referenced in this work.

https://t.me/DentalBooksWorld
v

Series editor preface

The Oxford Assess and Progress series is a groundbreaking development


in the extensive area of self-​assessment texts available for dental and
medical students. The questions were specifically commissioned for the
series, written by practising clinicians, extensively peer-​reviewed by stu-
dents and their teachers, and quality-​assured to ensure that the material
is up-​to-​date, accurate, and in line with modern testing formats.
The series has a number of unique features and is designed to be as
much a formative learning resource as a self-​assessment one. The ques-
tions are constructed to test the same clinical problem-​solving skills that
we use as practising clinicians, rather than only to test theoretical know-
ledge. These skills include:
● gathering and using data required for clinical judgement
● choosing the appropriate examination and investigations
● applying knowledge and interpreting findings
● demonstrating diagnostic skills
● ability to evaluate undifferentiated material
● ability to prioritize
● making decisions and demonstrating a structured approach to
decision-​making.
Each question is bedded in reality and is typically presented as a
clinical scenario, the content of which has been chosen to reflect the
common and important conditions that most dentists and doctors are
likely to encounter both during their training and in exams! The aim of
the series is to build the reader’s confidence in recognizing important
symptoms and signs and suggesting the most appropriate investigations
and management, and, in so doing, to aid the development of a clear
approach to patient management which can be transferred to the clinical
environment.
The content of the series has deliberately been pinned to the relevant
Oxford Handbook but, in addition, has been guided by a blueprint which
reflects the themes identified in the General Dental Council’s Preparing
for practice—​Dental teams learning outcomes for registration, including an
evidence-​based approach to learning, along with clinical, managerial, and
professionalism scenarios.
Particular attention has been paid to giving learning points and con-
structive feedback on each question, using clear fact-​or evidence-​
based explanations as to why the correct response is right and why the
incorrect responses are less appropriate. The question editorials are
clearly referenced to the relevant sections of the accompanying Oxford
Handbook and/​or more widely to medical literature or guidelines. They
are designed to guide and motivate the reader, being multi-​purpose
in nature and covering, e.g. exam technique, approaches to difficult
subjects, and links between subjects.

https://t.me/DentalBooksWorld
vi Series editor preface

Another unique aspect of the series is the element of competency


progression from being a relatively inexperienced student to being a
more experienced junior dentist. We have suggested the following four
degrees of difficulty to reflect the level of training, so that the reader can
monitor their own progress over time:
● graduate should know ★
● graduate nice to know ★★
● foundation dentist should know ★★★
● foundation dentist nice to know ★★★★

We advise the reader to attempt the questions in blocks as a way of


testing their knowledge in a clinical context. The series can be treated as
a dress rehearsal for life as a clinician by using the material to hone clin-
ical acumen and build confidence by encouraging a clear, consistent, and
rational approach, proficiency in recognizing and evaluating symptoms
and signs, making a rational differential diagnosis, and suggesting appro-
priate investigations and management.
Adopting such an approach can aid not only success in examin-
ations, which really are designed to confirm learning, but also—​more
importantly—​being a good dentist and doctor. In this way, we can deliver
high-​quality and safe patient care by recognizing, understanding, and
treating common problems, but at the same time remaining alert to the
possibility of less likely, but potentially catastrophic, conditions.
David Sales and Kathy Boursicot
Series Editors

https://t.me/DentalBooksWorld
vii

A note on single best


answer questions

Single best answer questions are currently the format of choice


being widely used by most undergraduate and postgraduate knowledge
tests, and therefore, the questions in this book follow this format.
Single best answer questions have many advantages over other
machine-​ markable formats, such as extended matching questions
(EMQs), notably the breadth of sampling or content coverage that they
afford.
Briefly, the single best answer or ‘best of five’ question presents a
problem, usually a clinical scenario, before presenting the question itself
and a list of five options. These consist of one correct answer and four
incorrect options, or ‘distractors’, from which the reader has to choose
a response.
All of the questions in this book, which are typically based on an evalu-
ation of symptoms, signs, results of investigations, or material inter-
actions, either as single entities or in combination, are designed to test
reasoning skills, rather than straightforward recall of facts, and utilize cog-
nitive processes similar to those used in clinical practice.
The peer-​reviewed questions are written and edited in accordance
with contemporary best assessment practice, and their content has been
guided by a blueprint pinned to all areas of the General Dental Council’s
document Preparing for practice—​Dental teams learning outcomes for regis-
tration, which ensures comprehensive coverage.
The answers and their rationales are evidence-​based and have been
reviewed to ensure that they are absolutely correct. Incorrect options
are selected as being plausible, and indeed they may appear correct to
the less knowledgeable reader. When answering questions, the reader
may wish to use the ‘cover’ test, in which they read the scenario and the
question but cover the options.
Kathy Boursicot and David Sales
Series Editors

https://t.me/DentalBooksWorld
https://t.me/DentalBooksWorld
ix

Author preface

Dental school can be a challenging and emotional time. The breadth of


experiences gained both professionally and socially cannot be rivalled,
and in hindsight, most come to look upon their time at university as
a thoroughly enjoyable experience. Needless to say, preparing for the
multitude of examinations and assessments throughout the programme
is never a favourite pastime, but a necessary evil nonetheless. Whether
it is the prospect of finals or postgraduate examinations on the horizon,
we remember the constant pressure to read and revise only too well.
Our own experiences frequently involved discussing a range of possible
questions which lacked informative answers. This led us to the Oxford
Assess and Progress Series and to the production of this book.
The Oxford Handbook of Clinical Dentistry (OHCD) was never far from
reach during dental school, and within this book, we have attempted to
provide a series of single best answer questions that link the OHCD with
real-​life practical scenarios to test reasoning and application of know-
ledge. Where possible, recommended reading and references to seminal
papers have been provided to encourage further reading and to support
evidence-​based practice. Within the book, we have selected Keywords,
where relevant, to help highlight specific clues or words that can assist
with recall in those high-​pressure situations. All questions have been
written and peer-​reviewed by clinicians working within each specialty,
and we have endeavoured to provide in-​depth justification for correct
and incorrect answers. Undoubtedly, some topics will remain conten-
tious, but, where necessary, we have explained our reasoning and hope
that this highlights the ‘grey’ areas in many dental scenarios. Chapters
are formatted by specialty, and we have attempted to maintain a clear
focus on clinically oriented scenarios that will be beneficial for finals and
beyond. A selection of questions on ‘Law and ethics’ have been written
and combined into the clinical specialty for which they are relevant.
As previously mentioned, we started this book with the hope of
providing an informative and supportive revision tool that encour-
ages further reading and evidence-​based practice. Looking back, we all
remember dental school with fond memories, and we hope that you find
this book useful and wish you the very best for your finals and future
careers beyond.
Nicholas Longridge, Peter Clarke,
Raheel Aftab, and Tariq Ali

https://t.me/DentalBooksWorld
https://t.me/DentalBooksWorld
xi

Acknowledgements

The authors would like to thank all of the contributors for their hard
work in producing the content for this book. Special thanks must go
to the authors of the Oxford Handbook of Clinical Dentistry David and
Laura Mitchell, for allowing us to use their excellent book as a guiding
framework and revision source. We would like to thank all reviewers—​
students and specialists—​for their detailed feedback and discussion
points, which we hope to have reflected in the final book. We are
also indebted to Geraldine Jeffers and Rachel Goldsworthy at Oxford
University Press for their support, guidance, and patience throughout
the entire project. Nick would like to thank his wife, Sarah, and his
parents for their endless support. Peter would like to thank his wife,
Tess, for her patience and understanding throughout the process. Tariq
would like to thank his family, friends, and colleagues for their constant
support throughout his career. Raheel would also like to thank his family.
All four authors would like to dedicate the book to their good friend
Andy Jones, who was taken from this world too soon and sadly passed
away in 207.

https://t.me/DentalBooksWorld
https://t.me/DentalBooksWorld
xiii

Publisher’s
acknowledgement

Thank you to the 27 dental lecturers and clinicians who participated in


our anonymous peer-​review process and kindly gave their time to this
project.
Thank you to Dr Karolin Hijazi, Clinical Lecturer in Oral Medicine,
University of Aberdeen Dental School, who reviewed the oral medicine
chapter.
Thank you to Professor Balvinder Khambay, School of Dentistry,
University of Birmingham, and Mr P J Turner, Consultant Orthodontist,
Birmingham School of Dentistry, who reviewed the orthodontic chapter.
Thank you to David and Laura Mitchell who gave their kind permission
for the reuse of a table and figure from the Oxford Handbook of Clinical
Dentistry.

https://t.me/DentalBooksWorld
https://t.me/DentalBooksWorld
xv

Contents

About the authors xvii


Contributors xix
Normal and average values xxi
Abbreviations xxvii
How to use this book xxxv

 Anatomy of the head and neck 


2 Preventative and paediatric dentistry 25
3 Orthodontics 47
4 Periodontics 6
5 Endodontics 8
6 Prosthodontics 05
7 Oral and maxillofacial surgery 35
8 Oral medicine and oral pathology 5
9 General medicine 75
0 Therapeutics and medical emergencies 9
 Analgesia, anaesthesia, and sedation 23
2 Dental materials 23
3 Head and neck syndromes 255
4 Radiology and radiography 265
5 Statistics, epidemiology, and dental public health 28

Index 295

https://t.me/DentalBooksWorld
https://t.me/DentalBooksWorld
xvii

About the authors

Nicholas Longridge is an Academic Clinical Fellow/​Specialty Trainee


in Endodontics at Liverpool University Dental Hospital. Alongside his
specialist training, Nicholas is completing a 3-​year Doctorate in Dental
Sciences (DDSc) in Endodontics. His current research interests are in
regenerative endodontics and pulp biology. Prior to commencing his
specialist training, Nicholas worked as a dental core trainee in a variety
of hospital settings across different specialties. He is a member of the
Royal College of Surgeons of Edinburgh and has a Bachelor of Science
degree in Anatomy and Human Biology.

Peter Clarke is a Specialty Registrar in Restorative Dentistry at the


University Dental Hospital of Manchester. Having completed his under-
graduate training, he proceeded to undertake a number of core training
jobs, covering a variety of disciplines. Having been involved at various
levels in undergraduate teaching and examining throughout his career,
he now plays an active role in coordinating the regional teaching pro-
gramme for dental core trainees.

Raheel Aftab is a general dental practitioner working within a multi-


disciplinary dental team in Kent. Following his undergraduate training,
Raheel passed the Membership of Joint Dental Faculties from the Royal
Colleague of Surgeons England examinations and soon after completed
a Postgraduate Certificate in Primary Dental Care from the University
of Kent. He is currently undergoing further training at King’s College
London in fixed and removable prosthodontics. Alongside his clinical
duties, Raheel is an Educational Supervisor for Health Education London
and Kent, Surrey, and Sussex. Raheel takes a particular interest in digital
dentistry, incorporating digital workflow and CAD/​CAM as part of rou-
tine dental care for his patients.

Tariq Ali is a dual-​qualified Oral and Maxillofacial Surgery Registrar


currently working in Queensland, Australia. He undertook both his
undergraduate degrees in the United Kingdom, having first completed
Medicine at the University of Birmingham and later completing Dentistry
at the University of Liverpool. He has worked in a broad range of sur-
gical specialties and emergency medicine, having completed his mem-
berships in Dentistry, Surgery, and Emergency Medicine. Tariq has also
completed his Diploma in Head and Neck Surgery at the Royal College
of Surgeons England, a prerequisite to otorhinolaryngology training. He
now plays an active role in clinical teaching for medical and dental stu-
dents at the University of Queenland.

https://t.me/DentalBooksWorld
xviii About the authors

Series editors
Katharine Boursicot BSc MBBS MRCOG MAHPE NTF SFHEA FRSM
is a consultant in health professions education, with special expertise in
assessment. Previously, she was Head of Assessment at St George’s,
University of London, Barts and the London School of Medicine and
Dentistry, and Associate Dean for Assessment at Cambridge University
School of Clinical Medicine. She is consultant on assessment to several
UK medical schools, medical Royal Colleges, and international institu-
tions, as well as an assessment advisor to the General Medical Council.

David Sales trained as a general practitioner and has been involved


in medical assessment for 30 years. Previously he was the convenor of
the MRCGP knowledge test, chair of the Professional and Linguistics
Assessment Board (PLAB) Part  panel, and consultant to the General
Medical Council Fitness to Practise knowledge tests across all medical
and surgical specialties. He has run item writing workshops for a number
of undergraduate medical schools, medical royal colleges including the
Diploma of Membership of the Faculty of Dental Surgery (MFDS) and
internationally in Europe, South East Asia, South Asia, and South Africa.

https://t.me/DentalBooksWorld
xix

Contributors

Nadia M Ahmed Thomas Albert Park


Specialist Orthodontist Clinical Dental Officer
Kettering General Hospital Pennine Care NHS
Northamptonshire, UK Foundation Trust
Gurpreet Singh Jutley Greater Manchester, UK
Rheumatologist
University Hospital Birmingham
West Midlands, UK

https://t.me/DentalBooksWorld
https://t.me/DentalBooksWorld
xxi

Normal and
average values

Haematology: reference intervals


Measurement Reference interval
White cell count (WCC) 4.0–​.0 × 09/​L
Red cell count M: 4.5–​6.5 × 02/​L; F: 3.9–​5.6
× 02/​L
Haemoglobin M: 3.5–​8.0 g/​dL; F: .5–​6.0
g/​dL
Packed red cell volume (PCV) or M: 0.4–​0.54 I/​L; F: 0.37–​0.47 I/​L
haematocrit
Mean cell volume (MCV) 76–​96 fL
Mean cell haemoglobin (MCH) 27–​32 pg
Mean cell haemoglobin concentration 30–​36 g/​dL
(MCHC)
Neutrophil count 2.0–​7.5 × 09/​L; 40–​75% WCC
Lymphocyte count .3–​3.5 × 09/​L; 20–​45% WCC
Eosinophil count 0.04–​0.44 × 09/​L; –​6% WCC
Basophil count 0.0–​0. × 09/​L; 0–​% WCC
Monocyte count 0.2–​0.8 × 09/​L; 2–​0% WCC
Platelet count 50–​400 × 09/​L
Reticulocyte count 25–​00 × 09/​L; 0.8–​2.0%
Erythrocyte sedimentation rate <20 mm/​hour (but depends on
age; see OHCM 0th edn, p. 372)
Activated partial thromboplastin time 35–​45 seconds
(VIII, IX, XI, XII)
Prothrombin time 0–​4 seconds

https://t.me/DentalBooksWorld
xxii Normal and average values

International normalized Clinical state (see OHCM 0th edn,


ratio (INR) p. 35)
2.0–​3.0 Treatment of deep vein thrombosis (DVT),
pulmonary emboli (treat for 3–​6 months)
2.5–​3.5 Embolism prophylaxis in atrial fibrillation
(see OHCM, p. 335)
3.0–​4.5 Recurrent DVT and pulmonary embolism;
arterial disease, including myocardial
infarction; arterial grafts; cardiac prosthetic
valves (if caged ball, aim for 4–​4.9) and grafts

Biochemistry
Alanine aminotransferase (ALT) 5–​35 IU/​L
Albumin 35–​50 g/​L
Alkaline phosphatase (ALP) 30–​50 IU/​L
Amylase 0–​80 U/​dL
Aspartate aminotransferase (AST) 5–​35 IU/​L
Bilirubin 3–​7 μmol/​L
Calcium (total) 2.2–​2.65 mmol/​L
Chloride 95–​05 mmol/​L
Cortisol 450–​750 nmol/​L (a.m.)
80–​280 nmol/​L (midnight)
C-​reactive protein (CRP) <0 mg/​L
Creatine kinase M: 25–​95 IU/​L
F: 25–​70 IU/​L
Creatinine 70–​<50 μmol/​L
Normal value
Ferritin 2–​200 μg/​L
Folate 2. μg/​L
Gamma glutamyl transpeptidase M: –​5 IU/​L
(GGT) F: 7–​33 IU/​L
Lactate dehydrogenase (LDH) 70–​250 IU/​L
Magnesium 0.75–​.05 mmol/​L
Osmolality 278–​305 mOsmol/​kg
Potassium 3.5–​5 mmol/​L

https://t.me/DentalBooksWorld
Normal and average values xxiii

Biochemistry
Protein (total) 60–​80 g/​L
Sodium 35–​45 mmol/​L
Thyroid-​stimulating hormone (TSH) 0.5–​5.7 mU/​L
Thyroxine (T4) 70–​40 nmol/​L
Thyroxine (free) 9–​22 pmol/​L
Urate M: 20–​480 mmol/​L
F: 50–​39 mmol/​L
Urea 2.5–​6.7 mmol/​L
Vitamin B2 0.3–​0.68 mmol/​L
Arterial blood gases
pH 7.35–​7.45
Arterial oxygen partial pressure >0.6 kPa
(PaO2)
Arterial carbon dioxide partial 4.7–​6.0 kPa
pressure (PaCO2)
Base excess ± 2 mmol/​L
Urine
Cortisol (free) <280 nmol/​24 hours
Osmolality 350–​000 mOsmol/​kg
Potassium 4–​20 mmol/​24 hours
Protein <50 mg/​24 hours
Sodium 00–​250 mmol/​24 hours

https://t.me/DentalBooksWorld
xxiv Normal and average values

The index of orthodontic


treatment need*
Grade  (none)
 Extremely minor malocclusions, including displacements of < mm.

Grade 2 (little)
2a Increased overjet 3.6–​6 mm with competent lips.
2b Reverse overjet 0.–​ mm.
2c Anterior or posterior crossbite with up to  mm discrepancy
between retruded contact position and intercuspal position.
2d Displacement of teeth .–​2 mm.
2e Anterior or posterior openbite .–​2 mm.
2f Increased overbite 3.5 mm or more, without gingival contact.
2g Pre-​normal or post-​normal occlusions with no other anomalies.
Includes up to half a unit discrepancy.

Grade 3 (moderate)
3a Increased overjet 3.6–​6 mm with incompetent lips.
3b Reverse overjet .–​3.5 mm.
3c Anterior or posterior crossbites with .–​2 mm discrepancy.
3d Displacement of teeth 2.–​4 mm.
3e Lateral or anterior openbite 2.–​4 mm.
3f Increased and complete overbite without gingival trauma.

Grade 4 (great)
4a Increased overjet 6.–​9 mm.
4b Reversed overjet >3.5 mm with no masticatory or speech difficulties.
4c Anterior or posterior crossbites with >2 mm discrepancy between
retruded contact position and intercuspal position.
4d Severe displacement of teeth, >4 mm.

*Reproduced from Peter H. Brook, William C. Shaw; The development of an index


of orthodontic treatment priority, European Journal of Orthodontics, Volume ,
Issue 3,  August 989, Pages 309–​320. Copyright © 989, by permission of Oxford
University Press.

https://t.me/DentalBooksWorld
Normal and average values xxv

4e Extreme lateral or anterior openbites, >4 mm.


4f Increased and complete overbite with gingival or palatal trauma.
4h Less extensive hypodontia requiring pre-​ restorative orthodontic
space closure to obviate the need for a prosthesis.
4l Posterior lingual crossbite with no functional occlusal contact in one
or both buccal segments.
4m Reverse overjet .–​3.5 mm with recorded masticatory and speech
difficulties.
4t Partially erupted teeth, tipped and impacted against adjacent
teeth.
4× Supplemental teeth.

Grade 5 (very great)


5a Increased overjet >9 mm.
5h Extensive hypodontia with restorative implications (more than
one tooth missing in any quadrant) requiring pre-​ restorative
orthodontics.
5i Impeded eruption of teeth (with the exception of third molars) due
to crowding, displacement, the presence of supernumerary teeth,
retained deciduous teeth, and any pathological cause.
5m Reverse overjet >3.5 mm with reported masticatory and speech
difficulties.
5p Defects of cleft lip and palate.
5s Submerged deciduous teeth.

Cephalometric values
Table  Cephalometric values: analysis of lateral skull tracings*
SNA = 8° (± 3)
SNB = 79° (± 3)
ANB = 3° (± 2)
-​Max = 09° (± 6)
-​Mand = 93° (± 6) or 20 minus MMPA
MMPA = 27° (± 4)
Facial proportion = 55% (± 2)
Inter-​incisal angle = 33° (± 0)
* Reproduced from Mitchell D, Mitchell L, Oxford Handbook Clinical Dentistry, 6th Edition,
Table 4., page 30, (204). By permission of Oxford University Press.

https://t.me/DentalBooksWorld
xxvi Normal and average values

Adult Dental Health Survey 2009


(some facts)*
● The proportion of adults in England who were edentate (no natural
teeth) has fallen by 22% from 28% in 978 to 6% in 2009.
● Only 7% of dentate adults had very healthy periodontal (gum) tis-
sues and no periodontal disease (i.e. no bleeding, no calculus, no peri-
odontal pocketing of 4 mm or more, and in the case of adults aged
55 or above, no loss of periodontal attachment of 4 mm or more
anywhere in their mouth).
● Overall 45% of adults had periodontal (gum) pocketing exceeding
4 mm, although for the majority (37%), disease was moderate, with
pocketing not exceeding 6 mm.
● Just under one-​third of dentate adults (3%) had obvious tooth decay
in either the crown or root of their teeth.
● There are social variations in dental decay, with adults from routine
and manual occupation households being more likely to have decay
than those from managerial and professional occupational households
(37% compared with 26%)
● The prevalence of decay (using the natural tooth crowns as the
measure) in England has fallen from 46% to 28% since 998, and this
reduction is reflected in all age groups.
● Moderate tooth wear has increased from % in 998 to 5% in 2009,
although severe wear remains rare.
● 75% of adults said that they cleaned their teeth at least twice a day and
a further 23% of adults said that they cleaned their teeth once a day.
● 8% of dentate adults had one or more untreated teeth with
unrestorable decay, and those who did had an average of 2.2 teeth in
this condition.
● The majority of dentate adults (85%) had a tooth affected by res-
toration. Among people with at least one restoration, 9% had some
secondary decay.
● Almost two-​thirds (6%) of dentate adults said that the usual reason
they attended the dentist is for a regular check-​up; 2% of adults who
had ever been to a dentist had an MDAS (Modified Dental Anxiety
Scale) score of 9 or more, which suggests extreme dental anxiety.

*Contains public sector information licensed under the Open Government Licence v3.0.
[http://​www.nationalarchives.gov.uk/​doc/​open-​government-​licence/​version/​3/​]

https://t.me/DentalBooksWorld
xxvii

Abbreviations

3D Three-​dimensional
Aa Aggregatibacter actinomycetemcomitans
ACE Angiotensin-​converting enzyme
ADJ/​EDJ Amelodentinal junction/​enamodentine junction
ADP Adenosine diphosphate
AED Automated external defibrillator
AFP Atypical facial pain
AI Amelogenesis imperfecta
AIDS Acquired immune deficiency syndrome
ALL Acute lymphoblastic leukaemia
ALP Alkaline phosphatase
ALT Alanine aminotransferase
AML Acute myeloblastic leukaemia
ANB A point, nasion, and B point
ANOVA Analysis of variance
ANS Autonomic nervous system; anterior nasal spine
ANUG Acute necrotizing ulcerative gingivitis
AO Atypical odontalgia
AST Aspartate aminotransferase
BD Twice daily
BEC Bioactive endodontic cement
BLS Basic life support
BNF British National Formulary
BOP Bleeding on probing
BP Bullous pemphigoid
BPE Basic periodontal examination
BPPV Benign paroxysmal positional vertigo
CAD/​CAM Computer-​aided design/​computer-​aided manufacture
CBCT Cone beam computed tomography
CCD Charge-​coupled device
CEA Cost-​effectiveness analysis
CEJ Cemento-​enamel junction
CH Calcium hydroxide
CHC Chronic hyperplastic candidiasis

https://t.me/DentalBooksWorld
xxviii Abbreviations

CKD Chronic kidney disease


Cl–​ Chloride ion
CLL Chronic lymphocytic leukaemia
CML Chronic myeloblastic leukaemia
CN Cranial nerve
CoCr Cobalt-​chromium
COPD Chronic obstructive pulmonary disease
CPA Cerebellopontine angle
CPD Continuing professional development
CPP-​ACP Casein phosphopeptides–​amorphous calcium phosphates
CRP C-​reactive protein
CUA Cost utility analysis
CVA Cerebrovascular accident
DCP Dental Care Professional
DI Diabetes insipidus
DIGO Drug-​induced gingival overgrowth
dL Decilitre
DLE Discoid lupus erythematosus
DMFT Diseased, Missing, Filled Teeth
DPH Dental public health
DPT Dental panoramic tomogram/​dentopantomogram
DVT Deep vein thrombosis
EAL Electronic apex locator
ECG Electrocardiogram
ED Emergency department
EDTA Ethylenediaminetetraacetic acid
ENA Extractable nuclear antibody
ENT Ear, nose, and throat
ESR Erythrocyte sedimentation rate
ESRF End-​stage renal failure
EWL Estimated working length
FBC Full blood count
FG Free gingival
fL Femtolitre
FPM First permanent molar
FWS Freeway space
g Gram
GA General anaesthesia
GCS Glasgow coma scale

https://t.me/DentalBooksWorld
Abbreviations xxix

GDC General Dental Council


GDPR General Data Protection Regulation
GGT Gamma glutamyl transpeptidase
GHIH Growth hormone inhibitory hormone
GIC Glass-​ionomer cement
GMP General medical practitioner
GORD Gastro-​oesophageal reflux disease
GP Gutta percha
GTN Glyceryl trinitrate
Gy Gray
HEBP -​hydroxyethylidene-​,-​bisphosphonate
HHV Human herpesvirus
HIV Human immunodeficiency virus
HSW Health and Safety at Work etc. Act
IAN Inferior alveolar nerve
ICDAS International caries detection and assessment system
ICP Intercuspal position
IgA Immunoglobulin A
IgE Immunoglobulin E
IgG Immunoglobulin G
IHD Ischaemic heart disease
IHS Inhalation sedation
IL-​ Interleukin-​
IM Intramuscularly
IMCA Independent Mental Capacity Advocate
INR International normalized ratio
IOTN Index of orthodontic treatment need
IRCP International Commission on Radiological Protection
IR(ME)R Ionising Radiation Medical Exposure) Regulations
IRR Ionising Radiation Regulations
IU International unit
IV Intravenous
JE Junctional epithelium
JSNA Joint strategic needs assessment
kPa Kilopascal
kV Kilovolt
L Litre
LDH Lactate dehydrogenase
LL4 Lower left first premolar

https://t.me/DentalBooksWorld
xxx Abbreviations

LL6 Lower left first molar


LL7 Lower left second molar
LL8 Lower left third molar
LLD Lower left first deciduous molar
LLE Lower left second primary molar
LMN Lower motor neurone
LMWH Low-​molecular-​weight heparin
LPS Lipopolysaccharide
LR4 Lower right first premolar
LR6 Lower right first molar
LR7 Lower right second molar
LR8 Lower right third molar
mA Milliampere
MAC Membrane attack complex (immunology)
MAF Master apical file
MCH Mean cell haemoglobin
MCHC Mean cell haemoglobin concentration
MCV Mean cell volume
MDAS Modified Dental Anxiety Scale
MDMA 3,4-​Methylenedioxymethamphetamine
MHRA Medicines and Healthcare Products Regulatory Agency
MI Myocardial infarction
mm Millimetre
mmol Millimole
MMP Mucous membrane pemphigoid
MOCDO Missing teeth, overjets, crossbites, displacement of contact
points, overbites
mOsmol Milliosmole
MPa Megapascal
MPTS Methacryloxypropyl-​trimethoxysilane
MRONJ Medication-​related osteonecrosis of the jaw
MTA Mineral trioxide aggregate
MTAD Mixture of tetracycline, citric acid, and a detergent
mU Milliunit
NADH Nicotinamide adenine dinucleotide
NAI Non-​accidental injury
NaOCl Sodium hypochlorite
NHS National Health Service
NIDDM Non-​insulin-​dependent diabetes mellitus

https://t.me/DentalBooksWorld
Abbreviations xxxi

NiTi Nickel titanium


nmol Nanomole
NOAC New oral anticoagulant
NSAID Non-​steroidal anti-​inflammatory drug
NSTEMI non-​ST segment elevation myocardial infarction
OD Once daily
OHCM Oxford Handbook of Clinical Medicine
OHNA Oral health needs assessment
OJ Overjet
OM Occipitomental
OM0O Occipitomental 0
OM30O Occipitomental 30
OMFS Oral and maxillofacial surgery
OPG Oral pantomogram
OPT Orthopantomogram
OVD Occlusal vertical dimension
PA Periapical; posteroanterior
PaCO2 Arterial partial pressure of carbon dioxide
PaO2 Arterial partial pressure of oxygen
PCV Packed cell volume
PD Peritoneal dialysis
PDL Periodontal ligament
PE Pulmonary embolism
PEFR Peak expiratory flow rate
pg Picogram
PHG Primary herpetic gingivostomatitis
PJP Pneumocystis jirovecii pneumonia
PMC Preformed metal crown
pmol Picomole
PMR Polymyalgia rheumatica
PP Pancreatic polypeptide
ppm Parts per million
PRR Pathogen recognition receptor
PSP Photostimulable phosphor
PVS Polyvinylsiloxane
QDS Four times daily
RANK Receptor activator of nuclear factor kappa-​Β
RAP Retruded axis position
RAS Recurrent aphthous stomatitis

https://t.me/DentalBooksWorld
xxxii Abbreviations

RBB Resin-​bonded bridge


RCA Root cause analysis
RCP Retruded contact point/​position
RCRG Royal College of Radiologists Guidelines
RCT Randomized controlled trial
ReRCT Revision root canal treatment
RFH Resting face height
RHP Radiation and Health Protection
RIDDOR Reporting of Injuries, Diseases and Dangerous
Occurrences Regulations 995
RMGIC Resin-​modified glass-​ionomer cement
RPA Radiation Protection Advisor
RPD Removable partial denture
RSD Root surface debridement
SAH Subarachnoid haemorrhage
SCC Squamous cell carcinoma
SIRS Systemic inflammatory response syndrome
SJS Stevens–​Johnson syndrome
SLE Systemic lupus erythematosus
SLM Selective laser melting
SNA Sella turcica, nasion, and A point
SNB Sella turcica, nasion and A point
spp. Species
SSC Stainless steel crown
STEMI ST segment elevation myocardial infarction
StR Specialty registrar
T4 Thyroxine
TB Tuberculosis
TBSA Total body surface area
TEGMA Tri-​ethylene glycol dimethacrylate
TEN Toxic epidermal necrolysis
THA Terminal hinge axis
TIA Transient ischaemic attack
TMD Temporomandibular dysfunction
TMJ Temporomandibular joint
TON Traumatic optic neuropathy
TPHA Treponema pallidum haemagglutination assay
TSH Thyroid-​stimulating hormone
U Unit

https://t.me/DentalBooksWorld
Abbreviations xxxiii

UK United Kingdom
UL Upper left permanent central incisor
UL2 Upper left lateral incisor
UL5 Upper left second premolar
UL6 Upper left first permanent molar
ULA Upper left primary central incisor
UMN Upper motor neurone
UR Upper right central incisor
UR3 Upper right permanent canine
UR5 Upper right second premolar
UR6 Upper right first permanent molar
USO Upper standard occlusal
WBC White blood cell
WCC White cell count
WHO World Health Organisation
WSN White sponge naevus
ZOE Zinc oxide eugenol

https://t.me/DentalBooksWorld
https://t.me/DentalBooksWorld
xxxv

How to use this book

Oxford Assess and Progress, Clinical Dentistry has been carefully designed to
ensure you get the most out of your revision and are prepared for your
exams. Here is a brief guide to some of the features and learning tools.

Organization of content
Chapter editorials will help you unpick tricky subjects, and when it is late
at night and you need something to remind you why you are doing this,
you will find words of encouragement!
Answers can be found at the end of each chapter, in order.

How to read an answer


Unlike other revision guides on the market, this one is crammed full of
feedback, so you should understand exactly why each answer is correct,
and gain an insight into the common pitfalls. With every answer, there is
an explanation of why that particular choice is the most appropriate. For
some questions, there is additional explanation of why the distractors
are less suitable. Where relevant, you will also be directed to sources
of further information such as the Oxford Handbook of Clinical Dentistry,
websites, and journal articles.

Progression points
The questions in every chapter are ordered by level of difficulty and
competence, indicated by the following symbols:
★ Graduate ‘should know’—​you should be aiming to get most of
these correct.
★★ Graduate ‘nice to know’—​these are a bit tougher, but not
above your capabilities.
★★★ Foundation dentist ‘should know’—​these will really test your
understanding.
★★★★ Foundation dentist ‘nice to know’—​give these a go when you are
ready to challenge yourself.

https://t.me/DentalBooksWorld
xxxvi How to use this book

Oxford Handbook of Clinical Dentistry


The OHCD page references are given with the answers to some ques-
tions (e.g. OHCD 6th edn → p. 340). Please note that this reference is the
sixth edition of the OHCD, and that subsequent or previous editions
are unlikely to have the same material in exactly the same place.

https://t.me/DentalBooksWorld
Sphenoethmoidal Crista galli
joint
Optic canal Cribriform plate
Superior orbital
fissure Orbital part of frontal
Pituitary fossa Lesser wing of
Foramen sphenoid
rotundum Greater
Foramen ovale wing of sphenoid
Foramen lacerum Dorsum sellae
Squamous part of
Foramen spinosum temporal
Internal auditory Petrous part of
meatus temporal
Jugular foramen Basilar part of occipital
Hypoglossal canal Condylar part of
occipital
Internal occipital
crest and Squamous part of
protuberance occipital

Figure .5
Reproduced from Atkinson Martin E, Anatomy for dental students, ­figure 22.5, page 23,
Copyright (203) by permission of Oxford University Press.

(a)

(b)

https://t.me/DentalBooksWorld
Figure 6.2a
Figure 6.3
Reproduced from Kidd, E. Essentials of Dental Caries (3rd Ed). Figure .4a, page 6, Oxford
University Press. Oxford. 206 by permission of Oxford University Press.

Figure 8.
Reproduced from Field E.A. & Longman W.R., Tyldesley’s Oral Medicine, 5th Edition,
Figure 6.8, page 70, Copyright (2003), by permission of Oxford University Press.

https://t.me/DentalBooksWorld
Figure 8.2
Reproduced from Field E.A. & Longman W.R., Tyldesley’s Oral Medicine, 5th Edition,
Figure .2, page 27, Copyright (2003), by permission of Oxford University Press.

Figure 8.3
Reproduced from Field E.A. & Longman W.R., Tyldesley’s Oral Medicine, 5th Edition,
Figure 2.4, pg 46, Copyright (2003), by permission of Oxford University Press.

https://t.me/DentalBooksWorld
Figure 8.4
Reproduced from Field E.A. & Longman W.R., Tyldesley’s Oral Medicine, 5th Edition,
Figure 4.2, page 33, (2003), Copyright (2003), by permission of Oxford University Press.

Figure 8.5
Reproduced from Field E.A. & Longman W.R., Tyldesley’s Oral Medicine, 5th Edition,
Figure .7, page 3, Copyright (2003), by permission of Oxford University Press.

https://t.me/DentalBooksWorld
Figure 8.6
Reproduced from Soames J.V. and Southam J.C., Oral Pathology, 4th Edition, Figure 9.2,
page 29, Copyright (2005), by permission of Oxford University Press.

Figure 8.7
Reproduced from Soames J.V. and Southam J.C., Oral Pathology, 4th Edition, Figure 6.24,
p79 Copyright (2005), by permission of Oxford University Press.

https://t.me/DentalBooksWorld
Figure 8.8
Reproduced from Robinson M et al, Soames’ and Southam’s Oral Pathology, Fifth edition,
­figure 2.58a, page 50, Copyright (208), by permission of Oxford University Press.

Figure 8.9
Reproduced from Field E.A. & Longman W.R., Tyldesley’s Oral Medicine, 5th Edition,
Figure 4.5, pg 39, Copyright (2003), by permission of Oxford University Press.

https://t.me/DentalBooksWorld
Figure 8.0
Reproduced from Field E.A. & Longman W.R., Tyldesley’s Oral Medicine, 5th Edition,
Figure 4.7, pg 4, Copyright (2003), by permission of Oxford University Press.

Figure 8.
Reproduced from Mitchell D, Mitchell L, Oxford Handbook Clinical Dentistry, 6th Edition,
Figure 0., Page 45, (204) by permission of Oxford University Press.

https://t.me/DentalBooksWorld
Figure 8.2
Reproduced from Soames J.V. and Southam J.C., Oral Pathology, 4th Edition, Figure 9.,
p20 Copyright (2005), by permission of Oxford University Press.

https://t.me/DentalBooksWorld
Chapter  1

Anatomy of the head


and neck
Nicholas Longridge

‘Which nerves do you need to anaesthetize?’


‘Which vessel are you trying to avoid?’
Both are routine questions encountered by students during their time
in dental school, and both require sound knowledge of the anatomy of
the head and neck. From wrestling with basic anatomical concepts and
planes to tracing the branches of the external carotid artery, anatomy
will underpin the rest of your practising career and is a fundamental
building block on which all other knowledge can be laid down. Basic ana-
tomical knowledge begins with the osseous structures of the head and
neck, blood vessels, lymphatics, and nerves. Interpretation of this know-
ledge is required for functional and clinical applications, which is a daily
occurrence for practising dentists and dental care professionals. Such a
large subject is difficult to assess in a small number of questions, but
this chapter touches on aspects of developmental embryology and tooth
formation, along with functional anatomical questions designed to test
the theory behind some common dental procedures and clinical presen-
tations. Undoubtedly, excellent knowledge of the innervation and blood
supply to the teeth and surrounding structures will be most beneficial
for dentists and dental care professionals during their practising careers.
Key topics include:
● Anatomical planes and terminology
● Craniofacial development
● The musculoskeletal system, including ossification and bony remo­-
delling
● Innervation and vascular supply to the head and oral cavity, including
the cranial nerves
● Structure of the eye, ear, nasal cavity, and oral cavity
● Odontogenesis
● Histology of the oral cavity.

https://t.me/DentalBooksWorld
2 Chapter  Anatomy of the head and neck

QUESTIONS

. A 25-​year-​old man attends for review. He has clinical signs of


Treacher–​Collins syndrome, and the consultant is discussing the
first and second pharyngeal arch syndromes. A junior dental student asks
which cranial nerve develops with, and goes on to innervate, the second
pharyngeal arch. Which is the single most appropriate response? ★
A Abducens—​VI
B Facial—​VII
C Glossopharyngeal—​IX
D Mandibular branch of the trigeminal—​V3
E Vagus—​X

2. A 22-​year-​old man undergoes a bimaxillary osteotomy to correct


a severe reverse overjet (Class III skeletal relationship). During the
procedure, the consultant asks the specialty registrar what single type of
ossification is involved in the formation of the mandible during embryo-
logical development. ★
A Ectopic
B Endochondral
C Endosteal
D Intramembranous
E Periosteal

3. A 23-​year-​old woman returns to the surgery 5 days after surgical


extraction of her lower right third molar. She reports loss of sen-
sation from the floor of the mouth on the right-​hand side. Which single
nerve is most likely to have been damaged during her extraction? ★
A Chorda tympani
B Hypoglossal
C Inferior alveolar
D Lingual
E Nerve to the mylohyoid

https://t.me/DentalBooksWorld
Questions 3

4. A 45-​year-​old man is receiving an inferior alveolar nerve block for


the restoration of a lower right first molar from a dental student,
who is administering the injection for the first time. They are using the
direct technique (over the contralateral premolar teeth), and their super-
visor asks them which single muscle will the needle pass through. ★
A Buccinator
B Masseter
C Medial pterygoid
D Palatoglossus
E Superior constrictor

5. A 25-​year-​old man attends with a buccal swelling next to the lower


right first molar. Incision and drainage of the abscess is planned.
Which single branch of the external carotid artery should be avoided,
whilst conducting this treatment? ★
A Ascending pharyngeal
B Facial
C Lingual
D Maxillary
E Parotid

6. An 8-​year-​old man is having a cranial nerve assessment performed


after a traumatic incident. Upon protrusion of his tongue, the tongue
deviates to the left-​hand side. Which single nerve has been affected? ★
A Glossopharyngeal
B Hypoglossal
C Lingual
D Mandibular branch of the trigeminal
E Pharyngeal plexus

7. A 33-​year-​old woman attends for extraction of the upper left


second premolar. The clinical tutor asks the student to anaesthe-
tize the patient using infiltrations prior to the procedure. Which nerves
require anaesthetizing? (Select one from the options listed below.) ★
A Anterior superior alveolar nerve + middle superior alveolar nerve
B Middle superior alveolar nerve + greater palatine nerve
C Middle superior alveolar nerve + nasopalatine nerve
D Posterior superior alveolar nerve + greater palatine nerve
E Posterior superior alveolar nerve + lesser palatine nerve
https://t.me/DentalBooksWorld
4 Chapter  Anatomy of the head and neck

8. A 73-​year-​old man has Paget’s disease and develops trigeminal


neuralgia in the lower jaw region. Compression of which single
cranial foramen is most likely to be causing his neuralgia? ★★
A Foramen lacernum
B Foramen ovale
C Foramen rotundum
D Foramen spinosum
E Stylomastoid foramen

9. A 45-​year-​old woman attends with localized right-​sided facial pain


of an aching nature and trismus. The pain has been present for 4
weeks and is localized to the pre-​auricular region. It was preceded by
recurrent clicking, with no restriction on opening. The patient is now
unable to open her mouth beyond 20 mm. Which single component of
condylar head movement is inhibited? ★★
A Abduction
B Circumduction
C Protrusion
D Rotation
E Translation

0. The presence of the anatomical attachment of which single


muscle visible in Figure . may obscure radiographic assess-
ment of the root apices? ★★
A Buccinator
B Lateral pterygoid
C Medial pterygoid
D Mylohyoid
E Posterior belly of the digastric

https://t.me/DentalBooksWorld
Questions 5

Figure .
Reproduced from Whaites, E, Essentials of Dental Radiography and Radiology 5th Ed.
Copyright 203, Fig 2.7a, Page 274, with permission from Elsevier.

. A 24-​year-​old man who has undergone a low-​volume cone


beam computed tomography (CBCT) scan of a tooth with a
complex dens invaginatus is being assessed (Oehler’s classification 3b).
The CBCT sofware is used to scroll through the full series of images in a
corono-​apical direction, to enable visualization of the complex anatomy
of the root in cross-​section. Using the method described above, which
single anatomical plane is being utilized to assess the image? ★★
A Coronal
B Frontal
C Longitudinal
D Sagittal
E Transverse

2. A 25-​year-​old woman has an intraoral swelling that has been


present for 2 days. The differential diagnosis for this swelling
would differ, based on its location. Other than the gingiva, which location
in the oral cavity would you be able to exclude salivary gland pathology
from the differential diagnosis? ★★★
A Anterior hard palate
B Buccal sulcus
C Floor of the mouth
D Soft palate
E Tongue
https://t.me/DentalBooksWorld
6 Chapter  Anatomy of the head and neck

3. An 8-​year-​old man attends the Maxillofacial Department, fol-


lowing an alleged assault to the right side of his head. He has
pain, bruising, and depression of the right cheek, with limited opening.
Radiographic assessment confirms a fracture of the right zygomatic arch.
Entrapment of which single muscle is most likely responsible for the re-
stricted opening? ★★★
A Buccinator
B Lateral pterygoid
C Medial pterygoid
D Masseter
E Temporalis

4. A -​day-​old boy has been born at full term in hospital with a
unilateral cleft lip. The parents are concerned and ask how this
has happened. Which two structures would have failed to fuse in this
scenario? (Select one answer from the options listed below.) ★★★
A Frontonasal prominence and the intermaxillary segment
B Frontonasal prominence and the maxillary prominence
C Lateral nasal prominence and the maxillary prominence
D Medial nasal prominence and the intermaxillary segment
E Medial nasal prominence and the maxillary prominence

5. A 7-​year-​old boy presents with signs of amelogenesis imper-


fecta (AI). Clinically, the enamel is of normal thickness but is
comparatively soft. Radiographically, the enamel is of similar radiodensity
to the dentine. Which single stage of tooth development is the disorder
most likely to have affected? ★★★★
A Bell
B Bud
C Cap
D Crown maturation
E Root development

https://t.me/DentalBooksWorld
Questions 7

6. An 82-​year-​old woman fails to attend her review appointment


at the dry mouth clinic. The consultant takes this opportunity to
discuss the autonomic nerve supply to the parotid gland with junior staff.
Parasympathetic nerve supply to the parotid gland is via the otic ganglion.
Which single cranial nerve supplies pre-​ganglionic parasympathetic nerve
fibres to the otic ganglion? ★★★★
A Accessory nerve
B Facial nerve
C Glossopharyngeal nerve
D Vagus nerve
E Vestibulocochlear nerve

7. A 68-​year-​old woman with unilateral facial paralysis is assessed


by a maxillofacial doctor in the Emergency Department (ED).
Her mouth and cheek are drooping on the right-​hand side, and she cannot
bare her teeth or purse her lips on that side when asked. However, she
can still wrinkle the skin of her forehead to command. What is the single
most likely cause for the signs that have been described? ★★★★
A Acoustic neuroma
B Bell’s palsy
C Otitis media
D Parotid tumour
E Stroke

8. A 34-​year-​old woman requires root canal treatment on an


asymptomatic lower left first permanent incisor (LL), which
has two canals. Having been given two buccal infiltrations for anaes-
thesia, she can still feel pain. Which single nerve is most likely to be pro-
viding accessory innervation to the tooth? (Select one answer from the
options listed below.) ★★★★
A Hypoglossal
B Lingual
C Long buccal
D Nerve to the mylohyoid
E Right inferior alveolar

https://t.me/DentalBooksWorld
8 Chapter  Anatomy of the head and neck

ANSWERS
. B ★
The pharyngeal apparatus comprises the pharyngeal arches, pouches,
membranes, and grooves. Specifically, several pairs of pharyngeal arches
begin to develop, following migration of neural crest cells into the head
and neck regions during the fourth embryonic week. Each arch has a car-
tilaginous component, which develops into skeletal structures and a mus-
cular component that develops into the muscles of the head and neck.
A cranial nerve also develops with each arch, which goes on to supply
the muscular and mucosal derivatives of the arch. Table . shows the
skeletal derivatives of the pharyngeal arches.
Keywords: embryological development, pharyngeal apparatus, cranial
nerve.

Table . Skeletal derivatives of the pharyngeal arches


Pharyngeal Cranial nerve Muscular Skeletal Vascular
arch structures structures structures
First V—​trigeminal Muscles of Meckel’s Maxillary
(mandibular) mastication, cartilage artery
anterior belly (contributes
of the digastric, to mandible),
tensor tympani, incus, and
tensor veli palatini malleus
Second VII—​facial Muscles of facial Styloid Hyoid and
(hyoid) expression, process, stapedial
posterior belly stapes, upper artery
of the digastric, part of body
stapedius, of the hyoid
stylohyoid and lesser
cornu
Third IX—​ Stylopharyngeus Lower part Internal
glossophary- of body of carotid
ngeal the hyoid and artery
greater cornu
Fourth X—​vagus Pharyngeal Thyroid Right
constrictors, cartilage subclavian
cricothyroid, artery and
levator veli palatini aorta
Reproduced from Atkinson Martin E, Anatomy for dental students, table 2., Copyright
(203) with permission from Oxford University Press.

2. D ★
Formation of the bony skeleton begins in utero as early as 6 weeks post-​
fertilization. Protection, support, and production of blood cells are vital
functions of bone. Bone formation (osteogenesis) relies upon the gradual
https://t.me/DentalBooksWorld
answers 9

replacement of tissue with bone by osteoblasts in a process known as


ossification. Two main types of ossification exist: endochondral and
intramembranous. Endochondral ossification occurs in the majority of
bones in the body and involves gradual replacement of a cartilage model
with bone. The genetically determined cartilage model grows via expan-
sion (endosteal ossification) and new cartilage production on the outer
surface (ectopic ossification), prior to fibroblasts differentiating into
osteoblasts and depositing bone. In contrast, intramembranous ossifi-
cation involves primitive connective tissue (mesenchyme) differentiating
into osteoblasts. The osteoblasts secrete a matrix which is subsequently
calcified. The mandible and maxilla are both formed via intramembranous
ossification, along with numerous other facial bones, the flat bones of
the skull, and the clavicles, to name a few. Remodelling (periosteal ossi-
fication) is heavily involved in both ossification processes, as the bones
formed are maintained via dynamic osteoblastic–​osteoclastic activity.
This process is responsible for the changes seen in the alveolar ridge fol-
lowing a tooth extraction. It is important to note that, whilst calcification
occurs during ossification, the terms are not synonymous, as calcifica-
tion can take place without the formation of bone. Intramembranously
ossified bones: skull vault, squamous part of the temporal, mandible,
premaxilla, maxilla, zygoma, wings of the sphenoid. Endochondrally ossi-
fied bones: petrous part of the temporal, ethmoid, body of the sphenoid,
basilar part of the occiput.
Keywords: ossification, primary ossification centres.

3. D ★
The lingual nerve supplies general sensation to the anterior two-​thirds
of the tongue, the floor of the mouth, and the lingual aspect of the
mandibular gingivae. It passes along the lingual aspect of the mandible
and is often in close proximity to the bone forming the sockets of the
mandibular wisdom teeth. As a result, it can be easily damaged during
extraction. The reported incidence of lingual nerve injury following third
molar surgery (when a lingual flap is not raised) varies significantly from
study to study; permanent injury is thought to occur in approximately 0–​
2% of cases. Lingual nerve damage following third molar surgery is more
likely than damage to the inferior alveolar nerve, and risk factors such as
distoangular impactions, inappropriate technique, or lack of experience
can increase a patient’s risk. Lingual nerve damage is often temporary,
and recovery is reported in the majority of cases. Lingual retraction
during lower wisdom tooth extraction remains controversial. However,
some clinicians claim a reduced risk of permanent lingual nerve damage
as justification for use of this technique. Incidence data do not support
this theory, with higher permanent damage documented. The hypo-
glossal nerve provides motor function to the muscles of the tongue. The
inferior alveolar nerve runs within the mandible to supply the mandibular
teeth and exits the mental foramen to become the mental nerve, which
innervates the skin, labial gingivae, and soft tissues of the chin. The nerve
to the mylohyoid provides motor control to the mylohyoid muscle. The
chorda tympani nerve supplies special sensory function (taste) to the
anterior tongue.

https://t.me/DentalBooksWorld
10 Chapter  Anatomy of the head and neck

Keywords: lower right third molar, loss of sensation.


→ Renton T. Oral surgery: part 4. Minimising and managing nerve injuries
and other complications. British Dental Journal. 203;25:393.

4. A ★
The direct technique for achieving inferior alveolar anaesthesia is
one of the most common procedures carried out in dentistry and
provides anaesthesia to the entire quadrant of mandibular teeth,
as well as associated soft tissues. The needle passes medially to the
internal oblique ridge and lateral to the pterygo-​mandibular raphe at a
height of approximately  cm above the occlusal plane. After passing
through the buccinator muscle, the needle travels along the inside of
the mandibular ramus into the pterygomandibular space and is in-
serted approximately 20–​25 mm until it touches bone. The needle
should be withdrawn slightly before local anaesthetic is deposited. At
this point, the needle tip will be superior to the mandibular foramen,
medial to the mandibular ramus and lateral to the medial pterygoid and
sphenomandibular ligament. The attachment for the sphenomandibular
ligament, or lingula, will be slightly inferior and anterior to the final
needle position. Upon withdrawing, continued injection of solution is
likely to anaesthetize the lingual nerve. It is important to remember the
bony reference points when administering the local anaesthetic. The
buccinator muscle passes anteriorly from the pterygomandibular raphe
to insert into the orbicularis oris anteriorly where it contributes to the
modiolus. It forms the lateral aspect of the oral cavity and inserts into
the lateral surfaces of the maxilla and mandible. It is supplied by the
facial nerve [cranial nerve (CN) VII)]. See Figure .2 which illustrates
the mandible.
Keywords: inferior dental block, direct technique.

Coronoid process Mandibular foramen


Condylar
Lingula process
Mylohyoid groove
and line
Sublingual fossa
Ramus
Submandibular fossa

Mental foramen
Angle
Body
Figure .2
Reproduced from Scully C, Oxford Handbook Applied Dental Sciences, ­figure 2.2, page 29,
Copyright (2003) by permission of Oxford University Press.

https://t.me/DentalBooksWorld
answers 11

5. B ★
Branches of the external carotid artery provide the vascular supply
to the majority of structures in the facial region. The maxillary
artery branches from the external carotid artery and passes deep
to the head of the mandibular condyle. It supplies numerous oral
structures, including the teeth and the muscles of mastication. The
inferior alveolar artery branches from the first part of the maxil-
lary artery and supplies the lower teeth. The teeth and associated
structures in the maxilla are supplied by the posterior, middle, and
anterior superior alveolar arteries. These are branches of the third,
or pterygopalatine, part of the maxillary artery, which branches in the
pterygopalatine fossa.
The facial artery and the transverse facial arteries supply the tissues of
the face, and the lingual artery supplies the tongue. The facial artery
runs in close proximity to the angle of the mandible before continuing
up into the cheek. Great care must be taken when performing surgery
(incisions) in the lower molar region, especially when localized swelling
may distort the soft tissues. Inadvertent injection of local anaesthetic into
the facial artery has also been reported, with subsequent blanching of
the soft tissues of the cheek. See Figure .3 which illustrates the blood
supply to the face.
Keywords: lower right first molar, carotid artery.

Supra-orbital a.
Supratrochlear a. Deep
temporal a.
Ophthalmic a.

Angular a.
Posterior
Infraorbital a.
auricular a.
Superior
alveolar a. Maxillary a.
(posterior, middle, Mylohyoid branch
and anterior of inferior alveolar a.
branches) Stylohyoid m.
Mental a.
Occipital a.
Submental a. Facial a.
Digastric m. Lingual a. External carotid a.
Figure .3
Reproduced from Scully C, Oxford Handbook Applied Dental Sciences, ­figure 3.4, page 38,
Copyright (2003) by permission of Oxford University Press.

https://t.me/DentalBooksWorld
12 Chapter  Anatomy of the head and neck

6. B ★
CN assessments are an important part of a junior dentist’s clinical know-
ledge. If asked to assess the CNs, the twelfth nerve (hypoglossal—​XII)
deviates to the affected side, i.e. if the patient had a large malignant
lesion at the base of the left side of their tongue, the tongue would
deviate to the left side. The glossopharyngeal nerve supplies sen-
sory function to the posterior part of the tongue. The lingual nerve
is a branch of the mandibular nerve of the trigeminal nerve (CN V),
which does not provide motor function to the tongue. The pharyngeal
plexus supplies the palatoglossus muscle of the tongue, which is a small
extrinsic muscle of the tongue that passes from the soft palate into the
tongue. Motor fibres of the pharyngeal plexus supply the majority of
the muscles of the soft palate and the constrictor muscles. The motor
fibres travel to the muscles via the vagus nerve but are derived from the
cranial root of the accessory nerve (CN XI).
Keywords: tongue, deviation.

7. B ★
Nerve supply to the maxillary teeth is via the maxillary branch of
the trigeminal nerve (CN V2). The nerve enters the pterygopalatine
fossa via the foramen rotundum. At this point, the posterior superior
alveolar nerve bifurcates and passes inferiorly along the infratemporal
surface of the maxilla to innervate the maxillary molars. The majority
of the maxillary nerve continues as the infraorbital nerve, which con-
tinues through the pterygopalatine fossa and inferior orbital fissure
to enter the infraorbital canal. The middle superior alveolar nerve
branches along this course and runs to supply to upper premolars
and, on occasions, the mesiobuccal root of upper first molars. The
anterior superior alveolar nerve branches further along this course
to supply the canine and upper incisors. Collectively, these branches
form a plexus which can be referred to as the superior dental
plexus. The upper anterior teeth can also be anaesthetized using an
infraorbital block.
For extractions, it is also necessary to anaesthetize the gingival tissues.
The palatine gingivae are supplied by the greater palatine nerves, and
the buccal and labial gingivae are supplied by the same superior alveolar
branch that supplies the tooth. Buccal or labial infiltration will therefore
anaesthetize the tooth and its labial or buccal gingivae, so only the palatal
gingivae require additional infiltration. Knowledge of these nerve path-
ways is essential for all dental practitioners and can assist in the diagnosis
of poorly localized, irreversible pulpitis when other special investiga-
tions have failed. See Figure .4 which shows the arterial supply and
innervation of the palate.
Keywords: upper, premolar, infiltrations, nerves.

https://t.me/DentalBooksWorld
answers 13

Nasopalatine Nasopalatine
artery nerve (V2)

Greater palatine Greater palatine


artery nerve (V2)

Lesser palatine Lesser palatine


artery nerve (V2)

Figure .4
Reproduced from Scully C, Oxford Handbook Applied Dental Sciences, ­figure 4., page 54,
Copyright (2003) by permission of Oxford University Press.

8. B ★★
Paget’s disease of bone is a rare chronic disease that involves disorgan-
ized bone remodelling. The disease can result in pain, fractures, and arth-
ritis. It most commonly affects bones of the axial skeleton and is often
localized to a small number of bones, including the pelvis. It can occa-
sionally affect the cranial bones, with potential compression of neural
tissue at the cranial foramina. Good knowledge of nerve pathways is
therefore important during diagnosis and assessment of symptoms. In
this particular scenario, trigeminal neuralgia is affecting the mandibular
distribution of the trigeminal nerve. See Figure .5 (see Colour Plate
section) which shows a superior view of the cranial base.
Foramen and associated CNs include:
● Optic canal—​optic nerve (CN II)
● Superior orbital fissure—​ oculomotor (CN III), trochlear (CN IV), abdu-
cens (CN VI), ophthalmic branches of the trigeminal nerve
● Foramen rotundum—​maxillary branches of the trigeminal nerve
● Foramen ovale—​mandibular branch of the trigeminal nerve
● Stylomastoid foramen—​facial nerve (CN VII)
● Jugular foramen—​glossopharyngeal (CN IX), vagus (CN X), branches of

the accessory nerve (CN XI)


● Hypoglossal canal—​hypoglossal nerve

Keywords: cranial foramina.

9. E ★★
The temporomandibular joint (TMJ) comprises the mandibular fossa and
articular eminence of the temporal bone and the head of the mandibular
condyle. An articular disc separates the two bones, and the joint is sur-
rounded by a joint capsule which contains synovial fluid. The articulating
surfaces are lined with fibrocartilage, and the articular disc is attached
to the capsule and to bone by anterior and posterior fibroelastic bands.
The anterior aspect of the articular disc and joint capsule is attached to

https://t.me/DentalBooksWorld
14 Chapter  Anatomy of the head and neck

the superior head of the lateral pterygoid muscle. The condylar head
undergoes two major components of movement during opening: rota-
tion and translation. These allow elevation, depression, protrusion, and
retraction of the mandible to take place.
Initial rotation of the mandible about the terminal hinge axis (THA) oc-
curs within the inferior compartment of the joint capsule and accounts
for the first 5–​20 mm of opening (total 45 mm). Following this, transla-
tion occurs in the upper compartment, and the mandibular head moves
anteriorly onto the articular eminence. Therefore, in the absence of
pathology, translation accounts for the majority of mouth opening. See
Figure .6 which shows a sagittal section of the TMJ, (a) the mandible
elevated, and (b) the mandible depressed.
Keywords: pain, pre-​ auricular, preceded, recurrent click, 20 mm,
condylar head.

Articular eminence (sectioned)

Upper (superior) compartment


TMJ articular disc

Lower (inferior) compartment

Area of retrodiscal pad


Mandibular condyle (sectioned)

Articular eminence (sectioned)


Upper (superior) compartment
Articular disc
Retrodiscal pad

Lower (inferior) compartment

Mandibular condyle (sectioned)

Figure .6
Reproduced from Scully C, Oxford Handbook Applied Dental Sciences, Figure 4.2 b & c, pg
63, Copyright (2003) by permission of Oxford University Press.

https://t.me/DentalBooksWorld
answers 15

0. D ★★
The bony attachment of the mylohyoid (mylohyoid ridge) is often seen
on lower posterior periapicals adjacent to, or overlapping, the root
apices of the posterior teeth. It is often visualized as a radio-​opaque
line within the premolar–​molar region. Knowledge of the anatomical
attachment of this muscle is important when diagnosing pathology
and assessing the proximity of the inferior dental nerve. Furthermore,
the location of the mylohyoid ridge can have significant ramifications
with regard to the spread of periapical infections. As the mylohyoid
represents the inferior boundary of the mouth, roots that lie above
the mylohyoid ridge are more likely to produce a sublingual swelling,
as infection is likely to remain above the mylohyoid. By comparison,
roots that pass below the mylohyoid attachment are more likely to
enable spread of infection below the floor of the mouth into the sub-
mandibular region, i.e. a submandibular space swelling. The origins and
insertions of the other muscles can be found in the recommended
reading.
Keywords: anatomical muscle attachment, lower posterior periapical.
→ Atkinson M. Anatomy for Dental Students (4th ed.). Oxford University
Press, Oxford; 203.

. E ★★
The anatomical planes are key references used to describe and
document clinical findings (see Figure .7 for anatomical position and
planes). More recently, in dentistry, the use of advanced radiography
techniques, such as CBCT, has gained popularity due to their ability
to provide high-​quality, three-​dimensional (3D) images of complex
anatomical structures. The technology is now frequently encoun-
tered in everyday scenarios, e.g. when assessing impacted teeth,
root canal anatomy, and residual bone for implant placement. Whilst
CBCT images can be reformatted to provide a 3D reconstruction,
they are frequently viewed individually in each of the three anatom-
ical planes:
. Coronal (frontal)—​a vertical plane that divides the body into anterior
and posterior segments
2. Sagittal (longitudinal)—​a vertical plane that divides the body into left
and right segments
3. Transverse (axial)—​ a horizontal plane that divides the body into
superior and inferior segments.
Keywords: CBCT, corono-​apical, cross-​section, anatomical plane.
→ Devlin H, Craven R. Oxford Handbook of Integrated Dental Biosciences.
Oxford University Press, Oxford; 208.

https://t.me/DentalBooksWorld
16 Chapter  Anatomy of the head and neck

Coronal or frontal plane

Sagittal plane

Horizontal or
transverse plane

Figure .7
Reproduced from Scully C, Oxford Handbook Applied Dental Sciences, Figure 2., pg 2,
Copyright (2003) by permission of Oxford University Press.

2. A ★★★
There are three pairs of major salivary glands: the parotid, subman-
dibular, and sublingual glands. As well as this, several hundreds of minor
salivary glands are located within the submucosa of the mouth. These
glands can become blocked or traumatized to form mucoceles or can
undergo neoplastic change to become benign or malignant neoplasms.
However, minor salivary glands are absent from the anterior hard palate
and the gingivae. As a result, swellings identified in these regions are
unlikely to have developed from salivary tissue, and this can be excluded
from the differential diagnosis. Understanding the histology of the oral
mucosa and soft tissues is important during assessment and diagnosis
of disease.
Keywords: salivary gland pathology, location.

3. E ★★★
General signs and symptoms of facial bone fractures include pain,
swelling, bruising, haematomas, and bony steps. Specific signs related to

https://t.me/DentalBooksWorld
answers 17

the local anatomy may include unilateral epistaxis (e.g. if an antral wall
is fractured), visual disturbances (if the orbit is fractured), paraesthesiae
(if nerves are damaged), or limited opening (if muscles are involved).
Limited opening may be as a result of muscle entrapment or muscular
spasm or purely due to localized swelling and discomfort. Fracture of
the zygomatic arch may occur in isolation or, more commonly, in com-
bination with a zygomatic complex fracture. The origin of the tempor-
alis muscle is the temporal line at the superior aspect of the parietal
bone of the skull. From here, it passes medial to the zygomatic arch
and inserts onto the coronoid process of the mandible. If the zygomatic
arch is depressed, the inferior fibres or the tendon of the temporalis
will be impinged at the insertion to the coronoid process, therefore
limiting the normal range of opening of the mandible on that side. Of
the remaining options, only the masseter muscle attaches directly to the
zygoma. However, its origin is the external surface of the zygomatic arch
and, as such, depression would not lead to entrapment. The remaining
options do not attach or pass under the zygomatic arch and therefore
would not be affected.
Keywords: limited opening, zygomatic arch, depression.

4. E ★★★
Children can be born with various kinds of cleft, which are reported
to occur in approximately  in 700. There is a 2: male-​to-​female ratio
when it comes to clefts involving the lip alone, and the ratio is inverted
for cleft palates alone. This may be due to the fact that the palatine
shelves elevate a week earlier in boys than girls.
The face derives from five facial prominences that form during facial
development:
● Two mandibular processes
● Two maxillary processes
● Frontonasal process.

The frontonasal process gives rise to the lateral and medial nasal pro-
cesses, which fuse with the bilateral maxillary prominences to form the
upper lip, nose, and philtrum. The two medial nasal prominences fuse
together at the midline to form the intermaxillary segment. As the maxil-
lary prominence grows inwards, it fuses with the lateral nasal prominence
to form the nasolacrimal groove. Failure of the maxillary prominence to
fuse with the medial nasal process (or intermaxillary segment) results in
a cleft lip, which can be uni-​or bilateral.
Inadequate fusion of the two nasal processes can result in a very rare
median cleft lip. See Figure .8 which illustrates facial development.
Keywords: unilateral cleft lip, failed to fuse, embryological development.

https://t.me/DentalBooksWorld
18 Chapter  Anatomy of the head and neck

5 weeks 6 weeks

Frontonasal
prominence

Nasal pit

Eye

Nasolacrimal
groove

Stomodeum

8 weeks 10 weeks

Eye

Nasolacrimal
groove

Philtrum

Mandibular prominence
Lateral nasal prominence
Medial nasal prominence
Maxillary prominence

Figure .8

https://t.me/DentalBooksWorld
answers 19

5. D ★★★★
Oral epithelial and mesenchymal cells, along with migrating neural crest
cells, are responsible for tooth formation. The interaction between
these cell layers is essential for tooth development. These cell layers
form the dental lamina, and within this lamina, the various stages of
tooth development occur. These stages are named the bud, cap, and
bell. During the bud and cap stages, the epithelial and mesenchymal
cells proliferate to form the enamel organ and the dental papilla,
respectively. The bell stage is defined by cell morphodifferentiation and
histodifferentiation when enamel-​and dentine-​producing cells develop.
Enamel-​forming ameloblasts develop from the cells of the inner enamel
epithelial cells, whilst odontoblasts (dentine-​producing cells) differen-
tiate from ectomesenchymal cells (neural crest cells) that lie adjacent to
the inner enamel epithelium. The resultant crown shape is determined
by the inner enamel epithelium. Ameloblasts begin depositing enamel
after dentine has begun to form, and this site of initial dentinogenesis
and amelogenesis is referred to as the amelodentinal junction (ADJ).
Ameloblasts subsequently pass through several stages, including a secre-
tory and maturation phase. AI is a group of hereditary conditions that
can present with multiple phenotypic variations.
The crown maturation (primary epithelial band) stage involves growth
of the enamel crystals that have been deposited, and this phase accounts
for the majority of the mineral found in enamel. Defects in the crown
maturation stage are likely to present with hypomaturation AI, which
would typically present in the manner described in the scenario. See
Figure .9 which shows the initiation stage of human tooth development.
Keywords: tooth development, amelogenesis.
→ Gadhia K , McDonald S, Arkutu N, Malik K. Amelogenesis imper-
fecta: an introduction. British Dental Journal. 202;22:377–​9.

https://t.me/DentalBooksWorld
20 Chapter  Anatomy of the head and neck

(a)

Transverse section

Longitudinal section

Longitudinal section Tranverse section


Preodontogenesis
Lingual
(b) Oral epithelium
Jaw mesenchyme
Mesial Distal Buccal Tongue

Initiation
(c)
Oral epithelium
Primary epithelial band
Jaw mesenchyme Tongue

(d) Dental lamina


Dental lamina
Vestibular fold
Tongue
Oral
epithelium

Jaw mesenchyme
Mesenchymal condensation

Figure .9
Reproduced from Scully C, Oxford Handbook Applied Dental Sciences, Figure 7., pg ,
Copyright (2003) by permission of Oxford University Press.

https://t.me/DentalBooksWorld
answers 21

6. C ★★★★
The glossopharyngeal nerve provides parasympathetic fibres to the otic
ganglion via the lesser petrosal nerve. Post-​synaptic parasympathetic
fibres then pass to the parotid gland via the auriculotemporal nerve. The
otic ganglion is one of four parasympathetic ganglia of the head, along
with the submandibular, ciliary, and pterygopalatine ganglia. These gan-
glia are the site of synapse for parasympathetic fibres of the autonomic
nervous system (ANS). The ANS can be subdivided into the sympa-
thetic, parasympathetic, and enteric nervous system. Of these subdi-
visions, the sympathetic (fight or flight) and parasympathetic (rest and
digest) divisions are heavily involved in everyday functions such as heart
rate, breathing, salivation, and swallowing. In the head, these ganglia
are primarily secretomotor in function, i.e. they induce secretions from
various glands (with the exception of the ciliary ganglion). Functions of
the parasympathetic ganglia of the head include:
● Pterygopalatine ganglion—​ secretomotor to the lacrimal gland and
mucous glands of the mouth, palate, and nose. Known colloquially as
the ‘hay fever’ ganglion. Supplied by CN VII—​facial
● Otic ganglion—​secretomotor to the parotid gland

● Submandibular ganglion—​ secretomotor to the sublingual and sub-


mandibular glands. Supplied by CN VII—​facial
● Ciliary ganglion—​motor to the constrictor muscle of the eye and the

ciliary body (responsible for adjusting the shape of the lens in the eye
to enable focusing on objects at various distances). Supplied by CN
III—​oculomotor.
All the other answers listed are CNs that carry out important functions
throughout the body, particularly the head and neck.
Keywords: autonomic, cranial nerve, otic ganglion, parasympathetic.

7. E ★★★★
The first action in this case would be to fast-​bleep the stroke registrar!
The signs and symptoms described indicate an upper motor neuron
(UMN) lesion because the forehead has been spared, which is most likely
to have been caused by an intracranial tumour or stroke. Knowledge
of neuroanatomy is fundamental for this question, specifically the facial
motor nucleus. The concept of UMN and lower motor neuron (LMN)
lesions can be complicated to grasp. A UMN is any neuron carrying
information within the central nervous system between the motor areas
and CN nuclei. An LMN is any neuron carrying motor information from
the CN nucleus to the target muscles and is the same as the peripheral
nerve. When put simply, the area of the facial motor nucleus controlling
the upper face receives its nerve supply from both sides of the cere-
bral hemispheres, whilst the part controlling the lower face receives its
supply only from the contralateral hemisphere. Therefore, when a right
UMN lesion occurs, the patient retains some control over the left fore-
head (from the left side of the brain), but loses motor control of the left
lower face. However, when an LMN injury occurs, all neurons leaving
https://t.me/DentalBooksWorld
22 Chapter  Anatomy of the head and neck

Cortex Cortex
(UMN) (UMN)
Facial nucleus
A of pons (LMN)
Upper face
Normal division
Wrinkles Lower face
forehead division
Shuts eye
B
Flares nostrils

Smiles

R L
Figure .0

the facial motor nucleus in the brainstem are affected, and as a result,
all five peripheral branches of the facial nerve on the same side are af-
fected. This results in complete paralysis of all the muscles on the same
side. Therefore, a left LMN lesion would affect the muscles on the left-​
hand side, rendering the left side of the face completely paralysed, e.g.
Bell’s palsy, which is a diagnosis of exclusion when all other causes have
been eliminated. See Figure .0 which shows: label A—​UMNs from
the right side are affected, but innervation from the left UMN remains;
label B—​both LMNs are affected, and therefore, there is no innervation
to the muscles.
Keywords: unilateral facial paralysis, retained use of forehead (‘forehead
sparing’).

8. D ★★★★
The normal innervation of the mandibular teeth is from the incisive
branch of the inferior alveolar nerve (IAN). The teeth on the left side
are supplied by the left IAN, and the teeth on the right side by the right
IAN. In reality, there is often crossover of innervation at the midline
of anatomical structures, and the mandible is no exception. It has been
reported that in 42% of patients, there is crossover of innervation of
the lower mandibular incisors from the contralateral IAN. However,
given that, in this scenario, a buccal infiltration has been given, crossover
supply from the contralateral IAN is unlikely, as the local anaesthetic has

https://t.me/DentalBooksWorld
answers 23

been deposited locally and this should anaesthetize both teeth. Although
the nerve to the mylohyoid is classified as a motor nerve, a number
of studies have identified that mandibular teeth can receive sensory
innervation from sensory fibres travelling within this branch. They are
thought to enter the mandible through the retromental foramina located
on the lingual aspect of the mandible, superior to the genial tubercles.
A lingual infiltration will help to anaesthetize accessory sensory neurons.
The lingual and long buccal nerves may supply accessory nerves to the
third mandibular molar teeth.
The hypoglossal nerve is a purely motor nerve and not known to carry
accessory sensory neurons.
Keywords: LL, buccal infiltration, accessory innervation.
→ Rosella LF, Buffoli B, Labanca M, Rezzani R. A review of the man-
dibular and maxillary nerve supplies and their clinical relevance. Archives
of Oral Biology. 202;57:323–​34.

https://t.me/DentalBooksWorld
https://t.me/DentalBooksWorld
Chapter 2 25

Preventative and
paediatric dentistry
Nicholas Longridge

‘Prevention is better than the cure.’


The child patient can be a challenging and daunting proposition for the
junior dentist and dental student. Whilst children can be anxious, unco-
operative, and unpredictable, they also present an extremely rewarding
opportunity, which, if managed correctly, may go on to influence their
healthcare experiences for the rest of their lives. Excellent behavioural
management of the child patient (and their parents!) is fundamental to a
successful clinical and patient-​reported outcome.
Aside from possible behavioural issues, paediatric patients may pre-
sent with a series of unique clinical presentations that require additional
skills and knowledge above and beyond those required for adult pa-
tients. Differences in the micro-​and macro-​structures of primary and
permanent teeth, coupled with variations in eruption dates, lead to an
evolving mixed dentition that can lead to some difficult diagnostic and
treatment planning scenarios. Furthermore, dental anxiety and the pre-
ponderance for dento-​alveolar trauma in children and young adults may
exacerbate the patient management of an already complex situation.
Prevention is central to paediatric dentistry. However, whilst signifi-
cant progression has occurred in some areas, poor dietary habits and
suboptimal oral hygiene regimes remain significant concerns for the pro-
fession, with large numbers of dental extractions still performed under
general anaesthesia each year.
Key topics include:
● Tooth anatomy and eruption patterns
● Abnormalities of structure and form
● Prevention and management of dental caries, including pulp therapy
● Dental trauma
● Dental extractions and space management
● Behavioural management
● Safeguarding
● Pharmacological management.

https://t.me/DentalBooksWorld
26 Chapter 2 Preventative and paediatric dentistry

QUESTIONS

. A 5-​year-​old boy attends your surgery for a routine examination.


His upper right permanent canine is yet to erupt. His upper left
permanent canine erupted at the appropriate age. At approximately
what age would you expect this tooth to have erupted? (Select one
answer from the options listed below.) ★
A 6–​7.4 years
B 7.5–​8.9 years
C 9–​0.4 years
D 0.5–​.9 years
E 2–​3.4 years

2. A 4-​month-​old girl is brought to the surgery by her mother. She


is concerned that a gap is present between some of her child’s
developing upper teeth. Which teeth would you expect to have erupted
in the maxillary arch of this child? (Select one answer from the options
listed below.) ★
A As and Bs
B As, Bs, and Ds
C As, Bs, Cs, and Ds
D As, Bs, Ds, and Es
E As, Bs, Cs, Ds, and Es

3. The babysitter of a 4-​year-​old boy telephones the practice, as


the child fell down a staircase and has knocked out his upper right
deciduous central incisor. The boy was momentarily unconscious but ap-
pears normal now. Which is the single most appropriate advice to give
to the babysitter over the phone? ★
A Bring the child to the practice to have it re-​implanted by a dentist
B Do not re-​implant, and reassure the babysitter not to worry
C Do not re-​ implant, and take the child to the local Emergency
Department
D Re-​implant the tooth at home, and bring the child to the practice
E Re-​implant the tooth, and take the child to the local Emergency
Department

https://t.me/DentalBooksWorld
Questions 27

4. A 3-​year-​old girl attends, following a recent fall. The crown of the


upper left primary central incisor (ULA) is significantly displaced
labially (protruded). The tooth is non-​mobile. A radiograph confirms the
clinical diagnosis of lateral luxation. Which is the single most appropriate
treatment? ★
A Extract the ULA
B Orthodontic repositioning
C Refer for specialist care
D Reposition the ULA
E Review after 2–​3 weeks

5. A 9-​year-​old girl attends for a single routine restoration with her


father. When asked, the father says that he was not married to the
child’s mother at birth, but that they subsequently married and that he
is named on the birth certificate. Which is the single most appropriate
option for obtaining consent for this procedure? ★
A Father and mother
B Father only
C Mother only
D No consent required
E The patient can consent for themselves

6. A 3-​year-​old girl attends for a routine dental examination, and


multiple white spot lesions are noted around the cervical region of
the upper and lower incisors. She reports frequent intake of sugary car-
bonated beverages (2 L per day). In this scenario, what is the single most
appropriate management to reduce her caries risk? ★
A Brushing the lesion
B Chewing sugar-​free gum
C Prescribing fluoride supplements
D Reducing the frequency of sugar attacks
E Topical fluoride

https://t.me/DentalBooksWorld
28 Chapter 2 Preventative and paediatric dentistry

7. A 0-​year-​old girl has marked plaque-​induced gingivitis. A ‘sim-


plified basic periodontal examination (BPE)’ is performed. This
procedure involves assessment of selective teeth termed ‘index teeth’.
These teeth include all permanent first molars. Which other teeth are
included in the simplified BPE? (Select one answer from the options listed
below.) ★
A Upper left and lower right permanent central incisors
B Upper permanent central incisors and lower permanent central
incisors
C Upper primary incisors and lower primary incisors
D Upper right and lower left lateral incisors
E Upper right and lower left permanent central incisors

8. A subdued 5-​year-​old boy is seen in the practice. He has mul-


tiple injuries on his hands, knees, chin, and back of the neck. The
parents report that the patient recently fell in the park. Injury to which
single region would raise the most suspicion of a non-​accidental injury
(NAI)? ★★
A Behind the ears
B Chin
C Forehead
D Hands
E Knees

9. A 7-​year-​old girl has had a pulpotomy performed on her lower


right second primary molar. Which single coronal restoration is
considered the most appropriate? ★★
A Amalgam
B Compomer
C Composite
D Glass ionomer cement (GIC)
E Preformed metal crown (PMC)

https://t.me/DentalBooksWorld
Questions 29

0. A grossly carious lower right permanent first molar (LR6) is ex-
tracted from a 9-​year-​old boy. The child’s parents are keen to
know how likely the lower permanent second molar (LR7) is to fill the
gap created. Which is the single best predictor that the LR7 will erupt
into a good clinical position? ★★
A Age of the patient
B Atraumatic extraction technique
C Early calcification at the LR7 root bifurcation
D Lack of periapical pathology around the LR6
E Presence of a space maintainer following LR6 extraction

. A 7-​year-​old girl has been referred for extraction of all of


her carious primary molars under general anaesthesia (GA).
Extensive oral hygiene instructions and preventive advice were given to
the child and her parents prior to their GA appointment. Which is the
single most appropriate future management for this child? ★★
A Refer for specialist care due to high caries rate
B Three-​month recall for 4-​yearly topical fluoride application
C Three-​month recall for twice-​yearly topical fluoride application
D Three-​month recall with fluoride application and fissure sealing of
permanent molars
E Three-​month recall with high fluoride toothpaste prescription
(2800 ppm)

2. A 5-​year-​old girl has a mid-​third root fracture of her upper


left permanent central incisor (UL). It is decided to splint the
tooth to reduce the risk of future tooth loss. Which is the single most
appropriate length of time for the tooth to be splinted with a flexible
splint? ★★
A 2 weeks
B 4 weeks
C 8 weeks
D 4 months
E Indefinitely

https://t.me/DentalBooksWorld
30 Chapter 2 Preventative and paediatric dentistry

3. A 3-​year-​old adolescent boy is complaining about crooked


teeth. Incidentally, a unilateral, severely infraoccluded and anky-
losed lower left second primary molar (LLE) is identified. The permanent
successor is not present, and the adjacent teeth are tipping. Which is the
single most appropriate initial management of the LLE? ★★
A Direct composite restoration
B Extraction followed by restoration of the space
C Indirect onlay
D Orthodontic assessment
E Reassure and monitor

4. An 8-​year-​old boy has occlusal caries in his lower right first
permanent molar. He is an irregular attender. Clinically, the
caries is minimal and confined to the distal section of the fissure system.
A bitewing radiograph shows the caries extends into the outer third of
the dentine. Which is the single most appropriate restorative manage-
ment strategy for this tooth? (Select one answer from the options listed
below.) ★★
A Amalgam restoration
B Composite restoration
C Fissure sealant
D Preformed metal crown (PMC)
E Preventive resin restoration

5. An 8-​year-​old boy attends, following a recent fall in the play-


ground. His upper right permanent central incisor (UR) has
been intruded by 5 mm. A radiograph confirms the UR is present with
incomplete root development. Which is the single most appropriate
course of action regarding the UR? ★★★
A Commence immediate root canal treatment
B Extraction
C Orthodontic repositioning
D Review after 2–​3 weeks
E Surgical repositioning

https://t.me/DentalBooksWorld
Questions 31

6. An 8-​year-​old cooperative boy has a routine examination, and


the patient’s mother has brought a 3-​year-​old sibling with no
dental experience to observe their first examination. Which is the single
most appropriate definition of this non-​pharmacological behaviour man-
agement strategy? ★★★
A Behaviour shaping
B Enhanced control
C Live modelling
D Positive reinforcement
E Systematic desensitization

7. An 8-​year-​old boy has a complicated crown fracture of his


upper right permanent central incisor (UR), which happened
the previous morning (>24 hours). The tooth is positive to sensibility
testing. Which is the single most appropriate management? ★★★
A Apexification
B Direct pulp cap
C Extraction
D Pulpotomy
E Root canal treatment

8. A 9-​year-​old boy has deep pitting of the palatal aspect of


both permanent maxillary lateral incisors. A diagnosis of dens
invaginatus is made. The boy reports no symptoms from the teeth,
which respond positively to sensibility testing. Which is the single most
appropriate management? ★★★
A Composite restorations of the upper lateral incisors
B Extract the upper lateral incisors
C Fissure-​seal the upper lateral incisors
D Oral hygiene instruction and monitor
E Root canal treatment on the upper lateral incisors

https://t.me/DentalBooksWorld
32 Chapter 2 Preventative and paediatric dentistry

9. A 9-​year-​old boy has an unrestorable, grossly carious lower


right first permanent molar (LR6) and a heavily restored upper
right first permanent molar (UR6). The remainder of the permanent
dentition is present radiographically, caries-​free and minimally restored.
They have a Class I incisal and molar relationship with well-​aligned
arches. Due to compliance issues, general anaesthesia is required. For
which single treatment should the patient be consented? ★★★★
A Extract the LR6 and all of the remaining first permanent molars (FPMs)
B Extract the LR6 and the contralateral LL6
C Extract the LR6 and the opposing UR6
D Extract the LR6 tooth only
E Extract the UR6 and extirpate the LR6

20. A 9-​year-​old boy attends for an examination. The enamel on all


his primary and secondary teeth appears dull, frosty and pitted.
There is considerable post eruptive breakdown of a number of teeth
and marked sensitivity. Bitewings show numerous carious lesions and
enamel that is less radiopaque than the underlying dentine. Which is the
single most likely diagnosis? ★★★★
A Amelogenesis imperfecta (AI)
B Dentinogenesis imperfecta (DI)
C Fluorosis
D Molar incisor hypoplasia (MIH)
E Taurodontism

2. A 5-​year-​old boy with uncontrolled diabetes has gross caries


in all of his primary molars. He is very anxious and considered
uncooperative. The teeth are unrestorable and asymptomatic. Which
is the single most appropriate treatment strategy to manage his carious
molars? ★★★★
A Await natural exfoliation
B Extraction using general anaesthesia (GA) as a day case
C Extraction using GA as an inpatient
D Extraction using inhalation sedation
E Extraction using local anaesthesia

https://t.me/DentalBooksWorld
Questions 33

22. A 7-​year-​old boy attends with his mother for a routine exam-
ination where he is assessed by a foundation dentist (first year
post-​qualification). The child had multiple primary teeth extracted under
general anaesthesia (GA) 8 months ago. Occlusal caries is diagnosed in
his lower right first permanent molar, which is partially erupted. The boy
was not brought to two subsequent appointments for the restoration.
What is the single most appropriate management strategy the dentist
should take in this scenario? ★★★★
A Contact the child’s general medical practitioner (GMP)
B Contact the local safeguarding nurse
C Discharge from the practice
D Discuss with their foundation trainer
E Refer to social services

https://t.me/DentalBooksWorld
34 Chapter 2 Preventative and paediatric dentistry

ANSWERS
. D ★ OHCD 6th ed. → p. 64
The average age for eruption of the upper permanent canines is
between  and 2 years. However, a range of 0–​2 years has
been identified in epidemiological studies. Variation occurs across the
population, and each case must be assessed based on the general
stage of development and eruption of all permanent teeth. As a very
basic mnemonic for assisting in the assessment of canine develop-
ment, the phrase ‘Big Canines for Big School’ can be of some help.
Knowledge of when the contralateral canine erupted or its degree
of eruption would provide crucial information regarding whether the
missing canine is likely to erupt spontaneously. Six months is con-
sidered a reasonable time period to monitor for contralateral tooth
eruption prior to investigation. However, further information should
be sought regarding:
● Previous extractions
● Previous trauma, particularly to the primary teeth
● Spacing or crowding in the relevant labial and buccal segments
● Whether the canine can be palpated buccally—​ this should be docu-
mented from the age of 9 years.
Radiographs are likely to provide the greatest information regarding
canine location. Parallax periapicals or an orthopantomogram (OPT)
with an upper occlusal would enable positioning of the canine to be de-
termined, whilst CBCT can be indicated where tooth resorption or cyst
formation is suspected. Table 2. shows the chronology of development
of primary dentition.
Keywords: upper permanent canine, appropriate age.

Table 2. Chronology of eruption of permanent dentition


Tooth Maxillary Mandibular
 6.7–​8. 6.0–​6.9
2 7.0–​8.8 6.8–​8.
3 0.0–​2.2 9.2–​.4
4 9.6–​0.9 9.6–​.5
5 0.2–​.4 0.–​2.
6 6.–​6.7 5.9–​6.9
7 .9–​2.8 .2–​2.2
8 7.0–​9.0 7.0–​9.0
Reproduced from Welbury R, et al, Paediatric Dentistry fourth edition, table 2., page 2,
Copyright (202) with permission from Oxford University Press.

https://t.me/DentalBooksWorld
Answers 35

→ Welbury R, Duggal M, Hosey M. Paediatric Dentistry (4th ed.). Oxford


University Press: Oxford; 202.

2. B ★ OHCD 6th ed. → p. 64


It helps to know the eruption dates of all teeth. Knowing which teeth
should be present at any given age will help you to identify the teeth
when charting and will alert you to abnormal eruption patterns which
may have an underlying cause.
In this particular scenario, the mother is likely to be concerned that a
gap exists between the Bs and Ds, and only reassurance is required at
this stage.
Average normal eruption dates of primary teeth are listed in Table 2.2.
The dates in the table are approximate and should be used as a guide-
line. The pattern of eruption may be more significant when assessing the
developing dentition.
Keywords: developing dentition, 4-​month old.
→ Welbury R, Duggal M, Hosey M. Paediatric Dentistry (4th ed.). Oxford
University Press: Oxford; 202.

Table 2.2 Chronology of eruption of primary dentition (months)


Tooth Maxillary Mandibular
A 8–​2 6–​0
B 9–​3 0–​6
C 6–​22 7–​23
D 3–​9 4–​8
E 25–​33 23–​3
Reproduced from Welbury R, et al, Paediatric Dentistry fourth edition, table 2.2, pg 3,
Copyright (202) with permission from Oxford University Press.

3. C ★
The child is only 4 years old and will be in the primary dentition. Primary
teeth must never be re-​implanted, as the apex of the primary tooth may
be pushed into the developing permanent tooth germ, causing damage.
Losing consciousness after trauma may be an indicator of an underlying
brain injury, so it is important to have the patient examined immedi-
ately by a medical professional. Other indicators of neurological damage
include vomiting, memory loss, confusion, and headaches.
Checking for neurological damage would take priority over the patient’s
oral health in most situations.
Reassurance only is incorrect in this scenario. Trauma must always be
followed up with a dentist to examine for oral injuries and to allow

https://t.me/DentalBooksWorld
36 Chapter 2 Preventative and paediatric dentistry

clinical and radiographic assessment, as symptoms or pathology may


have a delayed presentation.
Keywords: 4-​year old, unconscious.
→ Malmgren B, Andreasen JO, Flores MT, et al. International Association
of Dental Traumatology guidelines for the management of traumatic
dental injuries: 3. Injuries in the primary dentition. Dental Traumatology.
202;28:74–​82.

4. A ★ OHCD 6th ed. → p. 98


The history and examination describe a lateral luxation injury. These teeth
are often non-​mobile due to impaction of the root into the alveolar bone
and can be associated with vertical displacement. Dental trauma injuries
are common between  and 4 years of age. Preservation of the integrity
of the permanent successor is a fundamental concept when managing
primary dental trauma. Luxation with labial crown displacement carries
a high risk of damage to the permanent tooth, as the root is displaced
towards the developing tooth germ of the permanent successor, and
consequently, they are best managed with extraction. Severity of damage
to the permanent tooth has been linked with the age of the child at injury.
Radiographs are likely to show the tooth is elongated and would reaffirm
the need for extraction. Mild luxation injuries may be monitored when
damage to the permanent successor is not suspected. Orthodontic re-
positioning is not appropriate for this scenario, but referral to a specialist
may be indicated where compliance is challenging and general anaesthesia
is necessary, which may frequently be the case within this age bracket.
Keywords: lateral luxation, labially, significantly.

5. B ★ OHCD 6th ed. → p. 674


The Children Act 2004 sets out who has parental responsibility for a
child. The information provided in this scenario means that no date of
birth is actually required to interpret parental responsibility. Biological
fathers who are named on the birth certificate for births registered after
 December 2003 (in England) or who have subsequently married the
mother would be able to give legal consent. Therefore, in this scenario,
the father can provide consent for treatment. In addition to this, fathers
holding a court order granting them parental responsibility overrides any
issues relating to birth certificates and marital status. If a mother were to
choose to give her child up for adoption, she would also surrender par-
ental responsibility to the adopting parents. In situations where a patient
under the age of 6 presents for treatment, a clinician can use his or her
professional judgement to establish whether the patient understands the
nature of treatment, along with its benefits and limitations, and whether
they can compare alternative treatments to come to a decision. This is
called Gillick competence. It is unlikely that a child of this age can truly
understand and weigh up the information regarding their dental treat-
ment. Clinicians are advised to exercise caution when determining if a
patient is Gillick-​competent and, if possible, to delay treatment until an
adult with parental responsibility is available.
https://t.me/DentalBooksWorld
Answers 37

Keywords: subsequently married, named on birth certificate, consent.


→ Wheeler R. Gillick or Fraser? A plea for consistency over com-
petence in children: Gillick and Fraser are not interchangeable. BMJ.
2006;332:807.

6. D ★ OHCD 6th ed. → p. 28


Dental caries is a dynamic process. For demineralization to occur, the
following four factors must be present.
● Substrate (fermentable carbohydrate)
● Bacteria
● Time
● Host (tooth).

All of the options above may be involved in caries prevention. Clearly,


in this scenario, no amount of intervention or preventative advice will
reduce the caries risk, unless the frequency of sugar intake is reduced.
Each time food or drink (with sugars in) is consumed, substrate will be
available to bacteria in dental plaque. The result is a drop in acidity of
the local environment at the tooth surface, leading to demineralization.
Eventually, saliva buffers the acid formed by fermented carbohydrates
and tips the equilibrium in favour of remineralization. The more fre-
quent the intake of sugar, the greater the length of time that deminer-
alization will occur throughout the day. Therefore, in this scenario, it is
important to reduce the intake of sugar. Fluoride contained within oral
saliva is especially effective at remineralizing enamel when it comes in
contact with the enamel–​plaque interface. Constant supply of low-​level
fluoride in liquid form (i.e. topical fluoride) is most effective at preventing
caries. Brushing removes plaque and thus helps prevent further demin-
eralization, thus tipping the dynamic process in favour of remineraliza-
tion. Systemic fluoride is less effective in caries prevention than topical
fluoride, and it does not remineralize softened enamel. Systemic fluoride
used during tooth formation can help with caries prevention by strength-
ening the tooth structure and forming shallower fissures, which are less
susceptible to caries. Chewing gum will stimulate saliva after consump-
tion of food or drink.
Keywords: white spot lesions, frequent, sugary.
→ Welbury R, Duggal M, Hosey M. Paediatric Dentistry (4th ed.). Oxford
University Press: Oxford; 202.

7. E ★
The simplified BPE uses a WHO 62 probe to examine the index teeth
of children aged 7–​7. The index teeth are all permanent first molars
and the upper right and lower left permanent central incisors. It has been
recommended that between the ages of 7 and , BPE scores—​0, , and
2—​are used to assess gingivae for bleeding, calculus, and plaque reten-
tive factors. Prior to 2 years of age, periodontal false pocketing can be
evident, particularly around partially erupted teeth. From 2 to 7 years
of age, the full range of BPE scores—​0, , 2, 3, 4, and *—​should be used
https://t.me/DentalBooksWorld
38 Chapter 2 Preventative and paediatric dentistry

on index teeth. Assessment of all teeth is considered excessive, as true


periodontal disease would be unlikely to present without involving the
index teeth described above. Just as in adult patients, further investiga-
tions would be required if pocketing is identified, which would include
radiographs and more detailed periodontal charting. Specialist referral
may be required in these cases.
Keywords: simplified BPE, index teeth.
→ Clerehugh V, Kindelan S. Executive summary—​guidelines for periodontal
screening and management of children and adolescents under 8 years of
age. British Society of Periodontology and British Society of Paediatric
Dentistry. 202. Available at: http://​www.bsperio.org.uk

8. A ★★ OHCD 6th ed. → p. 96


It is unusual for anybody to hurt themselves behind the ears or the neck.
Accidental injuries usually occur on areas of the body that are exposed
(hands) or areas which protrude (chin, nose). Therefore, trips and falls
can easily traumatize a child’s chin, hands, or knees.
Other suspicious injuries include torn frena in non-​ambulatory children,
bruises of different ages (i.e. various shades and colours), and bruises
around the thighs and buttocks.
Assessment of the child’s general appearance, including cleanliness,
and their social interactions with family and the dental team will pro-
vide further information regarding the likelihood of neglect or an NAI.
Discussing the injuries with the child’s parents or the child themselves
may elicit important information regarding the mechanism of injury,
which may corroborate the clinical findings.
Escalation and discussion with a more senior clinician, a practice safe-
guarding lead, or a local safeguarding officer/​nurse would be appropriate
as a first-​line measure. It is not advisable to manage these situations on
your own or without additional support. You should always be aware of
your local child protection guidelines.
Keywords: subdued, back of the neck, fell in the park.
→ Department for Education, Home Office, Department of Health and
Social Care, et al. Safeguarding children. Available at: https://​www.gov.
uk/​topic/​schools-​colleges-​childrens-​services/​safeguarding-​children

9. E ★★ OHCD 6th ed. → p. 86


PMCs or stainless steel crowns (SSCs) are the restoration of choice in
teeth that have been pulp-​treated, either with pulpotomy or pulpectomy.
Asymptomatic carious primary molars with marginal wall breakdown
or caries involving two surfaces are also recommended for restoration
with a PMC. A systematic review of five randomized controlled trials
concluded that teeth restored with PMCs were less likely to develop
problems such as pain and abscess formation, as well as being less
uncomfortable to the patients during treatment. Plastic restorations with
amalgam, composite, compomer, and GIC generally record a poorer

https://t.me/DentalBooksWorld
Answers 39

performance by comparison to PMCs, which show lower failure rates


in many studies.
Keywords: pulpotomy, primary molar, coronal restoration.
→ Innes NP, Ricketts D, Chong LY, et al. Preformed crowns for de-
cayed primary molar teeth. Cochrane Database of Systematic Reviews.
205;2:CD00552.

0. C ★★
Unplanned early or late loss of a lower first molar can create numerous
occlusal problems and exacerbate malocclusions. The timing of the
extraction is therefore very important. The best predictor for this is
when there is radiographic evidence showing calcification of dentine at
the root bifurcation of the lower second molar. This is thought to occur
between 8 and 0 years. Analysis of the developing dentition and ortho-
dontic consultation should be sought, if possible. However, removal of
pain and infection should take priority, if present. Early loss of a lower
first permanent molar is associated with:
● Distal drifting, tipping, and rotation of the second premolar
● Potential for premolar spacing.

Late loss of a lower first permanent molar is associated with:


● Mesial tipping and mesio-​ lingual rotation of the lower second molar.
This results in poor occlusal contacts and buccal segment spacing
● The second premolar can drift distally.

Orthodontic consultation is required, as skeletal classification and


crowding can impact upon the designated treatment plan. See Figure 2.
which shows severe tipping of the lower secondary and over-​eruption of

Figure 2.
Reproduced from Welbury R, et al, Paediatric Dentistry fourth edition, Figure 4.3b,
page 286, Copyright (202) by permission of Oxford University Press.

https://t.me/DentalBooksWorld
40 Chapter 2 Preventative and paediatric dentistry

Figure 2.2
Reproduced from Welbury R, et al, Paediatric Dentistry fourth edition, Figure 4.4b,
page 286, Copyright (202) by permission of Oxford University Press.

the upper first molar. Figure 2.2 prevents over-​eruption of the opposing
first molar and reduces mesial tilting of the lower second molar.
Keywords: lower permanent second molar, erupt, predictor.
→ Cobourne M, Williams A, Harrison M. A guideline for the extraction of
first permanent molars in children. Royal College of Surgeons of England.
204. Available at: https://​www.rcseng.ac.uk/​dental-​faculties/​fds/​
publications-​guidelines/​clinical-​guidelines

. D ★★
This child has a high caries rate and should be managed accordingly.
An evidence-​based toolkit has been devised by the United Kingdom’s
Department of Health to assist in the management of oral health in chil-
dren and adults. The third edition of this toolkit was released in 204.
Children aged 7 years and older who are causing concern should be placed
on a 3-​month recall. They should have fluoride varnish applied twice or
more times a year, and all permanent molars fissure-​sealed using a resin
sealant, ideally within the first year post-​eruption. A fluoride mouth rinse
may also be considered from the age of 8 years. Diet advice and oral
hygiene instructions are fundamental in managing these cases and should
be reinforced at every opportunity. A significant proportion of this educa-
tion and advice needs to be directed towards the child’s parents or carers.
Keywords: caries, preventive, future management.
→ Public Health England. Delivering better oral health: an evidence-​
based toolkit for prevention. 204. Available at: https://​ www.gov.
uk/​ g overnment/ ​ p ublications/​ d elivering-​ b etter-​ o ral-​ h ealth-​ a n-
​evidence-​based-​toolkit-​for-​prevention

https://t.me/DentalBooksWorld
Answers 41

2. B ★★
When trauma causes a tooth to become mobile, splinting can pro-
vide stability to allow periodontal healing and comfort for the patient.
Splinting for too long can result in the tooth becoming ankylosed, and
not splinting for long enough may result in less favourable healing out-
comes. Generally, the type of healing is classified as: calcific healing
(dentine and cementum), fibrous healing (connective tissue), osseous
healing (bone and connective tissue), and non-​healing. The chances of
calcific healing are likely improved with good apposition of the fractured
segments—​potentially more so than the length of time for which the
tooth is splinted. Patient advice should include a soft diet for a week,
good oral hygiene, and short-​term use of chlorhexidine mouthwash for
optimal prognosis. Follow-​ups are recommended at 4 weeks, 8 weeks,
4 months, 6 months,  year, and 5 years.
Keywords: mid-​third root fracture, splint.
→ DiAngelis AJ, Andreasen JO, Ebeleseder KA, et al. International
Association of Dental Traumatology guidelines for the management of
traumatic dental injuries: . Fractures and luxations of permanent teeth.
Dental Traumatology. 202;28:2–​2.

3. D ★★
Treatment depends upon multiple factors, including the severity and
rate of infraocclusion and the patient’s age. However, an orthodontic
assessment is nearly always indicated to enable the correct long-​term
treatment plan to be decided, particularly when there is concomitant
malocclusion. Early diagnosis of infraoccluded teeth can help to reduce
potential complications. Delayed treatment can result in: tipping of
adjacent teeth, loss of arch space, over-​eruption of opposing teeth,
insufficient development of alveolar width or height, increased risk of
developing caries, and localized periodontal attachment loss. As a per-
manent successor is not present, orthodontics could help to close the
space, without the need for restorative treatment, or to maintain the
space to facilitate restorative treatment in the future. Reassurance and
monitoring of an infraoccluded tooth are only advised in the short term
if the tooth is not ankylosed, the successor is present, the tooth is not
having a detrimental effect on the patient’s oral health, and the degree
of infraocclusion is slight. This is usually more relevant in late-​presenting
adult patients. Composite build-​ups and indirect onlays can be used in
the medium term to maintain occlusal stability in cases of slight or mod-
erate infraocclusion. In this scenario, extraction will be likely due to the
severity and the patient’s age, but an orthodontic opinion is needed
first. Definitive restoration of the resulting space would typically be de-
layed until dento-​alveolar development is complete, or it may not be
necessary.
Keywords: 3-​year old, successor not present, crooked teeth, severely
infraoccluded.

https://t.me/DentalBooksWorld
42 Chapter 2 Preventative and paediatric dentistry

→ Arhakis A, Boutiou E. Etiology, diagnosis, consequences and treatment


of infraoccluded primary molars. Open Dentistry Journal. 206;0:74–​9.

4. E ★★
Minimal occlusal caries with evidence of dentine involvement requires
exploration and removal. Preventive resin restoration enables caries
to be identified and removed, whilst remaining minimally invasive.
Furthermore, fissure sealing of the remaining fissure pattern will protect
the tooth from further occlusal caries in the pits and fissures. Amalgam
is a suitable restorative material, but it is more destructive due to cavity
design requirements for retention and the need for a minimum thickness
of approximately 2 mm. Implementation of the Minamata Convention
recommendations is also decreasing amalgam usage. From  July 208,
in England, regulations state that amalgam should not be used in children
under 5 years of age and pregnant or breastfeeding women (unless
deemed strictly necessary by the clinician). A conventional composite
restoration may be required if the caries extends over a significant pro-
portion of the occlusal surface or through the marginal ridge. Dentists
using truly minimally invasive procedures have presented evidence for
sealing caries with resin fissure sealants alone. Theoretically, sealing the
carious lesion blocks substrate from reaching the bacteria. However,
caution is required during placement of the sealant, with careful long-​
term maintenance. Fissure sealants alone in irregular attendees would
not be advised.
Keywords: minimal caries, outer third of dentine.
→ Welbury R, Duggal M, Hosey M. Paediatric Dentistry (4th ed.). Oxford
University Press, Oxford; 202.

5. D ★★★ OHCD 6th ed. → p. 04.


In this case, the UR is considered to be moderately intruded and
importantly, root development is incomplete.
Severity of intrusion:
● Mild <3 mm
● Moderate 3–​6 mm
● Severe >6 mm.

Initial management for permanent incisors that have incomplete root


development is to allow for passive repositioning for 2–​ 3 weeks.
However, each clinical situation differs and some severe intrusions can
be managed surgically.
Active repositioning, including surgical and orthodontic repositioning,
should be considered if no movement is evident within the first 2–​3
weeks or if passive eruption ceases and the tooth remains infraoccluded.
Teeth with complete root development that are mildly intruded can
be monitored for passive repositioning for 2–​3 weeks prior to active
repositioning. Moderate and severe intrusions are best managed with
either orthodontic or surgical repositioning and may be best referred

https://t.me/DentalBooksWorld
Answers 43

for specialist care. Appropriate special investigations and trauma charts


should be carried out to monitor the vitality of the intruded tooth and its
adjacent teeth. It is likely that root canal treatment will be required, espe-
cially in teeth with complete root development, and regular radiographic
analysis should occur to confirm no inflammatory resorptive changes.
Keywords: permanent tooth, incomplete root development, 5 mm.
→ Albadri S, Zaitoun H, Kinirons MJ; British Society of Paediatric
Dentistry. UK National Clinical Guidelines in Paediatric Dentistry: treat-
ment of traumatically intruded permanent incisor teeth in children.
International Journal of Paediatric Dentistry. 200; 20(Suppl ):–​2.

6. C ★★★ OHCD 6th ed. → p. 60


Modelling is a branch of observational psychology that involves a behav-
iour being learnt through imitation. Specifically to dentistry, the young
or inexperienced child observes a non-​fearful child, or ‘model’, having
a positive experience at the dentist. This approach is thought to work
best if the patient can observe a ‘live’ model with whom they can
closely relate, i.e. they are a relative or friend of similar age. In reality,
the model is often an older sibling, parent, or family member. Behaviour
shaping utilizes selective reinforcement, in which positive reinforcement
is provided for behaviours that are closer to the desired behaviour, e.g.
praising a child for sitting nicely in the chair, even though they will not
open their mouth. Enhanced control is designed to empower the child,
so they feel in control of the situation. A stop signal or a countdown
are useful examples of this. Systematic desensitization is one approach
to achieving a desired goal in phobic patients. A hierarchy of steps are
outlined, with each step getting progressively closer to the ultimate de-
sired goal. Positive reinforcement is a branch of operant conditioning
whereby a behaviour is strengthened, based on its association with a
positive stimulus.
Keywords: sibling, observe, behaviour management.

7. D ★★★
At the age of 8 years, the UR would be expected to have incomplete
root development, as completion occurs typically 2–​3 years after erup-
tion. Although the coronal pulp has been contaminated, inflammation
is usually fairly localized due to the cellular structure of the pulp. If the
inflamed/​infected pulp tissue is removed, there is potential for healing to
occur and pulp vitality to be retained. The resulting benefits include con-
tinued root development and maintenance of normal repair, defence,
and sensory functions. The clinical technique for this is called a pulpotomy
(complete or partial). The tooth should be kept under review, as in the
long term, there is a risk of losing vitality. Where the apex is immature,
the potential for healing is considered to be better. A direct pulp cap
could be attempted if pulp exposure is not so severe (< mm and <24
hours). Extraction would not be an ideal first-​line treatment in this case
as the tooth is still restorable, and root canal treatment is inappropriate

https://t.me/DentalBooksWorld
44 Chapter 2 Preventative and paediatric dentistry

as the tooth is still vital. Apexification is useful when a tooth with incom-
plete root formation has lost vitality. Non-​setting calcium hydroxide can
be used to induce a calcific barrier at the apex before root canal treat-
ment can be performed, but more commonly a bioceramic barrier tech-
nique is now employed.
Keywords: 8-​year old, complicated crown fracture, >24 hours, positive
sensibility testing.
→ Cvek M. A clinical report on partial pulpotomy and capping with cal-
cium hydroxide in permanent incisors with complicated crown fracture.
Journal of Endodontics. 978;4:232–​7.

8. C ★★★ OHCD 6th ed. → p. 72


This developmental defect is caused by invagination of the enamel epi-
thelium into the dental papilla during tooth formation. It is most com-
monly associated with the upper lateral incisors and sometimes the
premolars. In this instance, the patient is young and has no symptoms,
so a more conservative approach would be suitable. Good-​quality fis-
sure sealants would reduce the chances of developing caries without
sacrificing tooth tissue. Flowable composite may be considered a suitable
alternative. However, fissure sealants are considered a more suitable
first-​line management strategy (many of which are resin-​based anyway).
Depending on severity, teeth with dens invaginatus are highly suscep-
tible to caries, so simple oral hygiene instructions are unlikely to prevent
decay. As the patient is asymptomatic, with positive sensibility tests and
no evidence of active caries, more complex restorative treatment is not
necessary at this stage. Teeth with dens invaginatus can be challenging to
perform root canal treatment on, with CBCT frequently used to diag-
nose and help plan treatment.
Keywords: palatal aspect of upper lateral incisors, no symptoms.
→ Bishop K, Alani A. Dens invaginatus. Part 2: Clinical, radiographic
features and management options. International Endodontic Journal.
2008;4:37–​54.

9. C ★★★★
An unrestorable molar tooth will require extraction. Therefore, root
canal treatment is not appropriate. When extracting FPMs, there is a risk
of undesirable tooth movement of adjacent teeth, which may provide
unfavourable over-​eruption or changes to tooth alignment. This should
be taken into account when planning treatment for the patient. Due to
the risks involved with a general anaesthetic, treatment plans are fre-
quently more radical in order to try and prevent repeat operations. It is
suggested that, with FPMs, when a lower molar is removed, removal of
the opposing upper FPM should be given close consideration to prevent
over-​eruption (i.e. a compensating extraction). Where a local anaes-
thetic can be used, these extractions are hard to justify and removal is
indicated only if there is unfavourable movement at review. Balancing

https://t.me/DentalBooksWorld
Answers 45

extractions (removal of the contralateral tooth) is unwarranted, as there


is no evidence to suggest removal of FPMs affects the midline. In the case
of severe crowding or complex malocclusions, orthodontic input is sug-
gested. The evidence supporting this is fairly weak, and each case should
be assessed individually.
Keywords: 9-​year old, unrestorable lower first permanent molar, general
anaesthetic.
→ Cobourne M, Williams A, Harrison M. A guideline for the extraction of
first permanent molars in children. Royal College of Surgeons of England.
204. Available at: https://​www.rcseng.ac.uk/​dental-​faculties/​fds/​
publications-​guidelines/​clinical-​guidelines

20. A ★★★★ OHCD 6th ed. → p. 68


AI describes a number of genetically controlled conditions that affect
the structure of enamel. The conditions are hereditary but can occur
following spontaneous genetic mutations and can present in numerous
different forms. Enamel derived from epithelial tissue in the developing
tooth germ is produced by ameloblasts. The timing of the defect with
regard to enamel development is fundamental to the type of enamel
formed, and ultimately the defect observed. The process is intimately
linked with genetics and can be classified based on the mode of inherit-
ance. However, the type of defect is often used to classify the condition.
● Hypoplastic—​ enamel is often thin or translucent. Radiographically,
normal enamel–​dentine contrast is observed.
● Hypocalcified—​ this results from inappropriate calcification and, as
such, the enamel is of normal thickness but is weak or chalky. This
structural change results in enamel that appears more radiolucent
than the underlying dentine.
● Hypomaturation—​ this results from inappropriate resorption of the
enamel matrix. The enamel is stronger, by comparison to hypocalcified
AI, but is generally softer than normal and is prone to wear. Mottling
can occur, and similar enamel–​dentine radiodensity can be observed.
Keywords: enamel, all, teeth, frosty, pitted, less radio-​opaque.
→ Gadhia K, McDonald S, Arkutu N, Malik K. Amelogenesis imper-
fecta: an introduction. British Dental Journal. 202;22:377–​9.

2. C ★★★★
GA should be reserved as a last resort; however, in this case, the main
concerns are the patient’s medical status and level of anxiety; for this
reason, a GA referral would be appropriate. Poorly controlled diabetes
would warrant extra precaution during the GA procedure; for this
reason, it is often best to keep the patient in overnight. However, this
decision is made at the discretion of the treating anaesthetist, and in
some cases, this may be managed as day-​case GA, providing inpatient
facilities are available, should they be required.

https://t.me/DentalBooksWorld
46 Chapter 2 Preventative and paediatric dentistry

Good cooperation is needed from patients during inhalation sedation, as


the patient is conscious during the procedure. This may be difficult for a
patient of such young age.
Performing extractions using local anaesthesia in multiple quadrants in
anxious patients may be difficult, as they may not wish to return after
the first appointment.
Keywords: uncontrolled diabetes, very anxious, unrestorable.
→ Adewale L, Morton N, Blayney M. Guidelines for the management
of children referred for dental extractions under general anaesthesia.
Association of Paediatric Anaesthetists of Great Britain and Ireland. 20.
Available at: https://​www.rcoa.ac.uk/​document-​store/​guidelines-​the-​
management-​of-​children-​referred-​dental-​extractions-​under-​general

22. D ★★★★ OHCD 6th ed. → p. 96


The scenario describes a potential case of dental neglect. At 7 years
of age, the child will rely on parents to attend appointments and, as
such, the term ‘was not brought’ is preferred over ‘did not attend’. As
a junior clinician, the foundation trainer should be the first port of call
to discuss the case. It is more than likely that the foundation trainer will
have experience in handling such cases, with pragmatic advice to assist
in handling the issue.
It is important to adhere to your local safeguarding policy and any guid-
ance provided by your local safeguarding children board. However, re-
peated failure to comply with reasonable appointment requests would
be a cause for concern, and your practice or local safeguarding nurse
would be an appropriate second step. This may require follow-​up with
the patient’s GMP or local Social Services Department, as the family may
be known to social services and may already have a designated support
worker assisting the family.
Keywords: foundation dentist, multiple extractions, GA, caries, not
brought, two, appointments.
→ Harris J, Sidebotham P, Welbury R. Child protection and the dental team:
an introduction to safeguarding children in dental practice. Committee of
Postgraduate Dental Deans and Directors (COPDEND), Sheffield; 2006.

https://t.me/DentalBooksWorld
Chapter 3 47

Orthodontics
Nadia Ahmed

‘Straightening things out.’


Orthodontics is ‘the specialty of dentistry concerned with growth of the
face, the development of dentition, and the prevention and correction
of occlusal anomalies. A malocclusion can be defined as ‘a deviation from
the ideal that may be aesthetically or functionally unsatisfactory, with a
wide range of occlusal traits’.
Orthodontics is a constantly evolving specialty, with ever changing
principles and techniques continuing to be developed. There has been
huge progress in orthodontics in recent times, with changes in the types
of brackets, archwire materials, and appliance systems (such as tem-
porary anchorage devices and aligner technology).
The key principles of orthodontics date back to 899 when Edward
Angle described ‘the key to a normal occlusion as the anteropos-
terior relationship between the upper and lower first molars’. In 972,
Lawrence Andrews described ‘six keys to an ideal static occlusion’. This
was the basis of early orthodontic treatment planning.
Knowledge of craniofacial development and growth is required as a
foundation for understanding the aetiology of a patient’s malocclusion,
to reach a diagnosis, and to plan orthodontic treatment. A basic under-
standing of the types of orthodontic appliances is beneficial (mainly fixed
appliances, functional appliances, some use of removable appliances, and
retainers).
In addition to the management of a malocclusion, orthodontic treat-
ment is often required in conjunction with other specialties, including
oral and maxillofacial surgery, paediatric and restorative dentistry
Key topics discussed in this chapter include:
● Fixed appliances
● Functional appliances
● Removable appliances
● Retention
● Index of treatment need
● Orthodontic assessment and diagnosis
● Cephalometric analysis
● Malocclusion
● Ectopic canines
● Dental anomalies.

https://t.me/DentalBooksWorld
48 Chapter 3 Orthodontics

QUESTIONS

. A 2-​year-​old boy attends for a check-​up. He has the following


features in his malocclusion:
Class II division  incisors
Overjet of 6 mm
Incompetent lips
Contact point displacement of 4.5 mm between LR3 and LR4
Contact point displacement of 3 mm between UL and UL2
Overbite of 70% complete to tooth.
What is his index of orthodontic treatment need (IOTN)? (Select one
answer from the options listed below.) ★
A 3a
B 3d
C 3f
D 4a
E 4d

2. A 9-​year-​old girl attends for a review appointment. Six months ago,


her upper left permanent central incisor had not erupted, and it is
still not present today. A radiograph reveals the presence of a super-
numerary overlying the unerupted tooth. She presents with the following
clinical features:
● Class II division  incisor relationship
● Overjet of 5 mm
● Increased overbite
● Lingual displacement of the partially erupted lower right lateral
incisor (LR2)
● Upper left first permanent molar (UL6) is in crossbite
Which single feature of the child’s dentition would lead you to score a
grade 5 in her IOTN? ★
A A midline supernumerary tooth
B An increased overbite
C An overjet of 5 mm
D Severe crowding of 4 mm associated with the LR2
E UL6 is in crossbite

https://t.me/DentalBooksWorld
Questions 49

3. A 2-​year-​old female, who is undergoing fixed orthodontic treat-


ment, attends for a routine examination. Her plaque control is par-
ticularly poor. Which single most important risk would encourage you to
contact the orthodontist to discuss discontinuing treatment? ★
A Decalcification
B Loss of vitality
C Pericoronitis
D Root resorption
E Tooth mobility

4. A 2-​year-​old girl attends for assessment. Her upper right central


incisor overlaps the coronal one-​third of the lower incisor crown
height, and there is contact between the lower incisal edges and the pal-
atal surface of the upper incisors. Which single option best describes the
patient’s overbite in this scenario? ★
A Increased and complete
B Increased and incomplete
C Normal and complete
D Reduced and complete
E Reduced and incomplete

5. An -​year-​old girl with a posterior crossbite is undergoing ortho-


dontic treatment with a removable appliance. Which single tooth
movement would a removable appliance not be able to achieve? ★★
A Bodily movement
B Canine retraction
C Reduction of an increased overbite
D Space maintenance
E Tipping of teeth

6. A 0-​year-​old boy is undergoing the initial stages of his orthodontic


treatment. He has a crossbite and has been provided with a remov-
able appliance in order to expand the maxillary arch. Which single com-
ponent should be adjusted to achieve the desired movement? ★★
A Acrylic base plate
B Adam’s clasps
C Midline expansion screw
D Offset southend clasp
E Posterior bite plane

https://t.me/DentalBooksWorld
50 Chapter 3 Orthodontics

7. A 7-​year-​old woman attends for a routine check-​up with her gen-


eral dental practitioner, having been discharged from her ortho-
dontist following comprehensive orthodontic treatment. She reports
that she has lost the instructions for wearing her vacuum-​formed re-
tainers. What is the single most appropriate piece of advice to give in
this scenario? ★★
A Wear the retainers full-​time
B Wear the retainers only during the day and remove for eating/​
drinking
C Wear the retainers for 2–​3 hours per day
D Wear the retainers only at night
E Wear the retainers only during the day and keep them in if eating/​
drinking

8. A 9-​year-​old boy has a Class II skeletal discrepancy and would


benefit from a functional appliance. What would be the most ideal
age range at which to begin his treatment in order to utilize his pubertal
growth spurt? (Select one answer from the options listed below.) ★★
A 0–​ years
B 2–​3 years
C 4–​5 years
D 6–​7 years
E Over 7 years

9. A 5-​year-​old boy with a palatally impacted canine is having a closed


exposure to enable alignment of the tooth, using a gold chain and
fixed appliances. The tooth requires significant three-​dimensional tooth
movement. Which single type of tooth movement requires the highest
force? ★★
A Bodily movement
B Extrusion
C Intrusion
D Rotation
E Tipping

https://t.me/DentalBooksWorld
Questions 51

0. A 2-​ year-​


old boy has an orthodontic assessment and his
incisor relationship is to be classified. His lower incisor edges
are posterior to the cingulum plateau of his upper incisors. The overjet
is 2 mm, and the upper incisors are retroclined. Which is the single most
appropriate British Standards Institute Incisor classification? ★★
A Class I
B Class II
C Class II division 
D Class II division 2
E Class III

. A 3-​ year-​


old boy attends the Orthodontic Department.
His upper right permanent canine erupted six months ago.
However, the upper left permanent canine is still unerupted and not
palpable buccally. What would be the most appropriate radiographic
investigation to assess the developing dentition and locate the upper
left permanent canine? (Select one answer from the options listed
below.) ★★★
A A cone beam computed tomogram (CBCT)
B A lateral cephalogram and an orthopantomogram (OPT)
C A periapical (PA) and an upper standard occlusal (USO)
D An OPT and a PA
E An OPT and a USO

2. A 4-​year-​old girl has had a cephalometric tracing performed


as part of her orthodontic assessment. Using this tracing, how
would her skeletal relationship be identified? (Select one answer from
the options listed below.) ★★★
A Measure the angle formed by the A point, glabella, and B point
B Measure the angle formed by the A point, nasion, and B point
C Measure the angle formed by the A point, porion, and B point
D Measure the angle formed by the A point, sella turcica, and nasion
E Measure the angle formed by the anterior nasal spine, nasion, and
sella turcica

https://t.me/DentalBooksWorld
52 Chapter 3 Orthodontics

3. An 9-​year-​old girl has been referred due to rotation and dis-
placement of her upper left central incisor. A periapical radio-
graph of the central incisors reveals an unusual radio-​opacity in the
midline. What is the single most likely cause of the presenting complaint
in this scenario? ★★★
A Complex odontome
B Compound odontome
C Conical supernumerary
D Supplemental supernumerary
E Tuberculate supernumerary

4. A 2-​year-​old boy attends your practice. The clinical features of


his malocclusion include:
● A 0-​mm overjet
● Class II division  incisor relationship
● A moderate Class II skeletal base

● Class II buccal segments

● Proclination of the upper incisors

● Well-​aligned upper and lower arches.

You decide to refer him to an orthodontist. What is the most likely man-
agement for his malocclusion? (Select one answer from the options listed
below.) ★★★★
A Delay treatment until facial growth has completed, then provide
orthognathic surgery
B Extraction of one unit in all four quadrants and fixed appliances
C Extraction of two units in the maxilla and non-​extraction in the man-
dible prior to fixed appliances
D Headgear retraction of upper buccal segments
E Provision of a twin block functional appliance

https://t.me/DentalBooksWorld
Questions 53

5. A 3-​year-​old girl has an increased, complete overbite that is


traumatizing the palatal mucosa. The intraoral orthodontic as-
sessment shows the patient is in the permanent dentition and has an
overjet of 8 mm, upper incisors of average inclination, mild upper and
lower arch crowding, and an increased curve of Spee in the lower arch.
Which single type of appliance is most appropriate in managing the first
stage of orthodontic treatment? ★★★★
A Lower removable appliance with a posterior bite plane
B Upper removable appliance with a flat anterior bite plane
C Upper removable appliance with a labial bow
D Upper removable appliance with a Robert’s retractor
E Upper removable appliance with Z springs on the upper incisors

6. A local dental practice is reviewing its storage requirements and


looking to dispose of old study models. The practice manager
has identified study models taken 4 years ago for a patient who received
orthodontic treatment at the age of  years, who is no longer seen at
the practice. What single piece of advice should be given to manage
these models? ★★★★
A Dispose of the models as clinical waste
B Dispose of the models as hazardous waste
C Securely send the models to her new dentist
D Store the models for a further 0 years
E Store the models for a further 4 years

https://t.me/DentalBooksWorld
54 Chapter 3 Orthodontics

ANSWERS
. E ★ OHCD 6th ed. → p. 26
The index of orthodontic treatment need (IOTN) comprises a dental
health component and an aesthetic component. The acronym MOCDO
(Missing teeth; Overjet; Crossbite; Displacement of contact points;
Overbite) can be used to help prioritize the single worst feature of a
malocclusion, in order to score the dental health component. The order
of the acronym is hierarchical. A score of 4d indicates severe displace-
ment of teeth of >4 mm.
● 3a—​increased overjet 3.6–​6mm with incompetent lips
● 3d—​displacement of teeth 2.–​4mm
● 3f—​increased and complete overbite without gingival trauma
● 4a—​increased overjet 6.–​9mm.

Keywords: IOTN, displacement of 4.5 mm.

2. A ★ OHCD 6th ed. → p. 26


The dental health component of the IOTN was developed from an index
used by the Dental Health Board in Sweden to reflect these occlusal
traits. Grade 5 indicates a very great need for treatment:
● 5a—​increased overjet greater than 9 mm
● 5h—​ extensive hypodontia with restorative implications (> tooth
missing in any quadrant), requiring pre-​restorative orthodontics
● 5i—​impeded eruption of teeth (except third molars) due to crowding,

displacement, the presence of supernumerary teeth, retained


deciduous teeth, and any pathological cause
● 5m—​reverse overjet greater than 3.5 mm, with reported masticatory

and speech difficulties


● 5p—​defects of cleft lip and palate and other craniofacial anomalies
● 5s—​submerged deciduous teeth.

The clinical scenario describes a 9-​year-​old child with delayed eruption


of a permanent central incisor. With the exception of the midline super-
numerary tooth impeding eruption, none of the other options would
score grade 5. Moreover, this patient is very young and not all of her
permanent dentition will have erupted. Therefore, an accurate assess-
ment of her IOTN is difficult, but a midline supernumerary tooth would
be picked up at around this time frame.
Delayed eruption of a permanent maxillary incisor tooth can be con-
sidered in the following circumstances:
● Eruption of the contralateral incisor occurred >6 months previously

● The maxillary incisors remain unerupted > year after the eruption of

mandibular incisors
● There is a significant deviation from the normal eruption sequence, i.e.

lateral incisors erupting prior to the central incisor.


Keywords: feature, IOTN, dental health component, grade 5.
https://t.me/DentalBooksWorld
answers 55

→ Yaqoob O, O’Neill J, Patel S, et al. Management of unerupted maxillary


incisors. Royal College of Surgeons (England). 206. Available at: https://​
www.rcseng.ac.uk/ ​ d ental- ​ f aculties/ ​ f ds/ ​ p ublications- ​ g uidelines/
​clinical-​guidelines/​

3. A ★ OHCD 6th ed. → p. 32


Decalcification occurs when cariogenic plaque is present with a high-​
sugar diet. A fixed appliance predisposes to plaque accumulation, as
cleaning around the fixed appliance can be difficult. Demineralization of
the enamel can occur if patients do not follow dietary advice and do not
brush their fixed appliances adequately, leaving a white outline around
the brackets. Some degree of tooth mobility during treatment is normal.
Other common risks discussed include relapse, root resorption and gin-
gival recession. Rarely, pulpal necrosis might occur if rapid and excessive
force is applied to the teeth that are already compromised.
Keywords: poor oral hygiene.

4. C ★ OHCD 6th ed. → p. 46


Overbite is described as the vertical overlap of the lower incisors by the
upper incisors, along with a description of the contact that the lower
incisor edge makes with the upper incisors or palate. The normal ver-
tical overlap is described as the upper incisor teeth covering one-​third to
one-​half of the lower incisor crown height. Less than this is described as
reduced, and greater than this is described as increased. An overbite is
complete if the edge of the lower incisors make contact with the upper
incisors or palatal mucosa, and incomplete if the edge of the lower in-
cisors do not make contact with the tooth surface or palatal mucosa.
An increased overbite is more commonly associated with Class II div-
ision 2 cases where the upper central incisors are retroclined.
An increased overbite does not necessarily require treatment, unless it is
causing trauma to the palatal mucosa.
Keywords: overlaps the coronal one-​third to half.

5. A ★★ OHCD 6th ed. → pp. 58 and 60


Fixed appliances can offer a wider range of tooth movements in com-
parison to removable appliances. Greater control over tooth movement
can be achieved, as brackets not only provide vertical or tilting move-
ments on the tooth surface, but also a force couple can be generated
by the interaction between the bracket and the archwire. Rotational and
apical movements are also possible. Removable appliances are capable
of the following tooth movements:
. Tipping
2. Movement of a block of teeth
3. Influencing eruption of opposing teeth, i.e. flat anterior bite plane,
buccal capping.
Keywords: removable appliance, tooth movements.
https://t.me/DentalBooksWorld
56 Chapter 3 Orthodontics

6. C ★★ OHCD 6th ed. → pp. 46 and 58


A removable appliance typically comprises four parts: one or more ac-
tive component, retention, anchorage, and a base plate. The active com-
ponent exerts the force required for the desired movement and can be
a spring, an elastic, or a screw, or a combination of any of these com-
ponents. For arch expansion, a midline expansion screw is commonly
used. There are various types of springs and screws that can be used,
depending on the tooth movement required.
● An acrylic base plate provides the framework of the appliance to

which all other components attach and provide anchorage.


● Adam clasps are a non-​active component which provide retention by

engaging undercuts, generally on the posterior teeth.


● An alternative retentive component is a southend clasp on the

anterior teeth.
● A posterior bite plane allows some disclusion to aid crossbite

correction.
Keywords: expand, maxillary arch, component, movement.

7. D ★★
Orthodontists may differ in the retention regime prescribed; however,
in the absence of clear instructions, vacuum-​formed retainers should be
worn every night (2 hours). There is a high risk of relapse if the patient
does not consistently wear them. There is a risk of caries and damage to
the enamel if food and drinks are consumed whilst wearing the retainers.
Only water can be consumed safely. The patient should return to their
orthodontist to seek clarification of their retention regime if they are
unsure.
Keywords: instructions, vacuum-​formed retainer.
→ Littlewood SJ, Millett DT, Doubleday B, Bearn DR, Worthington
HV. Retention procedures for stabilising tooth position after treatment
with orthodontic braces. Cochrane Database of Systematic Reviews.
206;:CD002283

8. B ★★ OHCD 6th ed. → p. 62


The ideal ages for the use of functional appliances to achieve the de-
sired jaw growth vectors is 0–​2 years for girls, and 2–​4 years for
boys. Studies have shown that an earlier treatment start with a twin
block prolongs treatment time for patients and can result in losing
patient compliance with lengthened treatment. Following a twin block
(approximately 9 months if worn well), fixed appliances are provided,
which is a further 8–​24 months of treatment. Cases where a twin
block may be fitted early include occasions when a child is being bul-
lied about their malocclusion or when a child is at increased risk of
trauma.
Keywords: pubertal growth, age.

https://t.me/DentalBooksWorld
answers 57

9. A ★★
Bodily movement requires 50–​20 g force, similar to the force required
to torque teeth into position, which is 50–​00 g. Rotational movement
and extrusion require a force of 35–​60 g. Tipping movements require
25–​60 g.
Bodily movement requires the highest force, as a greater area of the
periodontal ligament is involved, so the force is dissipated over a greater
area, and therefore, more force needs to be applied in order for optimal
force levels to be obtained.
Keywords: tooth movement, highest force.

0. D ★★
The definitions for Classes , II (), II (2), and III should be known, in
order to be able to do an orthodontic assessment in practice and make
appropriate referrals.
Class I—​the lower incisor edges lie immediately below the cingulum
plateau of the upper incisors.
Class II, division —​the lower incisor edges lie posterior to the cin-
gulum plateau of the upper incisors and the upper central incisors are
proclined or of average inclination with an increased overjet.
Class II, division 2—​the lower incisor edges lie posterior to the cin-
gulum plateau of the upper incisors and the upper central incisors are
retroclined. The overjet is usually minimal or may be increased.
Class III—​the lower incisor edges lie anterior to the cingulum plateau of
the upper incisors. The overjet is reduced or reversed.
Class II is always classified with a respective subdivision.
In this scenario, the lower incisors occlude posterior to the cingulum
plateau, indicating a Class II relationship, and the upper incisors are
retroclined.
Keywords: posterior to the cingulum plateau, retroclined.
→ Mitchell L. An Introduction to Orthodontics (4th ed.). Oxford University
Press, Oxford; 203.

. E ★★★
Parallax is a technique used to identify the position of an unerupted
tooth relative to a reference point such as an adjacent tooth. Two radio-
graphs must be taken, with a change in the position of the X-​ray tube
between the two radiographs. The object farthest away from the X-​ray
beam will move in the same direction as the tube shift. Two periapical
radiographs can be used for horizontal parallax, and an OPT and a USO
for vertical parallax. In this scenario, an OPT is also required to assess
the developing dentition.
The ‘SLOB’ rule can be used as a memory aid for this (same lingual,
opposite buccal). When using the parallax technique, if a tooth moves

https://t.me/DentalBooksWorld
58 Chapter 3 Orthodontics

in the same direction as the change in direction of the X-​ray beam, then
that tooth is lingually/​palatally positioned.
CBCT is a very useful contemporary tool for assessing impacted teeth.
However, due to the higher dose of radiation, it is not a first-​line imaging
technique and is reserved for cases where resorption of the adja-
cent teeth is suspected or further information is required for surgical
exposure of the canine.
Keywords: canine, unerupted, radiographic investigation.

2. B ★★★ OHCD 6th ed. → p. 30


The angle ANB represents the relationship of the maxilla and the man-
dible to the anterior cranial base and is obtained by subtracting the angle
formed by the sella turcica, nasion, and B point (SNB) from the sella
turcica, nasion, and A point (SNA).
ANB = SNA –​  SNB
The average value is 3° ± 2°, indicating a Class I skeletal relationship. An
increased ANB value is indicative of a Class II skeletal relationship, and
a decreased ANB is indicative of a Class III skeletal relationship. Broad
classification is as follows:
● ANB <2°: Class III
● 2° ≤ ANB ≤ 4°: Class I
● ANB > 4°: Class II.

Keywords: cephalometric, skeletal relationship.

3. C ★★★
Supernumerary teeth occur in 2% of the population in the permanent
dentition and are more common in males. Supernumerary teeth can be
described according to their morphology or position in the arch. Conical
supernumerary teeth are the most common form, and when a con-
ical supernumerary tooth is located in the midline, this is known as a
mesiodens.
Conical supernumerary teeth can cause displacement or failure of erup-
tion of a maxillary central incisor, or crowding. In some cases, super-
numerary teeth have no effect on adjacent teeth.
Typically, they require removal to facilitate alignment.
Complex and compound odontomes are disorganized masses of min-
eralized tissue that can impede eruption of teeth.
Keywords: rotated upper central incisor, radio-​opacity, midline.
→ Garvey MT, Barry HJ, Blake M. Supernumerary teeth—​an overview of
classification, diagnosis and management. Journal of the Canadian Dental
Association. 999;65:62–​6.

https://t.me/DentalBooksWorld
answers 59

4. E ★★★★ OHCD 6th ed. → p. 62


A twin block is a type of functional appliance. These guide the forces of
muscle function, tooth eruption, and growth to encourage correction
of a malocclusion. Commonly, twin blocks are used in growing patients
to correct Class II malocclusions with an increased overjet. A twin block
will correct a deep overbite by differential eruption of the posterior
teeth. As this patient is young, it is possible to utilize his growth in the
management of his malocclusion. The timing of twin block appliances
is important, and preferably they should be used just before the ado-
lescent growth spurt begins. However, they require adequate retention
such as using Adam’s clasps, labial bows, and ball hooks. Correction of
the malocclusion has been attributed to roughly 30% of skeletal and 70%
of dento-​alveolar change.
Orthognathic surgery is reserved for when facial growth is com-
plete but the skeletal pattern is too severe to consider orthodontic
camouflage alone.
Keywords: 2-​year old, 0 mm, Class II skeletal.
→ O’Brien K, Wright J, Conboy F, et al. Effectiveness of early orthodontic
treatment with the twin block appliance: a multicenter, randomized, con-
trolled trial. Part : dental and skeletal side effects. American Journal of
Orthodontics and Dentofacial Orthopedics. 2003;24:234–​43.

5. B ★★★★
In a growing patient, an upper removable appliance with a flat
anterior bite plane allows eruption of the posterior teeth and reduc-
tion of a deep overbite. The resultant reduction in vertical overlap
of the teeth (overbite) will allow a lower fixed appliance to be
placed simultaneously, without the lower brackets being in traumatic
occlusion.
Keywords: complete traumatic overbite, removable appliance, Class II,
division 2.

6. D ★★★★ OHCD 6th ed. → p. 684


Study models are an important part of case planning and assessment
in orthodontics. They are part of a patient’s clinical records and, as
such, should be kept for a minimum of  years or until the patient is
25 years old (whichever is longer). In this scenario, as the patient was
 years old at the time of the impressions, they should be stored until
the age of 25 because this is longer than the standard  years for
adults. As they have already been stored for 4 years, they need to be
kept for a further 0 years. This also applies to clinical notes, clinical
photographs, and radiographs, which all form part of a patient’s med-
ical records.

https://t.me/DentalBooksWorld
60 Chapter 3 Orthodontics

With the digital revolution in dentistry, scanning and storing models elec-
tronically can save vast amounts of space, but it is important to ensure
adequate security systems are in place and the data are regularly backed
up. The guidelines below illustrate time to keep records.
Keywords: study models.
Dental Protection. Record keeping in England. 207. Available at: https://​
www.dentalprotection.org/​uk/​articles/​record-​keeping-​in-​the-​uk

https://t.me/DentalBooksWorld
Chapter 4 61

Periodontics
Peter Clarke

‘We can give you a clean slate, but ultimately, it’s your job to keep
it clean.’
Periodontitis is estimated to be the sixth most prevalent disease in the
world, and clinicians are likely to encounter this disease and other gin-
gival conditions on a regular basis. It is therefore important to have a
sound understanding of both the pathophysiology and management of
periodontitis and related conditions.
Periodontal disease may also be seen as a manifestation of systemic
disease, so it may provide a window into the patient’s general health.
Common conditions, such as diabetes mellitus, have a well-​established
relationship with the progression of periodontal disease, but rare gen-
etic conditions, such as Ehlers–​Danlos syndrome, may produce unusual
findings. Therefore, the clinician should have a good breadth of know-
ledge and be able to examine the patient as a whole, relating oral signs
to systemic symptoms in order to diagnose and manage appropriately.
The general dental practitioner’s role will focus mainly on diagnosis
and non-​surgical management of these patients, but awareness of the
more advanced treatment will ensure appropriate referral and allow an
informed discussion with the patient. A key challenge in the successful
management of these patients is often getting them to obtain a suitable
level of plaque control to stabilize the disease and maintain health. Ability
to communicate this effectively and encourage excellent oral care is an
invaluable asset.
The questions in this chapter will test the readers’ knowledge of the
fundamentals of periodontal diagnosis and practical skills. Moreover,
questions are also presented examining the relationship with systemic
disease and advanced treatment concepts. It is hoped that the questions
in this chapter will test the readers’ baseline knowledge and promote
further reading around complex or contentious subjects.
Key topics include:
● Diagnosis/​disease classification
● Aetiology
● Systemic conditions
● Non-​surgical management/​cause-​related therapy
● Adjunctive therapies
● Surgical management/​corrective therapy
● Supportive therapy.

https://t.me/DentalBooksWorld
62 Chapter 4 Periodontics

QUESTIONS

. A 45-​year-​old man is being reviewed, following a course of non-​


surgical periodontal therapy. His current full mouth plaque score is
5%, and he has had a good response to the treatment. Following repeat
periodontal indices, he still has a number of isolated probing depths of
5 mm, which have reduced from 8 mm. Additional recession is present
in these areas, varying from  to 2 mm. There is no bleeding present at
these sites, and the root surfaces feel smooth. Which single clinical indi-
cator would suggest these sites are stable? ★
A Absence of bleeding on probing
B Plaque scores of <20%
C Presence of plaque-​retentive factors
D Recession
E Reduction in probing pocket depth

2. A 44-​year-​old woman is having non-​surgical root surface debride-


ment for chronic periodontitis. The instrument in Figure 4. is
being used to remove a subgingival calculus from a deep periodontal
pocket. These instruments have single cutting edges on the working tip
to enable more efficient removal of deposits and to avoid iatrogenic
damage. What is the optimal angle between the cutting edge and the
tooth surface when in function? ★
A 5°
B 30°
C 45°
D 70°
E 90°

Figure 4.

https://t.me/DentalBooksWorld
questions 63

3. A 28-​year-​old man presents for a routine examination. He is a


smoker (0 pack years) but is otherwise fit and well. His previous
clinical notes report a diagnosis of plaque-​induced gingivitis. However,
following clinical examination today, his diagnosis has changed to gener-
alized stage I, grade B periodontitis. Which single clinical finding is funda-
mental in differentiating between these two diagnoses? ★
A Gingival bleeding
B Loss of attachment
C Mobility
D Periodontitis only occurs after gingivitis has been present for 3 weeks
E Recession

4. A 78-​year-​old man complains of loose teeth. He has not attended


the dentist in many years, and a detailed examination reveals:
● Poor plaque control
● Widespread periodontal probing depths of >8 mm
● Furcation involvement of a number of teeth

● Generalized horizontal bone loss of up to 70%.

He is a smoker of 5 pack years (currently smoking five cigarettes a


day) but is otherwise fit and well. What is the single most appropriate
diagnosis? ★
A Generalized stage III, grade A periodontitis
B Generalized stage III grade C periodontitis
C Generalized stage IV, grade B periodontitis
D Localized stage III, grade A periodontitis
E Localized stage IV, grade B periodontitis

5. A basic periodontal examination is performed on a 42-​year-​old


woman. Her oral hygiene is generally good, but generalized reces-
sion of 2–​3 mm is noted. In the upper right posterior sextant, a 2* is
recorded. What single meaning does the * indicate in this context? ★
A Furcation involvement
B Loss of attachment of >7 mm
C Previous severe periodontal disease
D Probing depths of between 4 and 6 mm
E Probing depths of >6 mm

https://t.me/DentalBooksWorld
64 Chapter 4 Periodontics

6. A 27-​year-​old man attends, complaining of loose teeth. He is med-


ically fit and well and has never smoked. He reports rapid devel-
opment of mobility, affecting all of his lower anterior teeth and lower
right first permanent molar. Periapical radiographs provided by his gen-
eral dental practitioner show progression of horizontal bone loss from
20% to 50% in the last 2 months. His oral hygiene is good, and he is
aware that his father and paternal grandfather suffered from pyorrhoea.
Which single bacteria is most commonly associated with the disease
above? ★★
A Aggregatibacter actinomycetemcomitans
B Fusobacterium nucleatum
C Porphyromonas gingivalis
D Prevotella intermedia
E Tannerella forsythia

7. A 56-​year-​old man has swollen gums. Despite suboptimal oral


hygiene, the appearance of the gingivae is suggestive of gingival
overgrowth. He has controlled type 2 diabetes and hypertension. Which
single common medication is most likely to be responsible for this clinical
situation? ★★
A Amlodipine
B Bendroflumethiazide
C Glipizide
D Metformin
E Simvastatin

8. A 22-​year-​old woman has bleeding gums and severe pain around


her lower front teeth. She has type  diabetes mellitus, and she is
a heavy smoker (5 pack years). Her gingivae around the lower incisors
are acutely tender and inflamed, and there is ulceration, combined with
loss of the papilla. No significant probing depths or associated bone loss
on the radiographs are noted, but there is an unpleasant odour. Which is
the single most likely diagnosis? ★★
A Plaque-​induced gingivitis
B Molar–​incisor, grade C periodontitis
C Necrotizing gingivitis
D Periodontal abscess
E Periodontitis as a manifestation of systemic disease

https://t.me/DentalBooksWorld
questions 65

9. A 52-​year-​old man has generalized stage II, grade B periodon-


titis. Oral hygiene instruction was instigated immediately, along
with non-​surgical periodontal therapy, providing full mouth root sur-
face debridement (RSD). Upon a 3-​month review, he has demonstrated
no improvement in oral hygiene and his lack of motivation is apparent;
heavy plaque deposits are present on all teeth surfaces. What is the
single most appropriate next step in his management? ★★
A Commence a second round of  RSD
B Prescribe antibiotics to tackle periodontitis
C Provide another round of RSD and prescribe antibiotics to tackle
periodontitis
D Re-​address oral hygiene instruction
E Refer to a specialist periodontist for periodontal management

0. A 37-​year-​old woman has received a course of non-​surgical


periodontal therapy in the undergraduate clinic. At review,
loss of attachment charting demonstrates generalized reduction in peri-
odontal probing depths, combined with only minor amounts of associ-
ated recession. The clinical tutor asks the presenting student which type
of healing is most likely to have occured in this scenario. (Select one
answer from the options listed below.) ★★★
A Regeneration: normal bony and junctional architecture is regained
B Regeneration and repair: mainly new bone and periodontal ligament
(PDL) are formed, but some healing results from a long junctional
epithelium (JE)
C Regeneration and repair: the majority of reduction is from a long JE,
but some new bone height and PDL are formed
D Repair: a fibrous connection of scar tissue has formed
E Repair: a long JE attachment has formed

. A 5-​year-​old South Asian adolescent boy has severe peri-


odontal destruction and drifting of the upper incisors. His peri-
odontal disease had previously affected his primary dentition, and he has
palmar–​plantar keratosis. His sibling also suffers from severe periodontal
disease. Which single systemic condition is linked with their periodontal
disease? ★★★
A Chèdiak–​Higashi syndrome
B Hypophosphatasia
C Kabuki syndrome
D Leucocyte adhesion deficiency
E Papillon–​Lefèvre syndrome

https://t.me/DentalBooksWorld
66 Chapter 4 Periodontics

2. A 25-​year-​old man presents with generalized probing depths


of >7 mm around the upper incisors, all four first permanent
molars, and the lower premolars. Although he is an irregular attender, his
plaque score is 20% and he is medically fit and well, with no other peri-
odontal risk factors. What single adjunctive management option would
routinely be indicated for this patient? ★★★
A Antibiotics
B Chlorhexidine
C Complement inhibition therapy
D Photodynamic therapy
E Probiotic therapy

3. A 45-​year-​old man has severe maxillary anterior toothwear.


Marked sclerotic dentine is exposed, with  mm of supra-​
gingival tooth remaining. Dento-​alveolar compensation has occurred,
and there is no inter-​occlusal space for restorations. The surrounding
periodontium appears healthy, with no periodontal bone loss and
normal root lengths. Which is the single most appropriate procedure to
facilitate the direct restoration of these teeth? ★★★
A Apically repositioned flap
B Crown lengthening
C Gingivectomy
D Reduction of mandibular incisor height
E Overdenture provision

4. A 24-​year-​old woman has a Class II Millers recession defect,


for which a connective tissue graft and a coronally advanced
flap are planned. She has excellent oral hygiene, and all four wisdom
teeth. Which is the single most appropriate intraoral site for taking her
graft? ★★★
A Attached gingivae
B Buccal mucosa
C Buccal shelf
D Hard palate
E Retromolar pad

https://t.me/DentalBooksWorld
questions 67

5. A 46-​year-​old woman requires crown lengthening surgery to


facilitate the restoration of a fractured upper premolar. Based
on the clinical assessment, resection of 2 mm of gingiva is possible. The
junction between which two tissues is used to determine whether a
resective approach is possible when crown lengthening? ★★★
A Attached gingiva and alveolar mucosa
B Free gingiva and attached gingivae
C Free gingiva from sulcular epithelium
D Junctional epithelium and periodontal ligament
E Sulcular epithelium and junctional epithelium

6. A consultant delivers a seminar on the pathogenesis of peri-


odontal disease. Complement activation is discussed with
regard to innate immunity, and a junior dental student asks how bacteria
stimulate the complement cascade. Which single cell surface structure
stimulates this cascade? ★★★★
A Immunoglobulin G (IgG)
B Interleukin-​ (IL-​)
C Lipopolysaccharide (LPS)
D Membrane attack complex (MAC)
E Pathogen recognition receptors (PRRs)

7. A 34-​year-​old man is being reviewed, following two courses


of non-​surgical periodontal therapy. He has stopped smoking
 year ago and has no other risk factors. His recent plaque score is 23%,
and he has three localized areas of isolated probing depth, which radio-
graphically demonstrate angular vertical defects, with up to 50% of bone
loss. A range of infrabony defects are identified when probing, and he
has been treatment-​planned for regenerative surgery. Which single type
of defect has the most predictable outcome? ★★★★
A One-​walled defect
B Two-​walled defect
C Three-​walled defect
D Circumferential defect
E No walled defect

https://t.me/DentalBooksWorld
68 Chapter 4 Periodontics

8. A 46-​year-​old woman is reviewed 6 weeks after placing a


metallo-​
ceramic crown on her upper right central incisor.
The colour match is reasonable, although the crown appears slightly
opaque. She also has marked localized marginal gingivitis, with fine
subgingival calculus deposits that were not evident prior to crown
cementation. The crown margins are 0.5 mm intra-​sulcular and are
well adapted and smooth on probing. Which single additional aspect
of the prosthodontic treatment is most likely to be responsible for the
gingivitis? ★★★★
A Bulbous emergence profile
B Excess cement
C Negative crown margins
D Sensitivity to adhesive resin cement
E Supracrestal attached tissue encroachment

9. A 24-​year-​
old woman complains of a localized recession
defect around her lower left central incisor, which she is
struggling to keep clean. She has a history of orthodontic treatment.
Clinically, 4 mm of recession is present labially on the lower incisor,
with no loss of papilla height. A 2-​mm probing depth associated with
the defect and a high frenal attachment are also evident. Although not
sensitive, there is localized plaque build-​up around the defect, mar-
ginal inflammation, and a thin biotype with a -​mm band of keratinized
tissue present. The vestibular depth is also very shallow. Which is the
single most appropriate initial treatment if aesthetics are not a major
concern? ★★★★
A Advice and reassurance—​no intervention
B Connective tissue graft with coronally repositioned flap
C Free gingival graft
D Frenectomy
E Rotational pedicle graft

20. A 53-​year-​old man is dissatisfied with his recent privately con-


ducted periodontal treatment. He has submitted a formal com-
plaint to the practice. After working to resolve the complaint locally, no
solution has been reached. Which single piece of advice is most appro-
priate to give the patient in this scenario? ★★★★
A Contact a solicitor
B Contact the Dental Complaints Service
C Contact the General Dental Council (GDC)
D Contact the National Health Service (NHS) Ombudsman
E Contact your indemnifier

https://t.me/DentalBooksWorld
answers 69

ANSWERS
. A ★ OHCD 6th ed. → p. 88
Research by Lang et al. in the 980s and 990s highlighted the link
between absence of bleeding on probing (BOP) and periodontal sta-
bility. Recession and a reduction in probing pocket depths are likely to
represent successful treatment outcomes. However, these factors alone
do not indicate periodontal stability, as further periodontal destruction
could occur concomitantly.
Plaque retentive factors are likely to encourage biofilm formation and
aggravation of periodontal tissues. These should be addressed as part of
the overall periodontal management.
Plaque accumulation is a fundamental component in periodontitis.
Plaque indices, such as the O’Leary plaque index, are useful tools in the
assessment and management of caries and periodontitis. Whilst they are
useful in assessing patient compliance and prognosis, a low plaque score
does not indicate periodontal health, as plaque accumulation will inevit-
ably lead to gingival inflammation, which, depending on host response,
may progress to periodontitis.
Keywords: periodontal stability.
→ Lang NP, Adler R, Joss A, Nyman S. Absence of bleeding on probing an
indicator of periodontal stability. Journal of clinical periodontology. 990
Nov;7(0):74–​2.
→ O’Leary T, Drake R, Naylor J. The plaque control record. Journal of
Periodontology. 972;43:38.

2. D ★
Gracey curettes are site-​ specific hand scalers used for subgingival
debridement. The lower terminal shank should be inserted parallel to
the long axis of the tooth. The working tip is designed with a single lower
cutting edge to improve adaptation of the curette to the root surface and
the base of the pocket. If inserted correctly, this produces an optimal
angle of 70° between the tooth surface and the working tip. Absence of
a second cutting surface is designed to reduce trauma to the periodontal
tissues when scaling.
Keywords: Gracey site-​specific (displayed in image), cutting edge.

3. B ★ OHCD 6th ed. → p. 86


Periodontal disease is extremely prevalent in the population. Traditionally,
the disease process was classified into four stages: initial, early, estab-
lished, and advanced—​although ultimately, it is a continuous spectrum.
In the first three stages, signs of inflammation develop and the immune
response establishes. Vasodilatation occurs, bringing with it various cyto-
kines and leucocytes. As the disease becomes more established, there

https://t.me/DentalBooksWorld
70 Chapter 4 Periodontics

is a shift from the innate immune response to the adaptive immune


response, with T and B cells beginning to predominate.
The next stage is the advanced stage of the disease (periodontitis). In
this stage, there is a dense inflammatory infiltrate, with large numbers
of inflammatory mediators present. The mechanism of tissue destruc-
tion is a complex combination of direct bacterial action and indirectly
as a result of the host response. Not all cases progress from gingivitis
to periodontitis, but it is known that the host response plays a signifi-
cant part in the overall clinical picture. Around 0–​5% of the popu-
lation will be hypersensitive and suffer from severe disease, and about
0% of the population will be innately resistant to the disease process.
Recession and pocket depth measurements are combined to calculate
clinical attachment loss.
Keywords: plaque-​induced gingivitis, periodontitis.
→ Palmer RM, Ide M, Floyd PD. A Clinical Guide to Periodontology (3rd ed.).
BDJ Books, London; 203.

4. C ★
Until recently, classifications of periodontal disease were based upon
the work of Armitage and colleagues at the International Workshop for
Classification of Periodontal Diseases and Conditions in 999. A new
classification system was released in summer 208, based upon updated
understanding of periodontal biology, which also allows for incorpor-
ation of individual patient factors. It is a more pragmatic multidimen-
sional staging system that not only facilitates future adaption to emerging
evidence, but also permits personalized diagnosis that is crucial to con-
structing a comprehensive care plan. One of the big changes is ‘aggres-
sive’ disease and ‘chronic’ disease are no longer distinct entities and have
been incorporated into an overall umbrella term of ‘periodontitis’, which
is subsequently modified by the clinical findings for that individual patient.
Periodontal disease is now classified by:*
. The severity and complexity:
Stage I: initial periodontitis
Stage II: moderate periodontitis
Stage III: severe periodontitis with potential for additional tooth loss
Stage IV: severe periodontitis with potential for loss of dentition
2. The extent and distribution:
Generalized: >30% of dentition affected
Localized: <30% of dentition affected
Molar–​incisor distribution: affecting incisor and molar teeth
3. The risk of progression or anticipated treatment response:

*Reproduced from Caton et al. A new classification scheme for periodontal and peri-​
implant diseases and conditions –​Introduction and key changes from the 999 classifi-
cation. Journal of Periodontology. 208;89(Suppl ):S–​S8. Copyright © 208, John Wiley
and Sons.

https://t.me/DentalBooksWorld
answers 71

Grade A: slow rate of progression


Grade B: moderate rate of progression
Grade C: rapid rate of progression.
Severity is classified based on clinical attachment loss (mild, –​2 mm;
moderate, 3–​4 mm; or severe, >5 mm) and modified by the complexity
of treatment (e.g. local factors such as deep angular bony defects, root
grooves, furcation involvement, etc.).
The rate of progression and response to treatment is decided upon by
judging the rate of bone loss over time (if sequential radiographs are
present) or by estimating the bone loss/​age ratio. This is then adjusted
by examining the risk factors of the patient such as glycaemic control,
smoking habits, and suspected genetic susceptibility.
In this instance, there is no relevant medical history to link the diagnosis
to a systemic disease or any clinical evidence of necrotizing disease, and
therefore, the diagnosis would come under the classification of ‘peri-
odontitis’. Clinically, there is attachment loss of >5 mm affecting the
majority of the mouth and up to 70% of horizontal bone loss. Combined
with the increased mobility and the numerous furcation lesions, this puts
the dentition at risk of being lost and the patient is classified in the stage
IV category. Given that he is 78 but has been a lifelong smoker with
chronically suboptimal oral hygiene, the rate of bone loss is at an ex-
pected rate, and therefore, he would be in grade B category.
Keywords: poor plaque control, widespread pocketing of >8 mm, bone
loss of up to 70%.
→Tonetti M, Greenwell H, Kornman K. Staging and grading of periodon-
titis: Framework and proposal of a new classification and case definition.
Journal of Periodontology. 208;89(Suppl ):S59–​S72
→ Caton GJ, Armitage G, Berglundh T, et al. A new classification scheme
for periodontal and peri-​implant diseases and conditions: introduction
and key changes from the 999 classification. Journal of Periodontology.
208;89(Suppl ):S–​8.

5. A ★ OHCD 6th ed. → p. 22


The basic periodontal examination is a widely used screening tool to
assess for the presence of periodontal disease. It is conducted with a
ball-​ended probe that has black bands on it (WHO 62 probe) at 3.5–​
5.5 mm and 8.5–​.5 mm. When conducting the assessment, the mouth
is split into sextants and every tooth (apart from the third molars) is
examined; the worst score for each sextant is recorded. The score then
gives an indication of what treatment is required. Table 4. describes the
scoring system. The addition of a star to a sextant score indicates there
is furcation involvement. Historically, the star was a standalone score for
a sextant indicating either furcation involvement or attachment loss of
>7 mm; however, this was changed in 20.
→ British Society of Periodontology. Basic periodontal examination (BPE).
206. Available at: http://​www.bsperio.org.uk/​publications

https://t.me/DentalBooksWorld
72 Chapter 4 Periodontics

Table 4. The basic periodontal examination*


BPE code Indications
0 Pockets <3.5 mm (first black band completely visible)
No bleeding on probing or plaque-​retentive factors
 Pockets <3.5 mm (first black band completely visible)
Bleeding on probing is present, but there is no plaque-​retentive
factors
2 Pockets <3.5 mm (first black band completely visible)
Plaque-​retentive factors are present (e.g. calculus or overhanging
restorations)
3 Pockets 3.5–​5.5 mm (first black band partially visible)
4 Pockets >5.5 mm (first black band completely into the pocket)
* Furcation involvement
(NB. A star should be recorded, along with a number, if
furcation involvement is present in the sextant)
Reproduced from the British Society of Periodontology. Basic Periodontal Examination.
March 206. [Available at] http://​www.bsperio.org.uk/​publications.

6. A ★★
The scenario describes a young male patient with what was traditionally
known as localized aggressive periodontitis but would now be classified
as localized, grade C periodontitis (the stage would reflect the amount
of destruction at the specific time point). Aggressive periodontal dis-
ease described a group of diseases in which there is rapidly progressing
destruction of the periodontal attachment. It is classified as either local-
ized, generalized, or molar–​incisor, depending on its distribution. In the
majority of cases, patients tend to be younger in age with good plaque
control. There is also a propensity for familial aggregation, and patients
display a non-​contributory medical history.
Whilst multiple ‘red-​ complex’ pathogens have been implicated in
periodontitis, Aggregatibacter actinomycetemcomitans (Aa), formerly
Actinobacillus actinomycetemcomitans, has been identified as a key
pathogen in localized aggressive periodontal disease (now more cor-
rectly approximated to molar–​incisor, stage III, grade C periodontitis),
with patients demonstrating serum antibodies against the causative
agent. This bacterium possesses numerous virulence factors, enzymes,
and endotoxins which upregulate the connective tissue inflammatory
response, increase connective tissue destruction, and inhibit poly-
morphonuclear leucocytes function. Furthermore, this bacterium may
also invade epithelial cells of the periodontal pocket, increasing its resist-
ance to root surface debridement.
Keywords: progression, bone loss from 20% to 50%, bacteria, commonly
associated.
→ Alani A, Seymour R. Aggressive periodontitis: how does an understanding
of the pathogenesis affect treatment? Dental Update. 20;38:5–​2.

https://t.me/DentalBooksWorld
answers 73

7. A ★★
Diagnosing drug-​induced gingival overgrowth (DIGO) requires a thor-
ough history and examination. Suboptimal oral hygiene could precipitate
gingival swelling in the form of dental biofilm-​induced gingivitis. However,
the appearance of gingival overgrowth is generally distinct from that of
dental biofilm-​induced gingival swelling. Careful consideration of the
patient’s medications is prudent.
Anticonvulsants (phenytoin), immunosuppressants (ciclosporin),
and calcium channel blockers (nifedipine) have all been implicated in
the development of DIGO. All the drugs listed above could be used
in the management of this patient’s medical conditions. Of the drugs
listed, amlodipine, a calcium channel blocker, is frequently associated
with DIGO.
Whilst nifedipine may have been more widely reported in the literature
regarding the development of DIGO, amlodipine is an alternative calcium
channel blocker more regularly prescribed and also linked with DIGO.
Good knowledge of drug classifications and drug names is important in
determining the correct answer in this scenario.
Keywords: gingival overgrowth, commonly prescribed medication,
hypertension.
→ Seymour RA, Thomason JM, Ellis JS. The pathogenesis of drug induced
gingival overgrowth. Journal of Clinical Periodontology. 996;23:65–​75.

8. C ★★ OHCD 6th ed. → p. 93


Necrotizing periodontal diseases involve bacterial invasion of the
periodontium, leading to necrosis and tissue damage. The condition
is usually very painful and associated with distinct malodour. Amongst
others, bacteria thought to be associated with this disease are Prevotella
intermedia, Fusobacterium spp., and Treponema spp. Patients may also be
immunocompromised. Therefore, risk factors, such as smoking, stress,
malnutrition, poorly controlled diabetes, steroid use, and other medical
conditions which may compromise the immune system, should be noted.
Management of this condition is through a combination of antibiotics
and localized mechanical debridement of the biofilm (local anaesthesia is
often required in order to do this).
Keywords: smoker, diabetes, acutely tender, loss of the papilla,
unpleasant odour.
→ Rowland RW. Necrotizing ulcerative gingivitis. Annals of Periodontology.
999;4:65–​73.

9. D ★★ OHCD 6th ed. → p. 78


Plaque is the primary cause of periodontitis. If the patient is unable or
unwilling to maintain low plaque levels, another round of RSD is unlikely
to provide any long-​term benefit. Patient education and involvement
is fundamental to successful management of periodontal disease. This
discussion is dynamic and must adapt to each patient’s situation. It is

https://t.me/DentalBooksWorld
74 Chapter 4 Periodontics

important to emphasize that professional treatment is an adjunct to


excellent home care.
Further discussion of the aetiology, management, and progression of
periodontal disease, along with further oral hygiene instruction, should
take precedence over further treatment.
Referral for periodontal surgery is inappropriate, as periodontal surgery
is not suitable for generalized periodontitis in patients with poor com-
pliance. Antibiotics are not routinely used to treat periodontitis, unless
disease is rapidly progressing and suggestive of grade C (aggressive)
periodontitis.
Specialist periodontists are unable to provide any additional treatment
until suitable levels of plaque control are attained.
Keywords: grade B periodontitis, oral hygiene, lack of motivation.

0. E ★★★
The three attachment structures of the periodontium to the tooth are as
follows (coronal to apical):
. The junctional epithelium
2. The connective tissue attachment
3. The periodontal ligament.
In health, the JE is, on average, 0.97 mm and the connective tissue at-
tachment is .07mm; together, they comprise the supracrestal attached
tissues (formally the ‘biological width’). During active periodontal dis-
ease, the JE becomes ulcerated and there is damage to the connective
tissue attachment. As the process progresses, the periodontal pocket
deepens and the JE and CT attachments migrate apically. Following suc-
cessful non-​surgical therapy, the epithelial cells migrate the quickest and
are the first to colonize the root surface, starting at the apical portion of
the pocket and migrating coronally. This leads to the formation of a long
JE. The connective tissue repair then helps to stabilize this new attach-
ment. In regenerative therapies, the aim is to restore the original peri-
odontal architecture before disease and facilitate the formation of new
cementum, PDL, and bone. Healing following successful non-​surgical
periodontal therapy is therefore reparative whereby a long JE develops.
Keywords: reduction in probing depths, non-​surgical therapy, healing.
→ Gargiulo A, Wentz F, Orban B. Dimensions of relations of the
dentogingival junction in humans. Journal of Periodontology. 96;32:
26–​7.
→ Heasman P, Preshaw P, Robertson P. Successful Periodontal Therapy: A
Non-​Surgical Approach. Quintessence, London; 2004.

. E ★★★
Various systemic conditions are associated with rapid periodontal
destruction. Examples include: Chèdiak–​Higashi syndrome, Down’s

https://t.me/DentalBooksWorld
answers 75

syndrome, Ehlers–​ Danlos syndrome, hypophosphatasia, leucocyte


adhesion deficiency, and Papillon–​Lefèvre syndrome.
Papillon–​Lefèvre Syndrome is a rare autosomal recessive hereditary con-
dition, most common in the South Asian population. A genetic mutation
results in the loss of function of a lysosomal enzyme found in neutrophils
called cathepsin C; this has an important function in bacteria destruction.
Other clinical findings include: palmar–​plantar keratosis, arachnodactyly,
recurrent skin infections, pes planus (flat feet), onychogryphosis (hyper-
trophic nails), and acro-​ osteolysis (resorption of distal phalanges).
Management of these patients is difficult, and there is a high risk of tooth
loss and edentulism. Furthermore, implants placed in these patients are
high risk for peri-​implantitis.
Keywords: palmar–​plantar keratosis, sibling.
→ Chapple I, Hamburger J. Periodontal Medicine: A Window on the Body.
Quintessence, London; 2006.
→ Nickles K, Schacher B, Ratka-​Krüger P, Krebs M, Eickholz P. Long-​
term results after treatment of periodontitis in patients with Papillon–​
Lefèvre syndrome: success and failure. Journal of Clinical Periodontology.
203;40:789–​98.

2. A ★★★ OHCD 6th ed. → p. 92


The scenario above describes a typical presentation of a patient with
generalized aggressive periodontitis (AP)—​now classified as generalized
stage III, grade C periodontitis. AP is a relatively rare disease, associated
with rapid attachment loss. Plaque levels are often inconsistent with the
severity of bone loss, and there are strong familial tendencies. Patients
are often young (<35 years) and otherwise fit and well.
Antibiotics are important and often lifesaving medications. There are few
indications for antibiotic use within dentistry, especially in the manage-
ment of periodontal disease, but AP is one indication.
Localized AP is associated with particularly virulent pathogens, e.g.
Aggregatibacter actinomycetemcomitans (Aa), whereas serum antibodies
to infective agents in generalized AP are less robust. These pathogens
have been shown to invade local gingival tissues, and this makes elimin-
ation with RSD alone challenging.
The timing and use of antibiotics in AP remain controversial. However,
contemporary evidence shows a greater outcome with antibiotics pro-
vided immediately after the first round of RSD. Prior to commencing
any antimicrobial therapy, dentists should be confident in their diagnosis,
with referral to a specialist advised.
Complement inhibition therapy limits the effect of the complement
system and has been shown to reduce the amount of inflammatory
markers and minimize attachment loss and bone destruction in monkey
models. There is no current evidence on their use in humans.
Photodynamic therapy is an antimicrobial treatment modality where
a photosensitizer is administered to bacteria prior to application of a

https://t.me/DentalBooksWorld
76 Chapter 4 Periodontics

light source. The photosensitizer then breaks down upon activation


to produce free radicals which damage the cell membrane. Clinical
data do not show consistent long-​ term benefit over non-​ surgical
management alone.
Probiotic therapy involves taking a tablet containing primary colonizing
bacteria that are more conducive to periodontal health, shifting the eco-
logical environment. Again clinical results show minimal clinical significant
difference over non-​surgical treatment.
Keywords: 25 years, generalized probing depths of >7 mm, plaque score
of 20%.
→ Griffiths S, Ayob R, Guerrero A, et al. Amoxicillin and metronidazole
as an adjunctive treatment in generalized aggressive periodontitis at ini-
tial therapy or re-​treatment: a randomized controlled clinical trial. Journal
of Clinical Periodontology. 20;38:43–​9.

3. B ★★★ OHCD 6th ed. → p. 206


Toothwear is an increasing problem within the adult population.
Toothwear resulting in only  mm of remaining coronal tooth tissue is
severe and has significant consequences in terms of treatment predict-
ability. Composite restorations placed at an increased occlusal vertical
dimension (OVD) are now a relatively common treatment modality util-
ized in toothwear management. To facilitate restorations, crown length-
ening can be performed to apically reposition the gingivae and recontour
the bone (osseous recontouring). An apically repositioned flap without
osseous recontouring will likely result in gingival rebound with inad-
equate crown height gained.
Gingivectomy alone is also likely to encroach upon the supracrestal at-
tached tissues. Reduction of the mandibular incisors may provide space
for maxillary restorations, but no increase in bonding surface would be
gained. Direct restorations to manage toothwear are commonly placed
at an increased OVD, and the Dahl concept is utilized. Overdenture pro-
vision should not be overlooked as a potential management strategy;
however, overdenture provision would not facilitate the direct restor-
ation of teeth.
Keywords:  mm of supra-​gingival tooth remaining, dento-​alveolar com-
pensation, periodontium healthy.

4. D ★★★ OHCD 6th ed. → p. 2


Free gingival (FG) or connective tissue grafts involve complete detach-
ment of the graft from its donor site to the recipient site. An FG graft
involves removal of keratinized tissue, whilst a connective tissue graft is
removal of the subepithelial connective tissue only. Adequate prepar-
ation of the recipient site is imperative for successful integration, as is
careful handling of the graft during harvest and placement. The palate
is the most common site for harvest of a soft tissue graft. Subepithelial
connective tissue grafts are most commonly sourced from the palate, as
the available volume is greater. The tuberosity can be used, but it often

https://t.me/DentalBooksWorld
answers 77

lacks the necessary width of connective tissue required in graft proced-


ures. These flaps do not maintain their original blood supply and, as such,
rely heavily upon the vascularity of the recipient site for survival. A key
component of graft success is to ensure its immobility during the healing
period and allow undisrupted angiogenesis.
Alternatively, pedicle grafts are not detached and, as such, retain their
blood supply. Retention of blood supply is a significant advantage.
However, these flaps can only be repositioned locally (e.g. rotational flap
or coronal advancement flap) and sometimes lack sufficient volumes of
connective or keratinized tissue.
Keywords: recession, connective tissue graft, intraoral site.
→ Miller P. A classification of marginal tissue recession. International
Journal of Periodontics and Restorative Dentistry. 985;5:8–​3.

5. A ★★★
The mucogingival junction is a soft tissue landmark where the attached
gingiva meets the alveolar mucosa. The attached gingiva is keratinized
tissue, which results in a distinctly lighter colour than the adjacent, highly
vascular alveolar mucosa.The keratinized attached gingiva serves a vital
protective function during mastication and tooth brushing. Removal of
the entire attached gingiva is not advised for crown lengthening, and if an
inadequate attached gingiva is present (i.e. <2–​3 mm), then apical repo-
sitioning of the flap is advised.
The free gingival groove corresponds to the cemento–​enamel junction
of the tooth. At this point, the free gingiva meets the attached gingiva.
The free gingival groove is present in <50% of patients and is often lost
when periodontal tissues become inflamed. See Figure 4.2 which shows
macroscopic periodontal landmarks.
Keywords: crown lengthening, junction, two tissues, resective.
→ Devlin H, Craven R. Oxford Handbook of Integrated Dental Biosciences.
Oxford University Press, Oxford; 208.

1. Alveolar
mucosa
2. Attached
gingiva
3. Mucogingival
junction
4. Free gingiva
5. Free gingival
groove

Figure 4.2

https://t.me/DentalBooksWorld
78 Chapter 4 Periodontics

6. C ★★★★
The complement cascade is a general component of innate immunity.
Activation of the complement cascade is pro-​inflammatory. Activated
complement recruits immune cells and assists in bacterial cell destruc-
tion. Complement also binds bacteria in a process known as agglutin-
ation, which assists in destruction of the pathogens.
The membrane attack complex is formed during complement activation
and is heavily involved in the destruction of pathogens. The classical,
common, and alternative pathways have been identified. LPS (endo-
toxin), a component of bacterial cell walls, has been identified as a
potent complement activator in periodontal disease.
The classical pathway involves antigen–​antibody complex formation,
which activates C of the complement cascade. The alternative pathway
is activated by LPS and bacterial proteases. Both pathways converge
at C3 activation where the common pathway begins. IgG is an anti-
body produced by B cells, whilst IL-​ is a group of immune-​regulating
cytokines.
Keywords: complement activation, bacteria.
→ Eaton K, Ower P. Practical Periodontics. Elsevier, London; 205.

7. C ★★★★ OHCD 6th ed. → p. 20


Infrabony defects are usually classified by the number of remaining walls
surrounding them (i.e. 0 to 3). The predictability of the regenerative
outcomes is affected by wound stability and maintaining space for the
repopulating cells. Defects that are self-​contained are much more likely
to maintain both the membrane and regenerative material in position,
and therefore provide a stable clot during the healing period. In other
words, a three-​walled defect will support a membrane or regenerative
material better than a one-​walled defect. This is most relevant when
unreinforced biodegradable membranes or gel formations of growth
factor products are utilized, as they will displace more easily. Obviously,
titanium-​reinforced membranes may negate this problem, as they are
self-​supporting, but they do require a further surgical procedure to
remove them. Furthermore, challenges can arise in producing primary
closure with tented membranes, which is another factor that can affect
the success of regenerative procedures.
Keywords: infrabony defects, regenerative surgery, predictable.
→ Cortellini P, Tonetti M. Clinical concepts for regenerative therapy in
intrabony defects. Periodontology 2000. 205;68:282–​307.

8. A ★★★★
Localized gingivitis should be carefully examined to consider all possible
aetiologies. In this case, recent crown placement is evidently the causa-
tive factor. Poor crown margins are extremely common occurrences
that should be rectified, if present. By definition, crown margins that are
intra-​sulcular have not invaded the junctional epithelium and, as such,

https://t.me/DentalBooksWorld
answers 79

have not encroached upon the supracrestal attached tissues. Excess ce-
ment and poor crown margins were not identified upon clinical examin-
ation, and allergy to adhesive resin would be extremely rare.
Poor bulbous emergence profiles can hinder oral hygiene practices and
promote plaque and biofilm development. Where insufficient reduction
has been provided, the technician has to overbulk the restoration to
create sufficient thickness of porcelain for aesthetics and strength. In this
scenario, the opaque appearance is potentially suggestive of inadequate
reduction. The veneering ceramic is thinner than required for optimal
aesthetics, and the more opaque layer becomes visible. Restorations
with this design fault could represent a plaque-​retentive factor, which
would ultimately lead to localized gingivitis.
Keywords: crowns, marginal gingivitis, 0.5 mm intra-​ sulcular, well
adapted.

9. D ★★★★
Gingival recession following orthodontic treatment is relatively common.
Camouflage of the malocclusion can occasionally result in a tooth being
positioned outside of the bony envelope, leading to dehiscence or fen-
estration. If there is associated inflammation in the tissue and the biotype
is thin, then recession can result. Thicker biotypes with a greater bulk
of connective tissue tend to result in less recession, as they are more
robust.
When aesthetics is not a major concern and the patient has no com-
plaints of sensitivity, root coverage surgery is not necessary unless the
defect is progressing or excessive. If sensitivity is present, then desensi-
tizing agents may be used to remedy this initially. The next consideration
relates to the effect of the high frenal attachment. Previous investiga-
tions demonstrated little evidence to suggest a frenal pull during mus-
cular activity has any direct effect on gingival recession. However, the
high attachment may inhibit effective plaque removal. Some clinicians
also suggest there is a plunging effect which drives plaque into the gin-
gival sulcus or an effective gingival seal is prevented, either way perpetu-
ating further local inflammation. Anecdotally, therefore, relieving the
frenal attachment can help improve local plaque control and stabilize
the situation. Alternatively, if the patient can maintain good plaque con-
trol, then surgery may not be warranted. Moreover, where there is little
sulcal depth, apical displacement of the frenum during the procedure will
help to create greater sulcal depth and facilitate any future root coverage
procedures. Some clinicians may do this as a single-​stage procedure,
combined with root coverage, depending on the situation. It should be
appreciated that treatment planning in these situations is contentious and
very scenario-​specific.
Keywords: recession, high frenal attachment, vestibular depth, shallow.
→ Allen E, Irwin, C, Ziada H, Mullally B, Byrne PJ. Periodontics 6: the
management of gingival recession. Dental Update. 2007;34:534–​42.
→ Eaton K, Ower P. Practical Periodontics. Elsevier, London; 205.

https://t.me/DentalBooksWorld
80 Chapter 4 Periodontics

20. B ★★★★ OHCD 6th ed. → p. 670


Local resolution should be attempted for all complaints received by the
practice—​verbal or written. A clear complaints protocol should be in
place, which includes the time frame for acknowledgement of receipt
of complaint and the time frame for investigation. The GDC’s stand-
ards for dental professionals state practices require a clear and effective
complaints procedure. It is good practice to acknowledge a complaint as
soon as possible, and a full response should be given within 0 working
days. If the investigation takes longer, then the patient should be updated
at least every 0 days.
Where local resolution fails, the NHS ombudsman or the Dental
Complaints Service can be utilized for NHS and private treatment,
respectively. These services may investigate the complaint and advise
actions accordingly. It is unlikely that a well-​handled complaint should
require escalation to the GDC. However, issues of patient safety, illegal
practice, or concerns regarding a registered dentist’s or dental care pro-
fessional (DCP)’s conduct are more serious matters that are more likely
to require GDC involvement.
It may be advisable to contact your indemnity provider for advice before
responding to complaints, as their guidance can help to resolve matters
faster.
Keywords: privately conducted, complaint, no solution.
→ Dental Protection. Dental Advice Series—​ Complaints Handling
(England). 206. Available at: https://​ www.dentalprotection.org/​
docs/​librariesprovider4/​dental-​advice-​booklets/​dental-​advice-​booklet-​
complaints-​handling-​england.pdf

https://t.me/DentalBooksWorld
Chapter 5 81

Endodontics
Nicholas Longridge

‘There is a lesion and I need to fix it!’


Endodontics remains a rapidly advancing branch of restorative dentistry.
It is highly likely that, by the time this book is published, several new or
updated endodontic file systems will have been released. Despite the
fairly rapid technological advances that the profession has seen, the key
principles of endodontic treatment remain the same:
. Eliminate microorganisms from the root canal system
2. Prevent reinfection of the root canal system
3. Retain a functional natural tooth.
Whilst these principles are easy to discuss, they are consistently diffi-
cult to perform, due, in large part, to the complexity of the root canal
system. Multiple theories, principles, and approaches have been discussed
to help achieve an optimal technical and clinical outcome. However, evi-
dence to favour one specific stage or system over another is lacking,
and as such, a large degree of operator preference and experience will
ultimately influence the treatment planning and technical strategy. Much
like baking a cake, endodontic treatment relies upon a series of proced-
ural steps to achieve a desirable outcome, which, for the patient, often
equates to a functional, pain-​free natural tooth.
Good-​ quality magnification remains a key component of an
endodontist’s armamentarium, and dental loupes or a dental operating
microscope could not be recommended more highly.
Key topics include:
● Endodontic case assessment, including root canal anatomy
● Pain management, including local anaesthesia
● Access and canal identification
● Vital pulp therapy, including caries management
● Canal negotiation and instrumentation
● Root canal irrigation
● Root canal obturation
● Restoration of the endodontically treated tooth.

https://t.me/DentalBooksWorld
82 CHAPTER 5 Endodontics

QUESTIONS

1. A 28-year-old man attends with a continuous aching pain from his


upper left first permanent molar (UL6). It occurs spontaneously,
disturbs sleep, and is partly relieved by paracetamol. Chewing or biting
does not exacerbate the symptoms. The heavily restored UL6 was
hyperresponsive to refrigerant (EndoFrost®). All other special investiga-
tions were normal. Which is the single most likely diagnosis? ★
A Acute apical abscess
B Atypical facial pain
C Chronic apical periodontitis
D Irreversible pulpitis
E Periodontal abscess

2. A 45-year-old man has sharp pain to cold from his upper right
posterior teeth; the pain ceases immediately on removal of the
stimulus. He has multiple cervical abrasion cavities. Which single type of
sensory nerve fibre is primarily responsible for his pain? ★
A A-β
B A-δ
C C
D Parasympathetic
E Sympathetic

3. A 29-year-old woman is about to have primary root canal treat-


ment on her lower right first permanent premolar (LR4). Figure
5.1 shows burs selected by the clinician during root canal preparation.
At which single stage of root canal preparation would it be most appro-
priate to use these? ★
A Access
B Apical preparation
C Canal identification
D Coronal flaring
E Working length determination

https://t.me/DentalBooksWorld
questions 83

Figure 5.

4. A 45-​year-​old man is having root canal treatment on his upper


left first premolar. After scouting the canal with a small hand file, a
rotary file system is introduced with an ISO 25 tip size and a 0-​degree
taper (25/​.0) until resistance is felt. Subsequently, files with a smaller
taper are introduced until the working length is reached (25/​.06 → 25/​
.04). What single concept of endodontic preparation does this tech-
nique follow? ★
A Apico-​coronal
B Circumferential
C Crown-​down
D Modified double flare
E Step back

5. A 52-​year-​old woman has a sudden onset of severe pain and an


altered sensation under her eye, following irrigation during root
canal retreatment on a maxillary canine. Which is the single most likely
complication? ★
A Extrusion of sodium hypochlorite
B Inadvertent instrumentation of the infraorbital nerve
C Lateral perforation
D Root fracture
E Transportation of apical constriction

https://t.me/DentalBooksWorld
84 Chapter 5 Endodontics

6. A 32-​year-​old man has recently completed root canal treatment


on a lower molar that is missing both marginal ridges. Which is the
single most appropriate method to restore his tooth? ★★
A A bonded intracoronal amalgam restoration
B A bonded intracoronal composite restoration
C A ceramic inlay
D A full coverage gold crown
E. A gold inlay

7. A consultant is discussing the aetiology and management of dental


caries. On his slides, he highlights a histological slice from a pro-
gressing carious lesion. Specifically, the consultant discusses the import-
ance of a translucent zone identified histologically within the dentine.
A fellow colleague asks what this zone represents. (Select one answer
from the options listed below.) ★★
A The caries-​infected zone
B The zone of hypermineralized dentine known as tubular sclerosis
C The zone of hypermineralized surface enamel under which the
carious process takes place
D The zone of sound dentine immediately adjacent to the pulp complex
E The zone of subsurface caries spread along the amelodentinal
junction

8. A foundation dentist is conducting a root canal treatment on the


lower right second premolar of a 26-​year-​old woman. Whilst irri-
gating with chlorhexidine, she notes that the concentration is only 0.2%
and asks her nurse for an alternative irrigant. The nurse goes to see the
foundation trainer to see if they can borrow their sodium hypochlorite.
The foundation trainer says that it is not advisable to use both solutions
concomitantly because it might affect overall bacterial decontamination.
What is the single main reason for this advice? ★★
A A precipitate is produced, which can block dentinal tubules from fur-
ther irrigant effects
B A reaction between the irrigants results in the production of a bac-
terial growth factor
C Adjunctive sodium hypochlorite has no clinical benefits over
chlorhexidine alone
D The combination of solutions causes selective decontamination and
overgrowth of the more pathogenic bacteria
E The residual film produced causes degradation of gutta percha, al-
lowing bacterial leakage

https://t.me/DentalBooksWorld
questions 85

9. A 7-​year-​old adolescent man is referred for root canal treatment


of an upper central incisor with an immature apex. The tooth is
non-​vital and shows comparable root length to the contralateral incisor.
Which is the single most appropriate technique to endodontically treat
his tooth? ★★
A Apexification with repeated dressings of non-​ setting calcium
hydroxide
B Apical plug formation with bioceramic material
C Cvek pulpotomy
D Endodontic surgery
E Regenerative endodontic procedures

0. A 28-​year-​old man is having an obturation of a permanent


upper central incisor. A heated plugger (e.g. System-​ B®) is
being used during the ‘downpack’. Which is the single most appropriate
description of the obturation technique being used? ★★
A Carrier-​based thermoplasticized technique
B Cold lateral condensation
C Single cone obturation
D Thermoplasticized injection technique
E Warm vertical compaction

. A 54-​year-​old man is having root canal treatment on an upper


canine, using stainless steel hand files via the modified double
flare technique. An ISO size 25 hand file is the first to bind at the working
length. What should be the single most appropriate size of the master
apical file (MAF)? ★★
A ISO size 20
B ISO size 25
C ISO size 30
D ISO size 35
E ISO size 40

https://t.me/DentalBooksWorld
86 Chapter 5 Endodontics

2. A 47-​year-​old man is having root canal treatment on his upper


first molar, which has curved roots. Following coronal prepar-
ation of the mesiobuccal canal, an electronic apex locator (EAL) is used
and provides a reproducible reading. This is 2 mm shorter than the esti-
mated working length (EWL) from the preoperative radiograph. What is
the single most appropriate initial management? ★★
A Obturate to the EWL and review
B Obturate to the EWL, followed by apicectomy
C Prepare using the reading of the EAL
D Take a new radiograph to check for potential perforations
E Take a parallax technique radiograph of the molar to identify the apex

3. A 43-​year-​old woman has pain from a lower central incisor that
has previously received root canal treatment. Radiographically,
a small, diffuse periapical lesion is present, but the obturation follows
the root anatomy and appears adequate. A parallax radiograph indicates
asymmetrical distribution of the obturation. What is the single most
likely cause of failure? ★★
A Cyst formation
B Missed canal
C Perforation
D Root fracture
E Transportation of the apex

4. A 58-​year-​old woman with failed root canal treatment is seen


in a new patient consultation clinic. A junior dental student asks
about the process of root canal failure. She is aware that certain bac-
teria have significant dentine adherence capabilities and can survive harsh
environments by demineralizing dentine and degrading collagen. Which
is the single most likely causative organism in this scenario? ★★
A Aggregatibacter actinomycetemcomitans (Aa)
B Enterococcus faecalis
C Neisseria gonorrhoeae
D Prevotella intermedia
E Streptococcus mutans

https://t.me/DentalBooksWorld
questions 87

5. A 32-​year-​old man has pain from his upper right central incisor
(UR), having previously fallen off his bicycle 2 years ago.
Radiographically, the UR apex appears moth-​eaten and is 0% shorter
in root length than the adjacent central incisor. Additionally, there is
periapical radiolucency. What is the single most likely diagnosis? ★★★
A External cervical resorption
B External inflammatory resorption
C External replacement resorption
D External surface resorption
E Transient apical breakdown

6. A 58-​year-​old woman is seen in a new patient clinic, having


been referred with a fractured rotary instrument in the upper
right first permanent molar. Radiographically, the file has fractured high
up, occluding the majority of the canal. The consultant suggests that the
use of a reciprocating file system or balanced force technique might have
prevented this. For what single primary mechanical reason is this? ★★★
A Higher rotational speeds, increasing speed of preparation
B Limits heat from friction, increasing the cyclical fatigue limit of nickel
titanium (NiTi)
C No longer requires a pecking motion of use, thus reducing flexural
fatigue
D Non-​continuous rotation prevents file binding and reduces the risk of
torsional fracture
E Off-​centre rotational pattern, facilitating debris removal

7. An 8-​year-​old man is having root canal treatment on his lower


left first permanent molar. Following access to the pulp chamber,
three canals are identified: two mesial canals—​mesiobuccal (MB) and
mesio-​lingual (ML)—​and one distal canal. The distal canal is identified as
buccally positioned by 2 mm in relation to the mesiodistal midline of the
tooth. What is the single most appropriate next step? ★★★
A Coronal flare all three canals
B Determine the working lengths
C Investigate for a fourth canal orifice
D Place a rubber dam, as access has been achieved
E Take a periapical radiograph to exclude a perforation

https://t.me/DentalBooksWorld
88 Chapter 5 Endodontics

8. A 23-​year-​old man has discomfort from his upper left first pre-
molar but is keen to save it, if at all possible. Clinically, the tooth
has deficient crown margins but otherwise appears OK. A cone beam
computed tomography (CBCT) scan shows a missed palatal canal, a
well-​obturated buccal canal, and a periapical lesion. What is the single
most appropriate management to save the tooth? ★★★
A Antibiotics, a new crown, and monitoring of the apical lesion
B Apicectomy and a new crown
C Extraction
D Obturate the palatal canal and new crown
E Root canal retreatment and new crown

9. A 36-​year-​old man is having the restoration of a recently


endodontically treated permanent molar tooth. Only the pal-
atal wall remains in its entirety, but there is 2 mm of supra-​gingival den-
tine elsewhere. What is the single most appropriate method to retain
the core? ★★★
A Multiple dentine pins
B Nayyar core
C Post-​preparation in the mesiobuccal canal
D Post-​preparation in the palatal canal
E Split post-​technique

20. A 34-​year-​old woman presents having lost an amalgam restor-


ation. The tooth was previously painful when biting. The lower
right second permanent molar is endodontically treated and has lost
a mesio-​occluso-​distal restoration; an associated deep distal probing
pocket is noted. Radiographically, an optimal root filling with furcal radio-
lucency is evident. What is the single most likely diagnosis? ★★★★
A Adhesive resin failure
B Corrosion of the amalgam
C Lute dissolution
D Split tooth
E Supra-​gingival cusp fracture

https://t.me/DentalBooksWorld
questions 89

2. An undergraduate dental student is conducting re-​root canal


treatment on a 36-​year-​old man. During the session, the clinical
tutor notices that the student is cycling between sodium hypochlorite
(NaOCl) and ethylenediaminetetraacetic acid (EDTA) and advises
against doing this. Instead they suggest just to do a penultimate rinse with
EDTA prior to obturation. What is the single main reason for changing
the irrigation protocol? ★★★★
A Collagen dissolution
B Dentine erosion
C File corrosion
D Root resorption
E NaOCl extrusion

22. A 24-​year-​old woman has returned to the United Kingdom to


get married in 4 weeks. She is interested in whitening her teeth,
whilst she prepares for her wedding. She would like to know more about
the process of tooth whitening. Which single percentage of hydrogen
peroxide is likely to give the quickest results for this patient? ★★★★
A %
B 6%
C 0%
D 6%
E 30%

https://t.me/DentalBooksWorld
90 CHAPTER 5 Endodontics

ANSWERS
1. D ★ OHCD 6th ed. → p. 222
An exaggerated response to thermal testing indicates some vitality of
the dentino-pulpal complex of the UL6. There is no complaint of ten-
derness when chewing or biting. These findings should enable you to
exclude ‘chronic apical periodontitis’ and ‘acute apical abscess’, as these
diagnoses would likely result in a negative thermal test and a tooth that is
tender to bite. Tenderness when biting, whilst uncommon, has been re-
ported with irreversible pulpitis, more commonly in multi-rooted teeth
with an ambiguous pain history, due to the presence of vital and non-vital
tissue within different root canals.
The clinical signs clearly indicate a diagnosis of acute pulpitis. Reversible
pulpitis would present with sharp pain, which is often difficult to localize.
Spontaneous pain which lasts for long periods and can wake a patient
would represent irreversible pulpitis—as in this scenario.
Atypical facial pain (or chronic idiopathic facial pain) is a diagnosis of
exclusion. It can mimic dental pain, sinusitis, and headaches. However,
with careful examination, no pathology is found. These cases must be
treated with extreme caution, as extensive unnecessary dental treat-
ment can result.
Keywords: spontaneous, wakes at night, not exacerbated by biting.

2. B ★
A-δ fibres are myelinated nerve fibres responsible for mediating the
sharp/shooting pain associated with dentine hypersensitivity and revers-
ible pulpitis. Approximately 90% of A fibres are A-δ. Whilst A-δ fibres
are the narrowest A fibres by diameter, they are significantly larger than
C fibres, and their size (along with myelination) assists with propagating
action potentials through nerves rapidly.
A variety of sensory nerve fibres exist within the human body, with each
occupying a specific role in mediating pain, temperature, and propriocep-
tion, dependent upon their anatomical composition. The dentino-pulpal
complex is innervated by A-β fibres, A-δ fibres, and C fibres, as well as
some autonomic sympathetic fibres. Unmyelinated C fibres make up the
majority of the innervation of the pulp. They have slow conduction vel-
ocities and are responsible for the aching pain associated with irrevers-
ible pulpitis. A-β fibres are also believed to be involved in nociception,
but to a lesser degree than A-δ fibres.
Parasympathetic nerve fibres within the pulp have been postulated,
whilst sympathetic nerve fibres mediate circulation.
Keywords: sharp, ceases immediately.
→ Pashley DH. Dynamics of the pulpo-dentin complex. Critical Reviews in
Oral Biology and Medicine. 1996;7:104–33.

https://t.me/DentalBooksWorld
answers 91

→ Patel S, Barnes J. Principles of Endodontics (3rd ed.). Oxford University


Press, Oxford; 209.

3. D ★ OHCD 6th ed. → p. 332


Conducted after access and canal identification, coronal flaring pro-
vides a wide range of advantages during root canal preparation,
including: improving access to the apical region; removal of infected
tissue; improving irrigant placement; and improved accuracy in assessing
the working length. It also reduces file separation by decreasing the stress
placed upon files when preparing the apical portion of the root canal.
Gates-​Glidden burs have non-​cutting tips and are available in a range of
sizes. Scouting of the canal with some initial hand file flaring can facili-
tate their introduction. They are not flexible and can lead to endodontic
misadventure if used incorrectly, especially in curved canals. Many nickel
titanium rotary systems have specific files, or shapers, designed for a
similar purpose, which can be more conservative of radicular dentine.
Keywords: root canal preparation, Gates-​Glidden bur (pictured).
→ Darcey J, Taylor C, Roudsari RV, Jawad S, Hunter M. Modern endodontic
principles Part 3: preparation. Dental Update. 205;42:80–​22.

4. C ★ OHCD 6th ed. → p. 338


Multiple chemomechanical preparation techniques have been proposed
and utilized throughout the years. A large majority of modern rotary
systems have adopted a crown-​down approach for preparation, which
prepares in a stepwise manner from the canal orifice to the apical limit.
This is reported to increase access of the irrigant into the apical region
of the root canal, eliminate heavily contaminated coronal dentine, assist
with straight line access, and reduce instrument separation.
The procedure above describes the technique for a standard taper
file system, but others are available which can have variable taper. It is
important to understand the file system being used, to avoid iatrogenic
damage and to ensure an adequate glide path is created with hand files
prior to using automated systems.
Keywords: endodontic preparation, rotary endodontics.
→ Darcey J, Taylor C, Roudsari RV, Jawad S, Hunter M. Modern endodontic
principles Part 3: preparation. Dental Update. 205;42:80–​22.

5. A ★
Sodium hypochlorite’s ability to dissolve organic tissue is beneficial in
endodontics. However, apical extrusion of irrigant can cause significant
soft tissue damage. This is an uncommon occurrence, particularly if care
is taken, as irrigant generally only reaches –​2 mm beyond the needle
tip. Safety precautions to prevent extrusion include: using a side-​vented
needle, gentle irrigation pressure, and not taking the irrigating needle tip
closer than 2–​3 mm from the working length.

https://t.me/DentalBooksWorld
92 Chapter 5 Endodontics

Pain and swelling are the most notable symptoms of a hypochlorite


accident. Additional signs such as dysaesthesia or nosebleeds can relate
to local anatomy. Management of hypochlorite injuries depends on
the severity, but in general, immediate management includes: stopping
treatment, removing excess irrigant with paper points or aspirating
using an empty syringe, dressing with non-​setting calcium hydroxide,
and temporizing. Irrigation with saline to try and dilute the hypochlorite
is not advised, due to the risk of further extrusion of residual hypo-
chlorite. Pain relief, including a non-​steroidal anti-​inflammatory drug (if
acceptable), should be prescribed, in conjunction with cold compress
advice; an early review should then be arranged. For more severe
symptoms, liaison with the local maxillofacial unit is advisable, since
steroids, antibiotics, and close monitoring of any swelling progression
will be required.
Clinicians should be aware of their local policy for managing hypo-
chlorite accidents.
Keywords: severe pain, altered sensation.
→ Baldwin VE, Jarad FD, Balmer C, et al. Inadvertent injection of sodium
hypochlorite into the periradicular tissues during root canal treatment.
Dental Update. 2009;36:4–​9.
→ Farook S, Shah V, Lenouvel D, et al. Guidelines for management
of sodium hypochlorite extrusion injuries. British Dental Journal.
204;27:679–​84.

6. D ★★ OHCD 6th ed. → p. 346


Following root canal treatment, teeth that have lost a supporting mar-
ginal ridge are at higher risk of fracture. There are multiple proposed
reasons for this. Firstly, in non-​vital teeth, changes to the physical prop-
erties of the dentine can occur by chemical degradation with irrigants
and medicaments. Secondly, there is potential for a loss of sensory
feedback from root-​filled teeth, which can result in higher occlusal
forces being transmitted through the tooth. Thirdly, the loss of tooth
structure from access cavities or decay (particularly if a marginal ridge
is involved), combined with the lost architecture of the pulp chamber
roof, can drastically decrease the stiffness of the tooth. This last reason
is likely to be the main reason endodontically treated teeth are more
prone to fracture.
For these reasons, provision of cuspal coverage restoration has been
suggested. A recent study, over a 5-​year period, demonstrated lower
survival rates for root-​filled posterior teeth without cuspal coverage res-
toration. Modern dentistry has a focus on minimally invasive procedures,
and with better bonding systems, many practitioners are now prefer-
ring to prescribe onlay restorations, rather than full coverage crowns;
either is acceptable, assuming cuspal coverage is provided. Gold remains
the most conservative and biocompatible material for posterior teeth.
However, the demand for alternative aesthetic restorations has driven
an increase in all-​ ceramic onlay restorations, e.g. monolithic lithium
disilicate. Clinically, these restorations require greater preparation.
Keywords: root canal treatment, missing both marginal ridges.
https://t.me/DentalBooksWorld
answers 93

→ MacInnes A, Hall AF. Indications for cuspal coverage. Dental Update.


206;43:50–​8.
→ Tickle M, Milsom K, Qualtrough A, Blinkhorn F, Aggarwal VR. The
failure rate of NHS funded molar endodontic treatment de­livered in
general dental practice. British Dental Journal. 2008;204:254–​5.

7. B ★★
The carious process, which develops when a bacteria-​rich plaque bio-
film, fermentable carbohydrates, and a susceptible tooth surface interact
over time, is well described in the literature. Histological examination
of this process has identified distinct zones which display differing char-
acteristics. In dentine, these are, from outside in: zone of destruction,
zone of bacterial invasion, zone of demineralization, and zone of scler-
osis (or translucent zone). The translucent zone represents a reparative
process, in which odontoblasts deposit dentine in the tubules, as the
tooth attempts to reduce insult from the approaching carious process;
this is known as tubular sclerosis, which appears translucent under light
microscopy because of the reduced light refraction.
Dentine immediately adjacent to the pulp will be the last to undergo
the destructive changes of the carious process. It is here that tertiary
dentine is produced in response to carious insult. See Figure 5.2 which
shows histological changes in enamel and dentine before cavitation of
enamel.
Keywords: carious lesion, translucent zone.
→ Kidd E, Fejerskov O. Essentials of Dental Caries (4th ed.). Oxford
University Press, Oxford; 206.

S
Body
DZ
TZ
Dead tract
Translucent zone

Normal
Reactionary
dentine

Figure 5.2
Reproduced from Kidd, E. Essentials of Dental Caries (3rd Ed). Figure 2.2., page 3.
Oxford University Press. Oxford. 206 by permission of Oxford University Press.
https://t.me/DentalBooksWorld
94 Chapter 5 Endodontics

8. A ★★ OHCD 6th ed. → p. 334


Sodium hypochlorite is considered the gold standard endodontic irrigant.
It is relatively cheap and bactericidal and dissolves organic material. Some
practitioners prefer to use alternatives (e.g. chlorhexidine), as they do
not produce the side effects associated with apical extrusion of sodium
hypochlorite. Additionally, chlorhexidine also has fungicidal properties
and demonstrates substantivity for up to 2 weeks. It is important to
note that concentrations of chlorhexidine below 2% are only bacterio-
static (most over-​ the-​
counter mouthwashes have concentrations of
0.2%). In certain situations, clinicians may wish to use multiple irrigants
by using paper points or saline solutions to prevent cross-​contamination.
However, concurrent use of chlorhexidine and sodium hypochlorite
produces a brown precipitate (parachloroaniline), which is thought to be
carcinogenic. Furthermore, although the precipitate may not cause phys-
ical blockage or prevention of instruments from reaching the working
length, its presence can occlude dentinal tubules, preventing penetration
by further irrigants or sealers.
The other answers are incorrect; the by-​product is not known to influ-
ence gutta percha stability or act as a growth factor, and the two solu-
tions do not case selective decontamination. Sodium hypochlorite may
also have clinical benefits beyond that of chlorhexidine alone.
Keywords: irrigants, chlorhexidine, sodium hypochlorite, not in
conjunction.
→ Darcey J, Jawad S, Taylor C, Roudsari RV, Hunter M. Modern
endodontic principles Part 4: irrigation. Dental Update. 206;43:20–​33.

9. B ★★ OHCD 6th ed. → p. 0


Endodontic management of immature apices can be challenging, mainly
due to the wide apical foramina. Thin walls, potential inverse wall
tapers, and a lack of apical stop make mechanical debridement and ob-
turation fraught with difficulty and at greater risk of clinical complica-
tions. Previously, repeated changes of non-​setting calcium hydroxide
were required to try and induce a hard tissue barrier (apexification).
However, this takes considerable time (between 5 and 20 months),
and evidence suggests prolonged exposure to calcium hydroxide can
weaken dentine, predisposing to root fracture. Although an accept-
able technique, newer materials, like mineral trioxide aggregate (MTA),
allow the formation of an apical barrier in one visit, facilitating backfilling
with thermoplasticized gutta percha. This technique reduces the risk
of root fracture and clinical time, alongside improving patient compli-
ance as fewer appointments are required. Conversely, MTA can cause
discoloration of the tooth over time and some form of bleaching may
be required. Alternative bioceramics with reduced risk of discoloration
are also available.
Current research is exploring regenerative endodontic procedures with
early results reporting similar healing outcomes to apical plug tech-
niques. However, whilst research does demonstrate continued root

https://t.me/DentalBooksWorld
answers 95

development with this treatment strategy, it remains experimental and


is currently not the treatment of choice in a tooth with sufficient root
length.
Keywords: immature apex, comparable root length.
→ Bakland LK, Andreasen JO. Will mineral trioxide aggregate replace
calcium hydroxide in treating pulpal and periodontal healing compli-
cations subsequent to dental trauma? A review. Dental Traumatology.
202;28:25–​32.
→ Simon S, Smith AJ. Regenerative endodontics. British Dental Journal.
204;26:E3.

0. E ★★ OHCD 6th ed. → p. 342


A variety of techniques exist for obturation. The aim of obturation is
to produce a three-​dimensional hermetic seal of the root canal system,
without voids, extending to the apex. The most commonly used material
is gutta percha (GP), a rubber-​based material, although alternative plastic-​
based materials are available (e.g. Resilon®). Cold lateral condensation,
which involves using a finger spread to compact a master point sideways
and allow space for accessory points, was traditionally favoured.
In modern dentistry, more people are using matched-​point single cone
obturation or plasticized GP obturation techniques. During warm ver-
tical compaction, a heated plugger is inserted through the master GP
cone into the canal to within 5 mm of the working length, creating an
apical plug. Following this, either further segments of GP are compressed
down under heat or, alternatively, plasticized GP is injected in to ‘backfill’
the remaining space.
Carrier-​based systems have a central core (plastic or cross-​linked GP)
with a GP coating. The point is heated in a special oven and inserted into
the canal. In theory, thermoplastic techniques should produce better
three-​dimensional obturation. However, literature to support improved
outcomes is lacking.
Keywords: heated plugger, down-​pack, obturation.
→ Darcey J, Roudsari RV, Jawad S, Taylor C, Hunter M. Modern endodontic
principles Part 5: obturation. Dental Update. 206;43:4–​29.
→ Peng L, Ye L, Tan H, Zhou X. Outcome of root canal obturation by
warm gutta-​percha versus cold lateral condensation: a meta-​analysis.
Journal of Endodontics. 2007;33:06–​9.

. D ★★
Mechanical preparation aims to shape the canal to facilitate irrigant
exchange and obturation. Adequate preparation should enable the
irrigant to penetrate to the apical third of the canal and will secondarily
remove some infected dentine. Techniques focus on tapering either
from the ‘crown-​down’ or apico-​coronally in a ‘stepback’ approach.
The modified double flare technique is used with 2% ISO hand files
and combines both approaches. Firstly, the coronal two-​thirds are

https://t.me/DentalBooksWorld
96 Chapter 5 Endodontics

prepared in a crown-​down fashion. This has the benefits of removing


infected coronal dentine and greater irrigant access. Subsequently, the
apical third is prepared in a stepback fashion. The first file to bind at the
working length ‘gauges’ the size of the apical constriction. Next, the
file size is increased, most commonly by two sizes, to create a stop at
the working length. From here, the apical portion is flared backwards
by increasing the file size and reducing the length reached by  mm
each time, until it meets the coronal two-​thirds. The largest-​size file to
reach working length is known as the master apical file (MAF). In this
instance, as a 25 K file binds at length, the MAF would be a 35 K file. It
is important to understand the MAF principle, as successful obturation
relies upon selecting the correct master apical cone to correspond with
the MAF.
Keywords: modified double flare technique, master apical file.

2. C ★★ OHCD 6th ed. → p. 338


The debate on the apical limit of root canal preparation is contentious.
The minor apical foramen (apical constriction) is generally considered
to be the apical limit. On average, it lies 0.5 mm short of the major
apical foramen, but there is considerable variation in the relationship of
the radiographical apex to the major apical foramen. This is particularly
true in posterior teeth where discrepancies of up to 4 mm have been
reported. Modern EALs measure changes in impedance ratios and can
accurately identify contact with the apical tissues; they have been shown
to identify the major apical foramen more reliably than radiographs.
Therefore, in this instance, since the reading is reliable, reproducible, and
within an acceptable distance from the estimated length, the EAL reading
should be used. If there were concern regarding the measurement given
by the EAL, then this can be evaluated against a working length radio-
graph to make a definitive decision. It is advisable to assess the working
length in some form with at least one radiograph at some stage prior to
obturation.
Keywords: electronic apex locator, reproducible reading.
→ Gordon M, Chandler N. Electronic apex locators. International
Endodontic Journal. 2004;37:425–​37.
→ Martins JN, Marques D, Mata A, Caramês J. Clinical efficacy of
electronic apex locators: systematic review. Journal of Endodontics.
204;40:759–​77.

3. B ★★ OHCD 6th ed. → p. 348


Endodontic failures are common. The European Society of Endodontics
have classified success as the ‘absence of pain, swelling, and other symp-
toms, no sinus tract, no loss of function and radiological evidence of
a normal periodontal ligament space’. Success rates vary, depending
on a number of variables, but the key factors linked with success are
homogenous root filling without voids, obturation to within 2 mm of
the apex, absence of periapical lesion preoperatively, and a satisfactory

https://t.me/DentalBooksWorld
answers 97

coronal restoration. Mandibular incisors most commonly have one canal.


However, studies have suggested up to 45% of cases may have more
canals in some form, the anatomy of which is often complex and can
frequently coalesce, making debridement and obturation challenging;
Vertucci’s classification is commonly quoted. Parallax views or CBCT
can be utilized to help identify missed anatomy. Management of these
situations requires good access, good visualization, and patience!
Keywords: lower central incisor, asymmetric distribution of obturation.
→ Kartal N, Yanikoglu FC. Root canal morphology of mandibular inci-
sors. Journal of Endodontics. 992;34:40–​5.
→ Ng Y, Mann V, Rahbaran S, Lewsey J, Gulabivala K. Outcome of
primary root canal treatment: systematic review of the literature—​
Part 2. Influence of clinical factors. International Endodontic Journal.
2008;4:6–​3.
→ Vertucci FJ. Root canal anatomy of the human permanent teeth. Oral
Surgery, Oral Medicine, Oral Pathology. 984;58:589–​99.

4. B ★★
Multiple bacteria have been isolated in failed endodontic treatment.
However, E. faecalis is most frequently isolated in asymptomatic teeth
with long-​standing periapical infections. Its small size allows it to reside
within dentinal tubules, and it can survive long periods of starvation.
Furthermore, its ability to maintain and regulate its intracellular pH en-
ables it to withstand prolonged periods in strong alkali conditions (e.g.
those created by calcium hydroxide).
Numerous studies have looked into the irrigant effect on E. faecalis eradi-
cation. Sodium hypochlorite above 3% is effective if used in adequate
quantities and furthermore can penetrate biofilms. Chlorhexidine at 2%
concentrations is also effective and has the added benefit of substantivity.
A newer irrigant MTAD [a Mixture of a Tetracycline (doxycycline), citric
Acid, and a Detergent) also shows promise in being effective against
a wide number of endodontic pathogens. It is, however, important to
ensure suitable apical enlargement and adequate smear layer removal
prior to final irrigation regimes, so irrigants can access dentinal tubules
where the bacteria reside.
Additionally, as calcium hydroxide is ineffective in eliminating E.
faecalis, iodine or iodoform-​based intracanal medicaments are often
recommended.
Aa is commonly associated with aggressive periodontal disease; P. inter-
media is again more commonly associated with periodontal disease but
does have a role in endodontic infections. S. mutans is associated with
caries, and N. gonorrhoeae is the infective agent in gonorrhoea.
Keywords: harsh environments, failed root canal treatments.
→ Stuart C, Schwartz SA, Beeson TJ, Owatz CB. Enterococcus faecalis: its
role in root canal treatment failure and current concepts in retreatment.
Journal of Endodontics. 2006;32:93–​8.

https://t.me/DentalBooksWorld
98 Chapter 5 Endodontics

5. B ★★★ OHCD 6th ed. → p. 0


When classified simplistically, root resorption is classified by site
(internal or external) and type (surface, inflammatory, or replacement).
Additionally, resorption at the cervical margin is known as external cer-
vical resorption. Resorption occurs as a result of stimulation of osteo-
clasts/​odontoclasts, either via an inflammatory pathway or dysregulation
of normal homeostatic bone turnover. The protective mechanisms in
healthy teeth are thought to be: a vital periodontal ligament, an intact
cementum layer, and an extracellular pre-​dentine layer internally.
Internal resorption is rare and usually inflammatory in nature. Well-​
defined ballooning of the canal system occurs, which may be visible as a
pink hue if it occurs in the coronal aspect.
Surface resorption results from localized damage to the root surface;
frequently, it is subclinical and self-​ limiting. Prolonged inflammatory
stimuli, around a damaged root surface, can lead to external inflam-
matory resorption and is common after trauma. The root is gradually
destroyed, which can occur surprisingly quickly. Alternatively, if union
occurs between bone and dentine, normal bony remodelling can occur,
leading to replacement resorption.
In this case, the moth-​eaten appearance, periapical lesion, and history
should lead you to the diagnosis of external inflammatory resorption;
root canal treatment should be instigated immediately.
Keywords: fallen, 2 years ago, moth-​eaten, periapical radiolucency.
→ Darcey J, Qualthrough A. Resorption: part . Pathology, classification
and aetiology. British Dental Journal. 203;24:439–​5.

6. D ★★★
The development of NiTi rotary file systems has improved the effi-
ciency and ability to prepare the root canal system, even in the most
challenging of cases. In traditional rotary motors, the file motion is
a continuous 360° rotation. If the file binds to the walls of the root
canal, then torsional stress can occur within the file, causing deform-
ation along the long axis. Should this deformation exceed the elastic
limit of the material, then the instrument will fracture. In torsional
fatigue, the file will fracture at the junction between where the file is
bound and not bound, as the force will concentrate here. Conversely,
cyclical fatigue fractures occur at the point of maximum canal curva-
ture. Safeguards, such as torque limits and auto-​ reverse within
endodontic motors, are designed to stop torsional fatigue. However,
fractures can still occur, as average settings do not conform to the
environment of each canal.
Asymmetric reciprocation is effectively a mechanized version of the
balanced force technique (rotating approximately 50° clockwise, fol-
lowed by 30° anticlockwise). This repeated disengagement limits the
build-​up of torsional stress and reduces the number of rotations, thus
reducing instrument fracture. This design feature allows the develop-
ment of a single file system.

https://t.me/DentalBooksWorld
answers 99

Additional benefits of the single file system include: improved cost-​


effectiveness (due to fewer files used), simplified protocols (again fewer
files), and reduced cyclical fatigue because of the lower rotational speeds
required of the files.
Keywords: fractured high up, reciprocating file system.
→ Grande NM, Ahmed HM, Cohen S, Bukiet F, Plotino G. Current as-
sessment of feciprocation in endodontic preparation: a comprehensive
review–​Part I: historic perspectives and current applications. Journal of
Endodontics. 205;4:778–​83.

7. C ★★★ OHCD 6th ed. → p. 336


Commonly, mandibular molars have two roots with three canals: two
mesial canals—​mesiobuccal and mesio-​lingual—​and one distal canal.
However, it is not uncommon to have two or three distal canals
(approximately 35% have two).
Canal identification is often feared most by junior practitioners, and
the use of magnification and appropriate lighting cannot be underesti-
mated. There are several rules for assisting in the identification of ca-
nals. Firstly, the cemento-​enamel junction (CEJ) is considered to be
the most reliable anatomical predictor for the positioning of the pulp
chamber and pulpal floor. The CEJ is often protected from coronal
restorations and, as such, can assist greatly when teeth are heavily
restored.
Canals often display a high degree of symmetry (excluding the maxillary
molars), and the mesiodistal midline (running through the mid point of
the pulp chamber, from mesial to distal) can often be used to assist in
canal identification. A distal canal 2 mm buccally to the mesiodistal mid-
line is highly suggestive of a fourth distolingual canal. See Figure 5.3 which

Figure 5.3

https://t.me/DentalBooksWorld
100 Chapter 5 Endodontics

shows a diagrammatic representation of the law of symmetry in canal


location. The image on the left shows the distal canal to be in the midline,
whereas the image on the right shows one distal canal off to the left, indi-
cating the likelihood of a second canal in the hazed region.
Keywords: buccally positioned, mesiodistal midline.
→ de Pablo OV, Estevez R, Péix Sánchez M, Heilborn C, Cohenca N. Root
anatomy and canal configuration of the permanent mandibular first
molar: a systematic review. Journal of Endodontics. 200;36:99–​3.
→ Krasner P, Rankow H. Anatomy of the pulp-​chamber floor. Journal of
Endodontics. 2004;30:5–​6.

8. E ★★★ OHCD 6th ed. → p. 348


The long-​term management of any endodontic infection should be with
local treatment, and not antibiotics. Given the patient is in pain and
wishes to save the tooth, then treatment is required and only options
B, D, and E are viable.
Coronal leakage plays a significant part in the success of endodontic
treatment, with nearly twice the chance of failure in the presence of
an unsatisfactory restoration. Furthermore, a meta-​analysis suggested
that retreatment provides a better long-​term outcome than surgical
endodontics. Therefore, in these situations, endodontic retreatment is
preferred in the first instance. The decision becomes more complicated
when the original root filling appears satisfactory. If the clinician is con-
fident that all the intraoperative components have been optimized (i.e.
thorough chemo-​mechanical disinfection, use of dental dam) or the cor-
onal restoration would be challenging to remove, then apical surgery
may be warranted. In the above scenario, the treatment is suboptimal
and the coronal seal has been lost; therefore, retreatment is recom-
mended with replacement coronal restoration. Solely obturating the
missed canal would not be advised in this situation, as the buccal canal is
likely to be contaminated.
Keywords: deficient crown margin, missed palatal canal.
→ Ng YL, Mann V, Rahbaran S, Lewsey J, Gulabivala K. Outcome of
primary root canal treatment: systematic review of the literature—​
Part 2. Influence of clinical factors. International Endodontic Journal.
2008;4:6–​3.
→ Torabinejad M, Corr R, Handysides R, Shabahang S. Outcomes of
nonsurgical retreatment and endodontic surgery: a systematic review.
Journal of Endodontics. 2009;35:930–​7.

9. B ★★★ OHCD 6th ed. → p. 346


Definitive restoration of teeth should be considered prior to com-
mencing endodontic treatment. There is no merit in root-​treating an
unrestorable tooth, and, as such, restorability should be assessed first.
The decision on how to retain a core is based upon the clinician’s
judgement of the clinical scenario. Post-​preparation within molar teeth

https://t.me/DentalBooksWorld
answers 101

is feasible and can provide adequate retention for core materials.


However, complications from post-​preparations can occur, including
root fracture and perforation, making them less retrievable long term.
Furthermore, potentially damaging stress accumulation can occur with
dentine pin use. The Nayyar core utilizes inherent undercuts and diver-
gences within the root canal system for retention, although a certain
volume of coronal dentine is still considered necessary. The coronal 2–​
4 mm of gutta percha is removed from each canal, and an appropriate
restorative material placed to create a monoblock corono-​radicular
core. Amalgam is traditionally used, but composite is a suitable alterna-
tive. The authors would advocate avoiding post-​retained cores in molar
teeth where possible—​sufficient retention with an adhesive Nayyar core
can be gained in most scenarios.
Adhesive dentistry has added an additional treatment approach through
the use of endo-​crowns and bonded onlays. However, Nayyar core-​
style restorations continue to be the most widely accepted treatment
modality in this scenario.
Keywords: reduced coronal tooth tissue, root-​treated molar tooth.
→ Nayyar A, Walton RE, Leonard LA. An amalgam coronal-​radicular
dowel and core technique for endodontically treated posterior teeth.
Journal of Prosthetic Dentistry. 980;43:5–​5.

20. D ★★★★ OHCD 6th ed. → p. 02


The diagnosis of a vertical root fracture can be challenging. Symptoms
tend to be vague and inconsistent with descriptions of sharp pain on
biting and ‘on/​off ’ dull pains are frequently reported. Careful correl-
ation of symptoms to the clinical signs is important, especially in the
context of the biology. Clinically, after excluding other obvious causes
of pathology, investigation for indicators, such as obvious fracture
lines (with or without flexion), isolated deep probing depths, or pain
on release of biting with a tooth sleuth, should be conducted. Crack
identification with plaque-​disclosing dye or transillumination has been
suggested.
Radiographically, the presence of ‘j’-​shaped lesions, vertical bony de-
fects, or lateral/​furcal radiolucencies may indicate some form of longi-
tudinal tooth fracture. Fractures confined to the root can be diagnosed
as ‘vertical root fractures’, whilst fractures including the root and the
crown are diagnosed as a ‘split tooth’. Additionally, in this case, loss of
the restoration without loss of the cusps is unusual, and thought should
be given to the reason for loss of retention—​potentially from flexion of
the cusps. The furcal radiolucency and deep probing depth help differ-
entiate in this scenario.
Keywords: pain on biting, endodontically treated, isolated probing depth,
lost restoration.
→ Banerji S, Mehta SB, Millar BJ. Cracked tooth syndrome. Part : aeti-
ology and diagnosis. British Dental Journal. 200;208:459–​63.

https://t.me/DentalBooksWorld
102 Chapter 5 Endodontics

2. B ★★★★ OHCD 6th ed. → p. 334


NaOCl is a strongly antibacterial irrigant with organic tissue-​dissolving
qualities. EDTA is a weakly antibacterial, chelating agent that facilitates
smear layer removal. The synergistic effects of combining these two
irrigants have been widely researched. However, prolonged or repeated
use of EDTA has been shown to cause dentine erosion (through chela-
tion of the mineral component; NaOCl will dissolve the organic com-
ponent), and this practice should be avoided. Additionally, the mixing of
EDTA with NaOCl reduces the availability of free chloride (Cl–​) ions, and
in vitro studies have demonstrated reduced bactericidal properties. For
this reason, a single application of EDTA 7% solution for approximately
 minute, after mechanical preparation has been completed, has been
advocated. Many endodontists choose to follow this with one final rinse
with NaOCl in an attempt to penetrate into the patent dentinal tubules.
However, this remains controversial, as many argue the final rinse with
NaOCl will have a more profound effect upon the degree of dentine
erosion that occurs. Dentine erosion may have profound effects upon
the strength of the remaining root canal wall.
Some clinicians suggest the use of alternative mild chelators such as
-​hydroxyethylidene-​ ,-​bisphosphonate [HEBP (7%)] to avoid such
problems, but smear layer removal is reduced and less clinical outcome
data are available to support their efficacy.
Keywords: irrigation, cycling between, NaOCl, EDTA.
→ Bystrom A, Sundqvist G. The antibacterial action of sodium hypo-
chlorite and EDTA in 60 cases of endodontic therapy. International
Endodontic Journal. 985;8:35–​40.
→ Haapasalo M, Qian W, Shen Y. Irrigation: beyond the smear layer.
Endodontic Topics. 202;27:35–​53.
→ Zehnder M, Schmidlin P, Sener B, Waltimo T. Chelation in root canal
therapy reconsidered. Journal of Endodontics. 2005;3:87–​20.

22. B ★★★★ OHCD 6th ed. → p. 248


In October 202, the law relating to tooth whitening was amended, fol-
lowing changes to European Union law (EU Council Directive 20/​84/​
EU). As a result, bleaching products that contain or release up to 6%
hydrogen peroxide can be prescribed.
Whilst concentrations in excess of 6% hydrogen peroxide are utilized
worldwide, in the United Kingdom, concentrations in excess of 6% are
not allowed for cosmetic purposes only. Further advice should be sought
if concentrations in excess of 6% are deemed necessary to manage dis-
ease, but this is rarely the case, as the literature does not report superior
outcomes with higher concentrations. Standards of safety must be
complied with, and patients must be 8 years or above for cosmetic
whitening procedures.
It is important that close attention is paid to the type of product
contained within each bleach, as carbamide peroxide is frequently

https://t.me/DentalBooksWorld
answers 103

encountered. Carbamide peroxide 6% would release <6% if hydrogen


peroxide and is therefore safe for use.
Keywords: United Kingdom, tooth whitening, hydrogen peroxide.
→ Sulieman M, Addy M, MacDonald E, Rees JS. The effect of hydrogen
peroxide concentration on the outcome of tooth whitening: an in vitro
study. Journal of Dentistry. 2004;32:295–​9.

https://t.me/DentalBooksWorld
https://t.me/DentalBooksWorld
Chapter 6 105

Prosthodontics
Peter Clarke

‘I want one of them screw in ones!’


Prosthodontics comprises most of the routine restorative treatments
that practitioners perform on a daily basis. Much restorative work re-
sults from the impact of caries and periodontal disease. However, the
prevalence of toothwear is dramatically increasing and can be expected
to form a more prominent feature of the modern practitioner’s work-
load. There is a considerable theory base in prosthodontics, covering
all aspects of fixed and removable treatments, both conventional and
contemporary.
Although the individual management of teeth can be tricky, a challenge
many new practitioners struggle with is treatment planning on a patient
level. Treatment planning is rarely black and white, with considerable
variations in opinion among clinicians, even for more simple cases. The
staging of treatment planning is fairly consistent across the profession
(e.g. relief of pain first, then investigatory phase, etc.), but in complex
cases, a second opinion may be warranted. Not only is treatment plan-
ning a difficult skill, but so is the execution. It takes practice to become
adept at the variety of clinical skills in prosthodontics and the staging of
treatment, but this makes for a rewarding and fascinating discipline.
Modern dentistry has a much greater focus on minimal invasive treat-
ment, relying on dentine bonding and adhesive dentistry to limit the need
for aggressive preparations of teeth and protect the vitality of the pulp.
Moreover, the progression in digital dentistry is exponential, with newer
production methods and clinical techniques becoming increasingly
accurate and ever more accessible. As such, the modern practitioner
needs to have a good understanding of both conventional concepts and
modern alternatives in order to be able to apply the material and tech-
nique of choice to achieve an optimal outcome.
The questions in the chapter aim to cover a wide range of topics,
testing conventional concepts in both fixed and removable prostho-
dontics, whilst touching on contemporary materials and production
methods. It is hoped that the reader will be challenged and the more
difficult questions will promote wider reading.
Key topics include:
● Diagnosis and treatment planning
● Occlusion
● Toothwear
● Complete dentures
● Removable dentures (including denture design principles)

https://t.me/DentalBooksWorld
106 Chapter 6 Prosthodontics
● Direct restorations
● Crown and bridge
● Implant restorations
● Laboratory processes
● Digital dentistry.

https://t.me/DentalBooksWorld
questions 107

QUESTIONS

. A 47-​year-​old woman is undergoing planning for a full mouth


rehabilitation. Before taking the interocclusal record, the clinician
manipulates her into retruded contact position (RCP). Which is the
single most appropriate description of this position? ★
A The initial tooth contact following the rotational movement of the
mandible
B The initial tooth contact whilst closing around the terminal hinge axis
C The only clinically reproducible initial tooth contacts
D The position of the condyle whilst closing in the habitual position
E The position of the condyle whilst in its most relaxed functional pos-
ition during closure

2. A 79-​year-​old edentulous man has a loose upper complete den-


ture. After the primary impressions are taken, the dentist explains
that more impressions will be taken at the next appointment. The patient
asks why this is necessary. What is the single main reason for the second
impression? ★
A To accurately record soft tissue detail to improve the denture’s
adaption
B To correct faults in the primary impression
C To identify the path of insertion of the dentures
D To outline to the technician the neutral zone for tooth placement
E To record and define the full functional limits of the
denture-​bearing area

3. A 64-​year-​old woman is treatment-​planned to have an imme-


diate maxillary complete denture following a full clearance. Prior
to the clearance, she is warned that the dentures will need remaking in
6 months to  year. Which is the single most appropriate rationale for
this warning? ★
A Bony remodelling will result in occlusal irregularities
B Bony remodelling will result in overextended dentures
C Bony remodelling will result in poor denture adaptation
D Bony remodelling will result in poor tooth positioning
E The patient might complain if they are unaware they will have to
pay again

https://t.me/DentalBooksWorld
108 Chapter 6 Prosthodontics

4. An 87-​year-​old edentulous woman has worn dentures and is re-


questing a new set. As she has no complaints with her current set
and has worn them for 5 years, it is decided to provide a new set via
the copy technique. Which surfaces of the dentures can be changed
easily using this technique? (Select one answer from the options listed
below.) ★
A Fitting and occlusal surfaces
B Fitting and polished surfaces
C Occlusal and polished surfaces
D Occlusal surfaces only
E Polished surfaces only

5. An 80-​year-​old man attends his jaw registration appointment. He


is happy with the appearance and position of the anterior teeth on
his old maxillary denture, so it is decided to copy this to the new denture.
Which is the single most appropriate device to aid the clinician in doing
this at the chair side? ★
A Alma gauge
B Fox’s occlusal plane guide
C Iwanson gauge
D Steel ruler
E Willis bite gauge

6. A 45-​year-​old woman is having a new cobalt-​chrome partial den-


ture made. The primary models have been surveyed and returned
with the special trays. She has a lone-​standing upper right second molar
that would make a useful tooth for direct retention with a cast cobalt-​
chrome clasp. What is the single most appropriate clasp length and
undercut required for this type of clasp? ★
A 0.25 mm and 8 mm
B 0.25 mm and 5 mm
C 0.5 mm and 8 mm
D 0.5 mm and 2 mm
E 0.5 mm and 4 mm

https://t.me/DentalBooksWorld
questions 109

7. A 9-​year-​old woman is referred, following completion of her


orthodontic treatment for the replacement of her missing lateral
incisors. Medically she is fit and well and has a minimally restored den-
tition. There is insufficient interradicular space for dental implants, and
the patient is advised to have adhesive bridgework over conventional
bridgework. What is the single main reason for this advice? ★
A Aesthetic
B Cheap to produce
C Ease of construction
D Low biological cost
E Reduced clinical time

8. A 36-​year-​old man attends an emergency appointment with a


decoronated upper right second premolar. The tooth has been
previously root-​treated, but only  mm of supra-​gingival tooth tissue re-
mains. It is decided that a post crown is required, but before continuing,
the patient is warned the prognosis is guarded and that there is a risk of
root fracture long term. What single prognostic feature may increase the
risk of root fracture in this scenario? ★
A Limited ferrule
B Long post length
C Narrow post width
D Shallow retention grooves
E Unfavourable crown–​root ratio

9. A 35-​year-​ old man attends the practice for the first time.
A number of diagnoses are noted. Which single diagnosis should
be addressed first? (Select one answer from the options listed below.) ★
A Acute periapical periodontitis LR5
B Chronic periapical abscess LL6
C Clasp fracture of lower denture
D Generalized plaque-​induced gingivitis
E Occlusal caries UR67 and UL6

https://t.me/DentalBooksWorld
110 Chapter 6 Prosthodontics

0. A 8-​year-​old man attends with an enamel dentine fracture of


his upper right maxillary central incisor. He is treatment-​planned
to have a composite restoration placed to restore the tooth, and a
rubber dam is going to be used. When is the single most appropriate
time to take the shade of the composite? ★
A After bonding the tooth
B After etching the tooth
C After tooth preparation, e.g. bevelling
D At the beginning of the appointment
E Immediately after isolation with a rubber dam

. A 50-​year-​old man has recently had his maxillary incisors ex-
tracted following blunt trauma to the face. The rest of his den-
tition is present and sound. The treatment plan includes a cobalt-​chrome
denture to restore the space until implants can be considered. Which
single Kennedy classification does this situation represent? ★
A Class II
B Class II mod 2
C Class III mod 2
D Class IV
E Class IV mod 2

2. A 72-​year-​old man has just had a set of complete dentures de-
livered. The articulation is checked, and adjustments are made
to ensure bilateral balanced articulation. Which cusps should be adjusted
to prevent unwanted change to the occlusal vertical dimension (OVD)?
(Select one option from the options below.) ★★
A Buccal upper, lingual lower
B Distal upper, mesial lower
C Lingual upper, buccal lower
D Mesial upper, distal lower
E Palatal upper, buccal lower

https://t.me/DentalBooksWorld
questions 111

3. A 68-​year-​old woman attends for the try-​in of her cobalt-​


chrome denture framework. At the appointment, it does not
fit. Having spoken to the laboratory technician, it is discovered that
the wrong type of refractory model was used. What is the single main
reason this type of model is required in the production of cobalt-​chrome
dentures? ★★
A To assist in the flow of the molten metal over the cast, preventing
air blows
B To block out unwanted undercut
C To facilitate the placement of an investing sprue onto the wax pattern
D To maintain the registration previously recorded, as the original
model is often damaged during the casting process
E To withstand casting temperatures and expand/​contract in a similar
manner to the alloy

4. A 74-​year-​old man is having a new complete lower denture


constructed. He has worn an upper complete denture for
25 years but has never worn a lower one. His resting face height (RFH) is
78 mm. What is the single most appropriate occlusal vertical dimension
(OVD) to use during the jaw registration process? ★★
A 73 mm
B 75 mm
C 76 mm
D 8 mm
E 83 mm

5. A 72-​year-​old man is booked for the fit of his new upper com-
plete denture. Upon assessment of the denture, prior to the
appointment, a defect within the right buccal flange is noted. The defect
has a bubbly-​type appearance within the acrylic structure. What is the
single most likely manufacturing fault? ★★
A Contraction porosity
B Dilation porosity
C Gaseous porosity
D Granular porosity
E Moisture inclusion porosity

https://t.me/DentalBooksWorld
112 Chapter 6 Prosthodontics

6. A 74-​year-​old woman attends, following the fit of a new set


of conventional complete dentures. She complains of gener-
alized aching pain from her cheeks bilaterally, which started a few days
after getting the new dentures. She also feels that her teeth ‘clatter
together’ when she is speaking. What is the single most likely cause of
the problem? ★★
A An overcontoured base plate in the canine regions
B Inadequate freeway space (FWS)
C Increased overjet (OJ)
D Overextension of both the upper and lower dentures
E Unilateral, unbalanced contact in intercuspal position (ICP)

7. A 37-​year-​old man is having his four maxillary incisor metallo-​


ceramic crowns replaced. He has a Class II div 2 incisal rela-
tionship, and the anterior guidance is deemed satisfactory. The guidance
is planned to be copied from the preoperative mounted study models.
Which single element of a semi-​adjustable articulator can be used for
this purpose? ★★
A Condylar guidance angle
B Incisal guidance table
C Intercondylar width
D Occlusal table
E Progressive side shift

8. A 68-​year-​old woman attends for a routine check-​up. It is no-


ticed that her bridge replacing the upper right canine, which is
of a double abutment design, has failed. The upper right second pre-
molar has caries under the retainer. She is advised that the bridge will
need to be removed but that a new bridge design will be used because
double abutting retainers is no longer considered an appropriate design.
What is the single most important reason why this design is not con-
sidered appropriate? ★★
A Excessive costs
B Increased occlusal forces increase the risk of crown fracture
C Link retainers have a negative impact on periodontal health
D No mechanical benefit is gained over a cantilever design
E Partial debonding of the distal retainer frequently goes unnoticed

https://t.me/DentalBooksWorld
questions 113

9. A 29-​year-​old woman has severe localized anterior toothwear


affecting her mandibular incisors, with no inter-​occlusal space
present. Direct composite restorations have been placed, and posterior
teeth are no longer in contact. The patient is advised that her ‘bite’ will
feel different for a while, but it will eventually re-​establish. What single
movement is most likely to occur in the posterior teeth? ★★
A Eruption
B Intrusion
C No movement—​ restoration at new occlusal vertical dimension
(OVD) required
D Rotation
E Tipping

20. An 8-​ year-​


old woman has a failed single cantilever resin-​
bonded bridge (RBB) replacing her upper right lateral incisor.
What is the single most likely reason for failure of her bridge? ★★
A Abutment failure
B Debonding
C Emergence profile
D Shade match
E Pulp necrosis

2. A 25-​year-​old woman is having a routine examination. Dental


caries is suspected buccally on the lower right first permanent
molar. The tooth is dried with air, and a white spot lesion becomes more
evident. What is the single main explanation for this? ★★
A Change in caries assessment index
B Change in desiccation index
C Change in index of treatment need
D Change in light index
E Change in refractive index

22. A 5-​year-​old woman is having a gold shell crown placed on


her upper right first permanent molar (UR6). Following occlusal
reduction of  mm, a further 0.5 mm of height is reduced over the pal-
atal cusp. What is the single main reason for doing this? ★★
A Aids retention and resistance form
B Better structural durability
C Improved marginal integrity
D Maintains cleansability
E Preserves tooth structure

https://t.me/DentalBooksWorld
114 Chapter 6 Prosthodontics

23. A 59-​year-​old man is undergoing treatment for his failing denti-


tion. In the maxillary arch, he requires a number of new crowns
and a new cobalt-​chrome partial denture. Before he had any extracoronal
restorations placed, study models were surveyed and a preliminary den-
ture design was created. What is the single most important reason for
this? ★★
A So dissimilar metals are not placed in contact, leading to galvanic pain
B To allow incorporation of retentive/​supportive features into the
restoration
C To assess the amount of tooth reduction needed
D To ensure naturally occurring undercut is maintained
E To give the technician a reference point for the new occlusal table

24. A 38-​ year-​


old woman with post-​ eruptive breakdown sub-
sequent to amelogenesis imperfecta is undergoing full mouth
rehabilitation. Following planning on articulated models, the subsequent
restorations are provided at an increased occlusal vertical dimension
(OVD) in the retruded axis position (RAP). What is the single main
reason for using RAP in this scenario? ★★★
A It is the most reproducible position, allowing accurate information
transfer to the laboratory
B It reduces chair side time at cementation appointments
C Less damaging forces are transmitted to restorations where prema-
ture contacts exist
D RAP provides more space for the restorative material without the
need for preparation
E Rehabilitation in RAP is more comfortable for the patient in the
long term

25. A 68-​year-​old woman attends, complaining of a loose upper


denture. Clinically, she has an edentulous maxilla and retained
lower incisor teeth. She is not currently wearing a lower denture and
does not take her upper denture out at night. The upper denture is
loose, and the premaxilla is mobile and ‘flabby’. What is the single most
appropriate diagnosis? ★★★
A Apertognathia
B Bezold–​Brucke effect
C Bonwill triad
D Combination syndrome
E Kelly’s sign

https://t.me/DentalBooksWorld
questions 115

26. An 82-​year-​old man attends, wishing to have new dentures


made. He only has his mandibular incisor teeth remaining,
which are severely proclined, and the residual alveolar ridge is grossly re-
sorbed. It is decided to construct a swing-​lock denture to try and provide
a satisfactory lower partial denture. When designing the connectors,
which single combination would be the most appropriate? ★★★
A Fixed labial bar and hinged lingual plate
B Hinged labial bar and lingual plate
C Hinged labial bar and sublingual bar
D Lingual plate and coronally approaching clasps
E Lingual plate and gingivally approaching clasps

27. A 58-​year-​old patient attends with generalized tooth wear. His


teeth are vital and have previously been treated with composite
restorations, but these have repeatedly failed by wearing down and he
now requests an alternative. He is aware that he grinds his teeth at night,
and clinically he has very hypertrophic masseters. It is decided to pro-
vide full-​coverage crowns to restore his teeth. What would be the single
most appropriate choice of crown in this situation? ★★★
A Dentine-​bonded crown (feldspathic porcelain)
B Lithium disilicate all ceramic crown
C Metallo-​ceramic crown with metal palatal surface
D Monolithic zirconia crown with full coverage glaze
E Veneered alumina oxide all ceramic crown

28. A 53-​year-​old man has a deep disto-​occlusal-​lingual amalgam


restoration incorporating the cusp on his vital lower right first
permanent molar. It is due to be replaced. The tooth has historically
been affected by attachment loss, and the restoration margin is supra-​
gingival but extends below the cemento–​enamel junction. He returns
after 2 weeks, complaining of food packing. Given that it was very chal-
lenging initially to restore the contact point, what would be the single
most appropriate course of action? ★★★
A Addition of amalgam to the contact point
B Provide additional oral hygiene instruction
C Provide new direct composite resin restoration
D Replace the amalgam using a wedge
E Restore with an indirect extra-​coronal restoration

https://t.me/DentalBooksWorld
116 Chapter 6 Prosthodontics

29. A 56-​year-​old woman is having her single implant crown re-


stored. It is opposing a natural tooth. The static occlusal contact
is left ‘light’ (i.e. roughly 20–​30 μm of space between the prosthesis and
the opposing natural tooth during gentle occlusion). What is the single
main reason for doing so? ★★★★
A Account for periodontal ligament compression around the
natural teeth
B Compensate for the wear of the natural dentition over time
C Minimize the risk of potential excursive interferences
D Reduce stress accumulation in, and prevent fracture of, the
abutment screw
E Reduce the time taken to fit the crown

30. A 34-​year-​old woman is having a single implant crown provided


to replace a congenitally missing lower left second premolar.
The implant is well aligned, but there is limited inter-​occlusal space pre-
sent. A screw-​retained restoration is provided, and the patient is advised
that it will be more retrievable in the long term. What is the other single
main benefit of a screw-​retained restoration in this scenario? ★★★★
A Better aesthetics
B Improved retention
C Less time-​consuming
D Lower costs
E Reduced marginal leakage

3. A local colleague is overheard recommending a local personal


trainer who is offering tooth whitening with 0% carbamide
peroxide. This person’s name is not on the General Dental Council
(GDC) register. Which single UK law does this contravene, and how
would you manage the scenario? ★★★★
A Dentists Act 984—​contact the GDC
B Dentists Act 984—​contact Trading Standards
C Health and Medicines Act 988—​contact the GDC
D Health and Medicines Act 988—​ contact the Medicines and
Healthcare Products Regulatory Agency (MHRA)
E The Cosmetic Products Enforcement Regulations 203—​ contact
the MHRA

https://t.me/DentalBooksWorld
answers 117

ANSWERS
. B ★ OHCD 6th ed. → p. 226
During opening and closing, the mandible goes through two phases of
movement. Initially, it undergoes a rotational movement through the first
20–​25 mm of mandibular opening; this is then followed by a transla-
tion of the condyle down the articular eminence, as the individual opens
wider. The terminal hinge axis or retruded axis position is the fixed axis
of rotation joining the condyles, whilst they rotate in centric relation.
Centric relation has multiple definitions but generally would be con-
sidered to amount to the maxillomandibular relationship when the con-
dyles are in the glenoid fossa, in their most retruded, unstrained position.
When most people close, they come together into a position of max-
imum intercuspation that is habitually learnt (the intercuspal position—​
ICP). Alternatively, if the teeth contact with the condyle in centric
relation, it is known as the RCP. In 90% of the population, the teeth and
condylar head are located more posteriorly in the RCP, compared with
the ICP, hence retruded contact position. Convention dictates that when
planning a reorganized approach to treatment, the RCP is used, as it is
considered to be the only reproducible position to transfer information
to the laboratory.
Keywords: retruded contact position.
→ The Academy of Prosthodontics. The glossary of prosthodontic
terms: ninth edition. Journal of Prosthetic Dentistry. 207;7(5S):e–​05.
→ Wassell R, Naru A, Steele J, Nohn F. Applied Occlusion. Quintessence
Publishing, London; 2008.

2. E ★ OHCD 6th ed. → p. 296


The master (working) impression aims to not only record the full extent
of the denture-​bearing area, but also to record the width and depth of
the sulci in function. This provides a border seal and ensures the den-
ture contours correctly to the movements of the sulci and frenula, thus
preventing displacement of the denture by the soft tissues. Theoretically,
it also means that a minimal distance is kept between the denture and
soft tissues during function. This ensures the greatest pressure gradient/​
salivary surface tension (the pressure gradient is related to the thickness
of the salivary film thickness) is present, which improves retention and
optimizes the border seal. Generally, a special tray is required to accur-
ately modify and border mould the impression to the individual, as stock
trays frequently do not ‘fit’. However, with modern materials and tech-
niques, master impressions can be recorded using a stock tray with suf-
ficient time and skill. Naturally, some faults from the primary impression
will be corrected as the impression is refined. The surface detail may also
increase, depending on the material used for each impression, improving
the adaptation, but this is a property of material choice, and not the fun-
damental purpose of the second impression.

https://t.me/DentalBooksWorld
118 Chapter 6 Prosthodontics

The neutral zone can be recorded with a special impression technique


at the try-​in stage.
The path of insertion for complete dentures is not greatly relevant, un-
less there are significant areas of bony undercut to be engaged.
Keywords: complete denture, second impression.
→ Jacobson TE, Krol AJ. A contemporary review of the factors involved
in complete denture retention, stability, and support. Part I: retention.
Journal of Prosthetic Dentistry. 983;49:5–​5.
→ Massad JJ, Cagna DR. Vinyl polysiloxane impression material in remov-
able prosthodontics. Part : edentulous impressions. Compendium of
Continuing Education in Dentistry. 2007;28:452–​9.

3. C ★ OHCD 6th ed. → p. 292


Bone is an active structure, constantly being broken down and reformed
throughout life. Stimulation from occlusal forces and the connecting
periodontium around the teeth is required in order to maintain bone
height. When teeth are lost, the alveolar ridge resorbs over time down
to the basal bone. Work by Tallgren in 972 demonstrated that reduc-
tion is greatest in the first 6–​2 months, after which the rate slows. The
amount of bone loss is up to four times greater in the mandible than in
the maxilla.
Although the denture may potentially become overextended, the most
likely complication to negatively affect retention is poor adaptation of the
fitting surface, with loss of the border seal. Therefore, a reline, rebase,
or remake is likely to be required for immediate dentures 6–​2 months
post-​extraction. If a significant number of teeth are to be replaced, it is
likely to be a poor fit from the outset and need relining promptly. It is
wise to ask patients to consider immediate dentures as ‘temporary den-
tures’, as significant alteration is likely to be required.
Keywords: full clearance, immediate, denture, bony remodelling.
→ Tallgren A. The continuing reduction of the residual alveolar ridges in
complete denture wearers: a mixed-​longitudinal study covering 25 years.
Journal of Prosthetic Dentistry. 2003;89:427–​35.

4. A ★ OHCD 6th ed. → p. 32


A vital part of successful denture wearing is down to muscular control
of the prosthesis by the patient. The polished surfaces of the denture
(i.e. all of the denture surface that is not the teeth or impression sur-
face) are controlled by the surrounding soft tissue and musculature. The
copy technique aims to transfer this shape to the new denture to aid
habituation.
From the impression of the original denture that the clinician takes, the
technician will produce a ‘copy’ with wax teeth and an acrylic base. If
required, the occlusal vertical dimension (OVD) is corrected at the regis-
tration appointment, before new teeth will be added by the technician at
the correct height. A closed mouth wash impression is taken at the try-​in

https://t.me/DentalBooksWorld
answers 119

appointment, which the technician will use to reline and produce the
final product. As the denture base is produced in acrylic from the start,
modification of the polished surfaces is much more difficult than the wax
teeth (occlusal surface) or the fitting surface (impression surface), which
is corrected similarly to a reline. The closed mouth impression minim-
izes unwanted increases in the OVD. Small modifications to the shape
of the polished surface, or correcting the extensions, may be possible.
Unfortunately, bodily movements of the denture base are not possible,
and as such, significant alterations in tooth position would be easier with
a complete remake.
Keywords: worn dentures, copy technique.

5. A ★ OHCD 6th ed. → p. 32


The Alma gauge has a vertical spring ruler and horizontal scale. It is used
to measure the vertical and horizontal positions of the maxillary anterior
teeth of the old prosthesis, using the incisive papilla as a reference point.
These measurements can then be transferred to the new registration
block to help the clinician maintain the lip support and incisal level, if
it was deemed appropriate at the outset (see Figure 6. which shows
an alma gauge used for transferring the incisal position of old dentures
to the new registration rim). A Fox’s occlusal plane guide is used to as-
sess the angulation of the occlusal plane to the horizontal and antero-​
posterior planes; the reference lines are the interpupillary and alar-​tragal
lines, respectively. A Willis bite gauge measures the resting face height
(RFH) and occlusal vertical dimension (OVD), allowing assessment of
the freeway space; an alternative would be the two-​dot technique. The
thickness of material can be checked with an Iwanson gauge, similar to
calipers. This instrument is generally used in fixed prosthodontics for

Figure 6.

https://t.me/DentalBooksWorld
120 Chapter 6 Prosthodontics

measuring small measurement changes and ensuring that indirect restor-


ations are of adequate thickness; the thickness of chrome frameworks
can also be checked. A small steel ruler is a multipurpose tool used for
measuring distances clinically.
Keywords: position, upper anterior teeth, copy, chair side.

6. B ★ OHCD 6th ed. → p. 284


The high modulus of elasticity and lack of ductility of cobalt-​chrome mean
that the material is rigid, which is good for denture construction because
the denture base can be kept thin for comfort and yet maintaining suffi-
cient strength. However, the design of the clasp needs to consider the
amount of distortion the clasp undergoes when engaging the undercut.
This needs to be less than the proportional limit of the material to pre-
vent permanent distortion and subsequent lack of retention. Changing
the cross-​dimensional shape, length, or material of the clasp can increase
the flexibility. Alternatively, reducing the depth of undercut the clasp
has to engage reduces the amount of flexibility needed. In general,
cast chrome should engage undercuts of 0.25 mm and a clasp length
of 5 mm is recommended to allow sufficient flexibility (some clinicians
consider 4 mm to be an absolute minimum). Additionally, only the final
third of the clasp should engage the undercut.
Keywords: cobalt-​chrome clasp, length, undercut.
→ Bates JF. Retention of partial dentures. British Dental Journal.
980;49:7–​4.
→ Davenport JC, Basker RM, Heath JR, et al. Prosthetics: clasp design.
British Dental Journal. 200;90:7–​8.

7. D ★ OHCD 6th ed. → p. 278


All of the answers listed could be considered as advantages of resin-​
bonded bridges (RBBs) over conventional bridge work. Fundamentally,
RBBs are a reliable space management strategy with a low biological cost.
Tooth preparation for RBBs remains controversial, with a large number
of clinicians advocating ‘no-​ prep’ RBBs over minimal-​ preparation
RBBs. However, both techniques adopt a minimally invasive approach.
Preparation, if any, should remain within the enamel to provide greater
bond strengths and reduced insult to the dentin–​pulp complex.
Several materials and designs are available for construction of RBBs,
including metal, all-​ ceramic, and glass fibre-​ reinforced composite.
A recent systematic review reported the 5-​year survival of RBBs to be
9.4%. Careful consideration of occlusal issues can help to improve the
prognosis of RBBs. Light contact on the RBB pontic in the intercuspal
position and avoidance of involvement in excursions are thought to
be ideal.
Keywords: 9-​year old, minimally restored dentition.
→ Durey KA, Nixon PJ, Robinson S, Chan MF. Resin bonded bridges: tech-
niques for success. British Dental Journal. 20;2:3–​8.

https://t.me/DentalBooksWorld
answers 121

→ Thoma DS, Sailer I, Ioannidis A, Zwahlen M, Makarov N, Pjetursson


BE. A systematic review of the survival and complication rates of resin-​
bonded fixed dental prostheses after a mean observation period of at
least 5 years. Clinical Oral Implants Research. 207;28:42–​32.

8. A ★
The concept of incorporating a ferrule in post crown design is widely
accepted within dentistry. A ferrule is defined as any ring or bushing used
for making a tight joint. In dentistry, by having the margins of a crown
below the margins of the remaining tooth core, a ferrule is produced—​
the idea being to reduce the force concentration at the apex of the post
by distributing the lateral forces placed on the prosthetic tooth, theoret-
ically reducing the risk of root fracture. A ferrule height of 2 mm and a
width of  mm are desirable. Additionally, maintaining as much dentine
as possible is also very important in reducing root fracture.
A long post length and an unfavourable crown-​ to-​
root ratio may
increase the risk of fracture, because a greater volume of tooth tissue
is removed and greater leverage forces will be applied to the root,
respectively. However, a suitable ferrule is a more recognized prog-
nostic factor. A narrow post width will help retain dentine volume and
help minimize the risk of root fracture, but this needs to be balanced
against having sufficient material thickness to prevent a fracture of the
post itself. Shallow retention grooves will have little influence on root
fracture outcomes.
Keywords:  mm supra-​gingival tooth tissue, post crown, root fracture.
→ Eliyas S, Jalili J, Martin N. Restoration of the root canal treated tooth.
British Dental Journal. 205;28:53–​62.
→ Jotkowitz A, Samet N. Rethinking ferrule—​a new approach to an old
dilemma. British Dental Journal. 200;209:25–​33.

9. A ★
Despite numerous approaches to treatment planning, it is universally ac-
cepted that pain management should be the first item to be addressed
within any treatment plan. After the pain has been addressed, the patient
should then be stabilized with regard to the primary disease. This will
inevitably involve prevention advice, including oral hygiene advice,
dietary advice, and fluoride supplements, etc. Following on from this, a
provisional treatment planning stage will be undertaken to deconstruct
failing restorations and assess for restorability. There also needs to be a
period of reassessment of the patient’s compliance and motivation with
treatment. From here, a definitive treatment plan can be constructed,
taking into account the remaining tooth structure, the underlying peri-
odontal status, and the likely prognosis of the remaining dentition. After
treatment is completed, patients must be placed on an appropriate
recall interval. This will be based on the National Institute for Health and
Care Excellence (NICE) guidelines for recall and professional opinion.
Treatment planning can often be challenging, particularly when there

https://t.me/DentalBooksWorld
122 Chapter 6 Prosthodontics

are multiple diagnoses to consider. Time should be taken to construct a


definitive treatment plan and advice sought where required.
→ National Institute for Health and Care Excellence. Dental checks: inter-
vals between oral health reviews. Clinical guideline [CG9]. 2004. Available
at: https://​www.nice.org.uk/​guidance/​cg9

0. D ★
Anterior composite restorations are considered a reversible and aes-
thetic management strategy for fractured anterior teeth, with low bio-
logical cost and excellent patient satisfaction. Shade matching is crucial to
achieve a desirable outcome, and this should be carried out at the start
of the appointment, ideally under natural light.
Tooth preparation, etching, and placement of a dental dam can all
interfere with shade matching, as well as drying the tooth. Drying or
desiccating creates a greater discrepancy in the refractive index of the
tooth and can lead to inaccuracies in shade matching. It is considered
appropriate and good practice to place a small amount of composite
onto the tooth to be restored prior to beginning treatment. This can
be cured and aesthetics can be assessed, with the shade being altered,
if necessary. This composite can be removed easily, without damaging
the tooth.
Dental dam placement can influence the light passing through the tooth,
and the colour of the rubber dam can influence shade selection. For
this reason, shade matching should be recorded at the beginning of the
appointment.
Keywords: shade.
→ Beddis H, Nixon P. Layering composites for ultimate aesthetics in
direct restorations. Dental Update. 202;39:630–​6.

. D ★
The Kennedy classification system is used to categorize edentulous areas
in partially dentate patients. The classes are:
● Class I—​bilateral free end saddle
● Class II—​unilateral free end saddle
● Class III—​unilateral bounded saddle
● Class IV—​bounded saddle crossing the midline

The following rules aid identifying to which classification a patient belongs


if multiple saddle areas are present:
. You should classify according to the most posteriorly positioned
saddle. A free end saddle is considered more posterior than a
bounded saddle.
2. Once the classification has been decided, any additional saddles
should be denoted by ‘modifications’. The modification should
represent the number of additional saddles only, regardless of their
size or position.

https://t.me/DentalBooksWorld
answers 123

3. Class IV saddles cannot be modified. They would come under a


modification.
Third molars are not considered when deciding upon a classification.
Keywords: maxillary incisors, extracted, Kennedy classification.

2. A ★★ OHCD 6th ed. → p. 302


The BULL rule (Buccal Upper, Lingual Lower) is a simple acronym to
help remember which cusp to adjust to remove interferences in dynamic
lateral excursions. However, understanding the concept sometimes
takes more thought. When adjusting complete dentures to provide
balanced articulation, thought needs to be given to the function of each
cusp. The supporting or functional cusps (palatal maxillary cusp, buccal
mandibular cusp) maintain the OVD, and they should not be adjusted
when trying to gain balanced articulation, as alteration of the OVD
may result. Therefore, should these cusps cause an interference on
lateral excursions, then the balancing (non-​functional) cusp inclines of
the opposing arch (buccal upper, lingual lower) should be adjusted until
balanced working side contacts are achieved. It may not be possible to
apply this rule to the non-​working side, as the supporting cusps could
interfere during this movement. In this situation, it is usually possible to
remove the interference without completely removing the supporting
contact point.
Despite this concept being widely recognized and accepted, some
research has questioned the benefit of this occlusal scheme with regard
to masticatory efficiency, function, and patient experience. Results con-
cluded no difference between complete dentures setup with bilateral
balanced articulation and those in canine guidance. The authors consider
this plausible, because as soon as food is introduced between the den-
tures, the teeth will no longer be in contact and therefore, the role of
balanced articulation may become insignificant.
Keywords: balanced articulation, unwanted change, OVD.
→ Basker RM, Davenport JC, Thomason JM. Prosthetic Treatment of the
Edentulous Patient (5th ed.). Wiley-​Blackwell, Oxford; 202.
→ Farias-​Neto A, Carreiro A. Bilateral balanced articulation: science or
dogma. Dental Update. 204;4:428–​30.

3. E ★★ OHCD 6th ed. → p. 652


Casting of metal alloys needs to be conducted on special models for
a variety of reasons. The casting process needs to reach very high
temperatures—​ up to 000°C for cobalt-​ chrome. These extreme
temperatures produce challenges for the model, which needs to be
able to adapt by demonstrating various mechanical properties. Firstly,
the material needs to be able to withstand high casting temperatures
(thermal stability). Secondly, the material must be sufficiently porous
to vent gases produced during the casting process (porous). Finally, the
material needs to demonstrate a similar coefficient of expansion to the

https://t.me/DentalBooksWorld
124 Chapter 6 Prosthodontics

alloy, so as the metal cools, it will be of the correct size (expansion).


Refractory models are made of derivatives of silica that are bound to
gypsum or silica or phosphate. Phosphate-​bonded refractory models are
commonly used as adjustment of the colloidal silica component, which
means the setting expansion can be varied for the desired alloy.
The other answers are not related to the type of cast material, but
rather to other laboratory or clinical procedures.
Keywords: cobalt-​chrome, refractory model.

4. A ★★
This question requires good knowledge of the measurements used
during jaw registration. A Willis bite gauge is often used during this pro-
cess. The RFH, or resting vertical dimension, measures the physiological
rest position of the lower face. This is the position adopted by the teeth
and mandible when the muscles of mastication are at rest. Ideally, the
lips should be in contact and the head should be in an upright position.
Freeway space (or inter-​occlusal rest space) is the difference between the
RFH and the OVD (when opposing arches are in contact). Appropriate
freeway space is fundamental for chewing, speech, and comfort.
In the majority of complete denture cases, 2–​4 mm of freeway space is
deemed acceptable. However, in patients who have not worn a pros-
thesis before, or in a long time, increased freeway space has been re-
ported to improve habituation and, as such, a measurement of 5–​6 mm
is considered appropriate. However, it must be remembered that these
measurements are somewhat arbitrary and are unlikely to be accurately
measured clinically.
Moreover, these measurements are dynamic and change throughout
time. This includes the RFH, which is influenced by changes in the
stomatognathic system. For this reason, the RFH is often the starting
point for calculating the OVD.
Keywords: 74-​year old, never worn a lower denture.
→ McCord J, Grant A. Registration: stage 2—​intermaxillary relations.
British Dental Journal. 2000;88:60–​6.

5. C ★★ OHCD 6th ed. → p. 660


During the processing of acrylic dentures, three main faults can be
seen within the acrylic. These are contraction, gaseous, and granular
porosities. They result from errors with either the ratio of the acrylic
components or as a result of incorrect heating processes. A polymer
(powder) to monomer (liquid) ratio of 3.5: is required. However, if
insufficient monomer saturation occurs, then a dry, spongy/​crumbly
appearance of the acrylic results. Excess liquid may lead to unreacted
monomer present in the denture base, potentially causing irritation of
the mucosa. If too little volume of acrylic is packed into the flask, then
contraction porosities can occur, resulting in incomplete flange areas of
short gingival margins/​papilla. There may also be streaks seen within

https://t.me/DentalBooksWorld
answers 125

the material. Gaseous porosities occur when the temperature of the


resin exceeds its boiling point (00.3°C) before polymerization has com-
pleted, resulting in bubbles being included within the cured material. This
bubbly appearance should lead you to the answer of gaseous porosity.
Inclusion of moisture may lead to pale, cloudy areas within the acrylic,
and insufficient tightening would produce an excessively thick denture
base, increasing the occlusal vertical dimension from that prescribed.
Keywords: bubbly, appearance, manufacturing faults.
→ Van Noort R. Introduction to Dental Materials (4th ed.). Mosby Elsevier,
London; 203.

6. B ★★ OHCD 6th ed. → p. 308


A lack of FWS can produce an array of presenting complaints. However,
when combined with an increase in the patient’s occlusal vertical dimen-
sion (OVD), generalized aching around the mouth is often reported. The
pain is often non-​specific and vague. A complaint of ‘too many teeth’ or a
‘full mouth’ is highly likely to indicate errors within the OVD. Complaints
of teeth ‘clacking’ together are likely to relate to unretentive dentures
or inadequate FWS.
With an increased bulk of the baseplate in the canine regions, patients
may develop whistling sounds or difficulty with the ‘th’ or ‘ch’ sounds
when speaking. This is because the phonetic position of the tongue is
inhibited laterally by the denture base.
An increased OJ can affect speech, frequently fricatives (e.g. ‘f ’ sounds)
and ‘s’ sounds (sibilants). Additionally, some patients complain of diffi-
culty incising foods, which may be corrected with the prescription of a
flat anterior bite plane, depending on the situation. Overextension or
unbalanced occlusion are common faults that tend to produce localized
sores or localized/​unilateral aching, respectively; they are less commonly
associated with the generalized aching typically seen in OVD issues.
Keywords: generalized aching pain, clatter.
→ Basker R, Davenport J, Thomason J. Prosthetic Treatment of the
Edentulous Patient (5th ed.). Wiley-​Blackwell, Chichester; 20.

7. B ★★
When providing multiple indirect restorations, use of a semi-​adjustable
articulator and facebow transfer is recommended. The facebow allows
the maxillary cast to be related to the terminal hinge axis and also pro-
vides a horizontal plane of reference for future mountings of additional
casts. The horizontal reference point varies with each articulator used,
and the reader should refer to the manufacturer’s guidance. The semi-​
adjustable articulator has the benefit over an average-​value articulator in
that the condylar guidance angle, immediate side shift, and progressive
side shift can be adjusted (depending on the articulator model). These
can be set with the use of excursive check records. Greater freedom on

https://t.me/DentalBooksWorld
126 Chapter 6 Prosthodontics

the articulator allows more accurate replication of the mandibular move-


ments and, hopefully, the need for less intraoral adjustments.
Anterior restorations with adequate shared protrusive guidance can be
replicated, using a custom incisal guidance table, which is then used to
fabricate the palatal contours on the new restorations. Where inad-
equate or inappropriate guidance is identified, it would be prudent
to restore the anterior teeth with provisional restorations, which can
be modified chairside until an ideal anterior guidance is achieved and
copied. An alternative method of coping the anterior guidance is the
‘every other tooth method’, whereby definitive restorations are placed
on three alternating teeth at a time (e.g. UR3, UR, UL2). In this way,
the guidance in the definitive crowns can conform to the remaining tem-
porary restorations.
Keywords: guidance, copied, semi-​adjustable articulator.
→ Wassell R, Naru A, Steele J, Nohn F. Applied Occlusion. Quintessence
Publishing, London; 2008.

8. E ★★
In the past, double abutment was considered a sensible design for fixed
bridgework when the adjacent teeth were of poor quality or small
in size, in order for the bridge to have better mechanical properties.
Unfortunately, it was noticed that debonding of the distal retainer fre-
quently occurred without loss of the bridge, with subsequent caries
development in the abutment. This is because flexure of the super-
structure during loading leads to the mesial abutment acting as a ful-
crum, with the result being the breakdown of the lute under the distal
retainer. These days, efforts are made to avoid this type of design, as
it is a recognized complication. Careful assessment should be made of
the abutments if this type of bridge is seen. The presence of air bub-
bles developing at the crown margin of the retainers whilst pressing on
the pontic can be an indicator of debonding having occurred. See Figure
6.2a (see Colour Plate section) which shows a full-​arch fixed bridge that
debonded, leading to gross caries underneath the distal abutment. The
presence of bubbles under loading is indicative of the retainer having
partially debonded at the prosthesis at rest. Figure 6.2b (see Colour
Plate section) shows underloading air is displaced, forming bubbles at
the mesial aspect.
Keywords: bridge, double abutment, caries.
→ Hemmings K, Harrington Z. Replacement of missing teeth with fixed
prostheses. Dental Update. 2004:3:37–​4.

9. A ★★
The Dahl principle is a method for creating inter-​occlusal space, either
for restoring or for moving teeth. Traditionally, it utilized either a remov-
able prosthesis or a cast metal plate bonded to the back of the upper
teeth. These removable prostheses historically had issues with patient
compliance, affecting outcomes. Original work was conducted by Dahl

https://t.me/DentalBooksWorld
answers 127

and colleagues back in 975 on axial tooth movement to create space for
restoring worn anterior teeth. They demonstrated that this occurred by
intrusion of the anterior teeth (40%) and eruption of the posterior teeth
(60%). There may also be an element of condylar remodelling in some
cases. Re-​establishment of the posterior occlusion takes on average up
to 7 months but can take as long as 8 months. In modern dentistry,
bonding direct composite at an increased OVD to manage localized
anterior toothwear is a well-​accepted technique. Some clinicians also
utilize the Dahl principle when cementing resin-​bonded bridges to allow
a no-​preparation technique.
Keywords: posterior teeth, no longer in contact, re-​establish.
→ Ahmed KE, Murbay S. Survival rates of anterior composites in man-
aging toothwear: systematic review. Journal of Oral Rehabilitation.
206;43:45–​53.
→ Poyser N, Porter RW, Briggs PF, Chana HS, Kelleher MG. The Dahl con-
cept: past, present and future. British Dental Journal. 2005;98:669–​76.

20. B ★★
RBBs have been shown to display higher failure rates than conventional
full-​coverage coronal restorations. However, recent literature reports a
0-​year survival of around 80%. Furthermore, most failures tended to
occur within the first 4 years. Whilst conventional bridge designs may
offer greater mechanical properties for retention, the conservation of
tooth tissue associated with RBBs has made them a popular treatment
modality, especially for congenitally absent upper lateral or lower inci-
sors where space and a poorly developed alveolar process may compli-
cate implant placement.
Debonding of the retainer is the most common cause of failure, and
posterior teeth are more likely to fail than anterior teeth. Careful con-
sideration of the occlusion is fundamental to success. Pulp necrosis is
extremely unlikely, given the limited preparation.
Keywords: cantilever, RBB, failure.
→ King P, Foster LV, Yates RJ, Newcombe RG, Garrett MJ. Survival char-
acteristics of 77 resin retained bridges provided at a UK dental teaching
hospital. British Dental Journal. 205;28:423–​8.
→ Pjetursson BE, Tan WC, Tan K, Brägger U, Zwahlen M, Lang NP. A sys-
tematic review of the survival and complication rates of resin-​bonded
bridges after an observation period of at least 5 years. Clinical Oral
Implants Research. 2008;9:3–​4.

2. E ★★
The carious process results in porosities developing within the enamel,
which is usually filled with water. The refractive index describes how light
propagates through a medium. It is a ratio of how light passes through a
medium, compared to light travelling through a vacuum. The refraction
of light can affect how we interpret the colour of an object. In carious

https://t.me/DentalBooksWorld
128 Chapter 6 Prosthodontics

enamel, subsurface porosities are normally filled with water, through


which light travels more slowly than in air, and also more dispersion oc-
curs. When the tooth is air-​dried, water is expelled from the porosities
and replaced with air. This lowers the refractive index in this area, giving
a white appearance to the surface. Figure 6.3 (see Colour Plate section)
shows a white spot lesion upon air-​drying. A lesion visible without drying
represents more advanced disease. As the carious process develops to
dentine, exogenous staining, bacterial pigments, and denaturing of pro-
teins result in brown discoloration.
The caries assessment index is not a real index. Most epidemiological
studies examining caries will use diseased, missing, filled teeth (DMFT)
or the international caries detection and assessment system (ICDAS II).
There are multiple desiccation indexes in biology, but not one in den-
tistry. The Light index is used to assess the size of a pneumothorax. The
index of treatment need should be known as the index of orthodontic
treatment need (IOTN).
Keywords: dried, air, white spot, more evident.
→ Kidd E, Fejerskov O. Essentials of Dental Caries (4th ed.). Oxford
University Press, Oxford; 206.

22. B ★★
Shillingburg outlined five main principles of tooth preparation in his text-
book of fixed prosthodontics: preservation of tooth structure, retention
and resistance form, structural durability, marginal integrity, and main-
tenance of periodontal health. The functional cusp should be beveled
to allow for an adequate bulk of material to be placed in an area of high
loading. If insufficient reduction is given, then two outcomes may occur.
Firstly, the restoration will be too thin in this region and will be at risk
of fracture or an increased wear rate. Alternatively, the technician may
overbuild the restoration, resulting in occlusal interference.
Although this concept is well accepted throughout the dental com-
munity, there are some who debate the need for this added bulk of
material. However, generally, most practitioners will provide the extra
room in their preparations over the functional cusp.
Keywords: UR6, 0.5 mm, reduced, palatal cusp.
→ Shillingburg HT Jr, Sather DA, Wilson EL, et al. Fundamentals of
Fixed Prosthodontics (3rd ed.). Quintessence Publishing, Hanover Park,
IL; 997.

23. B ★★
A vital part of treatment planning for removable prosthodontics is to
include features to improve retention and support. Retention can be
gained from clasps to engage undercut, a path of insertion that differs
to the path of displacement, or the use of precision attachments. If a
removable partial denture (RPD) is planned and single or multiple indirect
restorations are required, then retentive/​supportive features, such as

https://t.me/DentalBooksWorld
answers 129

rest seats, guide planes, and desirable undercuts, can be incorporated.


Furthermore, use of intra-​and -​extracoronal precision attachments,
such as the FR system® (intracoronal) or Preci Clixc (extra-​coronal), can
be integrated into the casting. Therefore, a provisional denture design
should be developed prior to providing the indirect restoration.
It is important to remember that when preparing teeth where guide
planes or rest seats are planned, extra reduction in the preparation
is required to allow for sufficient material thickness and to prevent
overcontouring of the crown.
Keywords: new crowns, new cobalt-​chrome partial denture, preliminary,
design.
→ Burns D; Ward J. Review of attachments for removable partial den-
ture designs: . classification and selection. International Journal of
Prosthodontics. 990;3:98–​02.
→ Devlin H. Integrating posterior crowns with partial dentures. British
Dental Journal. 200;9:20–​3.

24. A ★★★ OHCD 6th ed. → p. 228


Over the years, various theories on occlusion have appeared in the lit-
erature. Although opinion varies, little firm scientific evidence is avail-
able to confirm whether one occlusal scheme is better than another.
However, convention dictates the use of the RAP when reorganizing
the occlusion. In function, mandibular movements are guided by the
condyles posteriorly and the teeth anteriorly. The retruded contact
position (RCP) is the first tooth contact on the RAP and is guided by
the condyles and surrounding musculature/​soft tissues. Recording a
location on the RAP primarily provides a reproducible position that is
defined by anatomical constraints, rather than muscular habituation.
Therefore, it allows more accurate transfer of information and changes
in the OVD between the patient and the laboratory. The literature
also argues that use of the RAP will provide a more harmonious mus-
cular position, with no damaging slides from premature contacts, redu-
cing the risk of temporomandibular disorders and restoration failure.
However, the evidence is inconclusive and contradictory evidence
exists. Moreover, it is known that a new RCP will re-​establish over time
in patients reorganized in RCP.
Reduction in chair time at cementation will come from the production of
accurate laboratory work, and more space maybe created by distalizing
the mandible into the RAP, but these are not the primary reasons for
its use.
Keywords: increased OVD, full mouth rehabilitation, RAP.
→ Becker C, Kaiser D. Evolution of occlusion and occlusal instruments.
Journal of Prosthodontics. 993;2:33–​43.
→ Davies SJ, Gray RM, Whitehead SA. Good occlusal practice in ad-
vanced restorative dentistry. British Dental Journal. 200;9:42–​4,
427–​30, and 433–​4.

https://t.me/DentalBooksWorld
130 Chapter 6 Prosthodontics

25. D ★★★ OHCD 6th ed. → p. 296


Combination syndrome was initially described by Ellsworth Kelly in
972, and Kelly’s syndrome (not sign) is a synonym. However, combin-
ation syndrome is the most commonly used name. The original features
described were: an edentulous maxilla associated with natural man-
dibular anterior teeth, including bone loss from the anterior maxillary
ridge; overgrowth of the tuberosities; papillary hyperplasia of the palatal
mucosa; extrusion of the mandibular anterior teeth; and loss of alveolar
bone height below the mandibular removable prosthesis [removable
partial denture (RPD)].
Further characteristics were added later by Saunders et al. There is
debate as to whether combination syndrome exists, as observed sci-
entific evidence of the condition is limited. However, the terminology
remains well grounded. It is well recognized that bone resorbs following
extractions and that a removable prosthesis can negatively impact upon
the rate.
Apertognathia is a synonym for open bite. The Bezold–​Brucke effect is
the apparent change in colour, depending on the intensity (brightness)
of the light source. The Bonwill triad should actually be the Bonwill’s
triangle, a 4-​inch equilateral triangle between the mandibular incisor tips
and the condyles—​a geometry that Bonwill used when designing his
early articulator.
Keywords: edentulous maxilla, retained lower incisors, flabby opposed
by natural dentition.
→ Palmqvist S, Carlsson GE, Öwall B. The combination syndrome: a lit-
erature review. Journal of Prosthetic Dentistry. 2003;90:270–​5.
→ The Academy of Prosthodontics. The glossary of prosthodontic
terms. Journal of Prosthetic Dentistry. 2005;94:0–​92.

26. B ★★★
The swing-​lock removable partial denture has been proposed as a cobalt-​
chrome denture design in patients with a mandibular Kennedy class I.
The cobalt-​chromium framework consists of a lingual plate opposed
by a hinged labial bar. This design utilizes labial alveolar undercut with a
swinging labial bar that closes following denture insertion. Moreover, a
lingual plate is used over a bar connector, as it will provide indirect reten-
tion. The labial component was traditionally constructed with anterior
struts that contacted the labial surface of the lower incisors. However, a
more aesthetic approach utilizes polymethylmethacrylate in the form of
gingival veneers. Disadvantages include: potential wear of the swing-​lock
components with extended use and possible negative consequences on
the remaining abutment teeth, which is usually the remaining mandibular
incisors.
Keywords: severely proclined, swing-​lock.

https://t.me/DentalBooksWorld
answers 131

→ Lynch CD, Allen PF. The swing-​lock denture: its use in conven-
tional removable partial denture prosthodontics. Dental Update.
2004;3:506–​8.

27. C ★★★
Choosing the most appropriate type of crown material can sometimes
be confusing, given the number of materials and production methods
available today; often there is not a correct answer. Judgement needs to
be made on what is best, given the presenting scenario. Consideration
needs to be given to the biological cost, aesthetic demands, and mech-
anical properties required in the given environment. In this situation, the
tooth is vital, visible, and needing to withstand high functional forces, and
a metallo-​ceramic crown is generally considered the most appropriate.
This gives the best compromise on aesthetics and durability, whilst min-
imizing the tooth reduction required.
All ceramic crowns and fully veneered metallo-​ceramic crowns require
significantly more reduction that metallo-​ceramic crowns with palatal
metal alone. Although feldspathic porcelain gains strength from bonding
to the tooth structure, it is less resistant to fracture under excessive
parafunctional loads. Monolithic zirconia, although being strong in thin
section, has compromised aesthetics. Alumina oxide cores are also very
strong, but the bond to the veneering ceramic is the weak link. Not only
will they require greater tooth reduction to provide adequate space
for the material, but under parafunctional loads, there is a risk that the
veneering ceramic will chip. Additionally, certain forms of porcelain can
be abrasive to enamel and cause wear of the opposing dentition when
the surface glaze is lost.
Keywords: vital, failed, wearing, grinds, hypertrophic masseters.
→ Shenoy A, Shenoy N, Dental ceramics: an update. Journal of
Conservative Dentistry. 200;3:95–​203.
→ Wassell R, Walls A, Steele J. Crowns and extra-​ coronal restor-
ations: materials selection. British Dental Journal. 2002;92:99–​202,
205–​.

28. E ★★★
This scenario describes a large direct restoration which has failed due
to inadequate contouring of the distal restoration surface. This has left
a deficient contact point which has resulted in food packing. This creates
an increased risk of caries or periodontal disease if left untreated, and
oral hygiene instruction alone is unlikely to be sufficient, especially where
previous attachment loss has been identified. Contact points are often
difficult to restore, using direct placement of restorative material, par-
ticularly where an adjacent cusp has been lost or where the restoration
depth is considerable. Contouring the matrix band can be challenging in
these situations and, as such, indirect extra-​coronal restorations should
be considered. Direct composite placement is likely to be equally as chal-
lenging as amalgam, with the added difficulty of moisture control and

https://t.me/DentalBooksWorld
132 CHAPTER 6 Prosthodontics

rubber dam placement with a deep margin. The use of wedges helps
to avoid ledges and assists in contact point formation by compressing
the periodontal ligament to compensate for the thickness of the matrix
band; particularly if pre-wedging is performed.
By comparison, indirect restorations can be appropriately contoured
in the laboratory and can be constructed in a wide range of materials,
including gold, ceramic, and resin composite. Whilst a dental dam may
still be required for bonding of the restoration, the length of time re-
quired for isolation is often shorter.
Keywords: deep restoration, food packing, challenging, contact point.

29. A ★★★★
Implant-retained restorations are becoming increasingly common
amongst the population. Osseointegration of the fixture rigidly secures
the implant to the bone. Implants differ from natural teeth in that they do
not have a periodontal ligament (PDL) surrounding them. With regard
to occlusion, this causes three main problems. Firstly, when the patient
occludes, the PDL of the natural dentition has the ability to compress;
implants only displace minimally (the distance bone flexes, approximately
3–5 μm vertically and 10–50 μm laterally).
Secondly, proprioception is provided predominantly by the PDL, which
is significantly greater than osseoperception around implants, limiting
the sensory feedback. Finally, the PDL has the ability to adapt to in-
creased occlusal loads, therefore limiting permanent damaging effects of
occlusal overload on natural teeth. If the space for PDL compression
of surrounding natural teeth is not accounted for, then there is a risk of
occlusal overload, which can lead to bone loss around the implant, screw
loosening, or implant component fracture.
These concepts vary slightly, depending on the opposing contact (e.g.
natural tooth, another implant, full arch implant restoration, etc.). The
clinician therefore needs to consider what the opposing contact is and
how this will impact on the occlusal design.
Keywords: single implant crown, natural tooth, static, contact, light.
→ Davies S, Gray R, Young M. Good occlusal practice in provision of
implant borne prostheses. British Dental Journal. 2002;192:79–88.
→ Gross M. Occlusion in implant dentistry. A review of the literature of
prosthetic determinants and current concepts. Australian Dental Journal.
2008;53:S60–8.

30. B ★★★★ OHCD 6th ed. → p. 316


One of the main benefits of screw-retained, compared to cement-
retained, restorations is retrievability. If there are problems with the
crown or the underlying abutment, it can be difficult to remove a
cement-retained restoration intact. The accurate fit of implant res-
torations and parallelism of the abutment mean retention is excellent,
even if weak luting cement is used. However, as in conventional crown

https://t.me/DentalBooksWorld
answers 133

preparations, adequate height is still required in order to produce suffi-


cient retention and resistance form with cement-​retained restorations.
Unfortunately, due to fixture angulations, screw-​retained restorations
are not always possible, as the access hole could emerge on the labial
surface (upper anterior incisors particularly). The recent development
of co-​axial implants and angled screw abutments is helping to resolve
this issue, although there are still drawbacks to these solutions. Marginal
leakage is less concerning at the crown abutment interface, as there is
no susceptibility to caries, but peri-​implant disease remains of concern.
There are advantages and disadvantages for both methods, and the clin-
ician should be aware of these to select the most appropriate one.
The difference in cost is unlikely to be a major consideration in this
scenario.
Keywords: cell aligned, reduced inter-​occlusal space, screw-​retained.
→ Michalakis K, Hirayama H, Garefis P. Cement-​retained versus screw-​
retained implant restorations: a critical review. International Journal of
Oral and Maxillofacial Implants. 2003;8:79–​28.

3. A ★★★★
In the UK, the practice of dentistry is protected for GDC-​registered
dentists and dental care professionals (DCPs) under the Dentists Act
984. Concentrations that release >0.% hydrogen peroxide must be
prescribed by a dental practitioner. This includes tooth whitening, which
has led to a large number of criminal prosecutions in recent years. The
Dentists Act also restricts the use of titles associated with practising den-
tistry, which could lead to prosecution, should a member of the public
claim they were a registered dentist or a DCP. These practices are re-
served for dentists. However, appropriately trained therapists, hygien-
ists, and clinical dental technicians can provide bleaching if prescribed
by a dentist. It is the dentist’s responsibility to conduct the initial clinical
appointment, which should routinely include photographs and a clearly
recorded shade. Reports regarding conduct of this nature should be re-
ported to the GDC, which follows up on the practice of illegal dentistry
within the UK.
Other laws and European Union regulations may be contravened by
practising dentistry illegally, and each case is assessed individually by the
GDC. Offering whitening gels that had concentrations in excess of 6%
hydrogen peroxide would also be deemed unlawful in the UK under the
Cosmetic Products Enforcement Regulations 203.
Keywords: personal trainer, tooth whitening, law.
→ General Dental Council. Position statement on tooth whitening. 206.
Available at: https://​www.gdc-​uk.org/​api/​files/​Tooth-​Whitening-​
Position-​Statement.pdf

https://t.me/DentalBooksWorld
https://t.me/DentalBooksWorld
Chapter 7 135

Oral and maxillofacial


surgery
Tariq Ali

‘All bleeding stops eventually.’


Oral and facial surgery has been practised in some form for millennia.
Hippocrates himself has described reducing jaw dislocations, and Sushtra
was performing reconstructive local facial flaps in India as early as sixth
century bc. The modern practice of oral and maxillofacial surgery (OMFS)
can be traced back to the battlefields of northern Europe in the early
twentieth century. Industrialized warfare produced horrific facial injuries
that were treated by frontline oral surgeons, and so the specialty of OMFS,
as a crossover between medicine and dentistry, developed. It became evi-
dent that both medical and dental education was necessary in order to
manage increasingly complex facial surgery that was being undertaken.
It can be argued that OMFS has evolved to be a truly general surgical
specialty, manipulating the hard and soft tissues of the head and neck and
having the skills to operate on neurovascular, glandular, and airway struc-
tures. There is variable exposure to OMFS during dental undergraduate
education and scarce exposure during medical undergraduate training.
Opportunities for postgraduate training in OMFS for dentists who are
not entertaining a career in the discipline are also limited. Knowledge of
the scope of OMFS practice is a bare minimum for any practising dentist,
as this can inform the limits of their individual competency, as well as
ensure a safe transfer of care for their patients.
The questions in this section are there to target the most commonly
tested and encountered aspects of OMFS for most junior dentists,
focusing on oral surgery, oral pathology, management of the medically
compromised patient, and trauma of the facial skeleton.
OMFS is an enormously satisfying endeavour and is recommended to
all junior dentists. It can form a granite-​like foundation of skills upon
which to build a career in clinical dentistry.
Key topics include:
● Trauma
● Oral surgery and exodontia
● Oral pathology
● Orthognathic surgery
● Temporomandibular joint surgery
● Cleft lip/​palate repair
● Craniofacial surgery
● Salivary disease
● Head and neck oncology and microvascular reconstruction
● Skin cancer
● Facial aesthetics.

https://t.me/DentalBooksWorld
136 Chapter 7 Oral and maxillofacial surgery

QUESTIONS

. A 74-​year-​
old man requires the extraction of his upper right
second premolar (UR5). He is currently taking warfarin for a
recent deep vein thrombosis. His international normalized ratio (INR)
was recorded as 2.6 when it was last assessed and has been stable over
the last 3 months. What is the maximum time recommended between
the extraction and the last INR reading? (Select one answer from the
options below.) ★
A 2 hours
B 24 hours
C 36 hours
D 48 hours
E 72 hours

2. A 24-​year-​old man with an 8-​pack year smoking history attends


in pain, following a routine extraction of his lower left first per-
manent molar (LL6) 3 days ago. He has deep-​seated, severe pain around
the extraction site. Medically, he is fit and well. Which is the single most
likely cause of his pain? ★
A Alveolar osteitis
B Fractured jaw
C Osteomyelitis
D Osteonecrosis of the jaw
E Retained root

3. A 64-​year-​old woman requires the extraction of her lower right


first molar. She previously received intravenous (IV) zoledronic acid
for breast cancer 6 years ago. Although no longer receiving zoledronic
acid, she is now taking denosumab. Which is the single most appropriate
management? ★
A Extract the tooth atraumatically
B Extract the tooth under antibiotic prophylaxis
C Maintain the roots as an overdenture abutment
D Perform surgical removal of the tooth
E Refer to your local Oral Surgery/​Oral Maxillofacial Department

https://t.me/DentalBooksWorld
questions 137

4. A 2-​year-​old man has been referred with recurrent bouts of


pericoronitis associated with his lower left third molar (LL8).
A dental panoramic tomogram (DPT) is taken, and incidentally, a large
unilocular lesion surrounding the crown of the unerupted lower right
third molar (LR8) is identified. What is the single most likely diagnosis
of this lesion? ★
A Calcifying epithelial odontogenic tumour (Pindborg tumour)
B Dentigerous cyst
C Odontogenic keratocyst
D Odontogenic myxoma
E Paradental cyst

5. A 23-​year-​old woman is having the extraction of an asymptomatic


upper right wisdom tooth. During the procedure, a large palatal
mucosal tear appears adjacent to the tooth, and the segment is now
mobile. Which single complication is most likely to have occurred?
(Select one answer from the options below.) ★★
A Crown fracture
B Oro-​antral communication
C Oro-​antral fistula
D Root fracture
E Tuberosity fracture

6. A 28-​year-​old man has received a blow to his right jaw. He reports


being unable to open his mouth wide or bite his teeth together. He
has no problems with his eyes and did not lose consciousness. He has
marked tenderness at the right angle of the mandible and over the left
parasymphysis. A right angle fracture and a left parasymphyseal fracture
are suspected. Which two plain film radiographs should be requested?
(Select one answer from the options below.) ★★
A Dental panoramic tomogram (DPT) and occipitomental 30 (OM30O)
B DPT and posteroanterior (PA) mandible
C Occipitomental 0 (OM0O) and OM30O
D OM30O and PA mandible
E Submentovertex and PA mandible

https://t.me/DentalBooksWorld
138 Chapter 7 Oral and maxillofacial surgery

7. A 46-​year-​old woman is reviewed following the extraction of an


upper right first molar (UR6). She says that she has been unable to
smoke properly since the extraction and has noticed a salty discharge.
A diagnosis of oroantral communication is made, and surgical correction
planned. Which is the single most appropriate surgical management in
this situation? ★★★
A Buccal advancement flap
B Modified three-​sided flap
C Palatal rotation flap
D Semi-​lunar flap
E Two-​sided flap

8. A 3-​year-​old woman is seen by the junior oral and maxillofacial


doctor in the Emergency Department with a large facial swelling
near the angle of her right mandible. Her temperature is 38.9°C. She
has 0-​mm mouth opening and is struggling to swallow her own saliva.
A grossly carious lower right second molar (LR7) is noted, and her floor
of mouth is raised and firm. The patient is asked to rest with her head
elevated above the level of her heart, and intravenous (IV) steroids and
antibiotics are commenced. The senior maxillofacial doctor asks that
the patient is discussed with another specialty. Which is the single most
appropriate specialty to call? ★★★
A Anaesthetics
B Ear, nose, and throat (ENT)
C Emergency medicine
D Haematology
E Intensive care medicine

9. A 27-​year-​old man received a blow to the right side of his head


during a rugby match and has blurry vision, which is corrected
by tilting his head. On ocular examination, his right eye appears ele-
vated and he reports double vision when he attempts to look straight
down to read. Which single cranial nerve is most likely to have been
traumatized? ★★★★
A Abducens
B Infraorbital
C Oculomotor
D Optic
E Trochlear

https://t.me/DentalBooksWorld
questions 139

0. A 3-​year-​old boy attends the Emergency Department (ED),


following a blow to the face during a recent cricket match. A ball
struck him directly on the left eye. He now has double vision on upward
gaze, with difficulty tracking objects with this eye. He feels unwell and
has vomited multiple times since the injury. There is no subconjunctival
haemorrhage or epistaxis, and his visual acuity is normal. What is
the single most likely diagnosis? (Select one answer from the options
below.) ★★★★
A Acute myospasm of the superior rectus
B Orbital floor fracture
C Retrobulbar haemorrhage
D Transection of the abducens nerve
E Traumatic brain injury

. A 35-​year-​old woman has pain behind their right eye, following
repair of an orbital floor fracture. The right eye is proptosed
and displays a fixed, dilated pupil. What is the single most likely cause of
these findings? ★★★★
A Dislodged fixation screw
B Retrobulbar haemorrhage
C Subconjunctival bleed
D Traumatic optic neuropathy
E White-​eyed blowout

2. A 36-​year-​old woman with a large submandibular abscess is


scheduled for extra-​oral drainage. An incision is made approxi-
mately 2 cm below, and parallel with, the body of the mandible on the
right-​hand side. Which single nerve is at greatest risk when carrying out
this procedure? ★★★★
A Cervical branch of the facial nerve
B Lingual branch of the mandibular nerve
C Mandibular branch of the facial nerve
D Mental branch of the mandibular nerve
E Transverse cervical of the cervical plexus

https://t.me/DentalBooksWorld
140 Chapter 7 Oral and maxillofacial surgery

3. A 26-​year-​old man attends the local Emergency Department,


following an alleged assault. The maxillofacial dental core trainee
is called to suture his 2-​cm lip laceration. The tissue forceps used display
the following symbol (see Figure 7.), and the new healthcare assistant
asks what the symbol means and what they should do with the item after
use. (Select one answer from the options listed below.) ★★★★
A Class II medical device—​wipe the surface and store for later use
B Class II medical device—​report to management that this item is not
suitable for use in this manner
C Do not reuse—​dispose in a sharps bin
D Do not reuse—​dispose in a clinical waste bin
E Send to the Central Sterilization Department for sterilization

Figure 7.
Permission to reproduce extracts from British Standards is granted by BSI Standards
Limited (BSI). No other use of this material is permitted. British Standards can be
obtained in PDF or hard copy formats from the BSI online shop: www.bsigroup.com/​
Shop

4. A 28-​year-​old woman had her lower left third molar extracted
6 months ago. The practice subsequently received a letter from
her solicitor requesting access to the patient’s clinical records, using a
subject access request. Under which single law is this request legally
valid? ★★★★
A Access to Health Records Act 990
B Access to Medical Reports Act 988
C Data Protection Act 208
D Dentists Act 984
E Freedom of Information Act 2000

https://t.me/DentalBooksWorld
answers 141

ANSWERS
. E ★ OHCD 6th ed. → p. 357
The INR should be checked ideally within 24 hours of the extraction.
However, this is often difficult to arrange, and in patients with stable INR
readings, up to 72 hours is deemed acceptable. Always check the local
working environment policy, as variations may exist.
Like all extractions, local haemostatic methods are fundamental in con-
trolling haemorrhage. The timing of the extraction and the day of the
week should also be considered. Appointments early in the morning and
towards the beginning of the week enable complications to be managed
more effectively.
A therapeutic range of 2–​4 is considered acceptable for patients under-
going routine dental surgery in primary care. Unstable INR, liver/​renal
impairment, significant alcohol intake, or the presence of a bleeding
diathesis should prompt referral to secondary care, as should combin-
ations of warfarin and antifibrinolytic drugs like aspirin and clopidogrel.
New oral anticoagulants (NOACs) are now available and patients
require careful management, as monitoring is still being developed and
immediate reversal is not currently possible. The cited recommended
reading contains a flow chart for the management of patients requiring
extractions who are taking NOACs.
Keywords: extraction, INR, maximum time.
→ Scottish Dental Clinical Effectiveness Programme. Management
of dental patients taking anticoagulants or antiplatelet drugs. 205.
Available at: http://​www.sdcep.org.uk/​published-​guidance/​
anticoagulants-​and-​antiplatelets/​

2. A ★
A dry socket is thought to be caused when a blood clot fails to form over
a socket; this can lead to bacterial ingress. Severe pain follows 2–​4 days
after an extraction. Predispositions include: smoking (vasoconstriction
and neutrophil dysfunction affect healing), traumatic extractions, poor
post-​operative care (poor care can dislodge a blood clot), local anaes-
thetic (vasoconstriction), bone diseases, and use of the oral contracep-
tive pill in females. It is commonly treated with irrigation and placement
of a sedative dressing such as Alveogyl® (iodine-​free).
Osteomyelitis is an infection of the bone. It is associated with local infec-
tion (dental abscess) or trauma (fracture or extraction). Osteomyelitis
can present radiographically with a moth-​ eaten appearance, after
roughly 3 weeks of infection. As the extraction in the question was
uncomplicated, it is unlikely to be the cause of the pain.
Small retained roots do not usually cause a problem, unless they are
infected, but again in this case, the extraction went as planned, with no
fractured apices.

https://t.me/DentalBooksWorld
142 Chapter 7 Oral and maxillofacial surgery

Osteonecrosis of the jaw can have a variety of causes (medication-​


related and radiotherapy-​induced). Patients at risk of this often have
underlying medical conditions (bone disease or cancer). The patient in
the question is fit and well.
Keywords: smoking, extraction, 3 days ago, lower molar.

3. E ★
Bisphosphonates are a group of drugs used in the management of sev-
eral diseases, including the skeletal manifestations of malignancy. These
agents serve to reduce osteoclastic activity and, as such, influence bone
healing following dental procedures, most notably dental extractions.
This can result in exposed necrotic bone that is difficult to manage—​
medication-​related osteonecrosis of the jaw (MRONJ).
Intravenous (IV) bisphosphonates would generally be classified as high
risk for MRONJ, whilst oral bisphosphonates used for <3 years are gen-
erally classified as low risk. Bisphosphonates have been shown to remain
inactive within bone for many years following cessation of treatment
and, as such, drug holidays confer no apparent benefit.
National guidance regarding bisphosphonate use is limited, and local pol-
icies may vary. However, a general consensus regarding high-​risk patients
would indicate referral to, or advice from, a specialist department to be
the most pragmatic management strategy. It would not be unreasonable
for a competent practitioner to atraumatically extract the tooth in prac-
tice, assuming they have fully informed consent and are able to manage
complications appropriately. Denosumab (monoclonal antibody) is an
alternative drug used to treat osteoporosis, which has also been impli-
cated in MRONJ.
Despite no high-​quality evidence, prophylactic antibiotic cover is often
given to high-​risk patients to reduce the risk of infection; no current con-
sensus is universally agreed upon.
Keywords: extraction, IV zoledronic acid, denosumab.
→ Ruggiero SL, Dodson TB, Fantasia J, et al. American Association of
Oral and Maxillofacial Surgeons position paper on medication-​related
osteonecrosis of the jaw—​204 update. Journal of Oral and Maxillofacial
Surgery. 204;72:938–​56.

4. B ★ OHCD 6th ed. → p. 392


These account for approximately 20% of all cysts and are common
in the posterior mandible of adolescents. Developing from the re-
duced enamel epithelium, they are associated with the amelodentinal
junction of unerupted teeth. These cysts are typically lined by thin,
non-​keratinized squamous epithelium. All of the answers can be asso-
ciated with unerupted or partially erupted teeth but are less common.
Pindborg tumours contain amyloid matrix that can calcify and are de-
scribed as having a driven snow appearance. Odontogenic keratocysts
are benign odontogenic tumours that are often multilocular and

https://t.me/DentalBooksWorld
answers 143

develop from the rests of Serres. They generally present in the


posterior mandible and show marked anteroposterior expansion.
Odontogenic myxoma are rare and benign neoplasms containing abun-
dant myxoid extracellular matrix. They are described as soap bubble/​
honeycomb appearance when visualized radiographically. Paradental
cysts usually occur in the mandible but are associated with partially
erupted wisdom teeth.
Keywords: unilocular lesion, surrounding the crown, unerupted,
third molar.

5. E ★★
Tearing of the palatal mucosa adjacent to an upper wisdom tooth during
an extraction is commonly associated with fracture of the maxillary
tuberosity. As such, this should be factored into any discussion when
obtaining informed consent.
Mobility of the tuberosity or an audible crack are additional signs fre-
quently detected, as mucosal tears can occur independently. Careful
assessment of the preoperative radiograph can help reduce the risk
of a tuberosity fracture occurring. Loss of the lamina dura, dense
trabeculation, or unfavourable root morphology could all predis-
pose to tuberosity fractures. Lone-​standing molars often pose the
greatest risk.
Management of the fracture is variable and can depend upon the status
of the tooth being extracted. Asymptomatic molars with no associ-
ated pathology could be splinted to adjacent teeth prior to rearran-
ging surgical removal following bony healing. Small tuberosity fractures
or unrestorable teeth with evident pathology would likely still warrant
removal. However, assessment of the size of the bone fragment and soft
tissue defect is important.
A crown fracture or root fracture can be ruled out because the tooth is
intact. An oroantral communication can result from a tuberosity fracture,
but this diagnosis usually would not be made unless a hole is visualized
following extraction or the patient returns with symptoms. If a commu-
nication is long-​standing and becomes epithelialized, then it would be
classified as a fistula.
Keywords: upper wisdom tooth, palatal tear, segment, mobile.

6. B ★★ OHCD 6th ed. → p. 470


The clinical history and examination suggest a mandibular fracture may
have occurred. DPTs provide excellent views of the majority of the man-
dible and are likely to be the first radiograph indicated. It is possible for
a fracture to be missed using a single film only, as the image is a two-​
dimensional representation of a three-​dimensional structure. Therefore,
a second parallax view (from a different position—​ideally perpendicular)
can identify fractures missed on single radiographs or yield more infor-
mation about them.

https://t.me/DentalBooksWorld
144 Chapter 7 Oral and maxillofacial surgery

A PA mandible (PA jaws) enables good visualization of the mandible,


from the condylar head to the body. Generally, these are the second
radiographs to be taken.
A standard occipitomental view is sometimes taken when fractures of
the coronoid process are suspected, and high condylar neck fractures
are often better visualized using a reverse Towne’s radiograph.
Note: oral pantomogram (OPG) is a term that can be used interchange-
ably with dental panoramic tomogram (DPT).
Keywords: mandible, fractures, radiographs.

7. A ★★★
The symptoms would indicate an oroantral communication or a fistula
is present. These can often resolve, following careful non-​surgical man-
agement and antibiotic use. However, should surgical treatment be re-
quired, the buccally advanced flap would be the most appropriate.
This involves creating a parallel or near parallel three-​sided flap that is
advanced across the defect, following excision of any epithelium lining
within the fistula. Scoring the periosteal surface of the flap enables
greater flexibility, although great care must be exercised when making
these incisions. Vertical mattress sutures are recommended for pri-
mary closure. A reduction in sulcus depth is one disadvantage of this
approach, which can pose problems with denture provision in the future.
Buccal fat pad can also be used to repair larger defects.
Palatal rotation flaps are an acceptable alternative [see Figure 7.2 which
shows a schematic of a buccal advancement flap (right) and a schematic
of palatal rotation flap (left)]. However, a large area of palate remains
exposed to heal by secondary intention, which often contraindicates its
use, especially in smokers. Smoking cessation is a fundamental compo-
nent to promoting wound healing with this complication.

Figure 7.2

https://t.me/DentalBooksWorld
answers 145

The remaining three options are flap techniques generally used for small
dento-​alveolar procedures.
Keywords: extraction, UR6, unable to smoke, salty discharge.
→ Scott P, Fabbroni G, Mitchell D. The buccal fat pad in the closure
of oro-​ antral communications: an illustrated guide. Dental Update.
2004;3:363–​6.

8. A ★★★
The clinical situation above requires immediate emergency management.
Inability to swallow saliva with marked trismus represents an airway risk,
and an anaesthetist is required to assess the patient prior to surgical
management. This is a time-​critical emergency, as severe swellings of this
kind can progress quickly, with loss of airway patency. An anaesthetist
can provide significant information regarding the severity of symptoms
and the urgency of treatment. They can also assess the patient prior to
general anaesthesia. Intravenous steroids and antibiotics can be given
to reduce the swelling and bacterial load, but the effects are delayed.
Awake, fibre-​optic intubation may be required if the swelling is extensive
and conventional intubation methods are unachievable.
Keywords: large facial swelling, struggling to swallow, floor of the mouth
raised and firm.

9. E ★★★★
The trochlear nerve (CN IV) carries efferent nerve fibres to the superior
oblique muscle. The muscle serves to depress and abduct the eyeball and
is the only extra-​ocular muscle supplied by the trochlear nerve. CN IV
is generally the most sensitive nerve to any insult, as it is the only cranial
nerve to emerge from the dorsal aspect of the brainstem and has the
longest intracranial course.
On examination, the patient would likely report blurred vision when
looking straight down and would have difficulty looking down and out on
the affected side. At rest, the eye may appear elevated and intorted, as
the stabilizing action of the superior oblique muscle against the superior
rectus is missing. These patients would often compensate with a head tilt
or chin tuck to the contralateral side to improve vision.
Damage to the abducens nerve would result in loss of function of the
lateral rectus muscle, and patients would be unable to look laterally from
the affected eye. Optic nerve damage would create visual field disturb-
ances, amongst other potential problems.
Damage to the oculomotor nerve, which supplies the remaining four
extra-​ocular muscles and the levator palpebrae superioris, would
result in drooping of the eyelid and a fixed, dilated pupil which would
fail to accommodate normally, with the eye gazing ‘down and out’.
Parasympathetic fibres from the ciliary ganglion control dilation of the
pupils and accommodation and can be an early sign of oculomotor nerve
damage. A good mnemonic is SO4 (superior oblique CN IV) LR6 (lateral
rectus CN VI).
https://t.me/DentalBooksWorld
146 Chapter 7 Oral and maxillofacial surgery

Keywords: corrected by tilting head (‘head tilt/​chin tuck’), elevated,


double vision on looking straight down.

0. B ★★★★ OHCD 6th ed. → p. 472


This clinical presentation in a young patient is likely to indicate an orbital
floor fracture and is commonly referred to as a ‘white-​eyed blowout’.
This occurs when soft tissue and muscle of the inferior rectus become
trapped within the orbital floor fracture, following a direct blow to
the eye.
Orbital floor fractures in children can often be more difficult to diagnose
due to the apparent absence of symptoms. The elastic nature of bone
in young patients can result in the orbital floor acting as a ‘trap door’,
essentially trapping the inferior rectus muscle following injury. This can
result in limitation of upward and downward gaze in the affected eye.
Oculocardiac/​oculovagal reflex can often obfuscate the diagnosis and
lead to a delay in treatment.
Ophthalmological assessment and CT scanning are appropriate investi-
gations prior to surgical correction of the fracture, with release of the
entrapped soft tissues. Delay in treatment can have serious ramifications
for the patient, and it is suggested that surgical treatment occurs within
48 hours of injury to avoid muscle ischaemia. All eye and facial trauma in
children should be approached with a high index of suspicion due to the
severity of complications.
Acute myospasm of the superior rectus is an unlikely diagnosis but might
cause difficulties looking up. Nausea and vomiting would discount this
diagnosis, unless combined with a traumatic brain injury. A traumatic
brain injury on its own would generally not present with these specific
eye signs. The abducens nerve controls the lateral rectus and so would
present with problems affecting the lateral movements of the eye.
Keywords: 3 years old, direct blow to the eye, double vision on upward
gaze, no subconjunctival haemorrhage.
→ Mehanna P, Mehanna D, Cronin A. White-​ eyed blowout frac-
ture: another look. Emergency Medicine Australasia. 2009;2:229–​32.

. B ★★★★ OHCD 6th ed. → p. 472


The signs and symptoms described most likely indicate a retrobulbar
haemorrhage, which is one of the few true maxillofacial emergencies.
Immediate surgical management with a lateral canthotomy is the treat-
ment of choice. However, these can often take a short amount of time
to arrange. As a junior dentist, immediate escalation to senior colleagues
and arrangement of medical management using mannitol, acetazolamide,
and a steroid (such as dexamethasone) will reduce the swelling in the
region and delay the progression of symptoms, thus improving the prog-
nosis. These drugs serve to decrease intravascular and intraocular pres-
sure, thus ‘buying time’ prior to surgery and improving the prognosis
post-​surgery. The protocol for medical management can vary by region,

https://t.me/DentalBooksWorld
answers 147

and the clinician should familiarize themselves with local trust policy early
on in a placement.
A dislodged fixation screw is unlikely at this time point if there was good
stability at placement and would not cause these symptoms.
Subconjunctival haemorrhage is a clinical finding indicative of trauma
and is not a diagnosis. If the posterior border cannot be seen, then
it is suggestive of a fracture, with bleeding coming from the ruptured
periosteum.
A traumatic optic neuropathy (TON) is where there is direct or indirect
damage to the optic nerve. The patient would have loss of vision and
pain, but the eye would not be tense and proptosed. A retrobulbar
haemorrhage should be ruled out if a TON is suspected.
A white-​eyed blowout is where the orbital floor has fractured and
recoiled, trapping the avulsed contents of the eye. This is generally more
of a concern in younger patients where the bone is more flexible. If not
managed rapidly, the trapped tissue and muscle can become ischaemic,
leading to long-​term diplopia. Patients would present with diplopia, re-
stricted upward gaze, and potentially a sunken eye. Patients can also have
an oculocardiac/​oculovagal reflex, in which upward gaze causes nausea,
bradycardia, and potentially asystole!
Keywords: pain, behind, eye, proptosed, fixed and dilated pupil.
→ Timlin H, Manisali M, Verity D, Uddin J, Osborne S. Traumatic orbital
emergencies. Focus—​Royal College of Ophthalmologists. 205. Available
at: http://​www.rcophth.ac.uk/​standards-​publications-​research/​ focus-
​articles/​

2. C ★★★★
Following a superficial incision, blunt dissection through the platysma is
carried out prior to navigating superiorly up to the abscess cavity. The
lingual border of the mandible is a useful surgical landmark to help locate
the abscess.
The mandibular branch of the facial nerve is most at risk with this ap-
proach. Stimulation of the nerve during the procedure would often
result, with visible contraction of the muscles surrounding the chin. The
cervical branch of the facial nerve runs lower down and should not be
near the incision site.
The mental nerve usually emerges between the roots of the mandibular
first and second premolar teeth, in line with the infraorbital nerve, the
supra-​orbital nerve, and the pupils and would not be at risk.
The lingual nerve branches off the posterior trunk of the mandibular
nerve. It is located medial to the ramus of the mandible, splitting from
the main trunk of the mandibular nerve just before it enters the man-
dible. It runs along the lingual aspect of the body of the mandible but
should not be at risk of damage, as the abscess cavity will be explored
by blunt subperiosteal dissection, and the nerve should be superficial
to this.

https://t.me/DentalBooksWorld
148 Chapter 7 Oral and maxillofacial surgery

The transverse cervical plexus arises deep to the sternocleidomastoid


muscle and supplies sensation to the skin on the front of the neck. They
are too far posterior from where the surgical incision is made to be
harmed. Figure 7.3a shows the facial nerve and its branches in the face.
The position of the parotid gland is indicated. Figure 7.3b shows cuta-
neous innervation of the head and neck and the cutaneous branches of
the nerves.
Keywords: extra-​oral, submandibular.

3. C ★★★★ OHCD 6th ed. → p. 744


The symbol in Figure 7. indicates that the medical device is ‘single use’
and not fit to be reused on subsequent patients. Additionally, as the
instrument has sharp tips, it should be disposed of in the sharps bin,
and not the clinical waste. Guidance from the Medicines and Healthcare
Products Regulatory Agency in 203 highlights the important legal and
regulatory implications surrounding items of single use, which refers to
a number of key legal documents, including the Medical Devices regula-
tions 2002, Health and safety at work Act etc. 974, and the Consumer
Protection Act 987, to name a few.
Patient safety is paramount in all dental surgeries and hospitals.
Throughout the past century, advances in medical and biological
research have highlighted a potential risk of persistent contamination
from a number of dental devices. As a result, in the United Kingdom, a
wide array of instruments and devices are no longer fit to be reused on
different patients, e.g. matrix bands, endodontic files, etc. Severe con-
sequences for both patients and practitioners can result if these regula-
tions are not adhered to. It would also be unethical and unprofessional
to knowingly put patients at risk and bring the profession into disrepute
with poor infection control standards.
There are three classes of medical devices, and this relates to the com-
plexity of the device and the stringency of regulation the device needs.
This is unrelated to the symbol shown.
Keywords: forceps, (symbol shown).
→ Medicines and Healthcare Products Regulatory Agency (MHRA).
Single-​use medical devices: implications and consequences of reuse. 208.
Available at: https://​www.gov.uk/​government/​uploads/​system/​up-
loads/​attachment_​data/​f ile/​403442/​Single-​use_​medical_​devices_​_​
implications_​and_​consequences_​of_​reuse.pdf

4. C ★★★★ OHCD 6th ed. → p. 732


Patients are able to access their own medical records under the Data
Protection Act 208. A ‘Subject Access Request’ can be submitted to
request health records. Third-​party data or data that may cause harm
to the patient on release could be redacted, if deemed necessary. Small
administration fees used to be able to be charged under the old Data
Protection Act of 998, but with the updated General Data Protection

https://t.me/DentalBooksWorld
answers 149

(a)

Temporal branch
Zygomatic
branches
Posterior
auricular branch Buccal
branches

Mandibular
branch
Cervical
branch

Figure 7.3a The facial nerve and its branches in the face. The position
of the parotid gland is indicated.
Reproduced from Atkinson Martin E, Anatomy for dental students, figure 23.12, page 236,
Copyright (2013) by permission of Oxford University Press.

(b)
Zygomaticotemporal
Supra-orbital
Zygomaticofacial

Supratrochlear
Greater occipital External nasal

Auriculotemporal
Infraorbital
Lesser occipital Buccal

Cutaneous Mental
branches of
dorsal rami Great auricular
of C3–C5 Transverse cervical
Supraclavicular

Figure 7.3b Cutaneous innervation of the head and neck and cuta-
neous branches of the nerves.
Reproduced from Atkinson Martin E, Anatomy for dental students, figure 23.13, page 238,
Copyright (2013) by permission of Oxford University Press.
https://t.me/DentalBooksWorld
150 Chapter 7 Oral and maxillofacial surgery

Regulation (GDPR) coming into force in 208, a fee is now not normally
chargeable, unless the request is excessive, unfounded, or requires mul-
tiple copies.
Requesting health records for a relative who has deceased, e.g. the
executor of a will, would be submitted under the Access to Health
Records Act 990. The Freedom of Information Act 2000 deals with
non-​personal data from public organizations. In medical settings, these
data can be of concern to patients, e.g. environmental issues.
Keywords: Subject Access Request, their own clinical records.
→ Information Commissioner’s Office. Rights of access. Available
at: https://​ico.org.uk/​f or-​organisations/​guide-​to-​the-​general-​data-​
protection-​regulation-​gdpr/​individual-​rights/​right-​of-​access/​

https://t.me/DentalBooksWorld
Chapter 8 151

Oral medicine and


oral pathology
Raheel Aftab

‘It doesn’t look like anything sinister, but we need to take a look
under the microscope just to be sure.’
The mouth has often been looked at as a window into the body, and this
is no truer than in oral medicine. The oral mucosa is a highly adapted
and robust tissue, which, at the same time, can be very susceptible to
changes in homeostasis. Dysregulation of the immune system, alter-
ations in cellular signalling pathways, or insult from exogenous stimuli can
lead to an array of weird and wonderful oral lesions.
Clinicians today are likely to see changes to the oral mucosa on a
regular basis—​from common ulcers, bony lumps, or white patches to
more exotic pigmented lesions or unusual lumps. It is therefore vital to
have good basic knowledge of common conditions and be able to iden-
tify lesions that need urgent referral and treatment. It is important to
take a thorough medical history, as many oral symptoms can be associ-
ated with systemic conditions or changes in medication. A temporal link
can be a good indicator of causation from medication changes.
Having a strategic method of constructing a list of differential diag-
noses, such as the surgical sieve, can be a great aide-​memoire to ensure
all the pertinent questions and investigations have been completed.
However, it must be noted that many conditions cannot be accurately
diagnosed without histological examination, and therefore, referral for
specialist input is commonplace.
Oral medicine can be a tricky discipline, fraught with challenging pa-
tients to manage, particularly those with chronic conditions. Conversely,
the diagnostic challenges make for a thoroughly rewarding and stimu-
lating discipline. The questions in this chapter will test your knowledge of
disease symptoms, links to medical conditions, and patient management.
Key topics include:
● Patient assessment and diagnosis
● Investigations
● Basic histology
● Infections (bacterial, viral, and fungal)
● Ulcers
● Soft tissue swelling
● Bony lumps
● Systemic conditions
● White, red, pigmented, and mixed patches
● Oral cancer
● Pharmacology.

https://t.me/DentalBooksWorld
152 Chapter 8 Oral medicine and oral pathology

QUESTIONS

. A 45-​year-​old woman has a sore tongue. She has red atrophic


areas on the dorsum and lateral border of the tongue with demar-
cated white borders (see Figure 8.) (see Colour Plate section). She says
that the appearance has been inconsistent over time. What is the single
most likely diagnosis? ★
A Ankyloglossia
B Geographic tongue
C Glossodynia
D Macroglossia
E Median rhomboid glossitis

2. A 33-​year-​old woman has recent-​onset pain from her jaw. She has
recently become a new mother. The pain radiates towards her ear
and is worse in the mornings. Her temporomandibular joint (TMJ) clicks
on opening and closing but is unrestricted and causes no discomfort. Pain
is elicited on palpation of the masseter muscles. What is the single most
appropriate first-​line treatment? ★
A Prescribe non-​steroidal anti-​inflammatory drugs
B Provide a soft occlusal splint
C Reassure and advise a soft diet with warm compresses
D Refer for acupuncture
E Refer for arthrocentesis

3. A fit and well 4-​year-​old adolescent girl has a well-​circumscribed


brown, pigmented area on her lower lip. She says that it is
unchanged since she noticed it 2 years ago. Clinically, it is an isolated le-
sion, flush with the labial mucosa, and measuring 3 × 2 mm in size. What
is the single most likely diagnosis? ★
A Amalgam tattoo
B Malignant melanoma
C Melanotic macule
D Pigmentation due to Peutz–​Jeghers syndrome
E Pigmented naevus

https://t.me/DentalBooksWorld
QUESTIONS 153

4. A 60-year-old man attends a primary care dental practice with an


ulcer on the ventral surface of his tongue for 4 weeks. The ulcer
bleeds and causes discomfort when touched. He has a 35-pack year
smoking history. He has sharp, fractured teeth on the same side as the
lesion. What is the single most appropriate management? ★
A In-house incisional biopsy
B Non-urgent referral to the local Ear, Nose, and Throat (ENT) Unit
C Smooth the edges of the sharp teeth
D Take clinical photographs and monitor
E Urgent referral to the local Head and Neck Maxillofacial Unit

5. A 44-year-old woman has painless white lesions bilaterally on her


buccal mucosa, which have been present for 4 months (see Figure
8.2) (see Colour Plate section). She recently had a patchy rash on her
shins, which has resolved. She has multiple amalgam fillings in the upper
right and lower right permanent molars. What is the single most likely
diagnosis? ★
A Chronic hyperplastic candidiasis (CHC)
B Lichenoid reaction
C Oral lichen planus
D Squamous cell carcinoma
E White sponge naevus (WSN)

6. A medically fit and well 25-year-old woman has recurrent soreness


under her tongue. She has approximately 20 ulcers, each of which
is a couple of millimetres in diameter on the floor of her mouth. What is
the single most likely diagnosis? ★
A Crohn’s disease
B Herpetiform recurrent aphthous stomatitis (RAS)
C Major RAS
D Minor RAS
E Traumatic ulcer

https://t.me/DentalBooksWorld
154 Chapter 8 Oral medicine and oral pathology

7. A 60-​year-​old man has a red patch limited to the denture-​bearing


area of his maxilla. His dentures have been recently remade and
are well fitting. A provisional diagnosis of chronic erythematous candid-
iasis is made. However, he has atrial fibrillation and is taking warfarin.
What is the single most appropriate management? ★
A Reinforce denture hygiene and prescribe amphotericin tablets
B Reinforce denture hygiene and prescribe chloramphenicol ointment
C Reinforce denture hygiene and prescribe chlorhexidine gel
D Reinforce denture hygiene and prescribe miconazole oral gel
E Reinforce denture hygiene and prescribe nystatin suspension

8. A 3-​year-​old girl has bloody crusty lips, widespread oral ulceration,


and a temperature of 38.5°C. Her mother reports that the con-
dition has lasted for over 3 weeks and says that she is struggling to eat.
What is the single most appropriate management? ★★
A Prescribe aciclovir tablets
B Prescribe benzydamine hydrochloride mouthwash
C Prescribe chlorhexidine gluconate mouthwash
D Reassure the mother and monitor
E Refer for a specialist’s opinion

9. A 28-​year-​old woman presents with this intraoral appearance (see


Figure 8.3) (see Colour Plate section). She says that her previous
dentist has suggested avoiding cinnamon in her diet. She has no systemic
symptoms, but her lower lip is enlarged. What is the single most likely
diagnosis? ★★
A Coeliac disease
B Crohn’s disease
C Linea alba
D Orofacial granulomatosis (OFG)
E Ulcerative colitis

https://t.me/DentalBooksWorld
questions 155

0. A 79-​year-​old man has sore, crusted skin lesions at the corner
of his mouth. He wears very old complete dentures, which have
a freeway space of 2 mm. Swab results indicate mixed Candida albicans
and Staphylococcus aureus infection, and blood tests exclude haematinic
deficiencies. Which is the single most appropriate management? ★★
A Prescribe fusidic acid cream, and arrange for new dentures to be
made with a decreased occlusal vertical dimension (OVD)
B Prescribe fusidic acid cream, and arrange for new dentures to be
made with an increased OVD
C Prescribe miconazole cream, and arrange for new dentures to be
made with a decreased OVD
D Prescribe miconazole cream, and arrange for new dentures to be
made with an increased OVD
E Prescribe nystatin suspension, and arrange for new dentures to be
made with an increased OVD

. A 57-​year-​old woman has a dry mouth and dry eyes. Recent
blood tests confirm anti-​Ro and anti-​La autoantibodies, and a
labial gland biopsy demonstrates multiple dense foci of lymphocytic infil-
trate in a 4 mm2 area. She is otherwise fit and well, with no other medical
conditions. What is the single most likely diagnosis? ★★
A Chronic bacterial sialadenitis
B Primary Sjögren’s syndrome
C Sarcoidosis
D Sicca syndrome
E Systemic lupus erythematosus (SLE)

2. A 39-​year-​old man attends the Maxillofacial Department with


sweating on eating and flushing of the skin. He had a pleo-
morphic adenoma surgically removed from his right parotid approxi-
mately 2 weeks ago. What is the single most likely diagnosis? ★★
A Frey’s syndrome
B Graves’ disease
C Horner’s syndrome
D Melkersson–​Rosenthal syndrome
E Sicca syndrome

https://t.me/DentalBooksWorld
156 Chapter 8 Oral medicine and oral pathology

3. A 50-​year-​old woman has right-​sided jaw pain. She describes it


as a recurrent, excruciating shooting ‘electric shock’-​like pain of
rapid onset and short duration. What is the single most likely preliminary
diagnosis? ★★
A Atypical facial pain
B Glossopharyngeal neuralgia
C Irreversible pulpitis
D Reversible pulpitis
E Trigeminal neuralgia

4. A 73-​year-​old man has been suffering with ongoing pain for the
past 3 months. He has severe, sudden-​onset unilateral head-
aches, which becomes worse when chewing. He has a temperature of
38.6°C. The area is tender to touch, and the pulse from the side of the
head on the affected side is not detectable. Which is the single most
likely diagnosis? ★★★
A Atypical facial pain
B Cluster headache
C Myofascial pain
D Temporal arteritis
E Trigeminal neuralgia

5. A 38-​year-​old man has diffuse bilateral white lesions on his


buccal mucosa and floor of his mouth. The lesions are asymp-
tomatic but have a rough texture. He has never smoked and says he
has had the lesions since 2 years of age. What is the single most likely
diagnosis? ★★★
A Frictional keratosis
B Leukoedema
C Lichen planus
D Stomatitis nicotina
E White sponge naevus

https://t.me/DentalBooksWorld
questions 157

6. A 53-​year-​old woman re-​attends with unilateral, pressure-​like


pain in the maxilla, radiating towards the ear and associated
with an itching sensation. She had a routine extraction of the upper right
second premolar 6 weeks ago, followed by root canal treatment of the
heavily filled adjacent first molar 3 weeks later. On both occasions, pain
initially settled but subsequently returned. What is the single most likely
diagnosis? ★★★
A Atypical facial pain (AFP)
B Atypical odontalgia (AO) (phantom tooth pain)
C Burning mouth syndrome (oral dysaesthesia)
D Temporomandibular dysfunction (TMD)
E Trigeminal neuralgia

7. A fit and healthy 27-​year-​old man is concerned about a lesion


on his tongue (see Figure 8.4) (see Colour Plate section). He
does not report any other lesions anywhere else. The lesion is non-​
tender, and no other oral lesions are seen, but cervical lymphadenopathy
is detectable. He is a non-​smoker. A blood test is requested to aid diag-
nosis for the suspected lesion. What is the single most likely provisional
diagnosis? ★★★★
A Major recurrent aphthous stomatitis
B Primary syphilitic chancre
C Squamous cell carcinoma
D Traumatic ulcer
E Tuberculosis

8. A 62-​ year-​ old woman has a recurrent swelling under her
tongue during mealtimes. She has a mobile, unilateral, firm sub-
mandibular swelling. She is allergic to iodine. Which is the single most
appropriate definitive investigation? ★★★★
A Computed tomography of the mandible
B Fine-​needle aspiration
C Magnetic resonance imaging of the mandible
D Sialography
E Standard true lower occlusal radiograph

https://t.me/DentalBooksWorld
158 Chapter 8 Oral medicine and oral pathology

9. A 55-​year-​old man has very painful, recurrent intraoral ulcer-


ations that have been ongoing for the past 6 months (see Figure
8.5) (see Colour Plate section). He says that the ulceration is preceded
by very short-​lived blisters. He also has skin lesions and an upper layer
of the skin shears with lateral pressure. To complement conventional
histology, direct immunofluorescence analysis of a recent biopsy is re-
quested. What is the single most likely result on this biopsy? ★★★★
A negative result
B Granular immunoglobulin A (IgA) deposits and C3 at dermal papillae
C Intercellular immunoglobulin G (IgG) and C3
D Linear IgA and C3 at the basement membrane
E Linear IgG and C3 at the basement membrane

20. A 65-​year-​old man attends, complaining of a red sore tongue.


He also reports backache and ongoing fatigue for the past
3 months. He had radiotherapy for an extramedullary plasmacytoma
in his upper respiratory tract 4 years previously. Blood tests from his
general medical practitioner confirm hypercalcaemia, normocytic and
normochromic anaemia, and elevated immunoglobulins. What is the
single most likely diagnosis? ★★★★
A Chondrosarcoma
B Langerhans cell histiocytosis X
C Metastatic squamous cell carcinoma
D Multiple myeloma
E Paget’s disease of bone

2. A 5-​year-​old woman has had a biopsy of a white patch from


the buccal gingivae. The pathologist has included the slides vis-
ible in Figure 8.6 (see Colour Plate section). What is the single most
likely diagnosis? ★★★★
A Chronic hyperplastic candidiasis
B Discoid lupus erythematosus (DLE)
C Frictional keratosis
D Lichen planus
E Mucous membrane pemphigoid (MMP)

https://t.me/DentalBooksWorld
questions 159

22. A 24-​year-​old man who is diagnosed with moderate autistic


spectrum disorder has bleeding swollen gums, which his carers
have been unable to brush because they are sore. He appears lethargic
and only eats crisps. Early loss of periodontal attachment is present, and
generalized hyperplastic gingivae are noted. His full blood count (FBC)
shows a white blood cell (WBC) count of 7.4 × 09/​L. The patient takes
melatonin. What is the single most likely diagnosis? ★★★★
A Acute lymphoblastic leukaemia (ALL)
B Acute necrotizing ulcerative gingivitis (ANUG)
C Drug-​induced gingival overgrowth (DIGO)
D Pyogenic granuloma
E Vitamin C deficiency

23. A 9-​year-​old man recently had his unerupted LR8 surgically


extracted. A cyst is also removed that is associated with the
lateral aspect of the unerupted crown. Histopathology shows a regular
cyst lining, consisting of stratified squamous epithelium. The basal cells
are palisaded columnar cells which show high levels of mitotic figures.
Parakeratinization is noted on the surface epithelial cells. The cyst con-
tents were pale, with a low protein content consisting mainly of albumin.
A copy of the slide is available in Figure 8.7 (see Colour Plate section).
What is the single most likely diagnosis? ★★★★
A Dentigerous cyst
B Eruption cyst
C Lateral periodontal cyst
D Odontogenic keratocyst
E Radicular cyst

24. A 62-​year-​old man attends for a review of his Behçet’s disease,


following a recent episode of uveitis. Upon arrival, he trips
on some torn carpet in the entrance of your practice and is taken by
ambulance to Accident and Emergency after losing consciousness briefly.
The fall results in a broken jaw, and the patient is admitted to hospital
for 2 days and requires 4 weeks off work. To which single organization
should this incident be reported? ★★★★
A Care Quality Commission
B Dental Indemnity Firm
C Environment Agency
D General Dental Council
E Health and Safety Executive

https://t.me/DentalBooksWorld
160 Chapter 8 Oral medicine and oral pathology

ANSWERS
. B ★ OHCD 6th ed. → p. 430
Geographic tongue (synonyms: benign migratory glossitis or erythema
migrans) is an inflammatory condition of unknown aetiology. This scen-
ario is typical of a patient presenting with this condition, with its name
explaining the classical clinical appearance of the lesions. The condition
can be associated with psoriasis. The condition may be symptomatic, and
analgesic mouthwashes may help relieve the pain in some cases.
Ankyloglossia, also known as tongue tie, is a developmental anomaly
which may decrease the mobility of the tongue tip and is caused by an
unusually short and thick lingual frenulum.
Macroglossia is the term given to an enlarged tongue, which may have
a congenital or an acquired component. It has been reported in acro-
megaly and Down’s syndrome.
Glossodynia, also known as burning mouth syndrome/​oral dysaesthesia,
may be accompanied by glossitis, but classically the tongue appears
normal. It can be caused by anaemia, lichen planus, and Candida infec-
tions. In cases where there is no precipitating factor, the condition may
be psychogenic in origin. The history given in this scenario is not typical
of glossodynia.
Median rhomboid glossitis is a form of chronic atrophic candidiasis af-
fecting the dorsum of the tongue, commonly anterior to the circumval-
late papillae. Clinically, a single and non-​migratory area of depapillation
is present, and the condition is more common in those using inhaled
steroids.
Keywords: sore tongue, atrophic, white borders, inconsistent.

2. C ★ OHCD 6th ed. → pp. 458–​60


This presentation is indicative of myofascial pain. The muscles are often
tender, and headaches can be an associated factor. Sometimes, limitation
of mouth opening may be a problem. Explain the likely pathogenesis
(often a parafunctional habit, potentially related to stress—​in this case,
the patient is a new mother), and provide reassurance that the condition
is often self-​limiting. Jaw rest, including eating a soft diet, will reduce the
workload for the masticatory muscles and should help reduce the symp-
toms. This can be combined with warm compresses, which promote
blood flow to the region. Simple over-​the-​counter analgesics, such as
ibuprofen, may be advised in the short term if safe to do so.
Occlusal splints or appliances can help manage TMDs but are usually
only considered if the patient’s problems persist.
Arthrocentesis is not required in this case. The TMJ may click in some
individuals. However, if there is no discomfort and opening is not sig-
nificantly limited, then this should only be documented and monitored.

https://t.me/DentalBooksWorld
answers 161

Acupuncture is not a first-​line treatment for myofascial pain and is re-


served for cases of localized, unresponsive myalgia.
Keywords: new mother, worse in morning, pain on palpation of muscles.
→ Durham J, Aggarwal V, Davies SJ, et al.; Royal College of Surgeons.
Temporomandibular Disorders (TMDs): an update and management guid-
ance for primary care from the UK Specialist Interest Group in Orofacial Pain
and TMDs (USOT). 203. Available at: http://​www.rcseng.ac.uk

3. C ★ OHCD 6th ed. → p. 424


Melanotic macules are caused by a localized, benign increase in melanin
by basal melanocytes. They are commonly found on the lips and are
< cm in size. See Figure 8.8 (see Colour Plate section) which shows
idiopathic oral melanotic macules.
Malignant melanomas are aggressive carcinomas but rarely affect the oral
mucosa. Orally, they are most commonly found on the hard palate or
the maxillary gingivae. Borders are poorly defined; the lesion generally
grows in size, and it can be ulcerated or bleeding. They typically present
in patients above the age of 30.
Amalgam tattoos can occur if fractured amalgam is left submucosally
during surgical procedures. They are most likely to be found on the
gingivae, appearing as blue/​grey lesions and sometimes detectable on
radiographs.
Mucosal pigmentation around the oral cavity is characteristic of Peutz–​
Jeghers syndrome; multiple small macules are present circumorally. The
condition is hereditary and is associated with intestinal polyps, which
have a low chance of malignant change.
Pigmented naevi look very similar to melanotic macules. However, they
are more commonly found on the palate or buccal mucosa, with the
average age of presentation being around 30. Naevi tend to have a less
well-​defined border than melanotic macules and are often slightly raised.
It is very difficult to differentiate between melanotic macules, melan-
omas, and naevi clinically, and a definitive diagnosis can only be made
with a biopsy.
Keywords: circumscribed, brown, unchanged, isolated, flush.

4. E ★ OHCD 6th ed. → p. 428


This clinical description suggests a squamous cell carcinoma (SCC). In
this scenario, the following should raise suspicion and warrant an urgent
referral to the local Head and Neck Unit:
● Use or consumption of cigarettes/​betel-​nut/​alcohol (alcohol and cig-

arettes have a synergistic relationship)


● Rolled margins of ulcer
● Bleeding ulcer
● Unexplained ulcer persisting for longer than 3 weeks

https://t.me/DentalBooksWorld
162 Chapter 8 Oral medicine and oral pathology
● Ulcers on the floor of the mouth or ventral or lateral surface of the
tongue.
The local Head and Neck Cancer Unit may be run by an ENT and
should be referred there if this is the local arrangement. However, a
Maxillofacial Unit would be more appropriate, if available.
An incisional biopsy is the first test that would be performed on such
a lesion, but it would be inappropriate to do it in practice. It should be
performed in a secondary care environment, as part of the cancer care
pathway.
Urgent referral and diagnosis are the most important factors affecting
the outcome of a suspected cancer. In the United Kingdom, urgent re-
ferrals must be seen by the Head and Neck Unit within 2 weeks. Due
to the aggressive nature of SCCs, you would not want to monitor this
lesion in primary care.
Sharp teeth may cause trauma, but the clinical signs and risk factors pre-
sent mean histological examination takes precedence.
Keywords: ulcer, ventral surface, 4 weeks, bleeds, smoking.
→ National Institute for Health and Care Excellence. Suspected cancer: rec-
ognition and referral. NICE guideline [NG2]. 205. Available at: https://​
www.nice.org.uk/​guidance/​ng2

5. C ★ OHCD 6th ed. → p. 422


Lichen planus is an autoimmune disease of the skin and/​or mucosa of
unknown aetiology. Mild forms are self-​limiting, whilst severe forms can
be managed with topical corticosteroids. There is, however, no cure. In
0.4–​3% of cases, oral lichen planus may undergo malignant change, par-
ticularly in patients with erosive lichen planus or associated risk factors.
Therefore, most cases are kept under long-​term review. Figure 8.2 (see
Colour Plate section) shows reticular, non-​erosive lichen planus of the
buccal mucosa, but other variants are well documented.
Lichenoid reactions are lichen planus-​like lesions with a known trigger.
Triggers could be any foreign objects (e.g. amalgam), which need to be in
direct contact with the mucosa to cause a lesion. Such lesions can also be
caused by non-​steroidal anti-​inflammatory drugs (NSAIDs) and antimal-
arial, antihypertensive, or diabetic medications. The patient only has uni-
lateral amalgams, which does not correlate with the bilateral presentation.
CHC is a fungal infection of the oral mucosa caused primarily by Candida
albicans. This type of Candida is usually seen at the buccal commis-
sures and is more common in smokers. It presents as a homogenous
or speckled white plaque and requires monitoring as it is considered
premalignant.
WSN is a relatively rare hereditary condition that presents as a symmet-
rical, corrugated white plaque.
Squamous cell carcinomas (SCCs) more commonly present on the
floor of the mouth or ventral surface of the tongue. SCCs are also

https://t.me/DentalBooksWorld
answers 163

exceptionally unlikely to present as multiple bilateral lesions. The patient


is also likely to have a history of smoking.
Keywords: bilaterally, buccal mucosa, rash on shins.

6. B ★ OHCD 6th ed. → p. 46


Herpetiform RAS occurs on both keratinized and non-​ keratinized
mucosa, with non-​keratinized sites being the most common. The ulcers
are usually very small but sometimes may coalesce to form larger ulcers.
Numbers vary, but patients may present with ten or more ulcers at a
time. Herpetiform RAS responds well to treatment with chlortetracyc-
line mouthwash.
Major RAS can affect keratinized and non-​keratinized tissue in the oral
cavity. Lesions are usually >0 mm in diameter and occur in small num-
bers (–​3). Lesions take longer to heal (up to 2 months), often leaving
behind scar tissue.
Minor RAS usually presents on non-​keratinized tissue. Lesions are usu-
ally under 0 mm in diameter and occur in small numbers (–​5). Healing
takes up to 2 weeks without scarring.
Traumatic ulcers usually present as isolated lesions and may have a
readily identifiable cause (sharp or fractured tooth/​denture compo-
nent). They also tend to have a white keratotic outline. The sulci and
palate may be affected in cases where patients hold tablets in their
mouth (e.g. aspirin).
Crohn’s disease may cause oral ulcers, as malabsorption can lead to defi-
ciencies in iron or vitamin B2. However, there is no indication of this in
this scenario, as the patient is medically fit and well.
RAS patients often have a family history of the condition. For mild
cases, symptomatic relief can be provided with an analgesic mouthwash.
Topical steroids can also be prescribed in primary care, whilst more
severe cases may require systemic immunomodulators; this is provided
in a hospital setting.
Key words: recurrent, under the tongue, 20 ulcers, couple of millimetres.

7. E ★ OHCD 6th ed. → p. 30


Chronic erythematous candidiasis (denture stomatitis) arises from a
fungal infection secondary to poor denture hygiene and tissue trauma.
Therefore, these factors should be rectified before issuing pharmaco-
logical treatment.
Topical antifungal treatment is required, and miconazole gel is the treat-
ment of choice, but in this instance, the patient is taking warfarin for
atrial fibrillation. Azole antifungals inhibit liver cytochrome P450 enzymes
and therefore increase the efficacy of warfarin. To avoid adverse drug
interactions, nystatin suspension would be a safer choice. Newer novel
oral anticoagulants (NOACs), such as apixaban, are becoming more
common and at present are not routinely monitored.

https://t.me/DentalBooksWorld
164 Chapter 8 Oral medicine and oral pathology

Table 8. Common cytochrome P450 inducers and inhibitors


Inducers Inhibitors
Carbamazepine Sodium valproate
Rifampicin Isoniazid
Alcohol (chronic use) Cimetidine
Phenytoin Azole antifungals (ketoconazole, miconazole,
fluconazole)
Erythromycin
Metronidazole

Amphotericin tablets were a traditional alternative but are no longer


manufactured. Chlorhexidine has some antifungal properties but is not
a conventional antifungal medication. Chloramphenicol ointment is an
antibacterial agent typically used in ophthalmology. See Figure 8.9 (see
Colour Plate section) which shows Candida-​associated, denture-​induced
stomatitis (chronic erythematous candidiasis) affecting the area of
mucosa covered by a partial denture. Some of the more common cyto-
chrome P450 inducers/​inhibitors are listed in Table 8..
Keywords: chronic erythematous candidiasis, warfarin.

8. E ★★ OHCD 6th ed. → p. 40


This is a classic description of primary herpetic gingivostomatitis (PHG).
See Figure 8.0 (see Colour Plate section) which shows a patient suf-
fering from PHG of the lips and perioral skin. The causative organism
is herpes simplex virus. Along with general malaise and generalized oral
ulceration, cervical lymphadenopathy may be present. However, the
condition usually resolves itself within 2 weeks. Failure to resolve within
this time period warrants a referral to a specialist to exclude underlying
blood dyscrasias or other systemic conditions.
Referring to a specialist to search for potential underlying health prob-
lems takes priority in this case over the other options, which may be
suitable for most other cases of PHG, as described below. In a fit and
healthy patient, reassuring and advising the patient to rest and take plenty
of fluids is sufficient, as PHG is self-​limiting and resolves within 2 weeks.
Aciclovir is ineffective once the infection is clinically detectable (vesicles
and pyrexia). However, aciclovir may be helpful for severe infections or
for immunocompromised patients.
The antiseptic properties of chlorhexidine gluconate can be helpful in
reducing secondary infection.
Benzydamine hydrochloride can help to relieve some discomfort from
the oral ulceration.
Keywords: bloody crusty lips, lasted for over 3 weeks, struggling to eat.

https://t.me/DentalBooksWorld
answers 165

→ Nair RG, Salajeghen A, Itthagarun A, Pakneshan S, Brennan MT,


Samaranayake LP. Orofacial viral infections-​
-​
an update for clinicians.
Dental Update. 204;4:58–​24.

9. D ★★ OHCD 6th ed. → p. 444


Figure 8.3 (see Colour Plate section) shows the buccal mucosa with a
cobblestoned appearance. This can occur in Crohn’s disease or in OFG.
OFG is a term used to describe a syndrome, which presents with facial
and oral tissue swellings that show non-​caseating granulomas on histo-
logical examination. The oral presentation is similar to that of Crohn’s
disease, but without any gastrointestinal problems. In this scenario,
the patient does not present with systemic symptoms, and so Crohn’s
disease can be provisionally eliminated. It is thought that OFG can be
exacerbated by certain foods; patients are therefore advised to avoid
cinnamon and benzoates in their diets.
Ulcerative colitis does not cause oral swellings. It is more likely to cause
aphthous-​like ulcers, along with gastrointestinal problems. Pyostomatitis
vegetans is an oral condition that is highly specific for ulcerative colitis.
Coeliac disease can present with oral ulcers or other symptoms asso-
ciated with nutritional deficiencies; these are not noted in this scenario.
Frictional keratosis along the occlusal plane (linea alba) is commonly
associated with cheek biting and may indicate a parafunctional habit.
Typically, it is less raised in appearance than the image above and is not
associated with orofacial swelling.
Keywords: avoid cinnamon, no systemic symptoms, enlarged lip.

0. D ★★ OHCD 6th ed. → p. 44


Sore, erythematous crusted lesions at the corner of the mouth are indi-
cative of angular cheilitis. Management involves excluding haematinic
deficiencies and underlying immunosuppression. See Figure 8. (see
Colour Plate section) for a clinical example of severe angular cheilitis as-
sociated with chronic oral candidiasis. Appropriate antimicrobial therapy
should be provided, and management of predisposing factors must be
addressed.
Dentures with a reduced OVD or excessive freeway space can predis-
pose to the development of angular cheilitis and should be managed by
providing new dentures at an increased OVD (approximately 2–​6 mm
of freeway space).
Extra-​oral infections can be managed with application of a suitable cream,
whilst intraoral infections will require use of a gel or rinse. Bacterial
Staphylococcus aureus infection will respond to fusidic acid cream. Mixed
infections will respond well to miconazole, which has antifungal and anti-
bacterial properties, although it is mainly an antifungal medication.
Nystatin (polyene class of antifungals) binds to ergosterol in the fungal
cell membrane, whilst miconazole (imidazole class of antifungals) blocks
ergosterol synthesis.

https://t.me/DentalBooksWorld
166 Chapter 8 Oral medicine and oral pathology

Keywords: crusted skin, corner of mouth, freeway space of 2 mm,


mixed Candida albicans and Staphylococcus aureus infection.

. B ★★ OHCD 6th ed. → p. 434


Subjective signs of dry eyes and dry mouth should always be investi-
gated. Sjögren’s syndrome results from immune destruction of glan-
dular tissue, which can lead to dry mouth (xerostomia) and dry eyes
(keratoconjunctivitis sicca). Commonly, the lacrimal and salivary glands
are damaged, but other body sites can be affected. Where a patient has
dry eyes and mouth, but no signs of autoimmune destruction, it is known
as sicca syndrome, but many use these terms synonymously.
Objective tests for dry eyes and dry mouth can be conducted, including
Schirmer’s test and salivary flow tests. Anti-​Ro and anti-​La autoanti-
bodies are extractable nuclear antibodies (ENAs), which can be detected
using special blood tests. Patients with primary Sjögren’s syndrome are
often positive for these tests, which, when combined with lymphocytic
infiltrates on a labial gland biopsy, are indicative of Sjögren’s syndrome.
The absence of an additional autoimmune condition would suggest a
diagnosis of primary Sjögren’s syndrome. In the presence of additional
autoimmune conditions, such as systemic lupus erythematosus (SLE), it
is referred to as secondary Sjögren’s syndrome.
Other objective tests used to investigate Sjögren’s syndrome include
sialography and ultrasound. Not all tests are required to be positive for
diagnosis, and the American-​European consensus Sjögren’s classification
criteria is cited above. Patients with Sjögren’s syndrome should have
long-​term follow-​up due to an increased risk for developing lymphoma.
Sarcoidosis is a granulomatous disease that can affect multiple organs,
most commonly the lungs and skin, with pulmonary sarcoidosis reported
in 90% of patients with the disease.
Bacterial sialadenitis would not account for the dry eyes or be an indica-
tion of labial gland biopsy.
Keywords: dry mouth, dry eyes, anti-​Ro, anti-​La, labial gland biopsy, mul-
tiple dense foci.
→ Vitali CB, Bombardieri S, Jonsson R, et al. Classification criteria for
Sjögren’s syndrome: a revised version of the European criteria proposed
by the American-​European Consensus Group. Annals of the Rheumatic
Diseases. 2002;6:554–​8.

2. A ★★ OHCD 6th ed. → p. 754


This question is a simple test for matching classical symptoms to a par-
ticular condition.
Horner’s syndrome consists of constricted pupils, drooping eyelids, uni-
lateral loss of sweating on the face, and the occasional sunken eye.
Melkersson–​Rosenthal syndrome consists of facial paralysis, facial
oedema, and fissured tongue.

https://t.me/DentalBooksWorld
answers 167

Sicca syndrome is a combination of dry mouth and dry eyes, in the


absence of any signs of autoimmune connective tissue disorders.
Graves’ disease is characterized by hypothyroidism as a result of the pro-
duction of autoantibodies to thyroid-​stimulating hormone. Classically,
there is exophthalmos.
Frey’s syndrome is a condition in which gustatory sweating and flushing
of the skin occur. It follows trauma to the skin overlying the parotid
gland, and it is thought to be the result of post-​traumatic crossover of
sympathetic and parasympathetic supply to the gland and skin.
Keywords: sweating on eating, flushing of skin, surgical intervention.

3. E ★★ OHCD 6th ed. → p. 438


The history suggests this pain is unlikely to be of dental origin.
Dental pain can be shooting or dull (depending on the stage of disease),
but often lasts from minutes to hours.
Reversible pulpitis, although being poorly localized, is not typically de-
scribed by patients as excruciating or electric shock-​like. Commonly, hot,
cold, or sweet stimuli induces pain in reversible pulpitis, and it is a sharp
pain that lasts as long as the stimulus is present or a few minutes longer.
Irreversible pulpitis presents as a dull, spontaneous ache which tends to
be poorly localized, and the involved teeth are usually vital. Very occa-
sionally, it can present in a manner similar to trigeminal neuralgia, and
therefore, a thorough dental assessment is required before definitively
diagnosing trigeminal neuralgia.
Glossopharyngeal neuralgia presents with a similar history to trigeminal
neuralgia, but pain is felt in the back of the throat and is usually triggered
by swallowing.
Information on atypical facial pain is available in other questions within
this chapter.
Intense, unilateral, shooting pain of very short duration is classic of
neuralgic pain. The pain follows the distribution of sensory nerves and
is brought on by trigger factors which the patient often self-​reports, e.g.
shaving or cold wind. Diagnosis is achieved through history, and carba-
mazepine is effective in treatment and often diagnostic. Gabapentin/​
pregabalin are alternative medications often prescribed when contra-
indications to carbamazepine are present (e.g. warfarin).
Keywords: 50-​year-​old woman, shooting ‘electric shock’, short duration.
→ Zakrzewska J, Linskey M. Trigeminal neuralgia. BMJ. 204;348:g474.

4. D ★★★ OHCD 6th ed. → p. 438


Temporal arteritis is a type of vasculitis, affecting the superficial tem-
poral artery. Pain may be elicited when chewing, as the temporal muscles
become ischaemic. The condition usually presents in older patients. If
the central retinal artery becomes involved, there is a risk of blind-
ness; therefore, treatment must be provided as soon as possible with

https://t.me/DentalBooksWorld
168 Chapter 8 Oral medicine and oral pathology

high-​dose steroids—​often prednisolone. Diagnosis can be clinical but


can be confirmed by a markedly raised erythrocyte sedimentation rate
(ESR) level and the presence of giant cells on arterial biopsy.
Trigeminal neuralgia can be triggered by chewing and is of sudden onset.
But the pain is short-​lived and classically described as lancinating, not
aching. Pyrexia is not associated with trigeminal neuralgia.
Myofascial pain may present in a similar manner, but the patient would
not be pyrexial.
Myofascial pain and temporomandibular disorders are also unlikely to
be of sudden onset.
Cluster headaches more commonly affect younger males from the age
of 20 years. The orbital, rather than the temporal, region is affected
more commonly.
Atypical facial pain does not follow the anatomical distribution of nerves
and is not associated with pyrexia.
Keywords: unilateral headache, worse on chewing, 38.6°C, pulse, not
detectable.

5. E ★★★ OHCD 6th ed. → p. 422


White sponge naevus is a genetic condition with childhood onset.
A rough or folded appearance of the mucosa is caused by thickening of
the epithelium. The condition often affects the buccal mucosa but can af-
fect any part of the mouth. Only reassurance is required. There is usually
a family history with similar lesions in siblings or parents. See Figure 8.2
(see Colour Plate section) which shows oral epithelial naevus affecting
the buccal mucosa.
Leukoedema causes a diffuse milky appearance of the oral mucosa. The
white patches do not rub off, but the white appearance may reduce
when the mucosa is stretched. This is a benign, asymptomatic condition
which does not require treatment.
Stomatitis nicotina affects the palate of smokers. The epithelium of the
palate thickens to produce a white/​grey patch, from which the minor
salivary glands protrude as red dots. This is not a premalignant condition
and usually resolves when the patient quits smoking. As the patient is a
life-​long non-​smoker, this answer is clearly incorrect.
Lichen planus can be diffuse, but it is unlikely to be present from
childhood.
Keywords: diffuse, asymptomatic, rough texture, since 2.

6. B ★★★ OHCD 6th ed. → p. 438


It can be difficult to diagnose vague pain, especially if the patient’s pain is
not resolved after treatment.
AFP and AO are diagnoses of exclusion, between which it can be diffi-
cult to differentiate. Characteristically, they are vague, aching pains dis-
tinguished by location and a careful history.

https://t.me/DentalBooksWorld
answers 169

As in the scenario, AO is persistent pain following extraction or root


canal treatment, in the absence of clinical or radiographic causes. The
pain is thought to be neuropathic, resulting from axonal sprouting and
neuroplasticity of localized nerve fibres, and it is therefore more local-
ized than AFP. This confusing presentation may be associated with tin-
gling or itching and can lead to multiple teeth being treated unnecessarily.
AFP is considered psychogenic and often has unusual presentations (e.g.
crossing anatomical boundaries) and does not respond to conventional
treatment or pain relief. The diagnosis is most common amongst women
over the age of 50 years. Associated factors include a history of depres-
sion, chronic illness, or irritable bowel syndrome. Treatment is often in
the form of cognitive behavioural therapy or antidepressants.
It is important to consider these diagnoses in cases where the pain his-
tory is erratic, to avoid unnecessary treatment.
Oral dysaesthesia is a burning sensation from the oral mucosa in the
absence of any oral pathology. Almost half of all cases of oral dysaes-
thesia are idiopathic. Other causes include vitamin B deficiencies, iron
deficiency, Candida infection (including dentures), and allergies.
Pain from TMD has several overlapping characteristics with atypical
pain. However, it is likely to present alongside other features such as
clicking, jaw locking, crepitus, trismus, or tenderness of the muscles of
mastication.
Trigeminal neuralgia is described as severe pain but is of very short dur-
ation and does not cross the midline. Patients are usually able to asso-
ciate with distinct triggering factors.
Keywords: 53-​ year-​
old woman, pressure-​ like pain, itching sensation,
extraction, followed by root canal treatment.

7. B ★★★★ OHCD 6th ed. → p. 408


Although relatively rare, syphilis should be considered in young pa-
tients presenting with a solitary, painless ulcer in the mouth when
no other cause can be identified. The most recent United Kingdom
epidemiology data have shown that peak incidence occurs in men
aged between 25 and 35 years and the incidence has been gradually
increasing since 2003.
A primary syphilitic lesion caused by infection from Treponema pallidum
is called a chancre. It presents as a firm, solitary, non-​tender ulcer at
the site of inoculation. The lesion is highly infectious but resolves within
–​2 months. The T. pallidum haemagglutination assay (TPHA) or dark-​
ground microscopy can be used to aid diagnosis of a syphilis infection.
Secondary (snail track ulcers) and tertiary (gumma) syphilitic lesions may
occur at later stages.
Squamous cell carcinoma is possible but unlikely, as the patient is young
and does not have a history of smoking.
Major recurrent aphthous stomatitis may present with a similar appear-
ance. However, the lesion is usually painful and there is usually a history

https://t.me/DentalBooksWorld
170 Chapter 8 Oral medicine and oral pathology

of recurrence. Lymphadenopathy is also unlikely in major recurrent aph-


thous stomatitis.
Traumatic ulcers are generally painful and would likely be accom-
panied by a history of localized trauma and frequently have a keratotic
outline.
Atrophic glossitis can result in a smooth, erythematous tongue, affecting
all or only part of the dorsum of the tongue. However, it would not
result in a positive TPHA result.
Keywords: 27-​year old, tongue, non-​tender, cervical lymphadenopathy,
blood test.

8. C ★★★★ OHCD 6th ed. → p. 490


Mealtime syndrome is commonly associated with salivary calculi
(sialolith). Recurrent swelling of the salivary gland can be a result of re-
stricted salivary flow. This is generally more common around mealtimes,
as saliva flow is stimulated. Sialoliths are more commonly associated with
the submandibular gland, due to the convoluted path of Wharton’s duct.
Normally, sialography would be the imaging method of choice in non-​
infected situations, due to the diagnostic and therapeutic information
the method provides (with regard to the cause and location). However,
sialography involves a radiograph, following injection of an iodine-​based
radio-​opaque medium, to reveal any obstructions or strictures within
the salivary gland or duct system.
As the patient is allergic to iodine, contemporary imaging with mag-
netic resonance would be most appropriate. Magnetic resonance
imaging has been reported to have positive and negative predictive
values of above 90%. Moreover, it provides accurate three-​dimensional
information on the location and can be used in cases of gland inflam-
mation. In contrast, resolution of secondary and tertiary branches of
the duct is worse than sialography, and therefore, it is not a first-​line
imaging choice.
Occlusal radiographs alone are useful, with around 80% of salivary cal-
culi detected by this investigation. They are normally taken as a primary
diagnostic test, and further imaging may be prescribed afterwards if a
calculus cannot be seen. Therefore, in answer to this question, mag-
netic resonance imaging would be more likely to produce a definitive
diagnosis.
Computed tomography imaging incurs a relatively high dose of radiation.
It also provides limited benefit over the above alternatives and therefore
is reserved for select cases.
Fine-​needle aspiration is used to biopsy superficial soft tissue swellings.
Keywords: recurrent swelling, mealtime, iodine allergy, definitive.
→ Rzymska-​Grala I, Stopa Z, Grala B, et al. Salivary gland calculi—​
contemporary methods of imaging. Polish Journal of Radiology.
200;75:25–​37.

https://t.me/DentalBooksWorld
answers 171

9. C ★★★★ OHCD 6th ed. → p. 48


This patient presents with pemphigus vulgaris. This is a bullous condi-
tion reported to be more common amongst middle-​aged people of
Ashkenazi Jewish origin. The disease is characterized by autoantibodies
that target the intercellular connections of the stratum spinosum layer
of the epidermis. This results in fragile, thin-​walled intraepidermal bullae
that rupture readily. Painful oral mucosal lesions and skin lesions result.
These weak cellular junctions can also lead to skin shearing with lateral
pressure, known as a positive Nikolsky’s sign. Immunofluorescence is
used to complement histology in the diagnosis. This would show inter-
cellular IgG and C3—​visually described as a ‘chicken wire’ or ‘basket
weave’ appearance.
Linear IgG and C3 at the basement membrane can be found by direct
immunofluorescence in mucous membrane pemphigoid (MMP)
and bullous pemphigoid (BP) disease. Orally, these two conditions
both resemble pemphigus vulgaris, but with different pathogenesis.
Autoantibodies target hemidesmosomes at the basement membrane, so
bullae are more robust and last a few days before rupturing. BP is pri-
marily a skin disorder, with oral lesions in only a third of cases. Nikolsky’s
sign is absent in BP. In MMP, skin lesions are uncommon, but Nikolsky’s
sign is present. Ocular involvement can occur, leading to blindness.
Granular IgA deposits and C3 at dermal papillae are found by direct
immunofluorescence in dermatitis herpetiformis. Orally, it presents in a
similar fashion to lichen planus.
Linear IgA and C3 at the basement membrane are found by direct
immunofluorescence in linear IgA disease. This condition is a variant of
dermatitis herpetiformis.
Keywords: very short-​ lived blisters, skin lesions, shear with lateral
pressure.
→ Scully C, Challacombe S. Pemphigus vulgaris: update on
etiopathogenesis, oral manifestations, and management. Critical Reviews
in Oral Biology and Medicine. 2002;3:397–​408.

20. D ★★★★ OHCD 6th ed. → p. 505


Multiple myeloma is cancer of plasma cells, which can present in mul-
tiple sites throughout the body. A solitary plasmacytoma affects a single
site and may indicate future disease. Malignant plasma cells proliferate,
commonly within the bone marrow, and produce a wide variety of
symptoms.
Bone pain and osteolytic lesions are common, due to an increase in
osteoclastic activity of plasma cells, which can sometimes be identified
as ‘punched-​out’ lesions on radiographs, e.g. pepper-​pot skull. Anaemia
may result, as more bone marrow becomes affected. Calcium levels also
rise, as bone is broken down.
Abnormal plasma cells often produce ineffective immunoglobulins
(paraproteins). Excretion of these products can be detected in the urine

https://t.me/DentalBooksWorld
172 Chapter 8 Oral medicine and oral pathology

and are often referred to as Bence-​Jones proteins (light chains of im-


munoglobulins). Renal problems can be linked to excessive production
of these paraproteins.
‘CRAB’ is a useful acronym for identifying the key symptoms of multiple
myeloma:
● Calcium increase
● Renal failure
● Anaemia
● Bone problems.

Keywords: plasmacytoma, hypercalcaemia, anaemia, elevated


immunoglobulins.

2. D ★★★★ OHCD 6th ed. → p. 422


The histology slides presented show classic pathology findings for
lichen planus, including a saw-​ tooth pattern to the rete processes
of the epithelium. The epithelium shows hyperkeratosis and can be
thicker (acanthotic) or thinner (atrophic). Rete processes are exten-
sions of the epithelium into the underlying connective tissue, which
can lengthen and increase in width. Subepithelial band-​like lymphocyte
infiltration is another pathological finding in lichen planus, largely com-
prising T lymphocytes. Liquefactive degeneration of basal cells can also
be seen, in addition to apoptotic cells. These degenerating cells (Civatte
bodies) can also be seen in other keratotic diseases, including lupus
erythematosus.
Frictional keratosis would show extremely thickened epithelium and
hyperkeratinization, with no subepithelial band-​like lymphocytic infil-
trate. Acute cases may present with an ulcerated epithelium. This lesion
would normally resolve with removal of the source.
DLE shows perivascular lymphocytic infiltration, along with liquefac-
tive degeneration of the basal layer. Immunofluorescence can be used
to detect immunoglobulin deposits. MMP has been discussed in other
questions within this chapter.
Keywords: gingivae, white patch.

22. E ★★★★
ALL is the most common type of leukaemia in children and young adults.
Spontaneously bleeding gums should always raise suspicion. However,
this patient’s WBC count returned within the normal range.
Pyogenic granuloma produces a hyperplastic lesion on the attached gin-
givae, which can ulcerate. This lesion is associated with local irritation,
and inflammation is exacerbated by the presence of plaque but is local-
ized to one area of the gingivae.
ANUG is a true infection of the gingivae. Presentation can include
necrosis of the interdental papillae. It can be painful and associated with
marked halitosis, but gingivae are unlikely to appear hyperplastic.

https://t.me/DentalBooksWorld
answers 173

Melatonin is taken to assist with sleeping and has no effect on gingival


overgrowth. Common drugs associated with gingival overgrowth are
phenytoin, calcium channel blockers, and ciclosporin.
Key clues in the history point to vitamin C deficiency or scurvy. The
patient only eats crisps and presents with generalized hyperplastic gin-
givae that bleed, along with early loss of periodontal attachment. Scurvy
is not common in developed countries.
Keywords: lethargic, only eats crisps (poor diet), loss of attachment,
hyperplastic gingivae.

23. D ★★★★ OHCD 6th ed. → p. 392


The clinical features given are most likely to represent a dentigerous
cyst, which are commonly found enveloping the crowns of unerupted
teeth. However, the histopathology described is most likely to indi-
cate an odontogenic keratocyst. It is important to recognize that
odontogenic keratocysts can present in a fashion similar to dentigerous
cysts (enveloping crowns), and it is therefore important to submit them
for histological analysis. The lining of a dentigerous cyst is also thin, but
the squamous epithelium is non-​keratinizing and often displays mucous
metaplasia.
Odontogenic keratocysts show specific growth in an anteroposterior
direction along the marrow spaces and can be responsible for resorbing
the roots of associated teeth. Small satellite cysts can develop in the cyst
wall, and this can account for the high recurrence rates identified.
Eruption cysts are extra-​alveolar dentigerous cysts.
Lateral periodontal cysts are thin-​walled developmental cysts associ-
ated with the lateral aspect of vital teeth. The cyst is non-​keratinizing
and is more commonly found in the canine and premolar region of the
mandible.
Radicular cysts are inflammatory cysts that commonly arise at the apex
of non-​vital teeth. They are non-​keratinizing cysts, which show marked
inflammation in the cyst lining. This results in a cyst lining that is much
more hyperplastic than that associated with developmental cysts.
Keywords: high levels of mitotic figures, parakeratinization, low protein
content (mainly albumin).

24. E ★★★★ OHCD 6th ed. → p. 720


The Health and Safety Executive is responsible for protecting employees,
employers, contractors, and the public within the work environment.
Employers and those in charge of premises are required to report adverse
incidents, including work-​related deaths, diseases, and near misses.
Whilst it is common practice for accidents at work to be recorded in
an accident log, the ‘Reporting of Injuries, Diseases and Dangerous
Occurrences Regulations 995 (RIDDOR)’ stipulates that injuries
which result in >3 consecutive days off work must legally be recorded.

https://t.me/DentalBooksWorld
174 Chapter 8 Oral medicine and oral pathology

Incidents which require the injured party to have >7 days off work must
be reported to the Health and Safety Executive within 5 days.
Since  April 205, a memorandum of understanding has existed between
the Health and Safety Executive and the Care Quality Commission to
ensure appropriate information is shared between the two bodies, and it
is likely that serious events that occur in the dental setting and reported
to the Health and Safety Executive under RIDDOR could be investigated
by the Care Quality Commission.
Keywords: trips, admitted to hospital, 4 weeks off work.
→ Care Quality Commission. Memorandum of Understanding (MoU)
between the Care Quality Commission (CQC), Health and Safety Executive
(HSE) and local authorities in England. 207. Available at: https://​www.
cqc.org.uk/​file/​82048
→ Health and Safety Executive. Reporting accidents and incidents at work.
203. Available at: http://​www.hse.gov.uk/​pubns/​indg453.htm

https://t.me/DentalBooksWorld
Chapter 9 175

General medicine
Tariq Ali

‘Teeth are quite often attached to people.’


The oral cavity is the largest and most used orifice in the human body. It
is the opening of the aerodigestive tract, as well as a region of the body
that is heavily involved in both sensing the outside world and communi-
cating with it. Simply put, our mouths are complex and deeply intimate
structures that can act as windows into the health of the rest of the
body. Many disease processes that are systemically invisible may display
quite overt oral manifestations.
There are deep associations between bodily diseases and oral dis-
ease counterparts, with considerable and mounting evidence to show
that oral health may have an impact on systemic wellness. It is there-
fore important to have an understanding of the wider human anatomy,
physiology, and pathology. Understand and treat the patient as a whole,
and think about all aspects of their health, whether it be routine pre-
ventative treatment for periodontally compromised diabetic patients or
polypharmacy patients requiring secondary dental care.
Often at times, patients can be unclear about their own health condi-
tion; having a fundamental understanding of general medicine will help
to make those difficult choices regarding your patients a little easier and
clarify when and whom to refer.
Key topics are not included for this chapter, as it is a vast topic, and not
the main focus of the dental undergraduate curriculum. It is, however,
important to have a good basic knowledge of general human diseases,
how they might interact with dental treatment, and the role that dentists
can play in both diagnosis and management

https://t.me/DentalBooksWorld
176 Chapter 9 General medicine

QUESTIONS

. A 5-​year-​old girl attends the Emergency Department at the local


dental hospital. She suffers from type  diabetes mellitus. Damage
to which single cell type in the pancreas would lead to the patient’s
condition? ★
A Alpha cells
B Beta cells
C Delta cells
D Epsilon cells
E Gamma cells

2. A 78-​year-​old man with poorly controlled type 2 diabetes re-


quires extraction of his lower right first permanent molar (LR6),
as he has a periapical abscess on the tooth. He is extremely nervous
and, uncommonly for him, did not eat breakfast. During treatment, the
patient begins to sweat profusely; he then begins to shake violently and
stops responding. What is the single most likely explanation for the
presentation? ★
A Brain tumour
B Epileptic seizure
C Hypoglycaemic attack
D Odontogenic sepsis
E Pseudoseizure

3. A 67-​year-​old man with chronic obstructive pulmonary disease


(COPD) attends for a routine restoration. During treatment, he
starts to cough and asks if the treatment can be stopped. He sits up and
is visibly working hard to breathe. His respiratory rate is 30 breaths per
minute. His cough is productive and appears to be clear, but his lips start
to develop a blue tinge. Whilst the nurse applies a pulse oximeter, what
is the next single most appropriate management? ★
A High-​flow oxygen at 0–​5 L/​minute (through a non-​rebreather mask)
B Low-​flow oxygen at 2–​3 L/​minute (through nasal cannulae)
C Medium-​flow oxygen at 5–​0 L/​minute (through a simple face mask)
D Titrate oxygen levels to between 88% and 92% (through a
non-​rebreather mask)
E Titrate oxygen levels to between 92% and 98% (through a simple
face mask)

https://t.me/DentalBooksWorld
questions 177

4. An unaccompanied 79-​year-​old woman attends her dentist for


extraction of her upper right first permanent molar (UR6). She
takes warfarin for atrial fibrillation, and her latest international normalized
ratio (INR) was 3. three days ago. Following the extraction, the socket
is packed with a haemostatic agent and sutured; however, after 20 min-
utes of pressure, the patient is still bleeding. What is the single most
appropriate next management strategy? ★
A Ask the patient to continue applying pressure to the area at home and
pause the warfarin until the bleeding stops
B Ask the patient to go to the local Emergency Department
C Continue applying pressure over the area and call for an ambulance
D Give high-​flow oxygen and suction the blood from the mouth
E Give the patient an intravascular injection of vitamin K

5. A 35-​year-​old man attends his orthodontist for a review, after


completing a course of orthodontic treatment 2 months ago. His
retainers no longer fit even, though he has worn them consistently every
other night. Clinically, spacing has developed between his anterior teeth,
and his facial appearance has changed from his post-​operative photos.
He now has more prominent supra-​orbital ridges and widening of the
nose and jaw, and has noticed tingling in both hands. What is the single
most likely cause in this scenario? ★★
A Addison’s syndrome
B Anabolic steroid use
C Conn’s syndrome
D Cushing’s syndrome
E Pituitary tumour

6. A previously fit and well 55-​year-​old woman attends her dentist,


complaining of recent-​onset gingival swelling. She apologizes for
being late to the appointment but explains that she has been chronically
tired for the past couple of months and slept through her alarm. She is
short of breath from walking up the stairs and has multiple bruises visible
on her arms as she takes off her coat. Clinically, she has bilateral cervical
lymphadenopathy and swollen pale gingivae, which bleed spontaneously.
What is the single most likely diagnosis? ★★
A Acute myeloblastic leukaemia
B Acute lymphocytic leukaemia
C Beta-​thalassaemia
D Chronic myeloblastic leukaemia
E Non-​Hodgkin’s lymphoma

https://t.me/DentalBooksWorld
178 Chapter 9 General medicine

7. A 70-​year-​old man attends for a routine examination. He has


recently fallen and fractured his foot. He takes time getting up
off the chair and shuffles into the surgery. His handwriting on the new
patient questionnaire is extremely small and spidery. He is expression-
less, whilst recounting the history of his recurrent falls. What is the single
most likely cause of the recurrent falls? ★★
A Benign paroxysmal positional vertigo (BPPV)
B Cerebellopontine angle (CPA) tumour
C Labyrinthitis
D Parkinson’s disease
E Posterior circulation infarct

8. A 37-​
year-​ old woman attends for a routine scale and polish.
Medically, she is under investigation with her general practitioner
for symptoms of excessive thirst, palpitations, excess sweating, and
recent weight loss. Clinically, she has a large lump in her neck. The lump
moves when she swallows, but not when she sticks her tongue out.
What is the single most likely cause for her neck lump? ★★★
A Diabetes insipidus (DI)
B Graves’ disease
C Hashimoto’s thyroiditis
D Thyroid cancer
E Tuberculosis (TB)

9. A 65-​year-​old man presents with a large dental abscess in the lower


right quadrant. He suffers from polymyalgia rheumatica (PMR), for
which he has recently been taking high-​dose prednisolone. He has not
taken his medication for the last 2 days, as he knows steroids ‘make in-
fections worse’. He is profusely sweaty and looks extremely unwell. His
current temperature is 39°C, and his blood pressure is 70/​50 mmHg.
His blood sugar is 2.9 mmol/​L. What is the single most likely underlying
cause for his presentation? ★★★
A Acquired immune deficiency syndrome (AIDS)
B Addison’s disease
C Adrenal crisis
D Hyperadrenalism
E Septic shock

https://t.me/DentalBooksWorld
questions 179

0. A 75-​year-​old man with hypertension attends for the fit of his
upper complete denture. During insertion of the denture, he
begins slurring his speech, and then the left corner of his mouth droops.
When he sees the abnormalities in the mirror, he becomes alarmed and
raises both his eyebrows in surprise. At his next review, he reports he
went to hospital, but the symptoms resolved after half a day, and he
is now under review with his doctor. What was the single most likely
diagnosis? ★★★
A Bell’s palsy
B Cerebrovascular accident (CVA)
C Subarachnoid haemorrhage (SAH)
D Transient ischaemic attack (TIA)
E Vertebrobasilar insufficiency

. A 45-​year-​old man presents to the dental hospital emergency


clinic with toothache. He denies any medical problems but ad-
mits to drinking 2 L of cider a day. The sclerae of his eyes are faintly
yellow; his abdomen is distended, and he appears to have red, spider-​like
patterns on the skin of his face and neck. On examination, the lower left
second molar (LL7) displays gross subgingival caries and requires extrac-
tion. The patient is in severe pain and wants immediate treatment. What
is the next single most appropriate course of action? ★★★
A Decline to extract the tooth and prescribe a course of metronidazole
B Decline to extract the tooth and extirpate to alleviate pain
C Extract the tooth as normal immediately
D Extract the tooth tomorrow, asking the patient not to drink until then
E Request a full blood count, liver function tests, and a clotting screen

2. A 59-​year-​old woman attends for the first time, complaining


of wobbly teeth. She has a smoking history of 40 pack years
but has recently quit. She has a chronic cough and is seen to be using a
bloodstained handkerchief. Her left eye is sunken in, with a smaller pupil,
compared to the right eye, and her left eyelid is also drooping. What is
the single most likely underlying diagnosis? ★★★★
A Mesothelioma
B Pancoast tumour
C Pneumothorax
D Pulmonary embolism (PE)
E Tuberculosis (TB)

https://t.me/DentalBooksWorld
180 Chapter 9 General medicine

3. A 32-​year-​old man attends for a routine appointment. He is a


recovering intravenous drug user, has a smoking history of 25
pack years, and drinks six cans of lager every day. He has had a cough
and been experiencing night sweats for the past 4 weeks. He says his
clothes feel a lot looser in the last 6 months. Clinically, a dark purple,
irregular lump is seen on the upper left buccal gingiva, which the patient
was unaware of. White plaques are present on his tongue, which can be
scraped off. What is the single most likely underlying diagnosis? ★★★★
A Acquired immune deficiency syndrome
B Candida albicans infection
C Malignant melanoma
D Oral squamous cell carcinoma
E Tuberculosis

4. An 85-​year-​old man is brought for a routine appointment in


his wheelchair, accompanied by his carer. He appears very
drowsy, and his carer states he has been sleeping excessively over the
last 2 days. He has been off his food and not drinking much. He wakes
up intermittently, complaining of toothache. The only new medication
he has started taking is over-​the-​counter ibuprofen. His other medica-
tions include bisoprolol, candesartan, aspirin, glaucoma eye drops, and a
morphine patch. His carer explains he has barely woken up over the last
2 days, not even to go to the bathroom. Attempts to rouse him are to
no avail. Upon performing a sternal rub, he opens his eyes and moves his
hands over the sternal area. He makes an attempt to mumble something,
but it is not comprehensible. What is the single correct Glasgow coma
scale (GCS) score for this gentleman? ★★★★
A 3
B 6
C 7
D 8
E 9

5. A 65-​year-​old woman requires an extraction due to advanced


periodontal disease. She suffers from stage 5 chronic kidney
disease (CKD) and has haemodialysis three times a week. When is the
single most appropriate time for the extraction? ★★★★
A Four hours after dialysis
B One day before dialysis
C The day after dialysis
D The morning of dialysis
E Two days prior to dialysis

https://t.me/DentalBooksWorld
answers 181

ANSWERS
. B ★ OHCD 6th ed. → p. 58
All of the above cells are part of the islets of Langerhans in the pancreas.
Beta cells produce insulin and amylin (60–​85% of pancreatic anabolic
activity). Both are essential for glucose homeostasis. Insulin promotes
absorption of glucose from the bloodstream, and subsequent glycogen-
esis (production of glycogen) in the liver and skeletal muscle and lipogen-
esis (production of triglycerides) in adipocytes.
Alpha cells produce glucagon (5–​20%) used for glycogenolysis (break-
down of glycogen into glucose), gluconeogenesis (formation of glucose
from amino acids, lactate, and glycerol), and lipolysis (breakdown of tri-
glycerides into free fatty acids). Amylin inhibits the effects of glucagon.
Delta cells produce somatostatin (3–​0%), also known as growth hor-
mone inhibitory hormone (GHIH). This, as the name suggests, inhibits
the release of pituitary growth hormone, as well as inhibition of insulin
and amylin.
Gamma cells produce pancreatic polypeptide (PP) (3–​5%). PP promotes
gastric secretion and plays a role in satiety.
Epsilon cells produce ghrelin (<%). It is a neuropeptide that acts as a
hunger stimulator and is usually secreted when the stomach is empty.
Keywords: diabetes mellitus, pancreas.
→ Devlin H, Craven R. Oxford Handbook of Integrated Dental Biosciences.
Oxford University Press, Oxford; 208.

2. C ★ OHCD 6th ed. → p. 58


Hypoglycaemia can cause a range of different autonomic and neuro-
logical symptoms, including tremors, clamminess, tachycardia, tachyp-
noea, anxiety, and behavioural changes. If prolonged, hypoglycaemia can
lead to seizures and coma. In normal circumstances, patients can recog-
nize the symptoms early and remedy the situation with consumption of
a rapidly absorbed carbohydrate.
Diabetics often lose their ability to recognize their own hypoglycaemic
symptoms because of their defective autonomic responses and there-
fore may not treat their hypoglycaemia in time.
Dental infection may place additional metabolic stress on the body, but
this on its own would not explain the incident.
Epilepsy cannot be diagnosed from this one incident and would require
additional investigations, as would the diagnosis of a brain tumour.
A pseudoseizure (conscious attempts by the patient to mimic a seizure,
often due to behavioural issues) should only be considered once all
other possibilities have been excluded.
A brain tumour may cause seizures but is unlikely to present in this
manner and usually would present with other neurological findings.

https://t.me/DentalBooksWorld
182 Chapter 9 General medicine

Keywords: type 2 diabetes, did not eat breakfast, sweat profusely, shake
violently.

3. A ★ OHCD 6th ed. → p. 50


COPD is characterized by progressive airway obstruction and worsening
breathlessness. It is an umbrella term for chronic bronchitis and emphy-
sema; smoking is the major cause. Chronic airway inflammation results
in hypertrophy of the mucus glands and destruction of distal bronchi-
oles, limiting gas exchange in the alveoli and resulting in hypoxaemia [low
blood partial pressure of oxygen (PaO2)] and hypercarbia [high blood
partial pressure of carbon dioxide (PaCO2)].
The patient is having difficulty oxygenating, hence the increased respira-
tory rate, and is hypoxaemic, as evidenced by the cyanosis. In this
situation, it is important to provide the greatest concentration of in-
spired oxygen as possible. Therefore, applying 0–​5 L of oxygen via
a non-​rebreather mask would be the best way to improve the situation
immediately.
In normal physiology, respiratory drive is stimulated by hypercarbia. In
patients with COPD, chronically high PaCO2 leads to the respiratory
drive being converted to hypoxia. Therefore, prolonged exposure to
high-​concentration oxygen can result in respiratory depression, and
ultimately loss of consciousness and death.
For this reason, it is recommended that patients with COPD have
their oxygen saturations titrated to 88–​92%, once any acute hypox-
aemia is alleviated. In normal patients, a target range of 92–​98% can
be titrated.
Keywords: COPD, 30 breaths per minute, blue tinge.
→ O’Driscoll BR, Howard LS, Earis J, et al. BTS guideline for oxygen use
in adults in healthcare and emergency settings. Thorax. 207;72(Suppl
):ii–​90.

4. C ★ OHCD 6th ed. → p. 506


Generally, when a patient is on warfarin, oral surgery procedures can be
performed as long as the INR is below 4.0. Some clinicians would suggest
an upper limit of 3.5 for block injections. It should also be checked within
72 hours of the procedure, as the INR is very liable to change, particu-
larly if the patient is forgetful with taking tablets or consumes substances
that might interact with warfarin (e.g. alcohol, grapefruit).
When a patient on warfarin has a problem with haemorrhage after a
minor surgical procedure, it is important to apply pressure for at least 20
minutes. This will allow time for a clot to form and stabilize. Additional
local measures could include additional local anaesthetic, placement of a
haemostatic agent, and suturing of the wound. In this case, the bleeding
was excessive and did not stop; the best option here would be to call an
ambulance and send the patient to a secondary care facility—​particularly
as they do not have anyone to escort them.

https://t.me/DentalBooksWorld
answers 183

Vitamin K can reverse the effect of warfarin, but this can take up to
6 hours to occur. If immediate reversal is required, then prothrombin
complex concentrate or fresh frozen plasma can be given in a hospital
setting.
Keywords: warfarin, still bleeding.
→ Scottish Dental Clinical Effectiveness Programme (SDCEP).
Management of dental patients taking anticoagulants or antiplatelet drugs.
205. Available at: http://​www.sdcep.org.uk/​published-​guidance/​
anticoagulants-​and-​antiplatelets/​

5. E ★★ OHCD 6th ed. → pp. 58–​9


Acromegaly develops as a result of overproduction of growth hormone
(GH), commonly caused by a benign tumour of the pituitary gland called
an adenoma. All of the signs described in the scenario are indicators of
an excess in GH. The classic aide-​memoire in dentistry is ‘hats and den-
tures’ that have become too small.
Complication include:
● Severe headaches
● Arthritis and carpal tunnel syndrome
● Hypertension
● Diabetes mellitus (excess GH leads to insulin resistance)
● Compression of the optic chiasm leading to loss of vision in the outer

visual fields, described as bitemporal hemianopia (tunnel vision).


Addison’s syndrome is used to describe hypoadrenalism, and therefore
inadequate corticosteroid production, leading to an electrolyte disturb-
ance (hyponatraemia, hypoglycaemia, and hypokalaemia).
Anabolic steroids can lead to iatrogenic hypogonadism and insufficient
endogenous production of androgens.
Conn’s syndrome, or primary hyperaldosteronism, leads to
hypernatraemia and hypokalaemia. This can manifest itself as high blood
pressure and muscular weakness or spasms.
Cushing’s syndrome is described as an excess of glucocorticoids, which
can lead to a number of symptoms such as high blood pressure, abdom-
inal obesity with thin arms and legs, abdominal striae (stretch marks),
a ‘moon-​shaped’ face, a hump between the shoulders, weak proximal
muscles, osteoporosis, acne, depression, and thin and fragile skin.
Keywords: spacing, prominent supra-​orbital ridges, widening of the nose
and jaw, tingling, hands.
→ Devlin H, Craven R. Oxford Handbook of Integrated Dental Biosciences.
Oxford University Press, Oxford; 208.

6. A ★★ OHCD 6th ed. → p. 504


The symptoms presented here are indicative of an underlying haemato-
logical malignancy. The presence of signs suggesting anaemia, coupled
with abnormal bleeding, should make the reader consider pathology of

https://t.me/DentalBooksWorld
184 Chapter 9 General medicine

both white and red blood cell lineages. Leukaemia is a neoplastic prolif-
eration of white blood cells in the bone marrow. The symptoms occur
because of bone marrow failure and crowding out of healthy leucocytes,
erythrocytes, and platelets. General manifestations include anaemia,
thrombocytopenia, liability to infections, and lymphadenopathy. There
are four main subtypes of leukaemia: acute and chronic lymphocytic
(ALL and CLL), and acute and chronic myeloblastic leukaemias (AML and
CML). ALL is the most common childhood leukaemia, whereas AML is
the most common acute adult leukaemia. In this scenario, the majority of
symptoms are common to all leukaemias, but the age of the patient and
the presence of gingival swelling are more suggestive of AML. Gingival
swelling from leukaemic infiltration is most commonly seen in AML (sub-
types M5 and M4).
Beta-​thalassaemia is an inherited anaemia resulting from decreased pro-
duction of the beta chains of haemoglobin. Although the shortness of
breath and fatigue could suggest this disease, the abnormal bleeding
would not be associated with this condition. Other symptoms of beta-​
thalassaemia include skeletal abnormalities, splenomegaly, and cardiac
abnormalities.
Non-​Hodgkin’s lymphoma could be responsible for these symptoms, as
leukaemia occurs in a small percentage of these patients, but it is usually
associated with immunocompromised patients.
Keywords: 55-​year old, tired, short of breath, swollen gingivae, spon-
taneous bleeding.
→ Wu J, Fantasia J, Kaplan R. Oral manifestations of acute myelomonocytic
leukemia: a case report and review of the classification of leukemias.
Journal of Periodontology. 2002;73:664–​8.

7. D ★★ OHCD 6th ed. → pp. 526–​7


The patient is demonstrating many of the classical features of
Parkinson’s disease. They have postural (proximal) muscle weakness,
as evidenced by difficulty getting up from his chair, a shuffling gait,
expressionless (mask-​ like) facies, micrographia (small handwriting),
and bradykinesia (slow movements). Other features include resting
tremor, very labile mood, and autonomic nervous system dysfunc-
tion (commonly presenting as loss of urinary continence or postural
hypotension).
Parkinson’s disease sufferers are susceptible to falls. Their shuffling gait
can lead to trips and slips; weak postural muscles limit their ability to
control their centre of gravity, and slowness of movement obtunds their
ability to break a fall.
BPPV and labyrinthitis are both benign conditions that affect the ves-
tibular system and can give the sensation of vertigo (subjective sensation
of the individual or the surroundings rotating). BPPV gives short-​lasting
(seconds to minutes) symptoms of vertigo, often precipitated by changes
in posture or movements of the head. Labyrinthitis (vestibular neuritis)
is inflammation of the vestibular apparatus, often secondary to an upper

https://t.me/DentalBooksWorld
answers 185

respiratory tract infection, that can lead to symptoms of vertigo that may
last from days to weeks.
CPA tumours can affect the cranial nerves that emerge from the region
between the cerebellum and the pons (CN V, CN VI, CN VII, CN VIII).
Although rare, CPA tumours should be excluded where there is any per-
sistent vertigo, loss of hearing, or tinnitus.
Posterior circulation infarct describes a stroke in the posterior circulating
artery and/​or the vertebrobasilar system of the brain. It can lead to ver-
tigo, ataxia, visual field defects, slurred speech, and paralysis.
Keywords: shuffles, handwriting, spidery, expressionless.

8. B ★★★
Graves’ disease is caused by an autoantibody that mimics thyroid-​
stimulating hormone (TSH), and this leads to overproduction of thy-
roid hormone (hyperthyroidism). Patients can present with palpitations,
weight loss, heat intolerance, and irritability, amongst other symptoms.
The main sign of Graves’ disease is diffuse swelling of the thyroid gland,
otherwise known as thyroid goitre.
Hashimoto’s thyroiditis is another autoimmune disorder that can also
present with swelling of the thyroid gland (goitre). However, in this case,
the anti-​thyroid antibodies slowly destroy the thyroid tissue, leading to
hypofunction of the gland (hypothyroidism). This would present as leth-
argy, weight gain, slow pulse, feeling cold, and depression.
DI can lead to weight loss and may present with polydipsia (frequent
thirst) and polyuria (frequent urination). As opposed to diabetes mel-
litus, where dehydration is caused by the diuretic effect of having glucose
in the urine, DI leads to loss of water in the urine because of a problem
with the kidney reabsorbing water from the collecting tubules. Diabetes
mellitus and DI are unlikely to cause swelling of the thyroid gland.
Thyroid cancer (there are many different forms) is a possibility with
any new presentation of a neck swelling that moves on swallowing and
tongue protrusion. However, given the clinical context in this case, it is
more readily explained by the answer given.
TB, caused by Mycobacterium tuberculosis, can affect any part of the body,
most commonly the lungs. Symptoms can include coughing, haemop-
tysis, fevers, night sweats, and weight loss. TB can also cause painless
swelling of the lymph nodes, including those of the neck. These are
known as cold abscesses due the absence of inflammatory signs.
Keywords: thirst, palpitations, sweating, weight loss, lump in neck.

9. C ★★★
The patient is suffering from an adrenal crisis, secondary to the with-
drawal of his usual steroids. Normal endogenous adrenal gluco-
corticoid and mineralocorticoid production will diminish as a result of
exogenous prednisolone administration (this causes a negative feedback
loop, limiting adrenal production). If there is sudden cessation of the

https://t.me/DentalBooksWorld
186 Chapter 9 General medicine

exogenous steroids, insufficient endogenous glucocorticoid produc-


tion will lead to inadequate lipolysis and gluconeogenesis, and there-
fore a low blood sugar level. Inadequate mineralocorticoid production
will lead to less retention of sodium (it is likely that the patient will be
hyponatraemic) in the blood, and therefore less retention of water,
leading to hypovolaemia and hypotension.
Steroids do have an immunosuppressive effect. However, the overriding
concern in this case would be compensating for the loss of exogenous
steroids. Anyone taking steroids for longer than a few weeks would
require a reducing regime before stopping completely. Patients taking
long-​term steroids often require increased doses if they develop sys-
temic infections.
Addison’s disease may have a similar presentation to this case, but the
underlying pathology in this scenario is based on inadequate endogenous
production of corticosteroids due to exogenous steroid consumption.
Septic shock is suggested by features of systemic inflammatory response
syndrome (SIRS)—​in this case, hypotension (low blood pressure) and
pyrexia (high temperature)—​in the presence of a source of infection.
This may be a compounding factor involved in the patient’s presentation,
but unlikely to be the main cause, given the history.
AIDS can have a varied presentation and is thought of as one of the
‘great pretenders’ in medicine, along with tuberculosis and syphilis. It
should always be thought of when other causes of pathology have been
excluded but is unlikely to present with these symptoms.
Keywords: abscess, high-​ dose prednisolone, not taken, sweaty, 70/​
50 mmHg, 2.9 mmol/​L.

0. D ★★★ OHCD 6th ed. → p. 526


A TIA is any vascular insult to the brain that causes a neurological deficit
(weakness of limbs, hemisensory loss, slurring of speech, or unusually
a reduction in conscious level) that lasts for <24 hours. All that can be
said from this scenario is that the patient has just begun to have a cen-
tral (upper motor neurone) neurological deficit, that has lasted for <24
hours. If the neurological deficit persists beyond 24 hours, it can be cat-
egorized as a cerebrovascular accident (CVA), also known as a stroke.
The insult may be as a result of an embolus—​a fragment of blood clot
occluding the vessel, causing ischaemia—​or a haemorrhage where blood
supply to certain parts of the brain may be disrupted due to an intracra-
nial bleed. The latter often results in a neurosurgical emergency.
Being able to raise both eyebrows differentiates this central neurological
deficit from a peripheral (lower motor neurone) neurological deficit such
as Bell’s palsy. The forehead is bilaterally innervated from the motor
cortex, so an ischaemic event in one hemisphere will not lead to loss of
function, as the other hemisphere will still be innervating the forehead
muscles.
A subarachnoid haemorrhage is usually a bleed from one of the major
intracranial blood vessels and is characterized by a sudden and severe

https://t.me/DentalBooksWorld
answers 187

occipital headache (often described as a ‘thunderclap’ headache), nausea,


and vomiting and can lead to confusion and coma. It generally does not
lead to unilateral neurological symptoms.
Vertebrobasilar insufficiency usually leads to a very transient loss of con-
sciousness (syncope), due to compression of the vertebral arteries or
basilar artery with certain neck positions. It can present as presyncopal
symptoms that include fading vision, vertigo, and confusion.
Keywords: slurring, left, droops, raise both eyebrows.

. E ★★★ OHCD 6th ed. → p. 54


The scenario alludes to a patient who is drinking excessive amounts
of alcohol and displaying signs of chronic liver damage. The liver is a
robust organ with numerous functions, most importantly detoxification
of compounds in blood, bile production, carbohydrate and lipid metab-
olism, storage of essential vitamins and minerals, and production of pro-
teins essential to life, including coagulation factors. It has a substantial
capacity for regeneration; however, prolonged toxic insults can cause
hepatocellular necrosis and normal liver architecture to be replaced with
fibrous tissue. The result is liver function deterioration and obstruction
of blood flow. There are multiple signs and symptoms that develop from
portal hypertension and reduced liver function, including jaundice (also
evident in palatal gingivae) and pruritus (itching), hepatomegaly, spleno-
megaly, peripheral and central oedema (ascites), finger clubbing, and
hyperdynamic circulation (palmar erythema, spider naevi).
In this scenario, the, patient has signs indicative of severe hepatic impair-
ment, and one should suspect derangement of liver functions and inad-
equacy of clotting capability even if the patient is trying to push for
treatment. Additional consideration should be given to alcohol-​induced
thrombocytopenia (low platelets), which will also affect the patient’s
ability to form a primary blood clot.
It is important to be safe, and therefore, a full blood count, liver func-
tion tests, and a clotting screen are required before any intervention is
performed. It may be possible to extirpate the tooth, but generally local
anaesthetic via a block injection will be required for a lower molar and
it is unlikely that a dental dam can be placed. In patients with clotting
impairment, even injections are contraindicated until the bleeding risk
can be assessed. Uncontrolled bleeding in the parapharyngeal area can
become a life-​threatening situation! Once the blood requests are con-
firmed, treatment can be planned.
Metronidazole can produce an unpleasant disulfiram-​like reaction and is
contraindicated in combination with alcohol. Additionally, it is metabol-
ized by the liver and, if prescribed, requires significant dose reduction.
Asking a chronic alcoholic to cease drinking alcohol immediately is not
advisable, as it may lead to adverse neurological consequences, coma,
and death (delirium tremens).
Keywords: cider, sclera, yellow, distended abdomen, spider-​like patterns.

https://t.me/DentalBooksWorld
188 Chapter 9 General medicine

→ Quach S, Brooke AE, Clark A, Ellison SJ. Blood investigations prior


to oral surgery for suspected alcohol-​induced coagulopathy. Are they
necessary? British Dental Journal. 205;29:2.

2. B ★★★★
Named after Henry Pancoast, an American radiologist, this is an apical
lung tumour located close to the superior ribs. Compression of the
superior part of the sympathetic chain causes the examination findings
described in the question: miosis (constricted pupil), ptosis (droopy
eyelid), and enophthalmos (sunken globe). Anhidrosis (decreased
sweating) of the ipsilateral face is also described in this syndrome. This
constellation of signs is otherwise known as Horner’s syndrome.
Other sequelae of a Pancoast tumour include weakness and wasting of
the muscles of the ipsilateral arm and hand (due to compression of the
brachial plexus) and alteration of the voice (due to recurrent laryngeal
nerve compression).
PE is a reasonable differential diagnosis, as it can produce haemoptysis
(coughing up blood) and dyspnoea (in addition to chest pain and cough),
but PE would not explain Horner’s syndrome.
Pulmonary TB can also cause dyspnoea and haemoptysis. Additionally,
TB can cause night sweats, anorexia, weight loss, and fatigue—​all of
which can be seen in any malignancy. In this case, the strong smoking
history and the presence of Horner’s syndrome should make you think
of cancer.
Mesothelioma is unlikely, as this carcinoma of the pleura almost exclu-
sively occurs in those with asbestos (a common building and insulation
material in the pre-​990s) exposure.
A spontaneous pneumothorax is unlikely to lead to haemoptysis.
Keywords: smoking, chronic cough, blood, eye, sunken, smaller pupil,
eyelid, drooping.

3. A ★★★★ OHCD 6th ed. → p. 534


AIDS is defined by a CD4+ T cell count of below 200 cells/​μL and the
presence of an array of different conditions that occur after the immune
system is compromised by the human immunodeficiency virus (HIV).
This question alluded to the patient’s previous intravenous drug use—​a
common cause for HIV transmission, along with sexual contact, blood
transfusion, and vertical transmission (from mother to child). However,
the rest of the signs and symptoms revealed in the question allude to
conversion of HIV infection to AIDS. A chest infection is suggestive of
either pulmonary tuberculosis (the symbiotic twin of HIV/​AIDS) and/​or
Pneumocystis jirovecii pneumonia (PJP), an AIDS-​defining condition.
The patient is also a heavy smoker and a drinker—​both high risk for oral
squamous cell carcinoma. Equally, a dark, irregular lesion in the mouth
or anywhere else on the body may also be suggestive of a melanoma.
However, in the context of this question, this lesion would fit better

https://t.me/DentalBooksWorld
answers 189

with Kaposi’s sarcoma—​a vascular lesion related to human herpesvirus 8


(HHV 8) that is another AIDS-​defining condition.
Keywords: intravenous drug user, cough, dark purple and irregular lump,
white plaques, scraped off.

4. E ★★★★
See Table 9..
This patient is not drinking fluids and is taking ibuprofen (non-​steroidal
anti-​
inflammatory drug) and candesartan (angiotensin II receptor
blocker); all three are risk factors for developing pre-​renal acute kidney
injury (loss of renal function after a drop in blood supply to the kidneys).
This is evidenced by a lack of urine production. As morphine is renally
excreted, it is likely this opiate has accumulated in the patient’s body and
is causing his narcosis (drowsiness due to high levels of opiate medica-
tion). He would need some naloxone (a μ-​opioid receptor antagonist)
to reverse his narcosis and monitoring with pulse oximetry, as well as
oxygen supplementation.
As a rule of thumb, one should seek anaesthetic assessment once the
GCS score is ≤8, as the patient may not be able to protect their airway.
Keywords: opens eyes, hands to sternal area, mumble,
incomprehensible, GCS.

Table 9. Glasgow coma scale


 2 3 4 5 6
Eyes Not Opening eyes Opening Opening N/​A N/​A
opening to pain eyes to eyes
eyes voice sponta­
neously
Verbal No Incomprehen­ Inapprop­ Confused Orientated N/​A
sound sible sounds riate
words
Motor No Extensor Abnormal Flexor Localizes Able to
move­ response to flexor response/​ to painful follow
ment pain response to withdrawal stimulus comm­
pain from painful ands
stimulus
The scores for this patient are: Eyes = 2; Verbal = 2; Motor = 5.

5. C ★★★★ OHCD 6th ed. → p. 56


CKD is classified by a reduction in the estimated glomerular filtration
rate. A rate of below 60 mL/​minute/​.73 m2 is considered abnormal in
adults. Below this, there are five stages of CKD, with stage 5 alternatively
being known as end-​stage renal failure (ESRF). At stage 5, dialysis or a
renal transplant is required.

https://t.me/DentalBooksWorld
190 Chapter 9 General medicine

There are two types of dialysis: peritoneal dialysis (PD) where the
patient’s peritoneal membrane is used as a semi-​permeable membrane
to filter blood, or haemodialysis where blood is taken out of the body
and filtered through a machine.
There are many clinical manifestations of CKD relevant to dentistry,
including: increased prevalence of periodontal disease, xerostomia,
oral ulcerations, increased bleeding tendency, reduced drug excretion
capabilities, etc. In the case of patients on haemodialysis, there is some
debate as to when the most appropriate time is to treat, but generally,
it is considered that the day after dialysis is best, as any accumulated by-​
products will have been cleared. However, this is always driven by the
degree of volume overload and serum levels of potassium/​urea.
Heparin (an anticoagulant used during haemodialysis) has a half-​life of 4
hours, and so treatment later in the day after dialysis is feasible, but the
so-​called ‘dialysis hangover’ (a collection of unpleasant post-​treatment
symptoms) may deter patients from undergoing treatment immediately.
Some authors do argue treatment on the day before dialysis is better
when major surgery is required, because it will remove post-​operative
inflammatory mediators or high protein loads from swallowed blood.
Either way, preoperative clotting screens, appropriate reduction in any
drug used, and a discussion with the renal physician are mandatory.
Keywords: extraction, CKD stage 5, haemodialysis.
→ Greenwood M, Seymour R, Meechan J. Textbook of Human Disease in
Dentistry. Wiley-​Blackwell, Oxford; 2009.​
→ Proctor R, Kumar N, Stein A, Moles D, Porter S. Oral and dental aspects
of chronic renal failure. Journal of Dental Research. 2005;84:99–​208.

https://t.me/DentalBooksWorld
Chapter 0 191

Therapeutics and
medical emergencies
Tariq Ali

‘Primum non nocere.’


Firstly, do no harm. This is held as the first law of clinical practice when
considering any intervention to improve the health of our patients. This
may at times be a difficult proposition, especially when the approach
to treating a condition is fraught with risks and can carry the danger of
adverse and unwanted side effects.
Prescribing therapeutics is the time perhaps when this maxim should
be most at the forefront of a clinician’s mind, as therapeutic interven-
tions may not cause any immediately discernible danger or harm in the
same way as operative interventions. It is therefore important for the
prescriber to understand the relevant pharmacodynamics (the effects
of the agent on the body) and pharmacokinetics (the effects of the
body on the agent). To add a further layer of complexity, the reader
should understand that pharmacological sciences are possibly the fastest
evolving part of medicine. It would be a fair bet to say that, within the
course of the reader’s undergraduate education, entire new classes of
therapeutics will have emerged and established perceptions of other
agents would have significantly changed.
Practically speaking, this does not mean that it is necessary to mem-
orize the nuances of all therapeutic agents (although you should have a
good grasp of those you prescribe regularly); rather it is more important
that a clinician understands how to recognize potential dangers and then
be resourceful enough to mitigate against them, given the best know-
ledge available at the time. Access to an up-​to-​date formulary and the
will to use it are the surest way to navigate any prescribing pitfalls.
‘We don’t rise to the occasion, we fall to the most basic level of
our training’.
Thankfully, medical emergencies occur infrequently in the general
practice setting. It is the rarity of such events that often leads to anxieties
when dealing with them. This reaction is amplified by the caregiver’s nat-
ural instinct to do something immediately, but often not knowing exactly
what to do because the diagnosis is not immediately clear.
The ABCDE approach, as advocated by the Resuscitation Council, is
a safe and methodical way to approach any emergency. ABCDE is not
only a hierarchy of importance for systems critical to life, but it also acts
as an aide-​memoire to undertake examinations and interventions when
necessary. Most importantly, it buys time whilst the diagnosis is found
or declares itself, without adversely affecting the outcome by inaction.

https://t.me/DentalBooksWorld
192 Chapter 0 Therapeutics and medical emergencies

Key topics include:


● Common drugs for common medical conditions
● Drug interactions
● Oral side effects of medications
● Basic life support
● Medical emergencies in dentistry (including the Resuscitation Council
UK guidelines)
● ABCDE approach.

https://t.me/DentalBooksWorld
questions 193

QUESTIONS

. Prior to the administration of a local anaesthetic, a 25-​year-​old


man with well-​controlled epilepsy begins to have a tonic–​clonic
seizure. The dental chair is laid flat, and high-​flow oxygen is delivered
through a non-​rebreather mask. His blood sugar is checked, whilst your
nurse has called for an ambulance; the result is 5.3 mmol/​L. After 5 min-
utes, the seizure has not self-​terminated. What is the single best inter-
vention that should be available in dental practices? ★
A Buccal midazolam 0 mg
B Intravenous lorazepam 4 mg
C Intravenous phenytoin infusion 40 mL (0%)
D Oral diazepam 0 mg
E Rectal diazepam 35 mg

2. A 40-​ year-​
old insulin-​
dependent diabetic man becomes very
sweaty and tremulous during a dental examination. He thinks his
sugars are low and asks whether any sweets are available. As the nurse
goes to get some, he rapidly deteriorates, developing speech slurring
before losing consciousness. He is still breathing and has a pulse. What
would be the single best course of action to take? ★
A Administer 0 mL of oral glucose gel
B Administer high-​flow oxygen and 300 mg of soluble aspirin dissolved
in water
C Infuse 20 units/​mL of insulin intravenously
D Inject glucagon  mg intramuscularly
E Use ammonium carbonate  puff nasally

https://t.me/DentalBooksWorld
194 Chapter 0 Therapeutics and medical emergencies

3. A nurse raises the alarm as she finds a 77-​year-​old man uncon-


scious. She has already called for an ambulance, but she wants
help managing him. Following the ABCDE approach, after 0 seconds,
no pulse or respiratory effort is noted. Another nurse has retrieved the
resuscitation trolley. What would be the single best immediate course
of action to take? ★
A Administer a precordial thump, whilst an automated external defibril-
lator (AED) is attached
B Begin chest compressions at a rate of 5:2 (5 compressions and
then 2 breaths), whilst an AED is attached
C Begin chest compressions at a rate of 30:2 (30 compressions and
then 2 rescue breaths), whilst an AED is attached
D Begin continuous chest compressions at a rate of 00–​0, whilst an
AED is attached and oxygen is continuously delivered
E Place the patient in the recovery position, and continuously deliver
oxygen until the ambulance arrives

4. A 29-​year-​old woman attends, concerned about the recent dark


staining of her teeth. She has acne vulgaris, iron deficiency anaemia
associated with heavy periods, hay fever, and gastro-​oesophageal dis-
ease (GORD). She has also been regularly using 0.2% chlorhexidine
digluconate mouthwash for periodontal disease for the past 3 months,
having seen a television advert. Her most recent prescription is
shown below:
Doxycycline 00 mg once daily (OD)
Diphenhydramine 25 mg four times daily (QDS)
Ferrous fumarate tablets 20 mg twice daily (BD)
Lansoprazole 5 mg OD
Which single medication is most likely responsible for the discoloration
in this scenario? ★★
A Chlorhexidine digluconate
B Diphenhydramine
C Doxycycline
D Ferrous fumarate
E Lansoprazole

https://t.me/DentalBooksWorld
questions 195

5. A 40-​year-​old woman enters the dental surgery, visibly breathless,


coughing, and wheezing. She expresses that her fingers feel tingly
but struggles to complete her sentence. She is a known asthmatic, so the
nurse exits the room to call for an ambulance. What is the single most
appropriate next step? ★★
A Administer beclomethasone 0 mg orally
B Administer ten puffs of salbutamol via a spacer
C Administer adrenaline 0.5 mg intramuscularly
D Measure her peak expiratory flow rate (PEFR) before administering
any drugs
E Oxygen via a non-​rebreather mask at 0–​5 L/​minute

6. A 68-​year-​old woman is waiting for a dental appointment. Whilst


waiting, they become flushed, clammy, and short of breath.
A dental team are called to help, at which point she tells them she has
some central crushing chest pain that radiates into her jaw and to her left
arm. She has tried her glyceryl trinitrate (GTN) spray, with no effect on
her symptoms. What is the single most likely diagnosis? ★★
A Acute coronary syndrome (ACS)
B Angina attack
C Aortic aneurysm rupture
D Oesophageal spasm
E Pulmonary embolus

7. A 64-​year-​old man attends his local dental practice for surgical


extraction of his upper right first molar (UR6). He is anxious as he
sits in the chair. Medically, he takes insulin for diabetes mellitus, ramipril
for hypertension, and atorvastatin for high cholesterol. After adminis-
tration of the local anaesthetic, he begins to experience crushing central
chest pain that radiates to his neck and left arm. He is noticeably clammy
and short of breath. Which medications from the emergency kit are the
most appropriate for managing this situation? (Select one answer from
the options listed below.) ★★
A Oxygen and midazolam
B Oxygen, aspirin, glyceryl trinitrate
C Oxygen, aspirin, salbutamol
D Oxygen, glucose, glucagon
E Oxygen, salbutamol, adrenaline

https://t.me/DentalBooksWorld
196 Chapter 0 Therapeutics and medical emergencies

8. A 70-​year-​old man attends as a new patient. He is being treated


for hypertension and ischaemic heart disease by his doctor. He
has had a coronary artery stent placed 3 years ago and a renal trans-
plant roughly 20 years ago. He takes bendroflumethiazide, nifedipine,
atorvastatin, clopidogrel, and ciclosporin. Clinically, his gingivae are very
enlarged, with a lobular shape, and there are deep periodontal pockets.
Which single medication is most likely responsible for the signs? ★★★
A Atorvastatin
B Bendroflumethiazide
C Ciclosporin
D Clopidogrel
E Nifedipine

9. A 23-​year-​old woman presents with severe facial swelling and signs


suggestive of systemic involvement. Antibiotics are prescribed and
administered at the dental practice, before referring her to the local
Maxillofacial Unit. Whilst awaiting the taxi, the receptionist notices a
deterioration in her situation. She has developed an urticarial rash, an
audible wheeze, cold sweaty hands, and a thready, rapid, pulse. What is
the single most appropriate next step to take? ★★★
A Administer five puffs of the Ventolin® inhaler via a spacer
B Administer 0 mg of chlorphenamine orally
C Administer hydrocortisone 200 mg intramuscularly (IM)
D Administer adrenaline 0.5 mg IM
E Administer 0.5 mL of :000 adrenaline intravenously

0. A 58-​year-​old woman, who has smoked 26 pack years, has


a past medical history of breast cancer, rheumatoid arthritis,
osteoporosis, and Crohn’s disease. She historically received radio-
therapy for breast cancer and is currently taking methotrexate, folic acid,
prednisolone, and twice-​yearly denosumab injections. She had an extrac-
tion 3 months ago, but there is no evidence of healing and exposed nec-
rotic bone is present. What is the single most likely causative agent for
the pathology? ★★★
A Denosumab
B Methotrexate
C Prednisolone
D Radiotherapy
E Smoking

https://t.me/DentalBooksWorld
questions 197

. A 24-​year-​old man attends as a new patient. He has just re-


turned from working as a holiday rep in Ibiza. He has been
taking numerous recreational drugs, caffeinated drinks, and his usual
antidepressant (citalopram). Clinically, he has scalloping of the lateral
borders of the tongue, occlusal wear facets on opposing cusps, and
hypertrophic masseters. Which single recreational drug is most likely
linked to these findings? ★★★
A Caffeine
B Cocaine
C Ketamine
D Marijuana
E MDMA (3,4-​methylenedioxymethamphetamine)

2. A 55-​year-​old woman attends, complaining of multiple large


mouth ulcers over the last 3 months. She sees her cardiologist
for severe angina who started a new oral medication 9 weeks ago. She
takes this regularly and uses her glycerin trinitrate spray, as necessary.
Clinically, the mouth ulcers are similar in appearance to major recurrent
aphthous stomatitis (RAS). It is suspected that the new medication is
linked to the mouth ulcers. Which single commonly prescribed cardiac
medication has the patient most likely been prescribed? ★★★
A Bisoprolol
B Naproxen
C Nicorandil
D Nifedipine
E Ramipril

3. A 34-​year-​old man attended the Emergency Department 2 days


ago, complaining of toothache. He was diagnosed (provision-
ally) with a dental abscess, prescribed amoxicillin, and advised to see
his dentist. Since then, he has been feeling progressively unwell, with
fever and sore throat. Numerous painful mouth ulcers have stopped
him eating and drinking normally, and he has developed a rash on his
abdomen. There are a number of lesions which display a red and white
target appearance, some of which have blistered. What is the single
most likely diagnosis? ★★★★
A Erythema multiforme minor
B Erythema multiforme major
C Primary herpetic gingivostomatitis
D Staphylococcal scalded skin syndrome
E Steven–​Johnson syndrome (SJS)

https://t.me/DentalBooksWorld
198 Chapter 0 Therapeutics and medical emergencies

4. An 82-​year-​old man with hypertension, angina, and epilepsy


presents for a routine examination. He is concerned that his
breath has begun to smell in the last 3 weeks. He does not feel he has a
dry mouth; his diet is unchanged, and routine dental examination does
not appear to offer a cause for his halitosis. His cardiologist has changed
one of his medications 2 months ago. Which single commonly prescribed
medication for his medical problems could explain his halitosis? ★★★★
A Bendroflumethiazide
B Isosorbide dinitrate
C Lamotrigine
D Ramipril
E Ranitidine

5. An 8-​year-​old woman presents as an emergency after having


had a simple extraction by a colleague earlier that day. Bleeding
has still not stopped, but because she was nervous, she forgot to men-
tion before the extraction that she has been started on ticagrelor fol-
lowing a coronary artery stent placement 4 months ago. Clinically, the
socket has been packed and sutured but oozes blood every time it is
agitated. Her blood pressure is 29/​78 mmHg, and her pulse is 72 beats
per minute. Which therapeutic intervention may be of benefit in this
situation? (Select one answer from the options listed below.) ★★★★
A Intravenous (IV) protamine sulfate
B IV prothrombin complex
C Oral vitamin K
D Topical 5.5% ferrous sulfate
E Tranexamic acid 5% mouthwash

6. A 62-​year-​old man with hypertension attends for a routine


dental examination. He begins to complain of sudden-​onset,
severe headache at the back of his head 5 minutes after walking into
your room. He mentions he has never experienced any headaches in his
life and that the pain feels like he was being hit with a cricket bat. The
nurse is asked to call for an ambulance. After 0 minutes of onset, the
pain becomes unbearable and he develops an aversion to light and says
that his neck feels stiff and he wants to vomit. What is the single most
likely diagnosis? ★★★★
A Idiopathic intracranial hypertension
B Meningitis
C Migraine
D Subarachnoid haemorrhage
E Trigeminal neuralgia

https://t.me/DentalBooksWorld
questions 199

7. A new dental nurse is unclear on the medical emergency proto-


cols within the practice. She informs the practice manager that
she has not carried out any continuing professional development (CPD)
during her first 2 months as a registered nurse. What is the single most
appropriate action in these circumstances? ★★★★
A Advise the nurse to complete a minimum of 0 hours of CPD in the
next 2 months, in line with her current personal development plan
B Advise the nurse to contact the General Dental Council (GDC)
about her situation
C Give the nurse the practice’s standard development plan for new
nurses, for the remaining 4 years of her CPD cycle
D Reassure the nurse and inform her she must complete 50 hours
of CPD by the end of her 5-​year cycle, of which 50 hours must be
verifiable
E Reassure the nurse and inform her she must complete 50 hours of
verifiable CPD in each 5-​year cycle

8. A 39-​year-​old woman has just had four dental restorations


performed, under intravenous (IV) conscious sedation with
midazolam. After completing treatment, there are 2 mL of midazolam
left. Which single option is the most appropriate for managing the re-
maining drug? ★★★★
A All remaining midazolam should be administered
B Decant into a sterile container for storage
C Denature or irretrievably dispose of the remaining midazolam
D Securely store all remaining midazolam in the original packaging
E Sign midazolam back into the local pharmacy/​controlled drug store

https://t.me/DentalBooksWorld
200 Chapter 0 Therapeutics and medical emergencies

ANSWERS
. A ★
Most epileptic seizures self-​terminate, without the need for pharmaco-
logical intervention and usually in under 5 minutes. Once the initial first-​
aid steps have been taken to ensure that the patient is lying in a safe
position, there is no obstruction to the airway, and high-​flow oxygen is
being administered, thoughts should turn to interventions that may help
terminate the seizure. Before assuming the aetiology is idiopathic epi-
lepsy, one should search for reversible causes such as low blood sugar
and electrolyte imbalance—​the latter is only practicable in the hospital
setting. In this case, the patient was not hypoglycaemic.
The initial agent of choice for seizure termination is a medium-​to long-​
acting benzodiazepine; ideally, this should be given intravenously (IV),
e.g. lorazepam 2–​4 mg. In the dental practice setting, buccal midazolam
should be available as part of the emergency kit, and this is a safe first-​
line intervention whilst waiting for emergency assistance to arrive. If
there is no effect from the first dose, it can be repeated. Oral diazepam,
commonly distributed in tablet form, is unlikely to be absorbed and
would be a significant aspiration risk.
A seizure lasting 5 minutes or multiple seizures without clear neurological
recovery is defined as status epilepticus and is an extreme neurological
emergency, as prolonged seizure times are associated with post-​seizure
neurological deficits and death. Continuous seizure activity carries a risk
of hypoxia from inadequate ventilation, which explains the potential
complications.
Keywords: epilepsy, seizure, after 5 minutes.
→ Resuscitation Council UK. Quality standards for cardiopulmonary
resuscitation and training. Available at: https://​ www.resus.org.uk/​
quality-​standards/​
→ The Scottish Government, National Dental Advisory Committee.
Emergency drugs and equipment in primary dental care. 205. Available
at: http://​www.scottishdental.org/​wp-​content/​uploads/​205/​0/​
Emergency-​Drugs-​and-​Equipment-​in-​Primary-​Dental-​Care-​205.pdf

2. D ★
It is important to remember that the response to all medical emergen-
cies should follow the ABCDE (Airway, Breathing, Circulation, Disability,
Exposure) approach. This patient continues to breathe and have a pulse,
and the progression of the scenario clearly points towards a hypogly-
caemic attack (defined as a blood sugar level of 3.9 mmol/​L or below).
Symptoms are as follows:
● Autonomic: sweating, palpitations, hunger, tremor
● General: headache, nausea
● Neurological: confusion, paralysis, seizures, coma.

https://t.me/DentalBooksWorld
answers 201

Glucagon is a hormone that antagonizes insulin and increases glucose


levels by glycogenolysis (breakdown of stored hepatic and muscular
glycogen into glucose) and gluconeogenesis (production of new glucose
from amino acid substrate or lipids) in the liver. Maximal plasma concen-
trations are achieved within 5 minutes, and the half-​life is between 8 and
8 minutes, so it is important to give further simple (sugary foods) and
complex (starchy foods) carbohydrates when the patient regains con-
sciousness to prevent rebound hypoglycaemia.
Oral glucose should be used in those with a preserved conscious level.
Despite the fact that it can be rubbed into the oral mucosa, there is an
aspiration risk and, if possible, other measures should be tried first. It
should be noted that prolonged periods of hypoglycaemia may lead to
irreversible brain damage.
There appears to be no clear indication for oxygen administration, but
it is good practice to give oxygen in emergency situations until a reliable
oxygen measurement can be undertaken.
Diabetics can have so-​called silent (asymptomatic) myocardial infarc-
tions, due to autonomic neuropathy, and aspirin may be helpful in these
acute ischaemic events. However, this patient’s symptoms can be better
explained by hypoglycaemia, and it may be unsafe to administer oral
medication to a patient with variable consciousness.
Ammonium carbonate are used as smelling salts and has previously been
used as a stimulus to reverse non-​serious causes of syncope (such as
orthostatic, vasovagal, or postural hypotension).
Keywords: diabetic, sugars are low, in and out of consciousness.
→ National Institute for Health and Care Excellence, British National
Formulary. Treatment of hypoglycaemia. Available at: https://​bnf.nice.
org.uk/​treatment-​summary/​hypoglycaemia.html

3. C ★
This question tests working knowledge of the basic life support algo-
rithm. All General Dental Council (GDC)-​registered dentists and dental
care professionals should undergo yearly basic life support (BLS) training.
The Resuscitation Council (UK) guidelines updated in 205 stipulate
chest compression at a ratio of 30:2 and a rate of 00–​0 for those
who are trained to do so.
For those who are not trained or not confident, there is increasing
emphasis on providing continuous chest compressions until the airway
can be secured. The reason for this approach is to improve the quality
of bystander chest compressions and this may lead to less neurological
morbidity in successful resuscitations. With a full emergency drug kit,
a pocket mask would be available and abstaining from breaths due to
cross-​infection reasons would not be appropriate. The emergency kit
in dental practices must comply with the latest GDC and Resuscitation
Council (UK) guidelines. The current Resuscitation Council (UK) minimal
requirements in a primary dental care setting include an AED.

https://t.me/DentalBooksWorld
202 Chapter 0 Therapeutics and medical emergencies

In situations where a patient is found unconscious, assessing for danger


and then following the ABCDE approach is highly recommended and
provides a systematic way to assess the patient in a high-​stress situation.
In cases of cardiac arrest, calling for an ambulance early and applying
defibrillation as soon as possible are vital.
Keywords: unconscious, no pulse or respiratory effort.
→ Resuscitation Council (UK). Adult basic life support and auto-
mated external defibrillation. 205. Available at: https://​ www.
resus.org.uk/​ resuscitation-​ g uidelines/ ​ a dult- ​ b asic- ​ l ife- ​ s upport- ​ a nd-​
automated-​external-​defibrillation/​

4. A ★★
Tetracyclines (which are used for acne vulgaris) and diphenhydramine
(the active ingredient of over-​the-​counter antihistamines like Benadryl®)
are known to cause intrinsic staining of the teeth when taken during
tooth development. Chlorhexidine mouthwash and liquid iron salts can
cause extrinsic tooth discoloration that can be reversed with cessation
of treatment and professional cleaning.
Lansoprazole, a proton pump inhibitor used to treat GORD, has
been reported to stain the tongue yellow but has no staining effect
on teeth.
In this situation, the staining has developed recently and is unlikely to
be intrinsic staining from medication. Although the iron salts can cause
staining, the patient is taking tablets, not a liquid form, and therefore
the chlorhexidine mouthwash is the obvious culprit. Chlorhexidine
mouthwash is known to cause staining of the teeth and recommended
to be used for periods no longer than 2 weeks at a time. If patients
are required to use chlorhexidine for longer periods, then they should
be warned of the risk of staining and other side effects (e.g. taste
disturbances) and should be advised to avoid food and drink imme-
diately after using the mouthrinse. Alternative daily formulations are
now available.
Keywords: recent, dark staining, chlorhexidine, 3 months.

5. B ★★
The patient is exhibiting signs and symptoms of a severe asthma at-
tack: shortness of breath, wheeze, inability to complete sentences, high
pulse rate, and high respiratory rate. PEFR is extremely important as
part of the assessment of asthma exacerbations. A PEFR of <50% of
the predicted value for this patient would indicate severe restrictive
airway disease. However, it is not the intervention that would improve
the patient’s condition.
In this situation, salbutamol (a short-​acting beta-​2 receptor agonist that
works to relax smooth muscles in the bronchioles) takes priority. Patients
above the age of 5 years can have ten puffs of salbutamol inhaler via
a spacer. Salbutamol administration can be repeated every 0 minutes
until the ambulance arrives. There is evidence that this has a comparable

https://t.me/DentalBooksWorld
answers 203

effect to 5 mg of nebulized salbutamol—​the likely first-​line treatment


this patient would receive from paramedics or at hospital.
Beclomethasone (inhaled steroid) via a spacer in an emergency setting is
a suboptimal treatment, as the amount delivered to the patient can be
very variable and they are slow-​acting. Oral steroids are the preferred
choice and may be given by medical practitioners but again are slow-​
acting in comparison to beta-​2 agonists.
High-​flow oxygen should be administered until the oxygen level can
be measured. Oxygen can be safely withdrawn if oxygen saturations
are above 94%. Again, in this scenario, salbutamol takes priority, as
bronchoconstriction should be reversed to give the best chance of
treating potential hypoxia. If a nebulizer were present, then both could
have been administered concomitantly. Alternatively, oxygen should be
given via a non-​rebreather mask, whilst salbutamol is not being inhaled.
Adrenaline, given as an intramuscular injection, is an important treat-
ment for anaphylaxis with cardiovascular compromise. This patient dis-
plays no other signs of anaphylaxis, and you have already deduced she
has airways disease from her past medical history.
Finally, it is important to state that an ambulance would need to be
called and the patient should be reassessed regularly after administering
the drugs.
Keywords: visibly breathless, wheezing, struggles to complete her sen-
tence, asthmatic.

6. A ★★
The patient has chest pain at rest, which is radiating to the left arm and
jaw. These features can be best described as ACS. Other features include
autonomic features (sweating, nausea) and the heart trying to compen-
sate for local hypoxia by increasing the cardiovascular rate (breathing
and heart rate).
ACS is the preferred nomenclature for the spectrum of conditions that
include:
● Unstable angina—​chest pain at rest in someone with known heart dis-

ease, due to cardiac muscle ischaemia (inadequate oxygen perfusion)


● Non-​ST segment* elevation myocardial infarction (NSTEMI) (death of

cardiac muscle)—​a minor heart attack that does not lead to electrical
changes on the electrocardiogram (ECG)
● ST segment* elevation myocardial infarction (STEMI)—​a major heart

attack leading to electrical changes on the ECG.


GTN is a vasodilator (widens artery diameter by causing smooth muscle
relaxation) and can relieve ischaemic pain associated with angina (cor-
onary ischaemic chest pain which is brought about by activity and re-
lieved by rest or nitrates).
Pulmonary embolism can lead to chest pain. But this is typically pleur-
itic (worsened by deep inspiration). Other features include haemoptysis,
*ST segment refers to a specific electrical deflection noted on the ECG.

https://t.me/DentalBooksWorld
204 Chapter 0 Therapeutics and medical emergencies

dyspnoea, tachycardia, features of deep vein thrombosis, cough, and


syncope.
Oesophageal spasm can present very similarly to ACS. There is normally
an absence of autonomic features, and if the patient lives an active life-
style, there would be an absence of preceding exertion-​induced chest
pain (this would be difficult to elicit in sedentary patients!).
Thoracic aortic aneurysm rupture presents with acute chest pain. It can
lead to chest pain that radiates to the back and is described as ‘tearing’
in nature. Other clinical features include a difference in blood pressure
between both arms.
Keywords: central crushing pain, radiates into jaw and left arm, GTN,
no effect.

7. B ★★
The scenario clearly alludes to an acute coronary syndrome (ACS)—​
the umbrella term for conditions that range from unstable angina to
acute myocardial infarction (MI). An electrocardiogram (ECG) would
be required to differentiate the severity of ischaemic heart disease,
the classical signs of which are chest tightness that may radiate into
the left arm or jaw, shortness of breath, nausea, vomiting, clammi-
ness, and a ‘sense of impending doom’. It should be noted patients
may present with atypical chest pain, particularly the elderly and
diabetics.
Oxygen is important in ensuring that there is adequate oxygenation of
blood perfusing the heart. There is evidence to suggest hyperoxygenation
can cause reperfusion injury and that it should only be administered if
oxygen saturations are below 96% on room air. In the absence of reli-
able pulse oximetry, it should always be administered. Oxygen will help
correct any hypoxia present and should be given until pulse oximetry
can be taken.
The most common cause for coronary ischaemia and infarction is rup-
ture of atherosclerotic (subendothelial calcified fatty plaques) lesions and
subsequent formation of a thrombus (blood clot) that occludes blood
supply to the cardiac muscle. Aspirin, which has antiplatelet effects and
minimizes the formation of a thrombus, must be given promptly at a
loading dose of 300 mg.
Glyceryl trinitrate (GTN) releases nitric oxide, a potent smooth muscle
relaxant that leads to vasodilatation and increased perfusion. This is the
intervention most likely to lead to the fastest symptom relief.
Morphine is also recommended for pain relief in a secondary care envir-
onment, but this will not be available in a primary care setting. MONA
is a useful acronym to remember the necessary medication in a hospital
setting—​morphine, oxygen, nitrate, aspirin.
Keywords: diabetes, hypertension, atorvastatin, crushing central chest
pain.

https://t.me/DentalBooksWorld
answers 205

8. C ★★★ OHCD 6th ed. → p. 436


Phenytoin, ciclosporin, and nifedipine (calcium channel blockers) interact
with epithelial keratinocytes, fibroblasts, and collagen, leading to an
overgrowth of gingival tissue which is exacerbated by gingival inflam-
mation. Gingival hyperplasia can be complicated by bleeding, pain, and
periodontal disease, as hyperplastic tissue may lead to plaque trapping.
Unfortunately, the presence of plaque may induce further enlargement
and lead to a vicious cycle of disease. If plaque control can be optimized,
then often gingival inflammation will resolve. However, surgical removal
of the enlarged tissue via gingivectomy is required to facilitate effective
oral hygiene. Further management may include liaising with the patent’s
general practitioner (GP) to change medication, where possible.
In this scenario, both ciclosporin and nifedipine are contributing to gin-
gival overgrowth, but the literature would suggest that gingival over-
growth is more common with ciclosporin. Anecdotally, it also tends to
be more excessive than with calcium channel blockers [phenytoin (50%
incidence) > ciclosporin (30%) > nifedipine (20%)].
Keywords: renal transplant, ciclosporin, gingivae, enlarged.
→ Seymour RA, Thomason JM, Ellis JS. The pathogenesis of drug induced
gingival overgrowth. Journal of Clinical Periodontology. 996;23:65–​75.

9. D ★★★
The patient is having an anaphylactic shock precipitated by the anti-
biotics. It is a type I [immunoglobulin E (IgE)-​mediated] hypersensitivity
reaction. Following an initial ABCDE approach, the most important step,
if this is suspected, is to give adrenaline 0.5 mg IM (0.5 mL of :000).
It can be repeated if the patient’s cardiovascular parameters do not
respond every 5 minutes.
Adrenaline can be administered intravenously, but the clinician must be
experienced with intravenous administration of adrenaline, and it shoud
be undertaken with cardiac monitoring. Therefore, it is not recom-
mended that dental professionals use this route. If you suspect hypo-
tension, then the patient should be nursed supine, with the feet raised.
High-​flow oxygen through a non-​rebreather mask is also an important
part of the patient’s management. Clearly, the patient needs to be sent
to an Emergency Department via an emergency ambulance.
The other options are worthwhile exploring after adrenaline is given.
For instance, antihistamines are important, but the recommended route
is intravenous, chlorphenamine (0 mg) being the agent of choice.
Salbutamol is reasonable if the patient is wheezy, preferably via a nebu-
lizer (5 mg); otherwise, 5–​0 puffs via a spacer can be very effective.
Another important treatment is corticosteroids (hydrocortisone 200 mg
intravenously). Although corticosteroids take approximately 6 hours to
work (independent of the route of administration), it is important to
administer this medication in a timely fashion.
Keywords: antibiotics, urticarial rash, wheeze, thready, rapid, pulse.

https://t.me/DentalBooksWorld
206 Chapter 0 Therapeutics and medical emergencies

→ Resuscitation Council (UK). Emergency treatment of anaphylactic reac-


tions: guidelines for healthcare providers. 2008. Available at: https://​www.
resus.org.uk/ ​ a naphylaxis/ ​ e mergency-​ t reatment-​ o f-​ a naphylactic-​
reactions/​

0. A ★★★ OHCD 6th ed. → p. 364


Medication-​related osteonecrosis of the jaw (MRONJ) is defined as
exposed or necrotic bone of the maxillofacial skeleton 8 weeks after
dental treatment in patients who have a history of using anti-​resorptive
(bisphosphonates and denosumab) or anti-​angiogenic agents and that
cannot be attributed to any other cause (i.e. jaw bone necrosis sec-
ondary to radiotherapy).
Bisphosphonates are a useful therapy for conditions that include osteo-
porosis and hypercalcaemia resulting from malignancies like myeloma
and breast cancer. It can happen spontaneously but more commonly
occurs after instrumentation or local trauma. Risk factors for MRONJ
include intravenous bisphosphonates, long-​term oral bisphosphonate
use, especially with concomitant steroid use (e.g. prednisolone), im-
munosuppressants (e.g. methotrexate), and a previous history of
MRONJ. Secondary risk factors include smoking, poor oral hygiene, pre-​
existing inflammatory disease (e.g. periodontitis/​periapical disease), and
denture trauma. These patients should ideally have their oral health op-
timized prior to commencing bisphosphonate therapy and less traumatic
treatments (root canal treatment or coronectomy versus extraction)
provided where possible. If extraction cannot be avoided, one should
consider the least traumatic method and frequent post-​operative re-
views, to ensure healing of the patient, or referral to an oral surgeon.
Denosumab [monoclonal antibody inhibiting the receptor activator of
nuclear factor kappa-​Β (RANK) ligand] is an alternative drug used to
treat osteoporosis, which has also been implicated in MRONJ. However,
as it has no bone-​binding affinity, and the actions only last for 6 months.
Patients receiving this medication may benefit from drug cessation for
6 months, but there is no consensus on drug holidays as yet and this deci-
sion must be made in conjunction with the patient’s physician.
Bone turnover markers like C-​ terminal telopeptide (CTX)/​ beta
crosslaps in serology have been suggested for use to stratify risk where a
serum value of >50 pg/​mL is suggestive of a safe threshold for invasive
bone procedures. However, the evidence for this is less than definitive.
Although radiotherapy can cause osteonecrosis, in this scenario, the pri-
mary beam would not have affected the head and neck region; therefore,
it is very unlikely to be implicated. A threshold value of 65 Gy of radi-
ation to the facial skeleton confers significant risk of osteoradionecrosis.
Keywords: denosumab, extraction, exposed necrotic bone.
→ Kunchur R, Need A, Hughes T, Goss A. Clinical investigation of C-​
terminal cross-​linking telopeptide test in prevention and management of
bisphosphonate-​associated osteonecrosis of the jaws. Journal of Oral and
Maxillofacial Surgery. 2009;67:67–​73.

https://t.me/DentalBooksWorld
answers 207

→ Ruggiero SL, Dodson TB, Fantasia J, et al. American Association of


Oral and Maxillofacial Surgeons position paper on medication-​related
osteonecrosis of the jaw—​204 update. Journal of Oral and Maxillofacial
Surgery. 204;72:938–​56.

. E ★★★
The findings are consistent with clenching and bruxism, both being
parafunctional activities of the jaw joint that can cause tooth wear, tooth
fracture, myalgia, joint pain, limited movement of the jaw, and head-
aches. Bruxism is a parafunctional process which involves ‘clenching
or grinding of the teeth and/​or bracing or thrusting of the mandible’.
Bruxism is considered a centrally mediated process (i.e. controlled
by higher processes in the brain). A number of psychotropic stimu-
lants and antidepressants (selective serotonin reuptake inhibitors) can
cause bruxism. Other agents which can cause bruxism include alcohol,
smoking, and caffeine. Non-​pharmacological causes include stress, anx-
iety, and sleep disorders. The influence of local occlusal factors is still
debated but is not widely considered a causative agent. It may, however,
be an aggravating factor.
In this case, the most likely causative agent is MDMA, the most potent
pharmacological agent to cause clenching and bruxism on the list. Rat
models suggest that MDMA inhibits the jaw opening reflex, therefore
allowing uninhibited action of the jaw-​closing muscles.
Keywords: recreational drugs, scalloping, tongue, occlusal wear facets,
hypertrophic masseters.
→ Milosevic A, Agrawal N, Redfearn P, Mair L. The occurrence of
toothwear in users of ecstasy (3,4-​methylenedioxymethamphetamine).
Community Dentistry and Oral Epidemiology. 999;27:283–​7.

2. C ★★★
Nicorandil is a second-​ line anti-​
anginal medication for symptomatic
benefit. It has a number of adverse drug reactions, including palpitations,
flushing, and toothache. Moreover, it is well known for causing large
painful mouth ulcers, similar in appearance to major RAS.
Naproxen belongs to a group of drugs known as non-​steroidal anti-​
inflammatory drugs. These drugs can cause upper gastrointestinal ulcer-
ation, including mouth ulcers. NSAIDs have a number of adverse drug
reactions, including kidney injury and increased cardiac risk. So it is
unlikely the cardiologist prescribed this.
Nifedipine is a calcium channel antagonist used primarily for hyperten-
sion, Raynaud’s phenomenon, and premature labour. It can cause gum
hyperplasia and hypotension.
Bisoprolol is a beta blocker, which is used primarily in arrhythmias and
ischaemic heart disease. It can be associated with a number of adverse
drug reactions, including hypotension, bronchoconstriction in asth-
matics, bradycardia, and lichenoid reactions.

https://t.me/DentalBooksWorld
208 Chapter 0 Therapeutics and medical emergencies

Ramipril is an angiotensin-​converting enzyme (ACE) inhibitor. It is used


primarily as an antihypertensive and has prognostic benefits in coronary
artery disease, cardiac failure, and left ventricular remodelling associated
with hypertension. It is associated with a dry cough, angio-​oedema, and
kidney injury. Some ACE inhibitors have been linked with oral ulceration,
but they are not specific anti-​anginal drugs and ulceration is less severe
than with nicorandil.
Keywords: large mouth ulcers, severe angina, oral medication.

3. E ★★★★ OHCD 6th ed. → p. 420


SJS or toxic epidermal necrolysis (TEN) is an immune complex-​
mediated (type III) hypersensitivity reaction that classically presents as
a mucocutaneous disorder, with febrile erosive stomatitis, severe con-
junctivitis, and disseminated cutaneous eruption. An important cause
of SJS is medication, along with infections and malignancies. Confusion
regarding the nomenclature has existed for some time. The following
describes the consensus in the literature:
● Erythema multiforme minor—​ target lesions or raised, oedema-
tous papules (circumscribed solid, raised lesion) distributed on the
extremities
● Erythema multiforme major—​ typical targets or raised, oedematous
papules distributed acrally, with involvement of one or more mucous
membranes; epidermal detachment involves <0% of total body sur-
face area (TBSA)
● SJS/​TEN syndrome—​widespread blisters predominantly on the trunk

and face, presenting with erythematous or pruritic macules and one


or more mucous membrane erosions. Epidermal detachment of 0–​
30% of TBSA is described as SJS TEN overlap syndrome, whereas
>30% of TBSA is described as TEN syndrome.
Staphylococcal scalded skin syndrome is a blistering eruption that oc-
curs with A and B toxins from disseminated staphylococcal infection. The
condition normally affects children but can also affect adults. In addition,
with time, the rash typically affects the whole body, and given the dur-
ation of this patient’s rash, it makes the diagnosis unlikely, but it is a rea-
sonable differential, given the history of abscess.
Keywords: amoxicillin, fever, sore throat, mouth ulcers, red and white,
target appearance.
→ Mockenhaupt M. The current understanding of Stevens-​ Johnson
syndrome and toxic epidermal necrolysis. Expert Review of Clinical
Immunology. 20;7:803–​3.

4. B ★★★★
Isosorbide dinitrate, a first-​line anti-​anginal medication, is associated with
halitosis. A number of other drugs are also associated with halitosis,
including:
● Chloral hydrate—​a sedative that can be misused recreationally
● Calcium channel blockers—​used for hypertension

https://t.me/DentalBooksWorld
answers 209

● Ranitidine—​a H2 antagonist used for reflux


● Statins—​used to treat high cholesterol
● Selective serotonin reuptake inhibitors—​used for treating anxiety and

depression.
Amitriptyline can cause xerostomia, which can lead to halitosis, but this
patient denies symptoms of xerostomia. Amitriptyline can cause ageusia
(losing taste).
Ramipril can cause lichenoid reactions in the oral mucosa and dysgeusia
(distortion of the correct taste). Phenytoin can cause gum hyperplasia,
and aspirin can cause white pigmentation on oral mucosa.
Development of new signs and symptoms after changes in medication
should lead you to investigate whether they may be a potential cause.
The British National Formulary (BNF) is an excellent resource and lists
common and rare known side effects of medications.
Keywords: angina, halitosis, cardiologist, changed, medications.

5. E ★★★★
Ticagrelor is a highly effective antiplatelet treatment used to min-
imize any clots that could form as a result of having a coronary artery
stent. The medication has a similar mechanism of action to that of
clopidogrel. It acts on adenosine diphosphate (ADP) receptors on
platelets, leading to their activation as part of the initial stages of clot
formation and subsequent cross-​linking with fibrin. It has a shorter
half-​life than other antiplatelet agents like aspirin, clopidogrel, and
prasugrel, and therefore, it is the only antiplatelet medication given
as a twice-​daily dose.
Tranexamic acid is a pro-​thrombotic agent that reversibly binds to plas-
minogen to prevent it from forming active plasmin (fibrin clot-​degrading
factor), thereby preventing the breakdown of cross-​links between ag-
gregated platelets and the resultant fibrin formed during the coagulation
cascade. It can be prescribed as a 5% mouthwash to be used up to five
times daily to allow for clot stabilization, or orally to provide a more
robust and systemic effect. In this case, it would be advisable to treat
this patient with a topical preparation, as a systemic agent may lead to
adverse coronary outcomes (blockage of the stent and heart attack).
Close review would be paramount if the patient was prescribed, and
sent home with, tranexamic mouthwash, along with advice on what to
do if bleeding does not stop.
Protamine sulfate is the reversal agent used for low-​molecular-​weight
heparins (LMWHs) like Clexane® and dalteparin. LMWH are a class of
anticoagulants that activate anti-​thrombin III in the coagulation cascade,
which inactivates factor Xa (thrombin), which ultimately prevents a fibrin
clot from forming.
Vitamin K is a reversal agent used for warfarin. It can be administered
orally or intravenously and works to replace vitamin K not reduced from
vitamin K epoxide (a by-​product of the production of coagulation fac-
tors II, VII, IX, and X).

https://t.me/DentalBooksWorld
210 Chapter 0 Therapeutics and medical emergencies

Ferrous sulfate 5.5% is the active ingredient in astringedent. Although


it is an effective haemostat, given the medication the patient is taking,
tranexamic acid would be more appropriate. Moreover, it produces a
precipitate that may impair wound healing.
Keywords: ticagrelor, oozes blood, therapeutic intervention.

6. D ★★★★
This patient has sudden-​onset occipital headache, with photophobia,
neck stiffness, and nausea. One could be forgiven for thinking this may be
meningitis. However, given the rapidity of onset in which the symptoms
have progressed, subarachnoid haemorrhage is the leading differential
diagnosis. Typically, the pain reaches maximal onset within 30 minutes.
Migraines normally present earlier in life, and this patient denies a pre-
vious history of headaches. They are also often associated with an aura.
The aura normally comes in the form of flashing lights, unusual smells,
or seeing blurred lines.
Idiopathic intracranial hypertension normally affects young, obese fe-
males who may be taking an oestrogen-​containing contraceptive pill.
Symptoms include headache which has features of raised intracranial
pressure (such as worse upon waking/​stooping), nausea in the morning,
papilloedema, and an increased blind spot.
Trigeminal neuralgia presents with severe intense stabbing pain, lasting
seconds, in the distribution of the trigeminal nerve. It is often unilateral,
affecting either mandibular or maxillary divisions of the trigeminal nerve.
Keywords: sudden-​onset severe headache, unbearable, aversion to light,
neck feels stiff.

7. A ★★★★
CPD is a mandatory requirement for registration with the GDC. It is
imperative that all clinicians remain up-​to-​date with new developments
and technologies. Persistent failure to comply may ultimately result in
erasure from the register. Verifiable CPD must have clear learning
objectives and outcomes and should be quality-​controlled, i.e. continu-
ally improved following participant feedback.
Enhanced CPD (commenced on  January 208 for dentists and 
August 208 for dental care professionals) instigated a number of
changes, including:
● Developing a personal development plan
● Changes in the number of verifiable hours
● Declaration of hours each year
● Formal CPD log
● Reflection for each activity
● An even spread of hours across a 5-​year cycle.

These changes follow the ‘plan, do, reflect, record’ model. In line with
these regulations, a minimum of 0 hours of verifiable CPD must be

https://t.me/DentalBooksWorld
answers 211

recorded every 2 years. Therefore, in this scenario, despite requiring 50


hours within a 5-​year cycle, the nurse must complete 0 hours of verifi-
able CPD in the next 2 months to comply with the regulations.
Non-​verifiable CPD may still be completed and reflected upon, but it is
not formally required.
Keywords: CPD, 2 months.
→ General Dental Council. Enhanced CPD guidance for dental profes-
sionals. 207. Available at: https://​www.gdc-​uk.org/​professionals/​cpd/​
enhanced-​cpd

8. C ★★★★
Midazolam is a Class C controlled drug, as defined by the Misuse of
Drugs Act 97, and comes under Schedule 3 of the Misuse of Drugs
Regulations 200. As such, there are strict rules regarding its disposal,
supply, possession, and prescribing, as well as rules regarding record-​
keeping of midazolam.
Any midazolam that has not been administered (no drug should be
unnecessarily administered to a patient), is out-​of-​date, or is no longer
required needs to be either denatured or irretrievably disposed of.
Midazolam must not be recognizable once disposed of nor should it be
deposited into the sewage system. Ideally, it needs to be incinerated.
The denaturing kits are usually a type of binding matrix that means, once
reacted with the matrix, the drug cannot be re-​extracted for use. This
requirement comes under the Misuse of Drugs Regulations 200.
The remaining answers are incorrect. Midazolam should not be stored
for future use and returned to the pharmacy.
Keywords: midazolam, completing treatment, leftover.
→ UK Medicines Information (UKMi) Pharmacists for NHS Healthcare
Professionals. How should dentists prescribe, store, order and dispose of con-
trolled drugs? 206. Available at: https://​www.sps.nhs.uk/​wp-​content/​
uploads/​206/​06/​NW-​QA78.4-​Controlled-​drugs-​for-​dentists-​.pdf

https://t.me/DentalBooksWorld
https://t.me/DentalBooksWorld
Chapter  213

Analgesia, anaesthesia,
and sedation
Thomas Albert Park

‘Can you not just put me to sleep?’


The ability to practise dentistry and provide invasive treatments to pa-
tients is based on the ability to make such procedures comfortable and
acceptable for patients to tolerate, as well as manage post-​operative
pain. A good working knowledge of the different treatment modalities
available, and analgesic agents that can be prescribed, is key to effective
management of patients. This must include the indications and limita-
tions of each modality.
The pharmacology of most drugs used in modern-​day dentistry has
changed very little since their introduction, some as far back as 00 years
ago. However, it is important to understand the processes regarding
their method of action, their effect on the human body, and their indica-
tions and contraindications. All of these factors must be considered to
maximize the clinical benefit to the patient. Several guidelines regarding
the use of conscious sedation in dentistry have recently been introduced,
and it is important that those wishing to provide conscious sedation and
refer patients appropriately familiarize themselves with these guidelines.
Key topics include:
● Principles of analgesia, anaesthesia, and conscious sedation
● Pharmacology and pharmacodynamics of commonly used pharmaco-
logical agents
● Indications and contraindications of commonly used pharmacological

agents
● Conscious sedation with nitrous oxide
● Conscious sedation with midazolam
● General anaesthesia
● Treatment planning for conscious sedation and general anaesthesia
● Managing complications and adverse reactions

https://t.me/DentalBooksWorld
214 Chapter  Analgesia, anaesthesia, and sedation

QUESTIONS

. A 27-​year-​old man has a left inferior alveolar nerve block prior to


a dental restoration. After 2 minutes, he reports he has lost feeling
along the left side of his face. Clinically, there is ptosis of the left eye,
which he cannot close fully, as well as drooping of the left corner of the
mouth. Into what single anatomical space has the local anaesthetic most
likely been administered? ★
A Buccal space
B Carotid sheath
C Parotid capsule
D Pterygomandibular space
E Submasseteric space

2. A 7-​year-​old fit and well boy attends with his mother for two
dental restorations under inhalation sedation with nitrous oxide.
The practice is equipped with a Matrix MDM® Flowmeter machine.
What is the maximum percentage of nitrous oxide that can be provided
to in this scenario? ★
A 65%
B 70%
C 75%
D 80%
E 85%

3. A 7-​year-​old fit and well girl is receiving treatment under inhalation


sedation. Nitrous oxide has been delivered at a concentration of
50% and a flow rate of 5 L/​minute, to achieve adequate sedation. All
planned dental treatment has been completed without complication, and
the session is about to be finished. What is the single most appropriate
action to complete sedation safely? ★
A Decrease the flow rate of the gases incrementally over 2 minutes
B Increase the oxygen concentration to 00% for 2 minutes
C Reduce the nitrous oxide concentration incrementally
D Remove the nasal hood
E Switch off the nitrous oxide delivery system

https://t.me/DentalBooksWorld
questions 215

4. A 50-​year-​old man is receiving an inferior alveolar nerve block for


a composite restoration. The clinician inadvertently needle-​sticks
his thumb, while withdrawing the needle. When the glove is removed,
there is a small puncture wound which is not actively bleeding. What is
the single most appropriate first stage of management for this event? ★
A Assess the relevant medical history of the patient involved
B Clean the wound under running water
C Contact your local Occupational Health Department
D Encourage the wound to bleed
E Place a sterile dressing on the wound

5. A 52-​year-​old woman is undergoing intravenous (IV) conscious


sedation for a simple restoration. She is anxious about dental injec-
tions. The final dose is titrated, based on her response. At which single
point would it be most appropriate to attempt treatment? ★★
A When the patient accepts local anaesthesia
B When the patient becomes motionless
C When the patient no longer responds to pain
D When the patient’s breathing rate is <0 breaths per minute
E When the patient’s oxygen saturation drops below 90%

6. A 45-​year-​old fit and well woman suffers with dental anxiety and
is offered the option of having her treatment completed under
intravenous sedation with midazolam. As part of the informed consent
process, the properties of midazolam are discussed. From which single
medicinal property will this patient derive the most benefit? ★★
A Amnesic
B Anaesthetic
C Analgesic
D Anticonvulsive
E Antiemetic

https://t.me/DentalBooksWorld
216 Chapter  Analgesia, anaesthesia, and sedation

7. A 35-​year-​old woman is under intravenous (IV) sedation with


midazolam for extraction of the lower left first premolar (LL4).
Before starting the extraction, her eyes have become closed and she fails
to respond to verbal commands. Which single pharmacological agent is
the most appropriate to administer in this scenario? ★★
A Activated charcoal
B Flumazenil
C Midazolam
D N-​acetylcysteine
E Naloxone

8. A 24-​year-​old fit and well, dentally anxious woman attends for an


extraction under intravenous sedation with midazolam. She has
recently been diagnosed with an allergy to ester compounds. Which
single commonly used dental anaesthetic would be contraindicated for
the management of this patient? ★★
A Benzocaine 20% topical gel
B Lidocaine 2% with :80,000 adrenaline
C Mepivacaine 3% plain
D Prilocaine 3% with felypressin
E Prilocaine 4% plain

9. A 0-​year-​old boy is being treated with inhalation sedation which


has been titrated to a dose of 60% nitrous oxide. During the treat-
ment, he begins to twitch, his pupils dilate, and his eyes become diver-
gent. When spoken to, he mumbles incomprehensibly. What would be
the single most appropriate first-​line action? ★★
A Ask the nurse to call for an ambulance
B Ask the patient to take deep breaths
C Continue treatment
D Decrease the flow rate of the nitrous oxide/​oxygen mix
E Give 00% oxygen

https://t.me/DentalBooksWorld
questions 217

0. A 2-​year-​old fit and well man is being prepared for treatment
under intravenous sedation with midazolam. Treatment in-
cludes extraction of the upper left second premolar (UL5) and ultrasonic
scaling. He arrives alone and plans to get public transport back home at
the end of his appointment. What is the single most suitable course of
action in this scenario? ★★
A Carry out the scaling under sedation, but defer the extraction
B Continue as planned, and arrange for your nurse to escort him home
C Do not carry out any sedation until the patient has an escort present
D Provide sedation as planned, but ask the patient to book a taxi first
E Provide sedation as planned, as this patient does not require an escort

. A 47-​year-​old woman has been referred for treatment under


intravenous conscious sedation. Her medical history includes
bipolar disorder, severe chronic obstructive pulmonary disease (COPD),
poorly controlled non-​insulin-​dependent diabetes mellitus (NIDDM),
hypertension, and ischaemic heart disease (IHD). She had ‘happy gas’ a
long time ago for dental treatment and requests this instead, as it worked
well previously. Which single medical condition is considered a contra-
indication for inhalation sedation (IHS) with nitrous oxide? ★★★
A Bipolar disorder
B Hypertension
C IHD
D Poorly controlled NIDDM
E Severe COPD

2. A 23-​year-​old woman attends the Emergency Department in


the evening. The on-​call maxillofacial doctor diagnoses irre-
versible pulpitis of her upper right first permanent molar (UR6). She
has an appointment with her dentist tomorrow. The doctor offers to
administer a long-​lasting anaesthetic to help with pain relief, as there is
no facility to extirpate or extract the tooth. Which commonly available
local anaesthetic is the most appropriate for this? (Select one answer
from the options listed below.) ★★★
A Articaine
B Bupivacaine
C Lidocaine
D Mepivacaine
E Prilocaine

https://t.me/DentalBooksWorld
218 Chapter  Analgesia, anaesthesia, and sedation

3. A 70-​year-​old man presents to the Oral Surgery Department


for consultation regarding the surgical extraction of a grossly
carious, partially erupted lower left third molar (LL8). Medically, he suf-
fers from Parkinson’s disease and moderate claustrophobia. He reports
dental anxiety after a previous bad experience as a child. What is the
single most appropriate strategy for anaesthesia? ★★★★
A General anaesthesia with sevoflurane
B Intravenous sedation with midazolam
C Local anaesthesia with prilocaine
D Oral sedation with diazepam
E Relative analgesia with nitrous oxide

4. An -​year-​old girl is referred to the Oral Surgery Department


for exposure of a palatally impacted upper right permanent
canine (UR3). She is not obviously anxious but has no previous experi-
ence of dental treatment and no experience of local anaesthesia. What
is the single most suitable treatment modality for this patient? ★★★★
A General anaesthesia
B Intravenous sedation with midazolam
C Local anaesthesia
D Nitrous oxide inhalation sedation
E Oral sedation with diazepam

5. A 9-​year-​old woman reattends, having had a dental restor-


ation completed an hour ago by a colleague. The notes record
the administration of 2.2 mL of 4% Citanest®. She reports to be fit and
well, with no known allergies. Clinically, the patient is cyanosed and
struggling to breathe. She now reports a similar, less severe occurrence,
following her previous restoration 6 weeks ago. From which single blood
disorder is she most likely to be suffering? ★★★★
A Haemophilia A
B Methaemoglobinaemia
C Pernicious anaemia
D Sickle-​cell anaemia
E Thalassaemia A

https://t.me/DentalBooksWorld
questions 219

6. A 42-​year-​old woman, who is non-​verbal and has severe autism,


attends for an examination with her brother, who is her carer.
She requires three simple restorations and one dental extraction. She
has accepted local anaesthesia in the past. You assess that she lacks cap-
acity to consent for dental treatment. Regarding the treatment required,
what is the single most appropriate next step? ★★★★
A As the patient lacks capacity to consent, treatment should be delayed
B Complete the required treatment as planned
C Perform the restorations, and defer the extraction until the patient
has capacity
D Discuss the options with the brother, and decide on treatment in her
best interests
E Liaise with a colleague before arranging for general anaesthesia; two-​
doctor approval is required

7. A sedation-​trained nurse is 8 weeks pregnant and informs her


line manager on the morning of a nitrous oxide sedation list
about her change in circumstances. What is the single most appropriate
course of action? ★★★★
A Advise the nurse that she is safe to continue normal duties
B Ask the nurse for proof of pregnancy
C Discuss the patient list with the clinical manager and cancel complex
treatment
D Ensure active scavenging is used on all patients throughout the list
E Liaise with the clinical manager to reassign your nurse to alternative
duties

https://t.me/DentalBooksWorld
220 Chapter  Analgesia, anaesthesia, and sedation

ANSWERS
. C ★ OHCD 6th ed. → p. 60
The answer to this question relies upon good knowledge of the anatomy
of the motor distribution of the facial nerve (CN VII). Within the
parotid gland, the main branch of the nerve divides into five terminal
branches: temporal, zygomatic, buccal, marginal mandibular, and cervical.
In this case, the needle of the local anaesthetic syringe has gone beyond
the ramus of the mandible and entered the medial aspect of the parotid
capsule. This allows the local anaesthetic to affect the branches of the
facial nerve. Aetiology may be due to patient anatomy (such as a low
sigmoid notch) or poor technique.
Local anaesthetic administration into the other listed spaces would not
result in facial palsy. However, injection of local anaesthetic into the
submasseteric space may lead to trismus, due to the volume of anaes-
thetic or haematoma formation secondary to trauma.
Management involves reassurance that the symptoms will resolve
when the anaesthetic wears off, taping the affected eye shut, as well
as prescribing eye drops/​ointment to prevent drying of the cornea
(which can result in permanent damage to the eye and vision), and
providing a protective dressing to prevent physical trauma to the
eye. Treatment may be continued or postponed, depending upon the
patient’s wishes.
Keywords: ptosis, drooping, anatomical space.

2. B ★ OHCD 6th ed. → p. 62


Inhalation sedation with nitrous oxide is widely used, as it is considered
very safe, with few contraindications. It has poor solubility in tissues and,
as such, reaches peak saturation in blood, and is then subsequently elim-
inated, very quickly. This means that the vast majority of patients will
become adequately sedated relatively quickly and at low concentrations.
Moreover, as nitrous oxide gas is expelled via expiration, recovery is
usually rapid.
Concentrations of nitrous oxide under 50% are usually sufficient for the
vast majority of patients to provide adequate sedation for treatment.
Higher concentrations can be given but are likely to result in inadvertent
over-​sedation, and close observation is required.
As a safety feature, most inhalation sedation machines, including those
fitted with a Matrix MDM® Flowmeter (a common machine used in many
UK clinics), do not allow the clinician to provide >70% nitrous oxide (i.e.
a minimum 30% oxygen). They also have a cut-​off feature, should the
oxygen supply run out. A ‘Pin Index Safety System’ ensures oxygen and
nitrous oxide cylinders cannot be attached incorrectly. This is to prevent
over-​sedation and, more dangerously, asphyxiation.
Keywords: inhalation sedation, nitrous oxide, Matrix MDM® Flowmeter.

https://t.me/DentalBooksWorld
answers 221

→ Intercollegiate Advisory Committee on Sedation in Dentistry.


Standards for conscious sedation in the provision of dental care and accredit-
ation. 205. Available at: https://​www.rcseng.ac.uk/​dental-​faculties/​
fds/​publications-​guidelines/​standards-​f or-​conscious-​sedation-​in-​the-​
provision-​of-​dental-​care-​and-​accreditation/​

3. B ★ OHCD 6th ed. → p. 62


One of the benefits of inhalation sedation with nitrous oxide over other
forms of conscious sedation is the rapid recovery from the effects of
sedation. This means that the small amount of nitrous oxide that pro-
vides sedation is quickly expired once the supply is discontinued.
A minimum period of 2 minutes on 00% oxygen is recommended to
reverse the effects of nitrous oxide. However, the patient should be
observed closely during this time. Any evidence of continuing sedation
after 2 minutes should alert the clinician that the patient requires further
administration of 00% oxygen.
Any concentration of oxygen of <00%, switching off the delivery
system, or removing the nasal hood immediately leaves the patient sus-
ceptible to the continued effects of residual nitrous oxide that could
make them unfit for discharge or delay recovery. Similarly, nitrous oxide
dissolved in tissue fluids could rapidly diffuse back into the alveoli of the
lungs, diluting oxygen and carbon dioxide concentrations in the lungs,
and prevent adequate reoxygenation of venous blood. This is termed
diffusion hypoxia.
Keywords: inhalation sedation, complete sedation safely.

4. D ★ OHCD 6th ed. → p. 354


A needle-​stick injury is a type of sharps or percutaneous injury, caused
when a medical instrument inadvertently breaks the skin. Unfortunately,
they are fairly common events and can potentially expose health pro-
fessionals to a number of blood-​borne viruses, including hepatitis B,
hepatitis C, and human immunodeficiency virus (HIV). The risk of trans-
mission for these diseases has been reported as 6–​30% (if unvaccinated),
.8%, and 0.3%, respectively.
Whilst the risk of transmission of these pathogens remains low, sharps
injuries remain incredibly emotive episodes for those involved and pre-
vention of these events is paramount. Local policies may vary from
region to region, but the following protocol has been suggested by the
UK’s Health and Safety Executive in their guidance on sharps injuries.
If you suffer an injury from a sharp which may be contaminated:*
● Encourage the wound to gently bleed, ideally holding it under

running water.
● Wash the wound using running water and plenty of soap.

* Contains public sector information licensed under the Open Government Licence v3.0.
[http://​www.nationalarchives.gov.uk/​doc/​open-​government-​licence/​version/​3/​]

https://t.me/DentalBooksWorld
222 Chapter  Analgesia, anaesthesia, and sedation
● Do not scrub the wound whilst you are washing it.
● Do not suck the wound.
● Dry the wound, and cover it with a waterproof plaster or dressing.
● Seek urgent medical advice (e.g. from your Occupational Health
Service), as effective prophylaxis (medicines to help fight infection)
are available.
Your Occupational Health Department may suggest completing a risk
assessment form with the patient, with their consent, to assess their risk
status. However, the standard procedure is that all patients should be
managed as potentially being unknown carriers of a blood-​borne virus,
regardless of a needle-​stick injury or not.
Keywords: needle-​sticks, puncture wound, first stage of management.
→ Health and Safety Executive. Sharps injuries. Available at: http://​www.
hse.gov.uk/​healthservices/​needlesticks/​

5. A ★★ OHCD 6th ed. → p. 66


Observing the patient’s response to any sedative agent is fundamental
to delivering safe and effective conscious sedation. This is achieved by
providing the sedative agent in small dose increments and evaluating the
patient’s objective and subjective responses, titrating the dose.
IV administration allows the response to be assessed almost immediately
(there is a short delay, as the agent travels through the blood vessels
to the brain, a journey approximately 20–​30 seconds long). This is dif-
ferent for oral or nasal administration of sedative agents, which can vary
considerably in terms of absorption rate and total dose absorbed, and
therefore the patient response.
A needle-​phobic patient who is willing to accept local anaesthesia is
showing signs of adequate sedation. Other signs of effective conscious
sedation include:
● An awake, communicative patient
● A calm demeanour
● Slow responses to questions
● Slurred speech
● Heart rate, oxygen saturation, and blood pressure all within normal

limits for the patient.


The other answers in this question suggest the patient has become over-​
sedated, and this should be managed appropriately.
Keywords: conscious sedation, titrated.

6. A ★★ OHCD 6th ed. → p. 64


Midazolam is a member of the benzodiazepine family of drugs. Its
pharmacological actions on the central nervous system result in disinhib-
ition, reduced anxiety, amnesia, muscle relaxation, respiratory depres-
sion, an anticonvulsant effect, and anaesthesia, amongst others.

https://t.me/DentalBooksWorld
answers 223

In this scenario, the patient will benefit from amnesia regarding her dental
treatment. The patient will not benefit from the anticonvulsant proper-
ties of midazolam, as she is fit and well, with no previous relevant diag-
nosis of a condition causing seizures.
Anaesthesia, the result of extreme over-​sedation, is the result of higher
doses of midazolam and is to be avoided. In contrast to nitrous oxide,
midazolam has no analgesic or pain-​relieving properties. Midazolam has
no proven direct antiemetic effects but can indirectly reduce a patient’s
gag reflex.
Keywords: anxiety, midazolam, informed consent.

7. B ★★ OHCD 6th ed. → p. 64


Over-​sedation can be more common with some patients, compared to
others (e.g. patients who have never had IV conscious sedation before,
with an unknown response). However, with a good, cautious titration
technique, this risk can be minimized. It is important that two sedation-​
trained members of staff are present during treatment, as the treating
clinician cannot continually assess the sedated patient.
When over-​sedation occurs, flumazenil is the reversal agent used. It is
a member of the benzodiazepine family but has no sedative effects and
competitively antagonizes other benzodiazepines. It has a shorter half-​
life than midazolam, around 40–​80 minutes, compared to –​6 hours for
midazolam.
A recommended dosing for flumazenil is 200 μg over 5 seconds, fol-
lowed by 00 μg every 60 seconds, until the sedation is reversed.
Naloxone is the reversal agent for overdose of opioid analgesics.
N-​acetylcysteine is the antidote to paracetamol overdose.
Activated charcoal is used to cause vomiting, thus limiting the absorp-
tion of substances without a specific antidote or reversal agent, when
overdose occurs.
Additional midazolam would be significantly detrimental in this situation.
Keywords: midazolam, fails to respond to verbal commands

8. A ★★
Local anaesthetics can be classified based on their chemical structure.
Ester local anaesthetics have an ester link. Conversely, amide local an-
aesthetics have an amide link between groups.
Ester local anaesthetics have a much longer history of use but are gen-
erally reserved as topical agents, due to a higher frequency of hyper-
sensitivity reactions with these compounds. It is very rare to have a true
allergy to amide local anaesthetics, with allergy to preservatives in the
amide local anaesthetic formulation being relatively more common.
Benzocaine is a common topical anaesthetic agent used on the mucosa
and skin prior to injections or cannulation. As it is an ester anaesthetic, its

https://t.me/DentalBooksWorld
224 Chapter  Analgesia, anaesthesia, and sedation

use is therefore contraindicated in this case. The other agents are amide
local anaesthetics and should be safe to use.
Keywords: allergy, ester compounds.
→ Meechan J. Local anaesthesia: risks and controversies. Dental Update.
2009;36:278–​83.

9. E ★★ OHCD 6th ed. → p. 62


These signs are indicative of over-​sedation. Verbal contact, one of the
defining features of conscious sedation, must be maintained at all times
during any form of conscious sedation.
Because of low tissue solubility of nitrous oxide, by giving the patient
00% oxygen, the inhaled nitrous oxide would be expelled rapidly and
replaced with oxygen, allowing a rapid recovery.
Asking the patient to breathe more will not help in this situation, as the
concentration of nitrous oxide you are delivering is too high and has
caused the patient to become over-​sedated in the first place. Moreover,
they are also unlikely to be able to respond to your requests.
Similarly, decreasing the flow rate will only reduce the volume of nitrous
oxide and oxygen the patient receives, not the concentration. To reverse
the sedation, you need to reduce the concentration of nitrous oxide and
increase the concentration of inspired oxygen.
You should not continue the treatment as planned because you have lost
verbal contact with the patient.
At this stage, the patient’s condition does not constitute a medical emer-
gency, and to ask for the assistance of the ambulance service is not a
first-​line action. Should the patient’s recovery be delayed or atypical, or
should they develop a concurrent medical emergency, then escalation
would be appropriate.
Keywords: 60% nitrous oxide, eyes, divergent, mumbles
incomprehensibly.

0. C ★★
Patients who have undergone conscious sedation can have variable
responses to the sedation that has been provided. This is especially
true with agents such as midazolam that has prolonged effects on pa-
tients’ cognitive function. As such, it is compulsory that patients attend
with an escort on the day of the procedure, who stays in the building
throughout the procedure (in the waiting room) and takes the patient
home immediately—​ preferably by car or taxi. The escort must be
a competent adult, must stay with the patient for the rest of the day,
and overnight, and must not be responsible for the care of anyone else
during this period.
Without an escort, the patient is at risk of harm on discharge, and no
compromise should be made in this regard. This information should be

https://t.me/DentalBooksWorld
answers 225

given to the patient as part of the preoperative information for the pro-
cedure, both verbally and in writing.
Keywords: intravenous sedation, alone.
→ Intercollegiate Advisory Committee for Sedation in Dentistry.
Standards for conscious sedation in the provision of dental care and accredit-
ation. 205. Available at: https://​www.rcseng.ac.uk/​dental-​faculties/​
fds/​publications-​guidelines/​standards-​f or-​conscious-​sedation-​in-​the-​
provision-​of-​dental-​care-​and-​accreditation/​

. E ★★★ OHCD 6th ed. → p. 62


In severe cases of COPD, respiration is controlled by hypoxic drive,
rather than higher concentrations of carbon dioxide. Extreme caution
should be exercised with such patients, as prolonged sedation, of any
kind, may cause respiratory depression, further reducing oxygen levels in
the blood. IHS with nitrous oxide provides relatively higher concentra-
tions of oxygen, compared to inspired air, for prolonged periods and so
can ‘switch off ’ the hypoxic drive, further reducing a patient’s ability to
adequately respire. Short-​term administration of oxygen is acceptable,
should a medical emergency arise.
Upper respiratory tract obstructions, such as colds or enlarged aden-
oids, can be considered a relative contraindication to IHS, because they
may make inhalation via the nose difficult or impossible.
IHS is beneficial to those with IHD and hypertension. Firstly, it decreases
stress and anxiety, which may exacerbate or precipitate acute episodes
of angina. Secondly, nitrous oxide is a vasodilator that may help to
reduce the risk of an ischaemic episode.
IHS is also beneficial for those with a prominent gag reflex. It acts both
as an anxiolytic and alters the functional trigger zone in the oropharynx
where the reflex is initiated. IHS makes the trigger zone smaller and less
sensitive. A reduction in anxiety, by any sedative means, will also gener-
ally reduce the sensitivity of the reflex.
Keywords: COPD, happy gas, contraindication.

2. B ★★★ OHCD 6th ed. → p. 606


The local anaesthetics listed have broadly similar chemical features. They
comprise a lipophilic group, a hydrophilic group, and an intermediate chain
that connects the two opposing groups. Chemically, the intermediate
chain can be an ‘ester’ or ‘amide’, and local anaesthetics are categorized
as such. The agents listed in the question are amide-​linked anaesthetics.
Variations in all three chemical components can influence the solubility,
affinity, and elimination of the anaesthetic. This influences the duration
of action and potency of the anaesthetic (see Table .). Bupivacaine
has the longest duration of action and is often used following surgery
where prolonged analgesia is desired.
Keywords: long-​lasting anaesthetic, commonly available.

https://t.me/DentalBooksWorld
226 Chapter  Analgesia, anaesthesia, and sedation

Table . Duration of action of local anaesthetics


Local anaesthetic Duration of action (hours)
Articaine –​3
Bupivacaine 3–​7
Lidocaine 0.5–​2
Mepivacaine 2–​2.5
Prilocaine 0.5–​
Adapted by permission from Springer International Publishing Switzerland: Frankhuijzen A.L.
(207) Pharmacology of Local Anaesthetics. In: Baart J., Brand H. (eds) Local Anaesthesia in
Dentistry, Second Edition. Copyright © 207.

→ Baart JA, Brand HS. Local Anaesthesia in Dentistry. Wiley-​Blackwell,


Oxford; 2008.

3. B ★★★★
Parkinson’s disease is a progressive disease of the nervous system,
marked by tremor, muscular rigidity, and slow and imprecise movement,
chiefly presenting in the middle-​aged and elderly. Benzodiazepines have a
muscle-​relaxant effect, reducing muscle tremors and stiffness, potentially
improving cooperation for such patients. However, as all benzodiazep-
ines are respiratory depressants, airway control is easily compromised
with these patients, so extreme care is required.
Inhalation sedation is of particular value in anxious patients undergoing
relatively atraumatic procedures and in children for whom the effects of
benzodiazepines are less predictable and can be paradoxically excitatory.
In the scenario presented, the patient may find the nasal hood difficult
to tolerate, due to his claustrophobia, and the level of sedation may be
insufficient for a challenging surgical extraction.
The extraction could be attempted with a local anaesthetic alone, but
the patient may find it difficult to tolerate the procedure due to his re-
ported anxiety and the effects of Parkinson’s disease.
A general anaesthetic is not appropriate as a first line of management in
this case, and should only be considered when other management op-
tions are contraindicated or are not successful.
Keywords: Parkinson’s disease, third molar, anxious.

4. A ★★★★ OHCD 6th ed. → p. 604


Impaction of upper canines are relatively common, and after lower third
molars, upper canines are the most common teeth to be impacted. The
clinician should be vigilant for this in all examinations of children from the
age of 8 onwards, as this tooth should be palpable in the buccal sulcus.
If the upper canine cannot be palpated in the buccal sulcus by the age
of 0, then referral should be made at the earliest opportunity for an
orthodontic assessment.

https://t.me/DentalBooksWorld
answers 227

Exposure of canines on the palate will likely require a palatal flap to be


raised. Although this procedure can be done under local anaesthesia,
with or without sedation, it can be a long and unpleasant procedure. The
patient’s relative inexperience at the dentist and how this may affect her
compliance during the procedure (particularly given her age) must be
considered. A negative experience could further affect her compliance
with subsequent orthodontic treatment.
Guidelines for the management of patients under general anaesthesia
are quite strict, but few centres in the United Kingdom would not con-
sider this procedure an acceptable indication for general anaesthesia.
Keywords:  years old, exposure, palatally impacted upper canine, no
previous experience of dental treatment.

5. B ★★★★ OHCD 6th ed. → p. 606


Several therapeutic agents, including local anaesthetics, can cause the
conversion of haemoglobin into a non-​ oxygen-​
binding form called
methaemoglobin. Usually methaemoglobin is converted back to haemo-
globin by the enzyme nicotinamide adenine dinucleotide (NADH)-​
methaemoglobin reductase. Deficiencies in this enzyme, or genetic
variants of haemoglobin, may mean this conversion does not happen as
readily. This resulting condition is called methaemoglobinaemia where
the oxygen-​carrying capacity of blood is reduced.
Prilocaine (Citanest®) and benzocaine reportedly carry the highest
risks of common dental anaesthetics. O-​toluidine, a liver metabolite
of prilocaine, can induce methaemoglobin. Signs such as cyanosis and
shortness of breath would be cause for concern, and the patient should
be referred immediately for medical assessment. Prilocaine use in these
patients is contraindicated, and caution should also be exercised with
benzocaine.
Haemophilia is an inherited bleeding disorder. Sickle-​cell anaemia and
thalassaemia A and B are disorders of the structure of the haemoglobin
protein, whilst pernicious anaemia involves autoimmune destruction of
stomach cells that produce intrinsic factor, which is responsible for the
uptake of vitamin B2 from the gastrointestinal tract. These disorders
may be associated with cyanosis and breathlessness, but they are unlikely
to present immediately following dental injections.
Keywords: Citanest®, cyanosed, struggling to breathe, similar, occurrence.

6. D ★★★★ OHCD 6th ed. → p. 674


The Mental Capacity Act (2005) states that all adults should be as-
sumed to have capacity until proven otherwise. It also states that no
other adult can consent for another. When determining capacity, the
first question that needs to be answered is ‘Does this patient have
an impairment or disturbance of the mind or brain?’ If the answer is
NO, then the patient therefore has the ability to make any decision
they wish, even if it is an unwise one, e.g. declining a lifesaving blood
transfusion.

https://t.me/DentalBooksWorld
228 Chapter  Analgesia, anaesthesia, and sedation

If the answer to this question is YES, then the following questions are
asked of the person in this order:
● Does the patient understand the information?
● Is the patient able to retain the information for long enough to make

the decision?
● Can the patient use or weigh up this information as part of a decision-​

making process?
● Can the patient communicate the decision by any means?

Should the answer to any one or more of these four questions also be
NO, then that patient does not have capacity to make that decision for
that particular decision. However, a patient may have capacity to make
other decisions, e.g. the patient may be able to make the decision to
have an examination, but not the extraction recommended from the
examination.
This process needs to be clearly written in the patient’s notes, including
any discussion with the family, carers, or senior staff. The reason for your
decision also needs to documented.
Any patient who does not have family or unpaid carers, or where there
is conflict over a proposed treatment between clinicians and involved
third parties, may benefit from being referred to an Independent Mental
Capacity Advocate (IMCA).
The patient’s brother is unable to provide consent, but would be in-
volved in decisions regarding her care.
It is not in the patient’s best interests to carry out no treatment or to
avoid/​ignore treatment where it is required. The brother should be con-
sulted as to what he feels is most appropriate, and then a best interest
agreement made. Two-​doctor agreement is not a fundamental require-
ment of the MCA but would be wise where radical or restrictive treat-
ment is proposed.
Keywords: Mental Capacity Act, capacity, consent.
→ Burke S, Kwasnicki A, Park T, Macpherson A. Consent and capacity—​
considerations for the dental team part : consent and assessment of
capacity. Dental Update. 207;44:660–​6.

7. E ★★★★
The use of nitrous oxide for conscious sedation is regarded as being safe
for patients, and acute exposure has not been demonstrated to present
a long-​term danger to clinical staff. However, chronic exposure to rela-
tively low levels of nitrous oxide (potentially received when conducting
inhalation sedation) can have a number of effects, which include: liver
disease, central nervous system toxicity, reduced blood cell production,
reduced fertility, and increased risk of miscarriage in females. Exposure
to nitrous oxide is managed under the Control of Substances Hazardous
to Health 2002, from the Health and Safety Executive.
Because of this, dental staff are in danger of side effects from chronic
nitrous oxide exposure. This may arise either due to leakage from poorly

https://t.me/DentalBooksWorld
answers 229

maintained equipment or through exhaled gases from the patient. To


mitigate these risks, scavenging, ideally active, is advised by many guide-
lines to help remove waste gas from the environment and limit exposure
to staff. Also, equipment should be maintained and serviced, as sug-
gested by the manufacturer.
For the above reasons, it is therefore prudent to find alternative duties
for your pregnant nurse in this situation. Cancelling patients on the day is
not ideal for either the service provider or the service user, but may be
necessary where no safe alternative is feasible.
Keywords: pregnant, nitrous oxide.
→ Donaldson D, Meechan JG. The hazards of chronic exposure to
nitrous oxide: an update. British Dental Journal. 995;78:95–​00.

https://t.me/DentalBooksWorld
https://t.me/DentalBooksWorld
Chapter 2 231

Dental materials
Raheel Aftab

‘How long will this last?’


Dental material science can be a daunting subject for most dentists, given
its origins in the pure sciences of physics and chemistry. Combining this
with human biology, and trying to see through the fog of material manu-
facturers’ commercial claims, can make it seem like a truly mystifying
subject. It is important that any student of material sciences maintains a
critical eye and an evidence-​based approach when it comes to material
selection and use.
Today we are lucky enough to work with the most advanced dental
materials we have ever had. But simply having such materials at your dis-
posal does not ensure success. Clinical procedural techniques are often
the prime focus in restorative dentistry; however, to achieve optimal
aesthetics, function, and longevity from restorations, a clear under-
standing of material sciences is required.
Ancient Roman engineers clearly understood this concept when con-
structing Rome. They had to work within the limitations imposed by the
materials they had at their disposal. However, the longevity and solidity
of the impressive infrastructure we see today can be attributed to their
expertise in exploiting the unique properties of the material resources
they had available. The Romans perfected concrete production (based
on volcanic ash and lime reacting with seawater to form tobermorite
crystals) to yield a water-​hardening material, so durable and resistant
to cracks that modern-​day concrete (based on Portland cement) is still
considered weaker.
It can be argued whether operator skill or advancements in dental ma-
terials have resulted in improvements in restorative dentistry. However,
few would disagree that it is the combination of good operator skill and
appropriate use of dental materials that is the key for successful long-​
term dentistry.
Key topics include:
● Adhesive dentistry concepts
● Understanding material physical properties
● Elemental make-​up of materials
● Manufacturing processing of materials
● Biocompatibility
● Appreciation of setting reactions and working time
● Appreciation of material aesthetic and optical properties.

https://t.me/DentalBooksWorld
232 Chapter 2 Dental materials

QUESTIONS

. A 22-​year-​old man attends for restoration of a large posterior


class V cavity with glass-​ionomer cement (GIC). Which single con-
ditioning agent is the most appropriate to improve bond strength of this
material to the tooth structure? ★
A Dentine bonding agent
B Fluorocarbonic acid
C Hydrofluoric acid
D Polyacrylic acid
E Resin-​modified GIC

2. A 26-​year-​old man returns  week after having a deep amalgam


restoration in his lower right first molar (LR6). The patient experi-
enced shooting pain that lasted for a few seconds when drinking cold
drinks. The restoration was removed by an emergency dentist and re-
placed with zinc oxide eugenol (ZOE). What is the primary benefit of
using this material in this situation? ★
A Antimicrobial
B Less microleakage
C Fast setting
D High compressive strength
E Obtundent

3. A 45-​year-​old man is having his upper left lateral incisor (UL2)


crowned. Prior to taking the impression with a monophase
addition-​cured polyvinylsiloxane (PVS) material, the tooth is thoroughly
air-​dried. What is the single main reason for this protocol? ★
A Absorption of residual surface moisture will lead to an inaccurate die
B Moisture acts as a separator between the light-​and heavy-​bodied
PVS, leading to folds
C Moisture may cause air blows in the impression, as PVS materials are
hydrophobic
D Moisture retards the polymerization of the material, potentially
causing drags
E Moisture stops the material being placed in the correct place, as it
slips off the teeth

https://t.me/DentalBooksWorld
questions 233

4. A 47-​year-​old woman has received revision root canal treatment


(ReRCT) on an upper central incisor. After chemomechanical
preparation, which single irrigant would be the most appropriate for
removing the residual inorganic matter from the root canal system? ★
A Acetone
B EDTA (ethylenediaminetetraacetic acid)
C MTAD (mixture of tetracycline, citric acid, and detergent)
D Saline
E Sodium hypochlorite

5. A 5-​year-​old boy attends for root canal treatment on a recently


traumatized upper central incisor. Following root canal treatment,
a significant proportion of the crown is missing and a glass-​fibre post is
used to retain the restoration. Which single type of cement should be
used when placing the post? ★
A Dual cured resin cement
B Flowable resin cement
C Light cured resin cement
D Resin-​modified glass-​ionomer cement
E Zinc phosphate cement

6. A 56-​year-​old man attends for construction of an upper partial


cobalt-​chromium (CoCr) denture. He has previously struggled
to tolerate a full palatal coverage acrylic denture because of his gag
reflex. The patient is a chemical engineer by trade and wishes to know
what property of CoCr allows provision of a thinner and less exten-
sive major connector. Which single property of CoCr can facilitate
this design? ★
A Good corrosion resistance
B High fatigue resistance
C High yield strength
D Low mobility
E Low thermal resistance

https://t.me/DentalBooksWorld
234 Chapter 2 Dental materials

7. A 2-​year-​old woman attends the Emergency Department, following


lateral luxation of her upper right central incisor. A self-​etch adhesive
bonding system is used for composite splint placement, as no dental chair
or water line is available. What single component of the bonding agent has
been modified to avoid the need for a separate etching stage? ★
A Bond
B Cement
C Conditioner
D Phosphoric acid
E Primer

8. A 40-​year-​old woman attends for another amalgam filling with her


new dentist. The dentist is a new graduate and, on a number of
occasions, has been struggling to pack the amalgam sufficiently before
it sets. Having spoken to their trainer, it is decided to order a lathe-​cut
amalgam instead—​for which primary reason? ★
A Easier to condense
B Hardens slowly
C Has more shrinkage
D Less difficult to achieve tight contact points
E Smoother finish after polish

9. A 46-​year-​old man has had four permanent lower molars prepared


for gold onlays to manage his generalized moderate toothwear.
The preparations have minimal retention or resistance form. What single
material is the most appropriate for cementation? ★
A Glass-​ionomer cement
B Polycarboxylate cement
C Resin cement
D Resin-​modified glass-​ionomer cement
E Zinc phosphate

0. A 44-​year-​old woman requires a mesio-​occlusal composite res-


toration in a lower first premolar. The supervising consultant
recommends a ‘wet bonding’ approach, and the assisting dental care
professional asks what this refers to. ★★
A Bonding in low humidity conditions
B Isolating without a rubber dam
C Irrigating with saline prior to surface priming
D Not excessively drying the dentine
E Not setting the bonding agent prior to composite placement
https://t.me/DentalBooksWorld
questions 235

. A 26-​year-​old man with severe learning difficulties is having an


amalgam restoration in his upper right first molar. Upon place-
ment into the cavity, the amalgam appears crumbly and is unusable. What
single procedural issue is most likely to have caused this to occur? ★★
A Amalgam contraction
B Moisture contamination
C Out-​of-​date
D Over-​burnished
E Under-​triturated

2. A 64-​year-​old woman is having a new upper complete den-


ture made. She has a hypermobile or ‘flabby’ anterior ridge,
and a window technique is planned for the master impression. What
primary property is desired of the impression material used in the flabby
area? ★★
A Dimensionally stable
B High elasticity
C High viscosity
D Low elasticity
E Low viscosity

3. A 24-​year-​old woman requires a replacement crown on her


upper right central incisor. as she dislikes the appearance,
describing it as ‘too dark’. A Vita 3D shade guide™ is used to select the
new shade. What element of colour needs to be adjusted, and in which
component of the shade guide is this reflected? (Select one answer from
the options listed below) ★★
A Chroma, the ‘M’ value
B Hue, numbers  to 5
C Hue, the ‘M’ value
D Value, the ‘L/​R’ value
E Value, numbers  to 5

https://t.me/DentalBooksWorld
236 Chapter 2 Dental materials

4. A 72-​year-​old woman attends a few days after the fit of a


replacement complete maxillary denture. She complains of a
burning/​itchy sensation of her palate. Following examination, it is be-
lieved to be a reaction to excess monomer. Which single processing
error has most likely occurred? ★★
A Curing temperature of 95°C not attained after an initial slow
heating period
B Excess powder in the resin mixture
C Final temperature of 50°C reached too quickly
D Lack of compression during flasking
E Packing process occurred during the ‘wet sand’ stage

5. A 2-​year-​old man has undergone endodontic treatment on


an upper central incisor with an open apex. He returns after
2 months, with marked discoloration of the crown. Non-​setting cal-
cium hydroxide (CH) was used as an intracanal medicament between
appointments, and sodium hypochlorite and ethylenediaminetetraacetic
acid (EDTA) were used as irrigants. Finally, mineral trioxide aggregate
(MTA) was placed as an apical barrier before completing the obtur-
ation with gutta percha and subsequently placing a coronal seal of resin-​
modified glass-​ionomer cement (RMGIC). What is the single most likely
cause of discoloration in this scenario? ★★
A Interaction of sodium hypochlorite with residual EDTA
B Interaction of CH with residual sodium hypochlorite
C Residual gutta percha within the canal
D Staining of RMGIC
E Use of MTA

6. A 39-​year-​old woman requires replacement anterior maxillary


crowns as part of a full arch rehabilitation. The crowns are to be
placed at an increased occlusal vertical dimension (OVD), and following
removal of the crowns, there is -​mm space between the residual core
and the desired incisal level. What is the minimum incisal reduction re-
quired for the provision of metallo-​ceramic crowns in this situation? ★★
A 0.5 mm
B  mm
C .5 mm
D 2 mm
E 2.5 mm

https://t.me/DentalBooksWorld
questions 237

7. A 3-​year-​
old woman attends with a mesio-​ incisal fracture
of an old composite restoration. The tooth was traumatized
playing sport when the patient was 6 years old, and over 80% of the old
composite restoration is remaining. Composite resin repair is planned,
and the old restoration is air-​abraded. After air abrasion, which single
agent should be used prior to placement of the bonding agent? ★★★
A Barium oxide
B Camphorquinone
C Hydroquinone
D Silanizing agent [e.g. gamma-​ methacryloxypropyl-​
trimethoxysilane
(gamma-​MPTS)]
E Tri-​ethylene glycol dimethacrylate (TEGMA)

8. The local dental laboratory has recently started providing


computer-​aided design/​computer-​aided manufacture (CAD/​
CAM) cobalt-​ chrome dentures, produced by selective laser melting
(SLM). They recommend clinicians change to them, as they are stronger.
What is the primary scientific reason for this? ★★★
A Greater σ-​phase levels of the alloy are produced
B Local heat treating from the laser work hardens the alloy
C Porosities are reduced
D Reduced heating time from the production process improves flexural
fatigue
E The quantity of molybdenum can be increased

9. A 52-​year-​old man recently required a disto-​occlusal amalgam


restoration adjacent to a gold shell crown. At a subsequent
appointment, a silver/​brown circular stain is noticed around the con-
tact point on the gold shell crown. Which single process has most likely
caused the discoloration? ★★★
A Electrochemical etching
B Galvanic corrosion
C Intergranular ion exchange
D Mechanical damage during cavity preparation
E Selective leaching

https://t.me/DentalBooksWorld
238 Chapter 2 Dental materials

20. A 7-​year-​old woman with taurodontism requires a direct pulp


cap, following an occlusal restoration on their lower right first
permanent molar. The supervising consultant recommends using a new
calcium silicate cement as pulp-​capping material, rather than the conven-
tionally used calcium hydroxide (CH). For which single biological reason
is this most appropriate? ★★★★
A Antibacterial
B Forms an ionic bond to the surrounding dentine
C High pH releases proteins and growth factors from the nearby den-
tine, leading to reparative dentine formation.
D Non-​cytotoxic and induces odontoblast proliferation and activity
E Obtundent

2. A 36-​year-​old man requires multiple composite restorations


to restore his worn anterior maxillary teeth. The restorations
are to be placed at an increased vertical dimension, using the Dahl ap-
proach. Which single type of composite is most appropriate for these
restorations? ★★★★
A Compomer
B Flowable composite
C Microfilled composite
D Nanohybrid composite
E Silorane composite

22. A 56-​year-​old man presents with mild asymptomatic erosion


of the palatal aspects of his upper incisors, secondary to acid
reflux. What single widely available active ingredient could be recom-
mended in his toothpaste as part of a protective measure? ★★★★
A Arginine 8%
B Casein phosphopeptides–​ amorphous calcium phosphates (CPP-​
ACP) 0%
C Sodium fluoride 0.3%
D Sodium lauryl sulfate 0.5%
E Stannous fluoride 0.45%

https://t.me/DentalBooksWorld
questions 239

23. A 66-​year-​old man is having his edentulous maxilla restored.


The residual ridge is a Class III ridge with minor anterior
undercut. He attends for an abutment-​level, open-​tray impression for a
full arch implant-​supported bridge. Which single impression material is
the most appropriate to use? ★★★★
A Addition-​cured silicone [polyvinylsiloxane (PVS)]
B Alginate
C Condensation-​cured PVS
D Impression plaster
E Polyether

24. A 32-​year-​old woman with a high smile line requires suturing of


the anterior soft tissues after periodontal surgery. Guided bone
regeneration has been attempted around the upper left lateral incisor
(UL2), using modern microsurgical techniques. Which single type and
size of suture would be the most appropriate in this scenario? ★★★★
A Black silk—​3.0
B Poliglecaprone—​2.0
C Polyester—​3.0
D Polyglycolic acid—​4.0
E Polypropylene—​6.0

25. A 9-​year-​old woman is having resin infiltration to manage her


aesthetic concerns regarding a developmental hypoplastic white
spot lesion on her central incisor. By what single mechanism would this
technique improve the appearance? ★★★★
A Absorbs blue wavelength light, leading to more light transmitted of a
yellow hue
B Increases the refractive index of the lesion by filling the internal
porosities
C Increases the transparency of the lesion by occluding the internal
porosities
D Lightens the surface colour of adjacent enamel by blending lesion
margins
E Provides a matt surface, reducing the reflection of light and masking
the appearance

https://t.me/DentalBooksWorld
240 Chapter 2 Dental materials

ANSWERS
. D ★ OHCD 6th ed. → p. 640
An attractive feature of GIC is that it can be placed in bulk and is less
technique-​sensitive than composite resin. Moisture control should still
be maintained, as contamination with blood or saliva can reduce the
bond strength.
The use of a conditioning agent on teeth prior to placement of GIC
has been demonstrated to improve the bond strength, but the evidence
is not conclusive. The main purpose of the conditioner is to remove
debris from the surface and facilitate a clean surface for bonding. Strong
agents that demineralize the tooth structure surface (such as citric acid
or phosphoric acid) should be avoided, as they will reduce the quality
of the ionic bond to apatite. Polyacrylic acid 0% is the most commonly
recommended conditioner for GICs.
Keywords: GIC, conditioning agent.
→ McCabe J, Walls A. Applied Dental Materials (9th ed.). Blackwell
Publishing Ltd, Oxford; 2008.

2. E ★ OHCD 6th ed. → p. 644


ZOE cements have a number of properties which facilitate their use
as dental material. Primarily, eugenol is thought to leach out from the
material and into the surrounding dentine where it is said to have an
obtundent or sedative effect upon the dentino-​pulpal complex. Its mode
of action is primarily attributed to its inhibition of capsaicin receptors.
The material is generally supplied as a powder and liquid which has a
relatively quick setting time once mixed. The material has historically
been used as a lining or base material in deep cavities, as well as a root
canal sealer. The material can have antimicrobial properties and has
been shown to have good sealability when placed in adequate thickness.
However, these would be secondary benefits in this scenario.
Evidence exists to show that dentine bond strengths are negatively af-
fected by ZOE materials. As such, their use should be avoided, or
further surface preparation will be required where future adhesive
restorations are to be placed. Non-​eugenol-​containing zinc oxide root
canal sealers are available to overcome this issue when restoring the
root-​treated tooth.
New tricalcium silicate-​based restorative materials (e.g. Biodentine™)
have a growing wealth of evidence to support their use over ZOE in
these clinical situations. However, these materials are more expensive
and are yet to be routinely adopted into this clinical scenario.
Keywords: deep amalgam restoration, shooting pain, ZOE.
→ Koch T, Peutzfeldt A, Malinovskii V, Flury S, Häner R, Lussi A. Temporary
zinc oxide-​eugenol cement: eugenol quantity in dentin and bond strength
of resin composite. European Journal of Oral Sciences. 203;2:363–​9.

https://t.me/DentalBooksWorld
answers 241

3. C ★
Addition-​ cured silicones are based on polymers of vinylsiloxanes.
One half of the material contains polydimethylsiloxane with some
methyl groups replaced with hydrogen, and the other half contains
polydimethylsiloxane with some methyl groups replaced with a vinyl
group. When mixed in the presence of platinum, a cross-​linking reac-
tion occurs between the single hydrogen groups and the vinyl groups.
Clinically, due to the inherently hydrophobic nature of these polymers,
the presence of moisture can result in air blows.
The addition of surfactants (a molecule which reduces the material’s sur-
face tension) is one method some manufacturers have used to combat
this. Surfactants are bidirectional molecules which have both a hydro-
phobic and a hydrophilic end. Their addition to PVS materials lowers the
surface tension and causes a reduction in the contact angle the material
makes with water. This improves the wettability of the material and the-
oretically decreases the risk of air blows. However, the authors would
still recommend that the impression surface is dried thoroughly (particu-
larly in crown and bridge work) to improve the quality of the impression.
Drags may occur when the material is removed before it is set or where
undercut is present and high-​viscosity material is used.
Folds may occur where the materials do not mix properly or where one
material has started to set before the other is placed (when using a two-​
viscosity technique).
Keywords: PVS, air-​dried.
→ Van Noort R. Introduction to Dental Materials (4th ed.). Elsevier,
Edinburgh; 203.

4. B ★
Sodium hypochlorite remains the gold standard for chemical disinfec-
tion of the root canal system. It is a highly effective antimicrobial agent
that dissolves organic matter and helps lubricate the root canal system.
The major disadvantage of sodium hypochlorite is that it is unable to
remove the smear layer created during mechanical cleaning of the canals.
The smear layer (composed primarily of inorganic material and approxi-
mately 5 μm in thickness) blocks dentinal tubules and remains a potential
reservoir for pathogens. Removal of the smear layer has been shown to
improve endodontic outcomes, especially in ReRCT. Alternative irrigants
are therefore available to help remove the inorganic matter that remains
(e.g. EDTA, citric acid). These irrigants chelate the inorganic compo-
nents that make up the smear layer and help remove the associated
bacteria. Furthermore, the underlying dentinal tubules are exposed, and
it is thought that this increases the penetration of the sealer and subse-
quently improves the adaptation of the root canal filling.
HEBP (-​ hydroxyethylidene-​ ,-​
bisphosphonate), or etidronate, is
gaining popularity within the endodontic community as a possible alter-
native chelating agent. However, EDTA remains the most widely used
and evidence-​based chelating agent at present.

https://t.me/DentalBooksWorld
242 Chapter 2 Dental materials

Keywords: inorganic matter, irrigant, root canal.


→ Hülsmann M, Heckendorff M, Lennon A. Chelating agents in root canal
treatment: mode of action and indications for their use. International
Endodontic Journal. 2003;36:80–​30.
→ Shavravan A, Haghdoost AA, Adl A, Rahimi H, Shadifar F. Effect of
smear layer on sealing ability of canal obturation: a systematic review and
meta-​analysis. Journal of Endodontics. 2007;33:96–​05.

5. A ★
Fibre posts should be bonded into the root canal system. This aids not
only retention, but also helps to distribute the forces along the root canal
system. In order to do this, resin-​based cement is required. Resin-​based
cements can be light-​cured, chemically cured, or dual-​cured. The reliable
curing depth of conventional composite is around 2 mm, which would
not normally be sufficient to cure to the full depth of the post prepar-
ation. However, translucent fibre posts can transmit the light along the
length of the post and initiate the curing reaction. The problem with
this is that the curing reaction will be towards the post. The enormous
c-​factor could easily lead to the dentine bond being broken. Although
there is evidence to suggest that light-​cured and dual systems have similar
bond strengths along the root regions for this type of post, the authors
would argue that dual-​cured cement (the chemically cured component)
is more likely to reliably achieve full curing throughout the lute and there-
fore is preferable. Another key element to a successful bond is the use of
a dual-​cured/​chemically cured bonding agent.
It is important to realize that achieving a reliable bond to the root dentine
can be challenging, and debonding of fibre posts is a common reason
for failure.
Keywords: glass-​fibre post, cement.
→ Barfeie A, Thomas MB, Watts A, Rees J. Failure mechanisms of
fibre posts: a literature review. European Journal of Prosthodontics and
Restorative Dentistry. 205;3:5–​27.
→ Giachetti L, Scaminaci Russo D, Baldini M, Bertini F, Steier L, Ferrari
M. Push-​out strength of translucent fibre posts cemented using a dual-​
curing technique or a light-​curing self-​adhering material. International
Endodontic Journal. 202;45:249–​56.

6. C ★ OHCD 6th ed. → p. 650


There are various factors influencing patients who have hypersensi-
tive gag reflexes. The gag reflex itself is ‘an involuntary contraction of
the muscles of the soft palate or pharynx that causes retching’. It is an
important protective reflex present in human physiology. It can be de-
scribed as either somatic (stimulated by touching areas of the mucosa)
or psychogenic. The severity of gagging can be graded  to 5. Above-​
moderate gagging (grade 3) routine dental treatment can be challen-
ging. For denture wearers with hypersensitive gag reflexes, coverage of

https://t.me/DentalBooksWorld
answers 243

the palate is a frequent complaint that they associate with the problem.
However, poor retention and an overly thick palate can contact the
dorsum of the tongue and stimulate the gag reflex. Provision of a CoCr
denture with good retention and a thin major connector may help from
both a physiological and a psychological perspective.
Yield strength is the point at which permanent plastic deformation (pro-
portional limit) occurs within the material. CoCr has a yield strength in
the region of 600–​700 MPa, giving it good flexural rigidity, and allows
denture bases of thin section. Strength in thin section means less palatal
coverage and smaller connectors are required.
Keywords: CoCr, property, thinner, major connector.
→ Al-​Jabbari Y. Physico-​mechanical properties and prosthodontic ap-
plications of Co-​Cr dental alloys: a review of the literature. Journal of
Advanced Prosthodontics. 204;6:38–​45.
→ Forbes-​Hayley C, Blewitt I, Puryer J. Dental management of the
‘gagging’ patient—​an update. International Journal of Dental and Health
Sciences. 205;3:423–​3.

7. E ★ OHCD 6th ed. → p. 638


The increased organic composition of dentine and the presence of den-
tinal fluid, compared with enamel, mean that a hybrid layer has to be
formed in order to allow the hydrophobic resin to bond to the den-
tinal surface. To allow this to happen, modern bonding systems have
three stages: etching, priming, and bonding. Etching demineralizes the
enamel or dentine surface to create microporosities or prepare the sur-
face for bonding. It also removes the smear layer. The primer has small
hydrophilic monomers, along with solvent molecules, that improve the
wettability of tooth structure and allow infiltration of the resin molecules
into the exposed collagen fibrils to form a hybrid layer when bonding to
the dentine. The bond component is either lightly filled or unfilled resin
that combines with the hybrid layer to form a hydrophobic surface for
the composite to bond to.
Manufacturers have tried to simplify the bonding process by combining
various stages. Self-​etching systems have had their priming monomers
modified to incorporate carboxylic or phosphate acid groups. When
bonding to enamel, in comparison to phosphoric acid, self-​etching pri-
mers do not produce such high bond strengths, as the acid primers
cannot etch as deeply or as uniformly. The results to the dentine are,
however, similar to conventional etching methods and furthermore may
reduce the technique sensitivity of bonding to the dentine, as there is
no risk of overdrying the dentine and collapsing of the exposed collagen
fibres. For these reasons, when using a self-​etching system for composite
restoration, selective etching of enamel is recommended.
Keywords: self-​etch adhesive bonding system, modified, etching.
→ Ozer F, Blatz M. Self-​etch and etch-​and-​rinse adhesive systems in
clinical dentistry. Compendium of Continuing Education in Dentistry.
203;34:2–​4.

https://t.me/DentalBooksWorld
244 Chapter 2 Dental materials

8. B ★ OHCD 6th ed. → p. 630


Amalgam is commonly classified by the shape of their silver-​tin par-
ticles, which can be spherical, lathe-​cut, or admixed. Lathe-​cut amalgam
hardens more slowly and gives a longer working time. The remaining an-
swers refer to properties of spherical amalgam, except answer D which
is another property of lathe-​cut amalgams.
Spherical amalgam can achieve a smoother finish because the alloy par-
ticles do not impinge upon each other as they do in lathe-​cut amalgams.
It is easier to condense because there is less friction between the spher-
ical particles than there is in lathe-​cut amalgams. Also, spherical alloys
have less mercury, which results in greater shrinkage, so contacts are
likely to be less tight.
Older amalgams underwent net expansion as the setting reaction pro-
gressed. Modern-​day amalgams show net contraction, the reasons for
this being:
● Older amalgam had larger alloy particles and a higher mercury content
● Before amalgamators were available, amalgam was mixed by hand trit-

uration, with more vigorous trituration, resulting in more contraction.


Admixed amalgam demonstrates properties between spherical and
lathe-​cut amalgams.
Keywords: struggling to pack, before it sets, lathe-​cut.

9. C ★
Gold onlays are commonly used in the management of toothwear to
restore posterior teeth because of the limited tooth reduction required.
However, their preparation design frequently lacks retention and resist-
ance form. They therefore rely on adhesive cements to retain them in
situ. From the list given, chemically active resin cements are the most
suitable option. All the other options exhibit insufficient strength to
resist the functional forces placed upon a restoration lacking resistance
or retention form. Resin cements are stronger, with high enough bond
strengths to retain onlay-​type restorations. It is important to remember
that resin cannot be light-​cured through gold restorations and, as such,
dual-​or chemically cured resin cement should be used. One of the chal-
lenges with bonding to gold is the inherently inert nature of the material
and the lack of an oxide layer produced. Chemically active cements, such
as Panavia®, can bond to noble metals, but methods such as heat treating
or tin plating have been advocated to improve the bond strength.
However, despite these suggestions, evidence exists to show that air-​
abrading (sand blasting) the fitting surface alone provides adequate
micromechanical retention to achieve a successful clinical outcome.
Keywords: gold onlays, minimal retention or resistance form,
cementation.
→ Chana H, Kelleher M, Briggs P, Hooper R. Clinical evaluation
of resin-​bonded gold alloy veneers. Journal of Prosthetic Dentistry.
2000;83:294–​300.

https://t.me/DentalBooksWorld
answers 245

0. D ★★
The bonding mechanism of the composite is micromechanical. Following
demineralization of the surface of the dentine with an etchant, a primer
and a bonding agent are used to create a hybrid layer to allow the hydro-
phobic composite to bond to the ‘wet’ or hydrophilic dentine surface.
This hybrid layer creates resin tags within the exposed dentinal tubules,
giving micromechanical retention. The reference listed below gives an
excellent summary of the evolution of dentine-​bonding agents. The con-
cept of ‘wet bonding’ was brought about when it was discovered that
overdrying of the surface led to a reduction in bond strength. At a micro-
scopic level, when the dentinal surface is etched, the demineralization
process leaves a matrix of collagen fibrils exposed, which is supported
by residual water. If overdried, this layer collapses (collagen collapse)
and the resulting hybrid layer is thin and weak, potentially decreasing the
bond strength. Conversely, if the residual water is not expelled, then
water globules will form within the resin layer, again resulting in a weaker
bond and the risk of future degradation. With modern combined pri-
mers and bonding agents, two coats should be applied to thoroughly
expel the residual water and optimize the bond. Prevention of collagen
collapse is key to the ‘wet bonding’ principle.
Keywords: composite, wet bonding.
→ Pashley D. The evolution of dentin bonding. Dentistry Today. 2003.
Available at: http://​www.dentistrytoday.com/​materials

. E ★★ OHCD 6th ed. → p. 630


Dental amalgam is an alloy composed primarily of silver, tin, copper,
and mercury. Its setting reaction has been refined over many years. The
reaction creates a fine balance between various compounds (referred to
as gamma phases), with each influencing the property of the amalgam.
A crumbly or grainy amalgam has been under-​triturated, and the material
is often unusable as insufficient wetting of the alloy has occurred. Over-​
trituration would result in an amalgam mix that is excessively sticky.
Amalgam contraction occurs with all amalgams during the first hour, and
this would not noticeably influence the workability of the material.
Moisture contamination remains a problem with zinc-​containing amal-
gams, as this can result in delayed expansion of the amalgam.
Burnishing improves the marginal integrity of amalgams and, as such, re-
duces the risk of leakage at the margins.
Keywords: amalgam, crumbly, unusable.

2. E ★★
The management of flabby ridges can be difficult. The basic principle
comes down to the argument of whether the denture-​bearing area
should be recorded under compression or relaxation (mucocompressive
or mucostatic). With fibrous ridges, it is generally considered that a
mucostatic impression of the hypermobile tissue is desirable, as it is

https://t.me/DentalBooksWorld
246 Chapter 2 Dental materials

more stable during rest (the soft tissue would recoil and dislodge the
denture if recorded under compression).
Therefore, a selective pressure technique is used to record the normal
tissues under slight compression and the flabby tissue at rest for optimal
support and retention. A variety of techniques have been suggested
in the literature, each one as an alternative method of achieving a
mucostatic impression. They all tend to employ a custom-​made tray,
with some form of venting or spacing over the flabby tissue. The choice
of material is important, as the viscosity is required to be low enough
that, when pressure is applied to seat the tray, the resistance of the
flabby tissue is greater than the inherent stiffness of the material causing
it to flow and not displace the tissue. Traditionally, plaster of Paris was
used, but the contemporary replacement is low-​viscosity silicone.
Keywords: flabby ridge, window technique.
→ Poonam SR, Agrawal H. A review of prosthodontic management of
flabby ridge conditions in maxilla and mandible by noninvasive techniques
and with the use of contemporary materials. Guident. 202;5:24–​32.

3. E ★★
The recognized way of classifying colour was devised by Albert Munsell.
He divided colour into three components to precisely define every
colour: hue, chroma, and value. Hue is the ‘actual’ colour and is either
red, yellow, green, blue, or purple, or somewhere within that circular
scale. This natural order of colour makes up Munsell’s colour wheel.
Teeth have hues of reds or yellows. This is reflected by the L and R
values on the Vita 3D shade guide™; ‘L’ represents a yellower hue, and
‘R’ more red (i.e. left and right shifts on the colour wheel). Chroma is
the saturation of a colour, i.e. the colour intensity. On the 3D shade
guide, this is denoted by the ‘M’ value. The value is how light or dark an
object is (i.e. white to black) and, on the shade guide, is represented by
the numbers  to 5. When using the shade guide, the value is selected
first, then the chroma (‘M’ value), and finally, if required, the hue is ad-
justed (‘L’ and ‘R’ values). The 3D shade guide provides a comprehen-
sive, systematic chart covering standard tooth colours. In comparison,
standard shade guides generally cover a selection of common colours.
Modern technological advancements include chairside spectrometers to
aid colour selection. Polarized or greyscale images (with shade tabs in
them) can help to select the correct hue and value, respectively.
Keywords: dark, element of colour, Vita 3D shade guide™.
→ Vita 3D shade guide user manual. Vita Zahnfabrik. Available at:
https://​www.vita-​zahnfabrik.com/​en/​VITA-​Toothguide-​3D-​MASTER-​
26230,27568.html

4. A ★★
Residual monomers in dentures can cause mucosal irritation and the-
oretical reductions in strength (as the polymerization conversion is
reduced). The international standard for residual monomer content is

https://t.me/DentalBooksWorld
answers 247

2.2%. Manufacturers’ curing cycles vary in method but typically have a


prolonged period of around –​2 hours at 70°C, followed by a final boil at
00°C for half hour to  hour. Despite several traditional studies investi-
gating curing cycle optimization, the reference below demonstrates that
there is no scientific basis to account for these two temperatures in the
curing cycle. Curing at 95°C overnight would achieve residual monomer
levels consistently below the required standard of 2.2%. Achieving
similar levels at 70°C would require curing times in excess of 300 hours.
The justification for using the protocol outlined above is for practicality,
energy efficiency, and concerns of rapid temperature increases boiling
the monomer.
Excess powder would lead to granular porosities from unreacted
polymer powder. Heating rapidly above acrylics boiling point would lead
to gaseous porosities. Packing at the ‘wet sand’ stage would likely lead to
an under-​filled mould, and under-​compression would result in an overly
thick denture base and an unwanted increase in vertical dimension.
Keywords: replacement, denture, burning/​ itching sensation, excess
monomer.
→ Lung C, Darvell B. Minimization of the inevitable residual monomer in
denture base acrylic. Dental Materials. 2005;2:9–​28.

5. E ★★ OHCD 6th ed. → p. 334


MTA has become commonly used for the formation of an apical plug
in endodontics, due to its biocompatibility. One of the adverse effects
is the resultant discoloration of the tooth. Although the exact mech-
anism is not yet fully understood, it is suspected to be related to the
presence of the radio-​opacifier bismuth oxide. A number of potential
interactions have been hypothesized. It has been suggested that bismuth
oxide can interact with collagen in the dentine to form a dark precipitate.
Alternatively, some research suggests that residual sodium hypochlorite
can react with bismuth oxide to form a dark precipitate. In efforts to
reduce the discoloration of MTA, manufacturers have tried to reduce
the content of heavy metals. Iron content has been reduced in white
MTA, yet discoloration remains a documented complaint.
Calcium silicate cements with alternative radio-​opacifiers, such as zirco-
nium oxide, may reduce discoloration risks in areas of aesthetic concern.
One further suggestion has been made that sealing the dentine prior to
placement of MTA may reduce discoloration.
Endodontic sealers may cause minor discoloration, but this is not as con-
sistent as with MTA. RMGIC is not known to stain teeth.
Keywords: marked discoloration, MTA.
→ Yun D, Park SJ, Lee SR, Min KS. Tooth discoloration induced by
calcium-​silicate-​based pulp-​capping materials. European Journal of
Dentistry. 205;9:65–​70.

https://t.me/DentalBooksWorld
248 Chapter 2 Dental materials

6. A ★★ OHCD 6th ed. → p. 252


For a long time, metallo-​ceramic crowns were the mainstay of aesthetic
indirect restorations due to their greater resilience than traditional por-
celain jacket crowns. The advent of dentine bonding and modern high-​
strength ceramic restorations has led to alternative options becoming
available that are generally considered to be more aesthetic due to their
optical properties. However, there is still a place for metal-​ceramic res-
torations as they have a proven track record, are durable in high-​stress
situations, and provide a good compromise between aesthetics and
tooth reduction. High-​strength ceramics generally require greater cir-
cumferential tooth reduction.
The mechanical and aesthetic properties of the materials in metal-​
ceramic crowns dictate the space. A relatively thin, uniform layer of por-
celain supported by metal is the strongest but often does not provide
acceptable aesthetics. The absolute minimum requirements for porcelain
is 0.7 mm, and metal 0.3–​0.5 mm. However, to avoid an opaque-​looking
restoration, .5–​2 mm of incisal reduction is advocated. Therefore, in
this scenario, as  mm of incisal clearance already exists, a minimum of
0.5-​mm further reduction is needed.
Keywords:  mm of space, desired incisal level, metallo-​ceramic crowns.
→ Ricketts D, Barlett D. Advanced Operative Dentistry: A Practical Approach
(st ed.). Churchill Livingstone, London; 203.

7. D ★★★ OHCD 6th ed. → p. 632


Composite is primarily composed of a resin matrix and an inorganic filler.
The filler improves the material characteristics by reducing shrinkage,
increasing compressive strength, and improving wear rate. Fillers are
commonly composed of glass particles (e.g. aluminoborosilicate), and
the size of the particles is used to classify the type of composite (e.g.
microfilled, nanofilled, nano-​hybrid, etc.). The composition of the filler
particles significantly influences the physical properties of the resultant
composite.
Unfortunately, resins and fillers do not bond chemically, and as such, an
intermediary silane coupling agent is required. Silanes have both an inor-
ganic and an organic end to their molecule, allowing the chemical linkage
of the resin and filler. Contact of the composite with air inhibits the
polymerization process, leaving an unreacted surface that can be used
for future bonding. However, in old composites, this is inevitably worn
(or polished) away, leaving a surface layer predominantly composed of
exposed inorganic filler. The original silane coating again will have been
worn away from exposure to the oral environment, and therefore, to
optimize the bond when repairing composites, a fresh silane layer should
be applied. Some newer universal bonding agents may already contain
silane.
Barium oxide is added for its radio-​ opacifying properties.
Camphorquinone is the source of free radicals for the polymerization
reaction of light-​cured composites. Hydroquinone prevents premature

https://t.me/DentalBooksWorld
answers 249

polymerization. TEGMA is a low-​ weight resin monomer added to


improve handling characteristics and reduce viscosity.
Keywords: composite, repair, prior to bonding.
→ Staxrud F, Dahl J. Silanising agents promote resin-​composite repair.
International Dental Journal. 205;65:3–​5.

8. C ★★★
Traditionally, dental alloys have been produced by casting methods util-
izing the lost wax technique; this method is labour-​intensive and time-​
consuming. CAD/​CAM has become readily available in dentistry for
prosthesis production. CAD/​CAM methods can either be subtractive
(i.e. milling of pre-​manufactured blocks) or additive (SLM, stereolithog-
raphy, etc.). SLM works by high-​powered lasers selectively fusing areas
of a metal powder, layer upon layer, to the designed 3D structure. This
process has the advantage over milling of being able to produce hollow
3D structures. In comparison to traditional casting methods, porosities
are greatly reduced, which significantly improves the alloy’s physical
properties. The accuracy is also comparable, if not better, than casting.
The σ-​phase of Co-​Cr makes the material more brittle. This is related
to the composition of the alloy, and not the production process. The
addition of molybdenum increases the γ-​phase (which demonstrates im-
proved strength), but this is not related to the manufacturing process.
The addition of tungsten helps to reduce the σ-​phase.
Keywords: CAD/​CAM, SLM, stronger.
→ Koutsoukis T, Zinelis S, Eliades G, Al-​Wazzan K, Rifaiy MA, Al Jabbari
YS. Selective laser melting technique of Co-​Cr dental alloys: a review of
structure and properties and comparative analysis with other available
techniques. Journal of Prosthodontics. 205;24:303–​2.

9. B ★★★
Various types of corrosion can affect dental materials, and the sur-
rounding environment plays a role in this. When dissimilar metals are
placed within an electrolytic solution, there is potential to create an elec-
trochemical cell (e.g. a chemical battery), otherwise known as a galvanic
cell. It has been reported that when amalgam is placed adjacent to gold
in saliva, a galvanic cell can be induced, leading to galvanic corrosion. This
leads to surface staining with a silver brown colour in a ring around the
contact point. However, it is suspected that it is the presence of free
mercury that results in galvanic corrosion. This reaction is short-​lived,
with little mercury remaining after about  hour and almost no residual
current after  day.
The effects of this corrosion are clinically insignificant with regard to the
alloy properties, but it has been reported that this galvanic current can
occasionally cause short-​lived mild pain. Replacement of the restoration
is not normally required.
Keywords: amalgam, gold, silver/​brown circular stain, contact point.

https://t.me/DentalBooksWorld
250 Chapter 2 Dental materials

→ Fusayama T, Katayori T, Nomoto S. Corrosion of gold and


amalgam placed in contact with each other. Journal of Dental Research.
963;42:83–​97.
→ Upadhyay D, Panchal AD, Dubey RS, Srivastava VK. Corrosion of
alloys used in dentistry: a review. Materials Science and Engineering.
2006;432:–​.

20. D ★★★★ OHCD 6th ed. → p. 334


In vital pulp therapy, CH was traditionally the gold standard until newer
materials, such as MTA [a type of bioactive endodontic cement (BEC)],
became available. When applied directly to pulp tissues, CH forms a
superficial layer of necrosis, which contains calcium deposits. These act as
foci for calcification, and subjacent pulp cells differentiate into odontoblasts
to produce reparative dentine and a hard tissue barrier. Furthermore, the
high pH is thought to solubilize proteins and growth factors trapped in
the adjacent dentine, which stimulates pulp cell differentiation. BECs have
been shown to release a greater number of growth factors than CH.
BECs differ in that they have been demonstrated to induce less inflam-
mation at the interface and cause greater proliferation of odontoblasts.
Some research suggests that the formation of CH within BECs is respon-
sible for this finding (again solubilizing cytokines from the surrounding
dentine). Additionally, BECs biocompatibility have been linked with the
material’s ability to form hydroxyapatite. Biologically, BECs are more
biocompatible, and results suggest that histologically dentine formation
is more regular, less porous, and in increased amounts.
Additionally, the ability to bond to dentine is important in the success
of pulp capping. Mechanically, BECs have been shown to have an ionic
bond to the dentine, which may limit bacterial penetration. Both mater-
ials display antibacterial properties, and neither material has been shown
to be obtundent.
Keywords: direct pulp cap, new calcium silicate, calcium hydroxide, bio-
logical reason.
→ Modena KC, Casas-​Apayco LC, Atta MT, et al. Cytotoxicity and bio-
compatibility of direct and indirect pulp capping materials. Journal of
Applied Oral Sciences. 2009;7:544–​54.

2. D ★★★★ OHCD 6th ed. → p. 632


Composites placed at an increased occlusal vertical dimension (OVD)
for toothwear are becoming common practice. The chosen material
needs to have good mechanical properties to resist the increased
occlusal loads placed on these ‘proud’ restorations, along with good aes-
thetic properties. Traditional microhybrid composites combine large and
small filler particulars to increase the filler content. Higher filler contents
generally improve the mechanical properties and reduce polymeriza-
tion shrinkage. Old microfill composites utilized smaller filler particles,
which reduced the size of surface irregularities and improved surface

https://t.me/DentalBooksWorld
answers 251

smoothness and appearance. However, this negatively impacted on


mechanical properties, as the glass content was reduced.
The introduction of nanoparticles to modern composites has been
claimed to maintain the mechanical properties of the microhybrid,
but also to harness the aesthetic properties of microfill compos-
ites. Therefore, in a case like in the scenario, a nanohybrid composite
would be the most appropriate choice of materials. A conventional
microhybrid material would be a suitable alternative, as it has a proven
track record.
Keywords: composite, worn, teeth, increased OVD.
→ Moraes RR, Gonçalves L de S, Lancellotti AC, Consani S, Correr-​
Sobrinho L, Sinhoreti MA. Nanohybrid resin composite: nanofiller
loaded materials or traditional microhybrid resins. Operative Dentistry.
2009;34:55–​7.

22. E ★★★★
Various active ingredients have been investigated to assess their poten-
tial to protect against erosion, with limited conclusive results. Some com-
pounds, such as CPP-​CAP, calcium nanophosphate, titanium fluoride,
and stannous fluoride, have shown potential in reducing the impact of
erosion. Over time, these compounds build a protective glaze/​coat-
ings on the tooth surface. Alternatively, some clinicians would prescribe
higher-​strength sodium fluoride (.% in patients over 6 years of age),
but this would help to remineralize the lost ions, rather than prevent dis-
solution. Although the actual quality of the evidence is weak and there
are numerous methodological challenges which reduce the external val-
idity of results, stannous fluoride toothpastes are readily available on
the high street and may provide greater benefit to patients over sodium
fluoride alone. Furthermore, as the product cost is low (compared to
CPP-​ACP or a prescription) and this type of fluoride formulation is not
detrimental, then the authors would suggest its use to supplement a pre-
ventative programme until further evidence is available. Clinicians should
also remember that standard dietary advice and any necessary medical
management are necessary and more important than the active ingre-
dient of toothpaste.
Arginine is generally used as a desensitizing agent, and sodium lauryl sul-
fate is a foaming agent in toothpastes.
Keywords: erosion, widely available, toothpaste, protective.
→ Carvalho FG, Brasil VL, Silva Filho TJ, Carlo HL, Santos RL, Lima BA.
Protective effect of calcium nanophosphate and CPP-​ACP agents on
enamel erosion. Brazilian Oral Research. 203;27:463–​70.
→ Hove LH, Holme B, Young A, Tveit AB. The protective effect of
TiF4, SnF2 and NaF against erosion-​like lesions in situ. Caries Research.
2008;42:68–​72.
→ Wang X, Megert B, Hellwig E, Neuhaus KW, Lussi A. Preventing ero-
sion with novel agents. Journal of Dentistry. 20;39:63–​70.

https://t.me/DentalBooksWorld
252 Chapter 2 Dental materials

23. E ★★★★ OHCD 6th ed. → p. 646


Fixed and removable prostheses rely upon the accuracy of working
impressions. In implant dentistry, this accuracy has greater relevance,
particularly where suprastructures are used to join implants. Any dis-
crepancies prevent passivity of fit and may cause complications (e.g. frac-
ture of veneering material or marginal bone loss).
Contemporary elastomeric materials have high accuracy and good
dimensional stability. Polyethers are generally favoured over addition PVS
in the scenario, as they have greater rigidity and therefore are thought
to better retain the impression copings in the correct position during
casting. It is important though that polyethers are poured within  hour,
as moisture absorption can cause distortion. PVS materials are suit-
able alternatives, as studies have shown similar accuracy to polyethers.
However, results are less reproducible.
Some clinicians may still use impression plaster for the above situ-
ation, but it is only applicable to edentulous cases with no undercut.
Furthermore, there is significant cumulative distortion, following the
pouring of the master model.
Hydrocolloids are not accurate or rigid enough for this application.
The era of digital dentistry is beginning to revolutionize prosthodontics,
and with future advances, intraoral scanning may supersede conventional
impression techniques.
Keywords: minor undercut, open-​tray impression, implants.
→ Donavon T, Chee W. Impression materials: a comparative review
of impression materials most commonly used in restorative dentistry.
Dental Clinics of North America. 2004;48:445–​70.
→ Hoods-​Moonsammy V, Owen P, Howes D. A comparison of the
accuracy of polyether, polyvinyl siloxane, and plaster impressions
for long-​ span implant-​ supported prostheses. International Journal of
Prosthodontics. 204;27:433–​8.

24. E ★★★★ OHCD 6th ed. → p. 362


A vast array of suture materials are available in modern medicine, and
often numerous materials will be appropriate for a given situation. In rela-
tion to modern periodontal microsurgery (particularly in the aesthetic
zone), a number of basic principles apply to material choice. In general,
monofilament sutures are preferential to multifilament fibres, as they
do not ‘wick’ bacteria into the wound, which aggravates the inflamma-
tory response. Secondly, in delicate tissues (e.g. the dental papilla), finer
and less traumatic sutures are recommended, as this promotes quicker
wound healing and causes less chance of disruption to the blood supply.
Synthetic monofilament fibres are usually considered the material of
choice, as they are least traumatic and only induce mild tissue reactions.
Despite this, some clinicians favour the use of polyglycolic acid, as good
knot tension can be achieved.

https://t.me/DentalBooksWorld
answers 253

Black silk is a braided multifilament material which produces one of the


higher tissue reactions, and although it has good handling properties, it
is infrequently used. Polyester is again a multifilament fibre and not fre-
quently used in dentistry. Vicryl® (polyglycolic acid) is a multifilament,
synthetic, resorbable material. In smaller sizes, it may be appropriate for
the task at hand; however, #4.0 is generally considered too large. It is
frequently used in dento-​alveolar surgery.
Poliglecaprone (e.g. Monocryl®) would be a more suitable resorbable
material, as it is a monofilament fibre and has high initial strength during
the first few days of wound healing. It is, however, more expensive.
The most appropriate choice would therefore be 6.0 polypropylene
(Prolene®), a small monofilament, non-​resorbable suture which induces
minor tissue reactions. Being non-​resorbable, there is less risk of dis-
lodgement and wound dehiscence during the critical early healing stages,
compared with resorbable sutures.
Keywords: periodontal surgery, microsurgical, suture.
→ Kurtsmann G. Suturing for success. Dentistry Today. Available at:
http://​ w ww.dentistrytoday.com/​ p eriodontics/​ 3 59-​ s uturing-​ f or-
​surgical-​success
→ Velvart P, Peters C. Soft tissue management in endodontic surgery.
Journal of Endodontics. 2005;3:4–​6.

25. B ★★★★
The management of developmental white spot lesions is frequently an
aesthetic concern and can be challenging to manage. The four traditional
management strategies are remineralization, masking with bleaching,
microabrasion, or restoration. Most clinicians would wish to avoid
formal restoration, if possible, particularly in younger patients. However,
the most conservative measures are inconsistent in their results, and fur-
thermore, microabrasion can remove up to 360 μm of enamel and leave
a rough surface.
Resin infiltration is a contemporary technique that uses low-​viscosity
resin to fill subsurface porosities. It was initially developed for
preventing progression of early carious lesions. In white spot lesions,
the resin fills the subsurface porosities, increasing the refractive index
to a value similar to that of the surrounding enamel (.52). Prior to
infiltration, the porosities are filled with air or water, which has a much
lower index (air has a refractive index of ) and makes the lesion more
obvious to the eye. Although results are not 00% successful, they are
promising and can provide acceptable results without the need for fur-
ther treatment.
Keywords: resin infiltration, aesthetic concerns.
→ Kim S, Kim EY, Jeong TS, Kim JW. The evaluation of resin infiltra-
tion for masking labial enamel white spot lesions. International Journal of
Paediatric Dentistry. 20;2:24–​8.

https://t.me/DentalBooksWorld
https://t.me/DentalBooksWorld
Chapter 3 255

Head and neck syndromes


Nicholas Longridge

‘How many teeth should I have?’


A comparatively small chapter, syndromes of the head and neck are gen-
erally rare. It is unlikely that the majority of clinicians will diagnose or
treat patients affected by many of the syndromes discussed. However,
it is highly likely that clinical findings or presenting features may find their
way into undergraduate academic examinations.
Most syndromes are presented in the literature as a specific list of
associated clinical findings, and many are still referred to eponymously.
Some of the syndromes with oral manifestations can also be associated
with more sinister conditions and, as such, a broad knowledge of the
common syndromes with orofacial manifestations is important. It is
highly advised to read the chapter on syndromes of the head and neck in
the Oxford Handbook of Clinical Dentistry.
Key topics include:
● Syndromes associated with hypodontia
● Syndromes affecting orofacial development
● Syndromes linked with gastrointestinal conditions
● Syndromes subsequent to infections.

https://t.me/DentalBooksWorld
256 Chapter 3 Head and neck syndromes

QUESTIONS

. A 9-​year-​old boy has multiple retained deciduous teeth, a prom-


inent brow, and a wide nasal bridge. Multiple supernumeraries are
evident on the orthopantomogram (OPT), and the mother informs you
that her son has recently seen a specialist regarding the lack of collar-
bones. Which single condition is this child most likely to have? ★
A Apert’s syndrome
B Cleidocranial dysostosis
C Melkerson–​Rosenthal syndrome
D Pfeiffer syndrome
E Pierre–​Robin sequence

2. A 7-​year-​old girl attends with her mother for a routine examination


(see Figure 3.). The patient’s mother has read that her child may
have congenitally missing teeth and that they have an increased preva-
lence for periodontitis. You also note the girl has low-​set ears, upward
sloping palpebral fissures, brachycephaly, and a tendency for tongue pro-
trusion. What is the single most likely diagnosis? ★
A 45,XO
B Trisomy 3
C Trisomy 8
D Trisomy 2
E XXY

https://t.me/DentalBooksWorld
questions 257

Figure 3.
Reproduced from Welbury R, et al, Paediatric Dentistry fourth edition, Figure 7.2,
page 286, Copyright (202) by permission of Oxford University Press.

3. A 27-​year-​old woman has marked mandibular prognathism and


multiple basal cell carcinomas. She has previously been diagnosed
with Gorlin–​Goltz syndrome (naevoid basal cell carcinoma syndrome).
An orthopantomogram (OPT) is taken to investigate cystic change in the
mandible. Which single type of lesion are you most likely to identify? ★★
A Ameloblastomas
B Dentigerous cysts
C Odontogenic keratocysts
D Radicular cysts
E Solitary bone cyst

https://t.me/DentalBooksWorld
258 Chapter 3 Head and neck syndromes

4. A 7-​year-​old adolescent girl has multiple hard, bony lumps on her


maxilla. She says that her mother had similar lumps and had a col-
ectomy. Which is the single most likely diagnosis? ★★
A Gardner’s syndrome
B Heerfordt syndrome
C Melkerson–​Rosenthal syndrome
D Paterson–​Brown-​Kelly syndrome
E Peutz–​Jeghers syndrome

5. A 6-​year-​old girl has pronounced facial asymmetry and large uni-


lateral café-​au-​lait spots on her skin. She has attended her dentist
for regular examinations since the early loss of her primary central in-
cisors, which exfoliated with her entire roots intact. She has recently
recovered from a fractured right femur. What is the single most likely
diagnosis? ★★★
A Crouzon syndrome
B Down’s syndrome
C Eagle’s syndrome
D Gardner’s syndrome
E McCune–​Albright syndrome

6. An 8-​year-​old boy attends for a new patient consultation. He is


under the care of the local Paediatric Maxillofacial Unit for a cleft
palate. His appearance can be seen in Figure 3.2. Which is the single
most likely diagnosis? ★★★
A Cherubism
B Ectodermal dysplasia
C Marfan’s syndrome
D Pierre–​Robin sequence
E Treacher–​Collins syndrome

https://t.me/DentalBooksWorld
questions 259

Figure 3.2
Reproduced from Traboulsi E.I., Genetic diseases of the eye, 2nd ed, Figure 2.3,
Chapter 2, Copyright (202), by permission of Oxford University Press USA.

https://t.me/DentalBooksWorld
260 Chapter 3 Head and neck syndromes

ANSWERS
. B ★ OHCD 6th ed. → p. 754
Cleidocranial dysostosis (cleidocranial dysplasia) is an inherited gen-
etic condition that typically presents with hypoplastic clavicles, de-
layed closure of fontanelles, and dento-​alveolar abnormalities. Dental
abnormalities can include a high-​ arched palate, multiple super-
numeraries, retained deciduous teeth, and crown/​root dilacerations.
Frontal and parietal bossing is common, with mid-​ f ace hypoplasia
also noted. The condition is autosomal dominant and mainly affects
membranous bones.
Apert’s and Pfeiffer syndromes are genetic conditions which result in the
premature closure of specific sutures of the skull vault.
Melkerson–​Rosenthal syndrome is an autosomal dominant condition
that can present with swollen or cracked lips, fissured tongue, and facial
paralysis.
Pierre–​Robin sequence is another first branchial arch genetic condition,
which can present with similar features to Treacher–​Collins. The man-
dibular body is shown to be considerably shorter in this condition.
Keywords: lack of collarbones, prominent brow, multiple
supernumeraries.

2. D ★ OHCD 6th ed. → p. 755


Trisomy 2 is more commonly referred to as Down’s syndrome. As the
most common of all malformations, the features described above are
most likely to be encountered with this syndrome. Prevalence is approxi-
mately  in 700 births, with a 50% increase in incidence when the mother
exceeds 40 years of age. It occurs as a result of an extra copy of all or
part of chromosome 2.
Developmental delay, auditory problems, and congenital cardiac de-
fects are common occurrences. Greater than 50% live to 50 years
of age. Hypodontia is a common feature and may complicate dental
management. An increased susceptibility to periodontal disease, infec-
tion, and haematological disorders, e.g. leukaemia, is also reported,
whilst caries incidence is reduced. Taurodontism, delayed eruption of
teeth, or teeth with short roots are additional oral features that may
be evident.
The other answers listed are alternatively known as the following:
● 45,XO—​Turner’s syndrome
● Trisomy 3—​Patau or D-​syndrome
● Trisomy 8—​Edward’s syndrome
● XXY—​Klinefelter’s syndrome.

Keywords: congenitally missing teeth, periodontitis, brachycephaly,


upward sloping palpebral fissures, tongue protrusion.

https://t.me/DentalBooksWorld
answers 261

3. C ★★
Gorlin–​Goltz syndrome (naevoid basal cell carcinoma syndrome) is a
rare autosomal dominant condition brought about by a mutation in the
PTCH gene. Patients have multiple skin lesions known as basal cell carcin-
omas that are locally invasive neoplasms. Jaw lesions are common and
occur in the majority of patients diagnosed with the syndrome. These
lesions are odontogenic keratocysts, which develop from remnants of
the dental lamina. They are locally aggressive cysts/​neoplasm (the true
classification is heavily debated within the profession!) that are often ex-
cised surgically.
Ameloblastomas are generally multiloculated tumours of the posterior
mandible. Unilocular ameloblastoma accounts for a small percentage of
ameloblastomas.
Dentigerous cysts are considered developmental cysts that arise from
remnants of the dental follicle (reduced enamel epithelium). They are
associated with the amelodentinal junction (ADJ/​EDJ) of unerupted
teeth, often third molars, and can vary significantly in size and expansion.
Radiographically, a follicular space exceeding 5 mm should be investi-
gated as a potential dentigerous cyst.
Radicular cysts are associated with the apices of non-​vital teeth. They
can vary in size quite significantly but are often well circumscribed, with
cortication at the margins.
Solitary bone cysts in the mandible are often unilocular in nature and pre-
sent with scalloping around the root apices of multiple teeth. They may
develop as a result of previous trauma and are often void of epithelium
upon surgical exploration.
Keywords: Gorlin–​Goltz, cystic change.
→ El-​Naggar AK, Chan JKC, Grandis JR, Takata T, Slootweg PJ. WHO
Classification of Head and Neck Tumours (4th ed.). International Agency
for Research on Cancer Press, Lyon; 207.

4. A ★★ OHCD 6th ed. → p. 755


Gardner’s syndrome (familial adenomatous polyposis) is a rare autosomal
dominant condition. Multiple osteomas, fibrous tumours, sebaceous
cysts, and polyposis of the colon are features of Gardner’s syndrome.
Multiple osteomas would require further investigation, as the intestinal
polyps in Gardner’s syndrome can undergo malignant change and require
surgical removal. Intraoral osteomas can impede or delay eruption and
can complicate denture provision. It is highly unlikely that Gardner’s
syndrome would be diagnosed following a dental visit, but investiga-
tion of gastrointestinal symptoms and referral to a Gastroenterology
Department may be required if suspicions were aroused.
Heerfordt syndrome is associated with sarcoidosis and can present with
uveitis, parotitis, fever, and possible facial palsy.
Paterson–​Brown-​Kelly syndrome is associated with oesophageal web-
bing, glossitis, cheilitis, and difficulty swallowing.

https://t.me/DentalBooksWorld
262 Chapter 3 Head and neck syndromes

Peutz–​Jeghers syndrome would present with multiple hyperpigmented


macules, often affecting the hands and perioral tissues. The syndrome
is associated with benign intestinal polyposis and has low malignant
potential.
Melkerson–​Rosenthal syndrome is discussed within other questions in
this chapter.
Keywords: multiple bony lumps, maxilla, colectomy.

5. E ★★★ OHCD 6th ed. → p. 754


McCune–​Albright syndrome is diagnosed with two of the following
three clinical traits:
● Polyostotic fibrous dysplasia
● Café-​au-​lait spots
● Autonomous endocrine abnormality.

The condition is caused by a genetic mutation and is rare, occurring in <


in 00,000. Fibrous dysplasia involves replacement of trabecular bone
with immature bone and fibrous tissue and can often result in significant
bone deformities or fractures. The craniofacial bones are thought to be
involved in approximately 50% of cases. The radiographic appearance
of bone in patients with fibrous dysplasia is often described as ‘ground-​
glass’ in appearance. Management of fibrous dysplasia can be complex
and protracted.
Skin pigmentation is often unilateral and can cover large areas of
the body.
Endocrine abnormalities are traditionally referred to as being preco-
cious puberty but can include goitres, hyperthyroidism, and Cushing’s
syndrome, to name a few. Hypophosphataemia has also been reported
with this syndrome.
Crouzon syndrome is a first branchial arch syndrome, which primarily
presents with craniosynostosis, exophthalmus, and hypertelorism.
Eagle’s syndrome results from a misshapen styloid process. It can present
with pain in the jaw and neck region that can occur with movement of
the neck or swallowing. Associated calcification of the stylohyoid liga-
ment can lead to syncope from temporary occlusion of the carotid ves-
sels when looking sideways.
Keywords: café-​au-​lait spots, facial asymmetry.

6. E ★★★ OHCD 6th ed. → p. 760


Treacher–​ Collins syndrome (mandibulofacial dysostosis) is an auto-
somal dominant condition that affects the first branchial arch, and thus
craniofacial development. Genetic mutation affects the TCOF-​ gene,
which encodes the Treacle protein. Structures derived from the first arch
are therefore affected. Hypoplastic zygomas and a retrognathic man-
dible, with downsloping palpebral fissures, are characteristic features.
Generally, normal cognitive development occurs. However, speech and

https://t.me/DentalBooksWorld
answers 263

hearing abnormalities can delay development. Cleft palate is identified in


approximately one-​third of cases. The mandibular ramus is particularly
affected, and a high gonial angle results.
Cherubism is often referred to as familial fibrous dysplasia and is associ-
ated with painless swelling of the jaws. Delayed eruption of the teeth is
common, and improvement can occur over time or with surgery post-​
puberty when the condition becomes self-​limiting.
Ectodermal dysplasia is a common condition which can have dramatic
effects on dentition, including hypodontia, small conical teeth, and maxil-
lary hypoplasia. Very fine or abnormal hair and the inability to sweat are
additional identifiable features.
Marfan’s syndrome is a connective tissue disorder that results in the
production of defective fibrillin-​. Several clinical features can be pre-
sent, but patients are often tall and thin, with arachnodactyly (spider-​like
fingers).
Pierre–​Robin sequence is discussed within other questions in this chapter.
Keywords: malar hypoplasia, deformed pinnas, coloboma.

https://t.me/DentalBooksWorld
https://t.me/DentalBooksWorld
Chapter 4 265

Radiology and
radiography
Raheel Aftab

‘X-​ray vision, your greatest superpower.’


When you consider that, with direct vision alone, you can only see the
coronal few millimetres of teeth and none of the surrounding alveolus,
it becomes clear that, without additional visual aids, we can only assess
and treat a relatively small proportion of our patient’s oral health needs.
In 895, only months after the very first medical radiograph, Dr Otto
Walkhoff recorded the very first dental radiograph. This exposure was
of his own dentition and lasted a lengthy 25 minutes.
Since then, radiography has become a staple tool of the profession
and refinement of the technology has allowed us to reduce exposure
times down to milliseconds, with radiation doses smaller than those ex-
perienced by people taking short-​haul flights. Further advances in dose
reduction and reformatting protocols have allowed for computed tom-
ography to become increasingly popular for diagnostics and treatment
planning in endodontic, oral surgery, and orthodontic cases.
The benefits of dental radiography make them an indispensable
resource, but since all types of radiation pose some degree of risk to
human health, the clinician must consider how useful the information
from the proposed exposure will be. There are no shortages of tragic
stories of employees working with radiation who suffered ill health years
after stopping work.
Today dental radiography can be performed routinely and safely as a
result of the valuable lesson learnt from the debilitating consequences
suffered by past medical professionals, nuclear workers, and even the
‘radium girls’ who painted luminous material onto watch faces.
Key topics include:
● Limitations of radiographs
● Image selection criteria
● Radiation physics, protection, and legislation
● Radiographic interpretation
● Types of dental radiographic imagery.

https://t.me/DentalBooksWorld
266 Chapter 4 Radiology and radiography

QUESTIONS

. A radiographer is discussing the importance of aluminum filtration


in standard X-​ray tubes. As part of the lecture, the dental students
are informed that X-​rays and gamma rays are classified as ionizing radi-
ation and occupy a specific section on the electromagnetic spectrum.
Following the seminar, one of the students asks what properties of X-​
rays enable them to avoid being attenuated by the aluminium filter. ★
A Long wavelength and high photon energy
B Long wavelength and low photon energy
C Medium wavelength and high photon energy
D Short wavelength and high photon energy
E Short wavelength and low photon energy

2. A 45-​year-​old woman has undergone post-​operative radiotherapy


following resection and selective neck dissection of a T2NM0
squamous cell carcinoma of the right retromolar pad. At her 6-​month
review, she is showing signs of xerostomia. Which single physical inter-
action between radiation and tissue would predominantly explain the
clinical signs? ★
A Attenuation
B Compton effect
C Ionization
D Photoelectric effect
E Unmodified scatter

3. A dental student has accidentally taken a right bitewing on an


average-​build 52-​year-​old man at 50 kV. The radiographer dis-
cusses the risk of low-​energy photons and advises that a kilovoltage of
60–​70 is generally advised. However, following the incident, the student
is now confused why an even higher kilovoltage (80–​90 kV) is not used
to investigate for interproximal caries. What is the main diagnostic disad-
vantage of using a kilovoltage of >70? ★
A Darkens the image
B Decreases the contrast
C Improves the resolution
D Increases the ionization
E Lightens the image

https://t.me/DentalBooksWorld
questions 267

4. A 53-​year-​old woman has vertical bitewings requested to assess


her bone levels, following a basic periodontal examination (BPE),
as shown in Table 4.:

Table 4.
3 2 3
3 2 3

The radiographer performing the exposure accidentally performed a


panoramic radiograph instead, because that is what is usually performed
in the department for periodontal bone-​level assessment. The patient
would now like to discuss any potential sequelae of this error. The risk
of which single effect has been increased? ★
A Carcinogenic
B Deterministic
C Heritable
D Metastatic
E Stochastic

5. A radiation protection advisor has recently visited a number of


local practices and has recommended that changes are made to
their X-​ray machines. He advises converting from a round to a rect-
angular collimator. Making this change to radiography equipment is
related to which single International Commission on Radiological
Protection’s (IRCPs) key principles? ★
A Authorization
B Justification
C Limitation
D Optimization
E Restriction

6. A 32-​year-​old woman attends for consultation in the local


Restorative Dentistry Department where she is assessed by a
specialty registrar (StR). She is subsequently sent to the Radiology
Department for the radiographs to be carried out by a radiographer.
Under the Ionising Radiation Medical Exposure) Regulations (IR(ME)R
208), the StR is acting as which of the following agents? (Select one
answer from the options listed below.) ★★
A Operator
B Operator and practitioner
C Practitioner
D Referrer
E Referrer and practitioner
https://t.me/DentalBooksWorld
268 Chapter 4 Radiology and radiography

7. A 44-​year-​old woman has had bitewing radiographs to investigate


for interproximal caries. The film was positioned using the ‘stick-​
on-​tab’ bitewing holding technique. Following development, it is noticed
all the mesial and distal contact points of all teeth appear blurry and
overlapped. If the image is to be retaken, which single factor should be
corrected to improve the image? ★★
A Film positioning
B Horizontal angulation of the collimator
C Length of exposure
D Patient movement
E Vertical angulation of the beam-​aiming device

8. A 4-​year-​old girl has a periapical radiograph taken of her lower


right first permanent molar (LR6) with a digital photostimulable
phosphor (PSP) plate. When the image is processed, there is a ghost
image overlying the LR6. Which single action is most appropriate to cor-
rect this problem? ★★
A Dispose of the PSP, and retake the image with a new plate
B Ensure that the PSP data are fully erased, and repeat
C Repeat the radiograph, and ensure the patient does not move during
the exposure
D Turn the PSP around to ensure the front surface is facing the X-​ray
tube, and retake
E Use the software to correct the fault

9. A foundation dentist is updating their practice radiation folder.


They notice that no radiation protection advisor (RPA) is recorded,
and their foundation trainer wishes to know which single piece of docu-
mentation makes this mandatory. ★★★
A Health and Safety at Work etc. Act (HSW) 974
B Ionising Radiation Regulations (IRR) 207
C Ionising Radiation (Medical Exposure) Regulations (IR(ME)R) 208
D Radiation and Health Protection (RHP) 998
E Royal College of Radiologists Guidelines (RCRG) 2008

https://t.me/DentalBooksWorld
questions 269

0. An -​year-​old boy is assessed in a Paediatric Trauma Clinic,


following a fall off his scooter. The referring general dental prac-
titioner has provided a periapical radiograph suggestive of a horizontal
mid-​third root fracture of the upper right central incisor (UR), but it is
not conclusive. The tooth is grade , mobile, and non-​tender and gave a
negative response to sensibility testing. What is the single most appro-
priate next step to confirm the diagnosis? ★★★
A Request a cone beam computed tomography (CBCT)
B Request a dentopantomogram (DPT)
C Request a supplemental periapical radiograph
D Request an upper midline occlusal radiograph
E Request to use an electronic apex locator to detect a fracture

. An 8-​year-​old man attends the local Emergency Department


(ED) after a blow to the right cheek playing rugby. Clinical signs
suggest a zygomatic complex fracture, and there is no diplopia, loss of
visual acuity, or restriction of eye movements. Which single plain film
radiographic images are most likely to provide the greatest diagnostic
information? ★★★★
A Dentopantomogram (DPT) and posteroanterior (PA) mandible
B DPT and reverse Townes
C DPT and submentovertex
D Lateral skull and PA skull
E Occipitomental (OM) and OM 30

2. A 56-​year-​old man reports recurrent swelling under the right


angle of the jaw every time he eats a meal. The swelling
presents with pain and eventually reduces in size after several hours.
Bimanual palpation reveals a firm bony lump immediately adjacent to
the submandibular gland. A salivary gland calculus or sialolith is made as
the provisional diagnosis. Which single plain film radiograph would be
the most appropriate first-​line image for visualizing the potential salivary
stone? ★★★★
A Dentopantomogram
B Lateral oblique
C Lower oblique occlusal
D Lower occlusal 45°
E Lower occlusal 90°—​‘true occlusal’

https://t.me/DentalBooksWorld
270 Chapter 4 Radiology and radiography

ANSWERS
. D ★
Each packet of X-​ray energy is termed a photon. X-​ray photons have
short wavelengths and high photon energy. These photons are produced
when high-​powered electrons are fired at a tungsten target. Resultant
deflection of these electrons or ejection of target electrons results in
photons of energy being emitted. Low-​energy photons are removed
within the X-​ray tube by an aluminum filter, to reduce irradiation from
photons with poor penetrating power.
Collision of these photons with different structures within the body can
have a variety of effects, which ultimately results in differential pene-
trance through tissues. This provides a radiographic image.
Radiowaves occupy a position at the opposite end of the electromag-
netic spectrum, characterized by long wavelengths and low photon
energy.
Keywords: X-​rays, gamma rays, electromagnetic spectrum.
→ Whaites E, Drage N. Essentials of Dental Radiography and Radiology
(5th ed.). Churchill Livingstone, London; 203.

2. D ★
Collision of an X-​ray photon with electrons within the tissues results in
the ejection of photoelectrons. The X-​ray photon gives up all energy
upon collision and is therefore absorbed by the tissue. Photoelectrons
can continue to collide with other electrons within the tissue, producing
further photoelectrons. Ionization brought about by these electrons is
thought to be linked to the damaging effect of X-​rays. This is known as
the photoelectric effect.
Attenuation refers to the process of absorption and scattering, as the
beam passes through organic tissues. This process reduces the intensity
of the X-​ray.
The Compton effect, or scatter, refers to the interaction of a moderate-​
to high-​energy X-​ray photon with an outer shell electron. The effects are
similar across all tissues. X-​ray photons can be scattered in any direction
but are dependent on the energy of the incoming photon. Compton
scatter is very difficult to minimize and can decrease the contrast of, or
fog, the resultant image.
Ionization refers to the process of producing a charged atom or ion.
Unmodified scatter refers to the change in direction of a low-​energy
photon, without ionization. This process may result in loss of energy but
has little effect upon the resultant radiographic image.
Keywords: radiotherapy, xerostomia, physical.
→ Whaites E, Drage N. Essentials of Dental Radiography and Radiology
(5th ed.). Churchill Livingstone, London; 203.

https://t.me/DentalBooksWorld
answers 271

3. B ★
Kilovoltage reflects the penetrating power of an X-​ray—​in other terms,
the higher the kilovoltage, the higher the energy of the beam. Contrast
refers to the optical density differences visualized on a radiograph—​think
fifty shades of grey—​it is the contrast that enables us to analyse the
different tissues irradiated on a radiograph. Density is the degree of
darkening of a film.
Increasing the kilovoltage will decrease the contrast of the resultant
image. Conversely, decreasing the kilovoltage will increase the contrast.
This is because low-​energy photons are more likely to be absorbed or
scattered by the tissues being irradiated. As a result, fewer photons
reach the film and the ratio between radio-​opaque and radiolucent areas
of the film will be more significant.
High kilovoltage will also affect the density of the film and, as such, a
balance between kilovoltage, milliamperage, and time of exposure
must be sought. Most modern periapical and bitewing radiographs use
between 60 and 70 kV. Kilovoltage is kept the same for most standard
intraoral views, but adjusted for panoramic radiographs where the time
cannot be changed.
In simple terms, altering the milliamperage changes the quantity of X-​
rays produced. The relationship between milliamperage and exposure
time is inversely proportional, i.e. increasing the milliamperage and
decreasing the exposure time by the same amounts will result in the
same image. Milliamperage is generally set (6–​8 mA) on most modern
X-​ray machines. Therefore, alterations in exposure time will be a major
factor in the resultant optical density of intraoral images.
Keywords: higher kilovoltage.

4. E ★
Stochastic effects occur by chance. The probability of these effects
occurring increases with exposure to more irradiation. However, they
could occur at any point and there is no known threshold which is as-
sumed to be safe. The relationship between dose and risk is linear in
nature, with no association between severity and dose. Cancer induction
(somatic effects and carcinogenic effects) and heritable diseases (genetic
effects) are two subcategories of stochastic effects.
Deterministic effects are dose-​ dependent and will occur following
a specified dose of radiation. These can include erythema, hair loss,
xerostomia, and cataracts, which are common deterministic effects after
radiotherapy to the head and neck region.
Metastatic effects refer to pathology that has spread from its primary
site to an additional site within the body. See Figure 4. which shows
the sequence of events when radiation is absorbed by a biological
medium.
Keywords: bitewings, panoramic, sequelae.

https://t.me/DentalBooksWorld
272 Chapter 4 Radiology and radiography

lonizing Energy Chemical


lonization
radiation absorbed changes
Direct action

Biological Indirect action


changes

Heredity Somatic Somatic


effects: effects: effects:
non-deterministic deterministic non-deterministic

Figure 4.
Reproduced from Mason R, and Bourne S, A Guide to Dental Radiography Fourth Edition,
Figure .5, page 7, Copyright (998) by permission of Oxford University Press.

5. D ★
Justification, optimization, and dose limitation are three key principles of
radiation protection, as outlined by the IRCP.
● ‘The Principle of Justification: any decision that alters the radiation

exposure situation should do more good than harm.


● The Principle of Optimization of Protection: the likelihood of incur-
ring exposure, the number of people exposed, and the magnitude of
their individual doses should all be kept as low as reasonably achiev-
able, taking into account economic and societal factors.
● The Principle of Application of Dose Limits: the total dose to any

individual from regulated sources in planned exposure situations other


than medical exposure of patients should not exceed the appropriate
limits specified by the Commission.’
In this scenario, changing from a round to a rectangular collimator would
reduce the field of exposure, thus lowering the received dose.
Keywords: round, rectangular collimator, IRCP’s key principles.
→ International Commission on Radiological Protection. The 2007 Re­­
commendations of the International Commission on Radiological Protec­
tion. 2007. Available at: http://​www.icrp.org

6. D ★★
The IR(ME)R 208 are designed to provide protection for patients who
require radiographic imaging. The regulations discuss the role of clin-
icians in justifying and taking radiographs, as well as diagnostic reference
levels. The regulations also provide information on how to report med-
ical exposures that have exceeded the required dose.
The role of individuals is broken down into the referrer, practitioner,
and operator. The referrer is the clinician who is entitled to refer for
an image and must supply sufficient information to justify the exposure.
The practitioner is the health professional who takes responsibility for

https://t.me/DentalBooksWorld
answers 273

the medical exposure and ensures that it complies with the principles
of IR(ME)R 208. The operator is an adequately individual who takes
the radiograph.
In many radiological departments, the operator would be a radiographer.
The referrer, as in this scenario, would be any dentist or medical profes-
sional who refers a patient for a radiographic exposure.
According to IR(ME)R 208, ‘The primary responsibility of the practi-
tioner is to justify medical exposures’. The practitioner in a hospital set-
ting is often designated by the employer. However, the power to justify
the exposure is often delegated to the operator, assuming the request
complies with pre-​existing local guidelines. General dental practitioners
could therefore occupy all three roles.
Keywords: sent to radiology department, IR(ME)R 208, agent.

7. B ★★
Overlapping of the contact points occurs when there is inappropriate hori-
zontal angulation of the X-​ray tube. The tube should be positioned at 90° to
the film, which needs to be carefully positioned to ensure the beam passes
directly through the interproximal regions. Use of a beam-​aiming device will
assist with positioning of the X-​ray tube at 90° to the film. However, incor-
rect placement of the film could still result in a poor radiograph.
Extremely curved or aberrant tooth positioning may result in more than
one bitewing being required.
Incorrect vertical angulation of the collimator could result in
foreshortening or lengthening of the teeth. Movement by the patient
would result in global blurring of the whole image. See Figure 4.2 which
shows a faulty bitewing radiograph with an inaccurate horizontal angula-
tion. Figure 4.3 shows the same region with the fault corrected.
Keywords: blurry/​overlapped, mesial and distal contact points.

Figure 4.2
Reproduced from Mason R, and Bourne S, A Guide to Dental Radiography Fourth Edition,
Figure 4.4a, page 60, Copyright (998) by permission of Oxford University Press.

https://t.me/DentalBooksWorld
274 Chapter 4 Radiology and radiography

Figure 4.3
Reproduced from Mason R, and Bourne S, A Guide to Dental Radiography Fourth Edition,
Figure 4.4b, page 60 Copyright (998) by permission of Oxford University Press.

8. B ★★
The majority of dental practices use digital radiography because pro-
cessing is fast and more economical and there is less chance of pro-
cessing errors. Additionally, the dose of radiation received per image
is typically less than for the conventional counterpart. Two main types
of digital systems exist: PSP plates and charge-​coupled devices (CCDs).
The benefits of PSP plates are that the films are of a similar size to con-
ventional films and are often better tolerated by the patient. CCDs are
linked to the computer by a lead and are bulkier. However, the image can
be viewed immediately without additional processing steps or a separate
processing machine.
For PSP plates, residual energy remains on the plate following scanning.
Exposure to a bright light source removes the residual energy and allows
the film to be reused. If this process has failed or has not been done,
then a residual ghost image can be seen the next time the plate is ex-
posed and processed.
Manufacturer-​dependent, a small metal disc on the back of the film
is involved in the automated mechanism that draws the plate into the
scanner. As a result, if the film is positioned back to front, a white disc
will appear on the image, which can obscure the view of the final radio-
graph. If no such disc is present, the image will be back to front. This
could be corrected using the computer software, but it might cause con-
fusion if multiple contralateral images are being taken.
The software can be used to change the contrast without altering the
kilovoltage settings but cannot remove double images.
In this scenario, the ghost image could have resulted from the processing
machine having not fully erased the previous image, or the plate might
accidentally not have been processed in the first place. For this reason,
option B is the most appropriate answer, although the plate might be

https://t.me/DentalBooksWorld
answers 275

faulty and might need replacing if no fault can be found with the pro-
cessor. PSP plates are expensive and reusable, so they should not be
thrown away.
Keywords: periapical PSP, ghost image.

9. B ★★★
The Ionising Radiation Regulations 207 (IRR7) is a piece of legislation
intended to protect members of staff working with radiation and the
general public. It is essential to comply with these guidelines. Dose limits,
risk assessment, staff training, and quality assurance are key components
of these guidelines, along with the appointment of an RPA. An RPA is a
legal requirement whose role is to ensure compliance and provide advice
on complying with the IRR 207. These may be an external person but
could also be a staff role within the company or practice.
IRR 207 superseded the IRR 999, whilst the Ionising Radiation
(Medical Exposure) Regulations 208 are designed to protect the patient
from ionizing radiation.
Keywords: radiation protection advisor, documentation.

0. D ★★★
Root fractures are a well-​recognized occurrence following dental trauma.
They are commonly described by their location within the root of the
tooth, with apical third root fractures displaying better outcomes than
coronal third root fractures.
Most fractures occur obliquely due to the vector of forces applied to
the tooth during the traumatic incident. Since radiographs are two-​
dimensional, unless there is displacement between the fractured seg-
ments or the X-​ray beam is travelling in the same direction as the fracture
line, the fracture will not be clearly seen.
For these reasons, two radiographs are recommended to ensure that
a fracture is not missed. Ideally, a vertical parallax view is advised, since
most oblique fractures will travel in a vertical direction. An upper mid-
line occlusal radiograph provides a wide field of view, which is useful
following trauma, but many practices will not have the facilities to per-
form this, so a periapical radiograph taken at an acute angle could be an
alternative. See Figure 4.4a which shows X-​rays not passing through a
fracture, so the fracture line is indistinct on the radiograph. Figure 4.4b
shows X-​rays passing through a fracture line, so it is more obvious on
the radiograph.
A panoramic radiograph (DPT/​OPG) is not advised due to the risk of
the C-​spine obscuring the midline. The dose of radiation from a CBCT
could not be justified at this stage, although it would provide a conclusive
diagnosis.
Keywords: periapical radiograph, root fracture, central incisor, confirm
diagnosis.

https://t.me/DentalBooksWorld
276 Chapter 4 Radiology and radiography

(a)

(b)

Figure 4.4

. E ★★★★
Like most trauma scenarios, two plain film radiographs are indicated
to ensure fractures are not missed. In this case, facial views are likely
to be requested, which usually comprise OM 0° and OM 30° views.
This is sometimes supplemented with a submentovertex or ‘jug-​
handle view’, which gives good visualization of the zygomatic arch.
However, isolated zygomatic arch fractures are less common than
complex fractures.
In this scenario, an OM 0° will provide the best image of the facial bones.
It provides good visualization of the orbital floor, naso-​ethmoidal com-
plex, paranasal sinuses, and zygomatic complex. See Figure 4.5a which
shows the position of the head for the OM projection. Figure 4.5b
shows an OM view to demonstrate fractures of the right zygomatico-​
maxillary complex.
Like the OM 0°, the OM 30° helps identify Le Fort fractures and fractures
of the coronoid processes. See Figure 4.6a which shows the position of
https://t.me/DentalBooksWorld
answers 277

(a)

(b)

Figure 4.5a, b
Reproduced from Mason R, and Bourne S, A Guide to Dental Radiography Fourth Edition,
Figure 0.5a, page 66, Copyright (998) by permission of Oxford University Press.

the head for the 30° OM projection. Figure 4.6b shows a 30° OM view
to demonstrate the fracture patterns in the floor of the orbit and on the
inferior surface of the zygomatic arch.
Lateral skull is good for identifying the cranial base and paranasal air
sinuses. A PA skull shows the calvarium and helps to identify conditions
such as multiple myeloma and Paget’s disease of the bone.
A DPT is a standard dental imaging system for dento-​alveolar issues
and mandibular fractures or condylar issues. This often combined with
either a PA mandible or reverse Townes when assessing for mandibular
fractures.
https://t.me/DentalBooksWorld
278 Chapter 4 Radiology and radiography

(a)

(b)

Figure 4.6a, b
Reproduced from Mason R, and Bourne S, A Guide to Dental Radiography Fourth Edition,
Figure 0.6a, page 67, Copyright (998) by permission of Oxford University Press.

Computed tomography is often used for complex mid-​facial fractures,


but it is unlikely to be a first-​line image requested in Accident and
Emergency (unless there are other neurological indications).
Keywords: zygomatic complex fracture, plain film, images.

2. C ★★★★
The history of the presenting complaint, combined with the clinical
examination findings, is highly indicative of salivary gland or duct path-
ology. Unilateral recurrent mealtime swelling would be pathognomonic
for salivary gland pathology. A lower oblique occlusal is a lower occlusal
view taken with the head rotated away from the X-​ray tube. This enables

https://t.me/DentalBooksWorld
answers 279

good visualization of the submandibular gland and the posterior sub-


mandibular duct, as the primary beam passes from the angle of the jaw
through the gland onto the film positioned between the maxillary and
mandibular teeth. Radio-​opaque calculi are often visible on these films
(see Figure 4.7 which shows a lower occlusal for the posterior part
of the submandibular duct, showing a calculi). However, not all block-
ages are radio-​opaque. Historically, these images have also been used to
identify impacted wisdom teeth; however, this has largely been super-
seded by panoramic radiographs and cone beam computed tomography
(CBCT).
A lateral oblique is more frequently used to assess caries in patients
unable to tolerate intraoral films or dentopantomograms (DPTs).
A lower occlusal 90°—​or lower true occlusal—​would be the first-​line
plain film radiograph to investigate a blockage within the anterior floor
of the mouth, closer to the opening of the submandibular duct. This
image would not visualize the area in question in this scenario, as it is
too far posterior.
A lower occlusal 45° involves setting the X-​ray tube at 45° to the film
through the anatomical menton to enable visualization of the anterior
mandible.

Figure 4.7
Reproduced from Mason R, and Bourne S, A Guide to Dental Radiography Fourth Edition,
Figure 3.d, page 204, Copyright (998) by permission of Oxford University Press.

https://t.me/DentalBooksWorld
280 Chapter 4 Radiology and radiography

A DPT is often taken to investigate large facial swellings and abscesses,


as well as for assessment of the developing dentition in orthodontics. It
would not be first line in the identification of calculi/​swellings in the floor
of the mouth; however, it is possible to identify calcifications within the
gland using this image.
Sialography involves injecting a radio-​opaque dye into the submandibular
duct prior to imaging. This would often be undertaken to assist in diag-
nosis and treatment planning, following suggestion of salivary gland
pathology. Ultrasound or magnetic resonance imaging (MRI) would be
another approach to investigate swellings of the neck.
Keywords: submandibular gland, calculus or sialolith, plain film, first line.

https://t.me/DentalBooksWorld
Chapter 5 281

Statistics, epidemiology,
and dental public health
Peter Clarke

‘There’s lies, more lies and then there’s statistics.’


The content of this subject is frequently overlooked, as it is often ‘not
seen as pertinent’ to practitioners’ day-​ to-​
day work. However, the
impact of dental public health (DPH) as a discipline can be far reaching.
DPH is concerned with improving the oral health of the population,
rather than the individual. It has been described as the science and art of
preventing oral disease, promoting oral health, and improving quality of
life through the organized efforts of society.
DPH teams have numerous responsibilities, including oral health sur-
veillance, developing and monitoring quality dental services, oral health
improvement, policy and strategy development and implementation, and
strategic leadership and collaborative working for health. As such, the
impact of DPH can frequently been seen at a local level, e.g. through
health promotion campaigns or provision of new/​redistribution of ser-
vices (in conjunction with commissioners) to meet local needs.
DPH is predominantly a postgraduate subject, and although the
undergraduate curriculum does not cover the whole topic, some core
knowledge is valuable. In particular, understanding research method-
ology and basic statistics is a useful skill to help interpret the dental lit-
erature appropriately. This is ever more necessary in the modern era of
evidence-​based dentistry.
The questions in this chapter will predominantly cover the fundamen-
tals of statistics relevant to medical research, along with the basics of
study design. Additional questions will touch on the concepts of health
promotion and epidemiology, with further reading suggested to supple-
ment the content.
Key topics include:
● Study design
● Data analysis
● Critical appraisal
● Epidemiology
● Health promotion
● Strategic working and collaboration
● Assessing evidence on oral health and dental interventions, pro-

grammes, and services


● Developing and monitoring quality dental services.

https://t.me/DentalBooksWorld
282 Chapter 5 Statistics, epidemiology, and DPH

QUESTIONS

. A local public health team is collecting data on caries from a group


of schoolchildren. A sample of the data collected is shown in Table
5.. Calculate the mean, mode, and median for the data collected
below. (Select one answer from the options listed below.) ★
A Mean—​6, mode—​6, median—​6
B Mean—​7, mode—​6, median—​6
C Mean—​7, mode—​7, median—​6
D Mean—​7, mode—​7, median—​9
E Mean—​8, mode—​6, median—​8

Table 5.
Name Diseased, missing, filled teeth
(DMFT)
Amit 3
Andy 9
Chloe 6
Francesca 0
Heather 6
Josephine 6
Kat 5
Kate 5
Mani 8
Matty 6
Peter 9
Vish 

2. A local Maxillofacial Unit are working on a clinical trial to develop a


screening blood test which will help identify patients at risk of oral
squamous cell carcinoma. The chief investigator informs the group that
current tests available have a very low sensitivity and a high specificity.
What single term is ‘sensitivity’ also commonly known as? ★★
A False negatives
B False positives
C True negatives
D True positives
E Type I error

https://t.me/DentalBooksWorld
questions 283

3. A senior lecturer in oral and maxillofacial surgery is discussing the


use of different types of data (variables) that can be recorded. Her
most recent epidemiological research collected the TMN stage of all oral
squamous cell carcinomas diagnosed in the Head and Neck Department.
Which single type of data does this staging system represent? ★★
A Continuous
B Discrete
C Nominal
D Ordinal
E Qualitative

4. A study aiming to identify whether alcohol consumption and


smoking during pregnancy increase the risk of the development of
cleft lip and palate is being conducted. After selecting 50 newborn babies
with cleft lip and palate from a local database, a questionnaire is pro-
vided to their parents to investigate the above health-​related behaviours.
A further 50 individuals born without cleft lip and palate are matched
for age, sex, and social demographics and provided with the same ques-
tionnaire. Medical records are also examined. What single type of study
design is being utilized in this scenario? ★★
A Case-​control
B Cross-​sectional
C Prospective cohort
D Randomized controlled trial
E Retrospective cohort

5. A junior dental student is presenting the outcomes from their


elective research project. They refer to the Ottawa Charter on
multiple occasions, and a colleague asks to which principle of public
health the Ottawa Charter refers. (Select one answer from the options
listed below.) ★★
A Amalgam safety
B Caries prevention
C Health promotion
D Infection control
E Sustainable development

https://t.me/DentalBooksWorld
284 Chapter 5 Statistics, epidemiology, and DPH

6. A cross-​sectional study investigating the prevalence of tooth


brushing abrasion is being conducted locally. The team develops
a new index, as it felt previous measurement tools were not fit for pur-
pose. During the clinical training and calibration exercise, examiner feed-
back stated that descriptors were too vague. What single characteristic
of the index needs to be readdressed? ★★
A Acceptability
B Objectivity
C Reliability
D Simplicity
E Validity

7. A dental public health consultant is running an oral health pro-


motion project focusing on periodontal disease reduction. They
decide to join with another public health team focusing on health pro-
motion for diabetes to help promote a unified message. What single
approach is being taken to manage the risk factors? ★★★
A Common risk factor approach
B Disease-​centred approach
C Medical model approach
D Settings approach to health promotion
E Single-​message approach

8. A longitudinal study aiming to identify the link between smoking


and periodontal disease is being carried out. During data col-
lection, a large number of participants did not return for follow-​up.
Statistical analysis showed no significant link between the two variables,
but previous research does. What single statistical problem may account
for this finding? ★★★
A Negative likelihood ratio
B Positive likelihood ratio
C True positive rate
D Type I error
E Type II error

https://t.me/DentalBooksWorld
questions 285

9. A local research group is conducting an observational study, as a


randomized controlled trial was deemed unethical. The group uses
the Bradford Hill criteria to help with its conclusions. What is the single
core aspect of observational studies that these criteria were developed
to evaluate? ★★★
A Bias
B Causation
C Incidence
D Randomization
E Selection

0. An observational study investigating the effect of different


toothbrushing regimes is being conducted within the local dental
school. The study involves two consultants observing a sample of dental
students brushing their teeth. During construction of the protocol, a
concern is raised that students may perform differently because they are
being observed (i.e. more likely to brush for longer). What single term is
used to describe this effect? ★★★
A Berksom effect
B Butterfly effect
C Central tendency effect
D Hawthorne effect
E Measurement effect

. Statistical analysis of data collected during a research project


investigating the age and onset of aggressive periodontitis is
being performed. Exploratory data analysis using a histogram identi-
fies a positive skew in the distribution of the variable ‘age’ (i.e. more
in younger patients). When presenting the data, which paired summary
measures are the most appropriate to report the location and spread of
age? (Select one answer from the options listed below.) ★★★
A Mean and interquartile range
B Mean and standard deviation
C Median and interquartile range
D Median and standard deviation
E Mode and proportion

https://t.me/DentalBooksWorld
286 Chapter 5 Statistics, epidemiology, and DPH

2. A study to compare the effect of two surgical interventions on


the reduction of mobility in periodontally involved teeth is being
conducted. Outcome data were recorded using Miller’s classification of
tooth mobility. Which single statistical test is the most appropriate to
compare tooth mobility between the two intervention groups? ★★★★
A Analysis of variance (ANOVA)
B Fisher’s exact test
C Kruskal–​Wallis test
D Mann–​Whitney U test.
E Spearman’s rank coefficient

3. Public Health England are working with the local clinical com-
missioning groups to reorganize the out-​of-​hours dental ser-
vices. Before continuing, the commissioners require some further
information on the current dental services. What primary investigation
should initially be conducted prior to the commissioning of any new
dental service? ★★★★
A Cost-​effectiveness analysis (CEA)
B Cost utility analysis (CUA)
C Joint strategic needs assessment (JSNA)
D Oral health needs assessment (OHNA)
E Root cause analysis (RCA)

4. During hospital induction, a member of the clinical governance


team discusses the importance of continually assessing stand-
ards to improve patient care. The Caldicott Principles are highlighted
multiple times. Following the talk, a junior colleague asks to which single
‘pillar’ of clinical governance these principles relate. ★★★★
A Audit
B Information governance
C Research
D Risk management
E Staff training

https://t.me/DentalBooksWorld
answers 287

ANSWERS
. B ★
The mean is the sum of the value of numbers divided by the quantity
of numbers (in this case, 84/​2 = 7). Commonly, the mean is referred
to as ‘the average’. However, the mean, mode, and median range are
all averages.
The mode is the most frequent value, which, in this case, is 6, which ap-
pears four times in the data set.
The median is the middle value in the data set when placed in sequential
order. In this sequence, the sixth and seventh number is 6, so the median
is 6. If the data set has an even number of data points, then the median
value is calculated as being halfway between the two middle data points.
Keywords: data, sample, mean, mode, median.

2. D ★★
Sensitivity refers to the ability of a test to correctly identify the propor-
tion of subjects with disease. A test which is considered to have ‘high
sensitivity’ is good at identifying patients with disease. A 00% sensitivity
would identify all patients with disease. If this test (with 00% sensitivity)
were negative, it is safe to assume that the patient is disease-​free.
Specificity reflects the ‘true negative rate’, i.e. the patients correctly iden-
tified as not having active disease. Tests with a high specificity help iden-
tify patients free from disease, who would record a negative result to the
test. A positive result in a test with 00% specificity would be a strong
indicator of disease.
False positives (otherwise known as type I error) and false negatives are
used to help calculate the specificity and sensitivity of tests. In reality,
tests combine sensitivity and specificity to reduce the risk of misdiag-
nosis. Tests which display 00% specificity and 00% sensitivity would
identify all healthy patients and all patients with disease. However, whilst
this is mathematically possible, it is practically highly unlikely. Frequently,
a choice has to be made as to whether to accept a test where the sen-
sitivity or specificity are less than ideal. For example, if a screening test
was being developed for diagnosing cancer, it would be better to have
a higher sensitivity and accept a lower specificity, so that cases are not
missed. The choice of what level of sensitivity and specificity to accept is
often dictated by the purpose of the test (i.e. screening or diagnosis) and
the severity of the disease.
Keywords: sensitivity.

3. D ★★
Ordinal data are data which are ranked in order of importance or
sequence. This is in contrast to nominal data, which are unordered cat-
egorical data. The numerical ranking provided is arbitrary and bears

https://t.me/DentalBooksWorld
288 Chapter 5 Statistics, epidemiology, and DPH

no relation to the actual measurement or distance between the data


points presented. The value assigned to each rank is intended to reflect a
sequence. TNM staging is one example of this (e.g. T, 2, 3, 4), which re-
flects the size and extent of invasion of the tumour. Council tax banding
(A, B, C, D, E) and the Glasgow coma scale (3–​5) are other examples
of ordinal data.
Data can generally be classified as ‘qualitative’ or ‘quantitative’.
Qualitative data are traditionally non-​numerical data, e.g. eye colour, and
are presented as categorical data. Quantitative data are numerical data
which can assume different forms. Continuous (synonym: scalar) data,
such as periodontal pocket depths or attachment loss, are data recorded
on a continuous scale.
Discrete quantitative data, such as diseased, missed, filled teeth (DMFT),
are numerical data with discrete data points.
Keywords: types of variables, TMN.

4. A ★★
A case-​control study design begins by identifying patients (cases) with the
condition under investigation (e.g. cleft lip and palate). Patients without
the condition (controls) are subsequently sought to enable comparisons
between the groups regarding the occurrence of possible risk factors.
This study design retrospectively analyses the various risk factors/​ex-
posures (e.g. smoking and alcohol intake) to identify a possible causal
relationship. They are more commonly used when uncommon and rare
conditions or diseases are the outcome of interest.
Case-​control studies are often quicker and less costly to conduct, but
are less robust in identifying true causal relationships, as multiple risk
factors may be involved and recall bias can be a significant confounding
factor. Randomized controlled trials (RCTs) provide a higher level of evi-
dence concerning causation, but like cohort studies, they are more time-​
consuming and considerably more expensive.
Odds ratios are often calculated from case-​control studies to estimate
the association between two factors. An odds ratio equal to ‘’ would
imply the risk for the disease is similar between exposed and non-​
exposed participants. An odds ratio above ‘’ would imply an increased
risk of those with the disease to be exposed to the risk factor. These
studies and statistics are frequently encountered in the dental literature,
and identifying the limitations of the study and interpreting the results
are important.
This is in contrast to a cohort study, which may be retrospective but is
typically prospective, in which those exposed to a certain risk factor are
identified and prospectively monitored for disease development over
time. RCTs involve random assignment of subjects into either a control
or an experimental arm, which are then prospectively followed for a
specified time period.
Keywords: risk, with, without, matched, questionnaire, medical records.

https://t.me/DentalBooksWorld
answers 289

→ Daly B, Batchelor P, Treasure E, Watt R. Essential Dental Public Health


(2nd ed.). Oxford University Press, Oxford; 203.

5. C ★★
The Ottawa Charter is an international document published in 986 by
the World Health Organization. Prevention of diseases and health pro-
motion should be the fundamental components of all public health or-
ganizations. The five key principles of health promotion include:
. Creating supportive environments
2. Building healthy public policy
3. Strengthening community action
4. Developing personal skills
5. Reorienting health services.
These principles were developed following international consultation in
Ottawa and are subsequently referred to as the Ottawa Charter. Since
986, these principles have been utilized to improve oral health, and
a number of different approaches and models of change have been
proposed.
Keywords: Ottawa Charter, principle, public health.
→ World Health Organization. The Ottawa Charter for Health Promotion.
986. Available at: http://​www.who.int/​healthpromotion/​confer-
ences/​previous/​ottawa/​en/​

6. B ★★
Different aspects should be considered when conducting an epidemio-
logical study, including costs, sample size, and ethical considerations.
In some circumstances, it is not feasible or appropriate to conduct a
full examination with specific tests on all participants. Therefore, re-
searchers often use a measurement tool, or a health index, to assess
health condition. To be effective, an index should ideally have the fol-
lowing properties:
● Simple—​the index should be easy to learn and apply. A large number

of clinicians can therefore be taught to use the tool quickly and obtain
consistent results.
● Objective—​ descriptors should leave little room for individual inter-
pretation; otherwise, results are likely to be unreliable.
● Valid—​the index should measure what it intends to; otherwise, meas-

urement bias can be introduced.


● Reliable/​reproducible—​ each time the index is used, the result should
be similar (consistent), assuming the disease state has not changed.
Reliability refers to any deviation inherent to the index, whereas repro-
ducibility relates to discrepancies with intra-​/​inter-​examiner use.
● Quantifiable—​the index should provide categorical or numerical data,

allowing statistical analysis.

https://t.me/DentalBooksWorld
290 Chapter 5 Statistics, epidemiology, and DPH
● Sensitive—​the index should allow the detection of small variations of
the condition between individuals and small changes over time (dis-
criminatory capacity).
● Acceptable—​the index should be acceptable to patients.

Few dental indices have all these properties. In the example in the scen-
ario, the examiner’s feedback suggested the category descriptors were
too vague. Therefore, they need to be made less subjective and more
objective.
Keywords: index, descriptors, too vague.

7. A ★★★
The traditional approach to health promotion was for individual organ-
izations to promote health messages for their individual disease (disease-​
centred approach); this is now outdated and no longer recommended.
Given the prevalence and relevance of several chronic diseases (such as
cardiovascular disease, diabetes, caries, periodontal disease, and obesity),
which all have common risk factors, a new public health proposal has been
conceived—​the common risk factor approach. This affords several bene-
fits. Firstly, it reduces wastage by avoiding duplication of similar health
messages for different diseases. Secondly, it reduces conflicting health mes-
sages from different organizations only focusing on ‘their’ disease. Thirdly,
it improves the reach of these shared health promotion messages to the
most deprived and socially excluded, due to improved cost-​effectiveness
of the programmes. Together, these benefits maximize the impact of pre-
ventive activities, increasing the effectiveness and efficacy.
The settings approach to health promotion refers to the implementation
of health promotion campaigns in specific environments or places, e.g.
oral health promotion in schools. This approach may be used as a subset
to the common risk factor approach or disease-​centred approach.
Keywords: health promotion, unified message.
→ Chestnutt I. Dental Public Health At a Glance. Wiley Blackwell,
Chichester; 206.
→ Sheiham A, Watt R. The common risk factor approach: a rational basis
for promoting oral health. Community Dentistry and Oral Epidemiology.
2000;28:399–​406.

8. E ★★★
In research, the null statistical hypothesis assumes there is no differences
between the two variables being investigated. A hypothesis cannot be
proven, only rejected, as a sample is being taken, and therefore, data for the
entire population are not available. Rejecting the null hypothesis means that
the difference between the two groups should not be discarded or denied.
As the statistical analysis is undertaken on a sample of the total population,
then the results may not actually reflect what happens in the population.
When the null hypothesis is falsely rejected, it is termed a type I error or
alpha (α) error, which is sometimes referred to as a ‘false positive’. In this
situation, the researchers would be incorrectly claiming that a difference is
https://t.me/DentalBooksWorld
answers 291

present. In contrast, a type II error or a beta (β) error (false negative) would
occur if the null hypothesis was incorrectly accepted.
The power of the study (sensitivity of the test) and significance level (p
value) will dictate the probability of type I and II errors. It is generally ac-
cepted that a power of 80% and a significance level of 0.05 are used, but
this may vary, depending on the investigation and the relative importance
of type I and II errors in the circumstance. These can be used to deter-
mine the sample size required for the study by conducting a sample size
calculation. During the sample size calculation, a dropout rate is usually
accounted for. However, if the dropout rate is greater than expected,
the sample size will be insufficiently large to detect the expected differ-
ences and the study is underpowered. This will increase the risk of a type
II error, and the sample may be too small to reliably detect a difference
between the two groups when it exists.
Keywords: participants, did not return, no significant link.

9. B ★★★
The Bradford Hill criteria aim to assist with identifying causation between
an exposure (risk factor) and a disease or health condition. Many factors
can occur together, which could erroneously be considered a causative
factor if not investigated correctly. Nine aspects were initially proposed:
. Strength of the association
2. Consistency with other investigators
3. Specificity
4. Temporality (time sequence)
5. Biological gradient (dose-​dependent relationship)
6. Plausibility (biological credibility)
7. Coherence
8. Experimental evidence
9. Analogy.
Modern epidemiological and statistical methods have changed consid-
erably since these considerations were proposed in 965, but whilst
the interpretation of each aspect may develop, the criteria are still con-
sidered relevant today.
Keywords: observational, Bradford Hill criteria.
→ Fedak K, Bernal A, Capshaw ZA, Gross S. Applying the Bradford Hill
criteria in the 2st century: how data integration has changed causal
inference in molecular epidemiology. Emerging Themes in Epidemiology.
205;2:4.
→ Hill AB. The environment and disease: association or causation?
Proceedings of the Royal Society of Medicine. 965;58:295–​300.

0. D ★★★
Observational studies can be considered descriptive (i.e. case reports)
or analytical (i.e. case-​control). These study designs are susceptible to
different types of bias. Observational bias represents errors in the study
https://t.me/DentalBooksWorld
292 Chapter 5 Statistics, epidemiology, and DPH

design or data collection that have not occurred by chance. There are
a number of types of observational bias, one of which is response bias.
Response bias occurs when the participants respond how they believe
they should respond.
The Hawthorne effect is a type of response bias in which participants
change their behaviour in response to their awareness of being observed.
Berksom (effect) bias refers to a selection bias that results when a study
design only assesses patients admitted to a certain facility. These subjects
are likely to be from a very specific group of patients and, as such, they
are unlikely to represent the population accurately.
Central tendency (effect) bias is the tendency for a participant to pro-
vide a score towards the middle of a scale, rather than the limits. For
example, if providing patients with a visual analogue scale to rate satis-
faction, they are more likely to place a mark in the middle of the scale.
Keywords: observing, perform differently.

. C ★★★
After data collection, it is important for researchers to review their data
and gain a greater understanding of the type of data they have collected.
This process is referred to as exploratory data analysis. A histogram en-
ables the researcher to examine continuous data (such as age) visually.
A typical bell-​shaped curve is considered a normal distribution, but data
can also be skewed to the left (positive) and to the right (negative), sug-
gesting a non-​normal distribution.
Summary measures or statistics enable researchers to present large
amounts of data in a format that is easy to understand and visualize,
e.g. mean and standard deviation. These measures help researchers and
readers to understand the location (centre) and spread of the data. The
mean, median, and mode are commonly used to present the average
or central values of the data set, whilst the standard deviation and
interquartile range present the spread of the data. Data that are more
spread out would have a wider standard deviation, and this may help
identify how accurately the data represent the population. Much greater
information is provided when both summary measures are presented
together, i.e. the mean and standard deviation.
Continuous data with skewed distribution are more accurately reported
using the median and interquartile ranges, whilst the mean and standard
deviation are recommended for normally distributed continuous data.
Categorical data are often presented using percentages.
Keywords: exploratory data analysis, positive skew, summary measures.

2. D ★★★★
There are various statistical tests available for hypothesis testing. Overall,
the choice of the most appropriate statistical test will depend on the:
● Normality of the distribution (parametric vs non-​parametric)
● Type of data (continuous, ordinal, nominal)

https://t.me/DentalBooksWorld
answers 293

● Number of comparison groups


● Dependence of the data (e.g. independent variables, paired data,
etc.).
For ordinal or categorical data (such as Miller’s classification—​ a
common classification for tooth mobility based on severity), the dis-
tribution is generally non-​parametric (i.e. not normally distributed),
and therefore, a non-​parametric test is the correct choice. When as-
sessing two independent groups, the Mann–​Whitney U test is appro-
priate. Should there have been more interventions and then more
groups to compare, then the Kruskal–​Wallis test would be the most
appropriate.
Where data are binary, then the chi-​squared test tends to be employed.
To compare two groups of binary data with a small sample size (where
the expected value is <5), the Fisher’s exact test is more accurate than
the chi-​squared test.
ANOVA is used for normally distributed continuous data where there
are >2 independent groups to compare.
Spearman’s coefficient is used to assess the correlation between a
dependent variable and one or more independent variables (i.e. a known
risk factor and the outcome of interest). It is a non-​parametric test.
When data are normally distributed, Pearson’s correlation coefficient
can be used (parametric test).
Keywords: two interventions, Miller’s classification (ordinal data).
→ Gosall N, Gosall G. The Doctor’s Guide to Critical Appraisal (4th ed.).
PasTest, Knutsford; 205.

3. D ★★★★
Before treating an individual patient, a clinical examination and diagnostic
phase is undertaken. Similarly, in public health, when commissioning new
dental services, an examination of current services is needed; for dental
services, this will be an OHNA. This process will look at elements such
as the population demographics (e.g. ageing population), the current
dental service provision (availability, cost, location), disease occurrence
(prevalence, trends), service users (demands and priority groups), and
current evidence base. This information is then used in the planning
process. In the example given in the scenario, an OHNA would advise
the commissioners on the options available and how to contract out-​of-​
hours emergency dental care. Ideally, health service planning should be
considered as a cycle. Needs assessment should precede policymaking
and also should be performed after evaluation of the new implementa-
tion, to restart the cycle.
The JSNA looks at the wider picture, beyond dental health. With the
202 Health and Social Care Act reforms in the UK, it became a statu-
tory requirement for local health and well-​being boards to conduct
them. The JSNA looks at the health of the population, including negative
health behaviours (e.g. smoking), identifies health inequalities, and at-
tempts to define what the health and social care needs of the population

https://t.me/DentalBooksWorld
294 Chapter 5 Statistics, epidemiology, and DPH

are now and in the future. The aim is to aid planning and commissioning
of services to meet the needs of the local population.
Options A and B are more related to health economics, rather than
commissioning of services. RCA is a problem-​solving technique used fol-
lowing adverse incidents, to help identify potential human and systemic
factors that may have contributed to the event.
Keywords: commissioning, reorganize, new service.
→ Chestnutt I. Dental Public Health At a Glance. Wiley Blackwell,
Chichester; 206.
→ Department of Health and Social Care. Joint strategic needs assess-
ment and joint health and wellbeing strategies explained. 20. Available at:
https://​www.gov.uk/​government/​publications/​joint-​strategic-​needs-​
assessment-​and-​joint-​health-​and-​wellbeing-​strategies-​explained

4. B ★★★★
The Caldicott Principles were established following the Caldicott Report
into the ‘use of patient identifiable data’ in 997. These guidelines relate
to information governance, i.e. the way in which we use, handle, and
share patient information.
The key principles are:
. Justify the purpose(s)
2. Do not use patient-​identifiable information, unless it is absolutely
necessary
3. Use minimal necessary patient-​identifiable information
4. Access to patient-​ identifiable information should be on a strict
need-​to-​know basis
5. Everyone with access to patient-​identifiable information should be
aware of their responsibilities
6. Understand and comply with the law.
Since 998, NHS organizations have appointed a Caldicott guardian who
is responsible for ensuring an organization uses the personal informa-
tion of its service users in a legal, ethical, confidential, and appropriate
manner. The underpinning legal framework for information sharing
within the UK is currently the Data Protection Act 208. Legislation is
always liable to change, and the authors would advise all clinicians to
keep up-​to-​date with any changes within their country.
Keywords: Caldicott.
→ UK Caldicott Guardian Council. A Manual for Caldicott Guardians. 207.
Available at: https://​www.gov.uk/​government/​g roups/​uk-​caldicott-
​guardian-​council

https://t.me/DentalBooksWorld
295

Index

Tables and figures are indicated by t and f following the page number.

A adhesive dentistry,
0, 05
disto-​occlusal-​lingual
amalgam
ABCDE approach ADJ (amelodentinal restoration,
anaphylactic shock, 205 junction), 9 5, 3–​32
medical adrenal crisis, 78, 85–​86 fillings, 234, 244
emergencies, 9 adrenaline, asthma, 203 lathe-​cut amalgam,
unconsciousness, 94, AFP (atypical facial pain), 234, 244
20, 202 90, 67, 68 loss, 88, 0
abducens nerve Aggregatibacter moisture contamination,
trauma, 45 actinomycetem­ 245
abscess drainage, 3,  comitans infection, paediatric
Access to Health Records 64, 72, 75 dentistry, 42
Act 990, 49 aggressive periodontitis restoration, 235, 245
acetazolamide, 46–​47 (AP), 75 spherical amalgam, 244
N-​acetylcysteine, age and onset, 285, 292 tattoos, 6
paracetamol AI see amelogenesis ameloblastomas, 26
antidotes, 223 imperfecta (AI) ameloblasts, 9
aciclovir, 64 AIDS, 80, 86, 88 amelodentinal junction
acid reflux, 238, 25 see also HIV infection (ADJ), 9, 26
acromegaly, 83 airway patency loss, 45 amelogenesis imperfecta
acrylic dentures, alcohol consumption (AI), 6, 9
, 24–​25 alcoholism, 79, 87 full mouth
ACS (acute coronary induced thrombo­ rehabilitation,
syndrome), 95, cytopenia, 87 4, 29
203, 204 pregnancy in, and cleft paediatric
activated charcoal, 223 lip, 283, 288 dentistry, 32, 45
acupuncture, 6 ALL (acute lymphoblastic American-​European
acute apical abscess, 90 leukaemia), consensus
acute coronary syndrome 72, 83–​84 Sjögren’s
(ACS), 95, all-​ceramic only classification
203, 204 restorations, 92 criteria, 66
acute lymphoblastic Alma gauge, 08, amitriptylene,
leukaemia (ALL), 9f, 9–​20 xerostomia,
72, 83–​84 alpha cells, islets of 209
acute myeloblastic Langerhans, 8 AML (acute myeloblastic
leukaemia (AML), alumina oxide cores, 3 leukaemia), 77,
77, 83–​84 aluminum filtration, 83–​84
acute necrotizing radiography, amlodipine, adverse
ulcerative gingivitis 266, 270 effects, 64, 73
(ANUG), 72 alveolar mucosa, ammonium
acute periapical gingival tissue carbonate, 20
periodontitis, attachment, 67, 77 amnesia, intravenous
09, 2–​22 alveolar osteitis, 36, 4 sedation, 25,
Adam clasps, 56 alveolar ridge 222
Addison’s disease, resorption, 8 amphotericin, 64
83, 86 amalgam anabolic steroids,
addition-​cured burnishing, 245 iatrogenic
polyvinylsiloxane, contraction of, 245 hypogonadism,
232, 24 deep amalgam 83
addition-​cured restorations, anaemia
silicones, 24 232, 240 iron deficiency
adenoma, disto-​occlusal amalgam, anaemia, 94
pleomorphic, 55 237, 249 leukaemia, 83–​84

https://t.me/DentalBooksWorld
296 Index

anaesthesia, 23 anticoagulant drugs, autism, anaesthesia


consent, 29, 227 36, 4 consent, 29, 227
general see general medical emergencies, autistic spectrum disorder
anaesthetic 98, 209 (ASD), 59, 72
inability to swallow, anticonvulsant drugs, 73 autonomic nervous system
38, 45 antifungals, topical, 63 (ANS), 2
inferior alveolar nerve antihistamines, anaphylactic autonomous endocrine
block, 25, 22 shock, 205 abnormalities,
inhalation sedation, anti-​La autoantibodies, McCune–​Albright
26, 224 55, 66 syndrome, 262
intravenous anti-​Ro autoantibodies,
anaesthesia, 222
left inferior alveolar
55, 66
ANUG (acute necrotizing B
nerve block, ulcerative bacterial sialadenitis, 66
24, 220 gingivitis), 72 balancing (non-​functional)
local see local AO (atypical odontalgia), cusp, 23
anaesthesia 57, 68 barium oxide, 248–​49
methaemoglobinaemia, AP see aggressive basal cell carcinoma
28, 227 periodontitis (AP) (BCC), 26
nitrous oxide, 24, 27, apertognathia, 30 basic life support,
220, 22, 225 Apert’s syndrome, 260 unconsciousness,
Parkinson’s disease, apexification, 94 20
28, 226 apical movements, basic periodontal
see also sedation orthodontic fixed examination (BPE),
analgesia, 23 appliances, 55 28, 37–​38
acute periapical apical periodontitis, bone level assessment,
periodontitis, chronic, 90 267, 267t, 27
2–​22 apical plug formation, BCC (basal cell
local anaesthetics, 27, endodontics, 85, 94 carcinoma), 26
225, 226t apixaban, 63 BDC (bioactive endodontic
opioid analgesics, 223 A point, nasion and B point cement), 250
anaphylactic shock (ANB) angle, 58 beclomethasone,
ABCDE approach, 205 arginine, 25 asthma, 203
antibiotics, 96, 205 arthrocentesis, 60 Behçet’s disease, 59, 73
antihistamines, 205 ASD (autistic spectrum benign paroxysmal
facial swelling, 96, 205 disorder), 59, 72 positional vertigo
high-​flow oxygen, 205 aspirin, acute coronary (BPPV), 84–​85
hydrocortisone, 205 syndrome, 95, 204 benzocaine, 227
ANB (A point, nasion assault, right zygomatic contraindications,
and B point) arch fracture, 26, 223
angle, 58 6, 6–​7 benzodiazepines, 200
Andrews, Lawrence, 47 asthma, 95, 202 characteristics, 226
Angle, Edward, 47 asymmetric benzydamine
angular cheilitis, Plate 7, reciprocation, 98 hydrochloride, 64
65 asymptomatic (silent) Berksom (effect) bias, 292
ankyloglossia, 60 myocardial beta cells, islets of
ANOVA, 293 infarctions, 20 Langerhans,
ANS (autonomic nervous atherosclerotic lesion 76, 8
system), 2 rupture, coronary Bezold–​Brucke effect, 30
anterior bite plate, ischaemia, 204 bias
maxillary arch atrophic glossitis, 70 Berksom bias, 292
expansion, 56 attachment loss, central tendency
anterior hard palate, periodontitis vs. bias, 292
intraoral gingivitis, 63, 69–​70 Big Canines for Big
swelling, 5, 6 attenuation, 270 School, 34
anterior maxillary teeth atypical facial pain (AFP), bilateral cervical
crowns, 236, 248 90, 67, 68, 69 lymphadenopathy,
restoration, 238, 250–​5 atypical odontalgia (AO) 77
antibiotics (phantom tooth bimaxillary
airway patency loss, 45 pain), 57, 68 osteotomy, 2, 8–​9
anaphylactic shock, auditory problems, trisomy binary data, 293
96, 205 2 (Down’s bioactive endodontic
plaque, 66, 75 syndrome), 260 cement (BEC), 250

https://t.me/DentalBooksWorld
Index 297

Biodentine, 240 BULL rule (Buccal Upper, causation, statistics,


biofilm-​induced gingivitis, Lingual Lower), 285, 29
63, 69–​70 0, 23 CBCT see cone beam
bisoprolol, 207 bupivacaine, 27, 225, computed
bisphosphonates, 206 226t tomography
oral and maxillofacial burnishing, amalgam, 245 (CBCT)
surgery, 36, 42 CCBs see calcium channel
bitewing radiograph, occlusal
caries, 30, 42 C blockers (CCBs)
CCDs (charge-​coupled
black silk, 253 CAD/​CAM (computer-​ devices), 274
bleaching products, 89, aided design/​ cemento-​enamel
02, 253 computer-​aided junction, 99
bleeding on probing manufacture), cements
(BOP), 69 237, 249 bioactive endodontic
body movement, café-​au-​lait spots, 258, 262 cement, 250
orthodontic calcium channel calcium hydroxide
removable blockers (CCBs) cements, 238, 250
appliance, 49, 55 drug-​induced gingival calcium silicate
bone overgrowth, 73 cements, 238, 247,
alveolar osteitis, 4 halitosis, 208 250
cysts, 26 calcium hydroxide chair time reduction, 29
level assessment, 267, cements, 238, 250 dual-​cured resin cement,
267t, 27 calcium nanophosphate, 233, 242
pain in multiple 25 glass-​ionomer cement,
myeloma, 7 calcium silicate cements, 232, 240
remodelling in denture 238, 247, 250 resin cement, 234, 244
adaptation, Caldicott principles, central tendency (effect)
07, 8 286, 294 bias, 292
turnover markers, 206 candesartan, 80, 89 cephalometrics, xxvt
Bonwill’s triangle, 30 Candida albicans infection, orthodontics, 5, 58
BOP (bleeding on Plate 7, 55, 62, ceramics
probing), 69 65 all-​ceramic only
BPE see basic periodontal chronic erythematous restorations, 92
examination (BPE) candidiasis, 63 metallo-​ceramic crowns,
BPPV (benign paroxysmal chronic hyperplastic 5, 3
positional candidiasis, 62 cerebropontine
vertigo), 84–​85 canines angle (CPA)
Bradford Hill criteria, Big Canines for Big tumour, 85
statistics, 285, 29 School, 34 cerebrovascular accident
brain tumours, 8 palatally impacted (CVA), 86
breast cancer therapy, 36 canines, 50, 57 cervical branch of facial
breathlessness, upper permanent canine nerve, 47
asthma, 95 eruption, 26, 34 chair time reduction,
bridges, 09, 20 Care Quality cementation, 29
double abutement Commission, 74 charcoal, activated, 223
bridge, Plate , caries charge-​coupled devices
2, 26 assessment index, 28 (CCDs), 274
resin-​bonded bridges, development, 93 CHC (chronic
3, 20, 27 occlusal caries, 30, hyperplastic
British National Formulary 33, 42, 46 candidiasis), 62
(BNF), 209 paediatric Chédiak–​Higashi
buccal advancement dentistry, 27, 37 syndrome, 74–​75
flap, oral and prevention of, 37 cherubisms (facial fibrous
maxillofacial surgery, translucent zone, 84, 93 dysplasia), 263
09, 44f, 44 white spot lesions, 3, chest compression,
buccal capping, 55 9f, 27–​28 unconsciousness,
buccal mandibular carrier-​based systems, 95 94, 20
cusp, 23 case-​control study designs, Children Act (2004),
buccinator muscle, 3, 0 283, 288 consent issues, 36
bullous emergence profile, casein phosphopeptide–​ chin tuck, blurred
gingivitis, 68, 78–​79 amorphous vision, 45
bullous pemphigoid calcium phosphates chi-​squared test, 293
(BP), 7 (CPP-​ACP), 25 chloramphenicol, 64

https://t.me/DentalBooksWorld
298 Index

chlorhexidine, 94, 64, 202 CLL (chronic lymphocytic computed tomography


Enterococcus faecalis leukaemia), 83–​84 (CT), 70, 278
infection, 97 closed mouth wash orbital floor
root canal impression, 8–​9 fracture, 46
treatment, 84, 94 cluster headaches, 68 see also cone beam
tooth discoloration, CML (chronic myeloblastic computed
94, 202 leukaemia), 83–​84 tomography
chlorphenamine, 205 cobalt–​chrome dentures, (CBCT)
choral hydrate, 0, 22, 237, 249 computer-​aided design/​
halitosis, 208 framework, , 23–​24 computer-​aided
chronic apical partial dentures, 08, manufacture
periodontitis, 90 4, 20, 28–​29 (CAD/​CAM),
chronic erythematous upper partial cobalt-​ 237, 249
candidiasis (denture chromium conditioning agents, 240
stomatitis), 63 (CoCr) denture, condylar head, translational
chronic hyperplastic 233, 242–​43 movement, 4,
candidiasis coeliac disease, 65 3–​4, 4f
(CHC), 62 cold, pain to, 82, 90 cone beam computed
chronic kidney disease cold lateral tomography
(CKD), 80, 89 condensation, 95 (CBCT), 58
chronic liver damage, 87 collimator complex dens
chronic lymphocytic horizontal angulation, invaginatus, 5,
leukaemia 268, 273, 274f 5, 6f
(CLL), 83–​84 vertical angulation, 273f, root canal pain, 88,
chronic myeloblastic 273, 274f 00
leukaemia colon, polyposis, 26 congenital cardiac defects,
(CML), 83–​84 combination syndrome, trisomy 2 (Down’s
chronic obstructive prosthodontics, syndrome), 260
pulmonary disease 4, 30 conical supernumerary
(COPD), 76, 82 common risk factor teeth, 52, 58
nitrous oxide sedation approach, dental connective tissue
contraindications, public health, grafts, 76–​77
27, 225 284, 290 periodontium
ciclosporin complement, 67, 78 attachment
adverse effects, 96, 205 C3 deposits, 7 structures, 74
drug-​induced gingival classical complement Conn’s syndrome (primary
overgrowth pathway, 78 hyperaldo­
(DIGO), 73 inhibition therapy, 75 steronism), 83
ciliary ganglion, 2 complete maxillary consent
Citanest (prilocaine), denture, anaesthesia, 29, 227
28, 227 236, 246–​47 paediatric
Civette bodies, 72 complex dens invaginatus, dentistry, 27, 36
CKD (chronic kidney cone beam Consumer Protection Act
disease), 80, 89 computed (987), 48
classical complement tomography, 5, continuing professional
pathway, 78 5, 6f development
class II skeletal complexity, periodontal (CPD), 99, 20
discrepancy, 50, 56 disease continuous data, skewed
claustrophobia classification, 70 distribution, 292
inhalation sedation, 226 compliance, acute contrast, high voltage
intravenous sedation, periapical radiography,
28, 226 periodontitis, 2–​ 266, 27
CLE (discoid lupus 22 controlled bleeding,
erythematosus), composite, 248 77, 82
72 composite resin repairs, COPD see chronic
cleft lip, 6, 7, 8f 237, 248 obstructive
risk factors, 283, 288 composite restoration pulmonary
cleft palate, Treacher–​ enamel dentine fracture, disease (COPD)
Collins syndrome, 0, 22 coronal flaring, 82, 83f, 9
258, 259f, 262 prosthodontics, coronal leakage, 00
cleidocranial synostosis, 0, 22 coronal plane, cone
256, 260 Compton effect (scatter beam computed
clinical records, 59 effect), 270 tomography, 5

https://t.me/DentalBooksWorld
Index 299

coronary ischaemia, lateral periodontal glass-​ionomer cement


atherosclerotic cysts, 73 restoration,
lesion rupture, 204 paradental cysts, 42–​43 232, 240
corticosteroids, radicular cysts, 73, 26 gold onlays, 234, 244
anaphylactic solitary bone cysts, 26 impression materials,
shock, 205 cytochrome P450 235, 239,
coughing, asthma, 95 inducers/​ 245–​46, 252
CPA (cerebropontine inhibitors, 64t mesio-​occlusal
angle) tumour, 85 composite,
CPD (continuing
professional D 234, 245
mineral trioxide
development), Dahl principle, 3, 26–​27, aggregate, 236, 247
99, 20 238, 250–​5 resin cement, 234, 244
CPP-​ACP (casein data resin infiltration, 239, 253
phosphopeptide–​ binary data, 293 self-​etch adhesive
amorphous calcium continuous data, skewed bonding system,
phosphates), 25 distribution, 292 234, 243
CRAB acronym, 72 discrete quantitative shade guide, 235, 246
cranial nerves data, 288 silanizing agent, 237, 248
assessment, 3, 2 exploratory data sutures see sutures
cranial nerve IV analysis, 292 zinc oxide eugenol
(trochlear nerve) ordinal data, 283, 287 (ZOE), 232, 240
damage, 38, 45 qualitative data, 288 dental public health
cranial nerve V quantitative data, 288 (DPH), 28
(trigeminal Data Protection Act 208, common risk factor
nerve), 2 40, 48–​49, 294 approach, 284, 290
cranial nerve VII, 2, 8 DCPs (dental-​care primary investigations,
damage, 38, 45 professionals), 33 286, 293
craniofacial bones, debonding, 3, 27 responsibilities, 28
McCune–​Albright distal retainer, 2, 26 standard assessment,
syndrome, 262 decalcification, 49, 55 286, 294
craniofacial deep amalgam restorations, see also epidemiology;
development, 47 232, 240 statistics
CRD (continuing deep periodontal pockets, dentigerous cysts, 37,
professional 96, 205 42–​43, 73, 26
development), 20 delayed eruption of dentine, 93
Crohn’s disease, 7, 65 incisor, 54 bonding, 05, 250
crooked teeth, 30, 4 delta cells, islets of bonding agents, 245
crossbites, 59 Langerhans, 8 erosion, 89, 02
Crouzon syndrome, 262 denosumab, 36, 42 minimal occlusive
crowns, 232, 24 adverse effects, 96, 206 caries, 42
fractures, 3, 43–​44, 43 osteoporosis, 206 odontoblasts, 9
lengthening, 62, 66, dens invaginatus, 3, 44 physical properties, 92
67, 76, 77 dental-​care professionals Dentists Act (984),
CT see computed (DCPs), 33 6, 33
tomography (CT) Dental Complaints dentopantomograms
C-​terminal Service, 68, 80 (DPTs), 3, 37,
telopeptide, 206 dental dam placement, 22 43, 277
curing temperature, dental loupes, 8 dento-​pulpal complex, 90
complete dental materials, 23 dentures
maxillary denture, acid reflux resistance, acrylic dentures,
236, 246–​47 238, 25 , 24–​25
Cushing’s syndrome, 83 amalgam see amalgam cobalt–​chrome dentures
cuspal coverage anterior maxillary see cobalt–​chrome
restoration, 92 crowns, 236, 248 dentures
cusps, 23 anterior maxillary complete maxillary
CVA (cerebrovascular teeth restoration, denture,
accident), 86 238, 250–​5 236, 246–​47
cysts composite resin repairs, copying of, 8–​9
bone cysts, 26 237, 248 fitting surface
dentigerous cysts, 37, crowns, 232, 24 adaptation, 8
42–​43, 73, 26 direct pulp cap, 238, 250 hygiene, Plate 6,
eruption cysts, 73 glass-​fibre post, 233, 242 09, 63

https://t.me/DentalBooksWorld
300 Index

dentures (cont.) distal drifting, paediatric electronic apex locator


immediate maxillary dentistry, 39 (EAL), 86, 96, 8
complete denture, distal retainer, partial embolus, transient
07, 8 debonding, ischaemic attack
loose, 07, 7 2, 26 (TIA), 86
master (working) distribution enamel
impression, 7 non-​normal ameloblasts, 9
partial dentures, 28–​29 distribution, 292 invagination of
see also cobalt–​ non-​parametric epithelium, 44
chrome dentures distribution, 293 enamel dentine fracture,
swing-​lock dentures, periodontal disease composite
5, 30 classification, 70 restoration,
upper complete denture, DMFT (disease, missing 0, 22
235, 245–​46 filled teeth), 28 ENAs (extractable nuclear
worn dentures, 08, 8 double abutement bridge, antibodies), 66
denture stomatitis (chronic Plate , 2, 26 endochondral
erythematous double vision ossification, 8–​9
candidiasis), 63 oral and maxillofacial endocrine
dermatitis surgery, 39, 46 abnormalities, 262
herpetiformis, 7 trochlear nerve (CN IV) endodontics, 8
desensitizing agents, 79 damage, 38, 45 apical plug
deterministic effects, Down’s syndrome (trisomy formation, 85, 94
radiography, 27 2), 74–​75, 256, caries management, 84,
diabetes insipidus (DI), 85 257f, 260 93f, 93
diabetes mellitus, paediatric doxycycline, 94 fractured rotary
dentistry, 32, 45 DPH see dental public instruments, 98,
diabetes mellitus health (DPH) 2, 20
alpha cells, islets of DPT (dental plastic infections, 86, 97, 00
Langerhans, 8 tomogram), 37 magnification, 8
beta cells, islets of DPTs see dentopantomo­ missed canal, 84, 96–​97
Langerhans, grams (DPTs) principles, 8
76, 8 drainage, abscesses, 3,  root canal treatment,
medical emergencies, drug-​induced gingival 82, 83f, 83, 84, 85,
93, 200 overgrowth 86, 87, 9, 92, 94,
type , 64, 73, 76, (DIGO), 73 95–​96, 99, 8
8, 200 dry eyes, Sjögren’s obturation, 85, 95
type 2, 76, 8, 82 syndrome, 55, 66 endodontic sealers, 247
hypoglycaemia, 76, dry mouth enteric nervous
8, 82 clinic, 7, 2 system, 2
dialysis, chronic kidney Sjögren’s syndrome, Enterococcus faecalis
disease, 80, 89 55, 66 infection, 86, 97
diet, paediatric dual-​cured resin cement, epidemiology, 28
dentistry, 25 233, 242 elective research
diffusion hypoxia, 22 projects, 283, 289
digital dentistry, 252
digital radiography, 274 E longitudinal studies,
284, 290
DIGO (drug-​induced Eagle’s syndrome, 262 observational studies,
gingival EAL (electronic apex 285, 29
overgrowth), 73 locator), 86, study design, 283, 288
diphenhydramine, 94 96, 8 tooth brushing abrasion
direct pulp cap, 238, 250 ecstasy (MDMA), 97, 207 prevalence,
crown fractures, 43–​44 ectodermal dysplasia, 263 284, 289
discoid lupus ectopic ossification, 8–​9 see also dental public
erythematosus EDTA (ethylenedi­ health (DPH);
(DLE), 72 aminetetraacetic statistics
discrete quantitative acid), 233, 24 epilepsy, 8
data, 288 sodium hypochlorite epsilon cells, islets of
disease, missing filled teeth (NaOCl) cycling, Langerhans, 8
(DMFT), 28 89, 02 eruption
dislodged fixation Edward’s syndrome cysts, 73
screws, 47 (trisomy 8), 260 delayed of incisor, 54
disposable item signage, Ehlers–​Danlos syndrome, erythema multiforme
40f, 40, 48 6, 74–​75 major, 208

https://t.me/DentalBooksWorld
Index 301

erythema multiforme family history, recurrent fractured rotary


minor, 208 aphthous instruments, 98,
erythrocyte sedimentation stomatitis, 7 2, 20
rate (ESR), 67–​68 faults, prosthodontics, Freedom of information
ethylenediaminetetraacetic , 24–​25 Act 2000, 49
acid (EDTA) FBC see full blood free gingival (FG)
see EDTA count (FBC) grafts, 76–​77
(ethylenediam­ ferrous fumarate, 94 free gingival (FG) groove,
inetetraacetic acid) ferrous sulfate, 20 77f, 77
etidronate fibrous dysplasia, freeway space (FWS), 24
(-​hydroxyethylidene-​ McCune–​Albright pain from
, -​bisphosphonate syndrome, 262 prosthodontics,
(HEBP)), 24 fibrous tumours, Gardner’s 2, 25
European Society of syndrome, 26 frenal pull, 79
Endodontics, 96–​97 fillers, 248 frenectomy, 68, 79
exploratory data film density, high voltage Frey’s syndrome, 55, 66
analysis, 292 radiography, 27 frictional keratosis,
external carotid fine-​needle aspiration 65, 72
artery, 3,  (FNA), 70 frontotemporal process,
external cervical first permanent molars cleft lip, 7
resorption, 87, (FPMs), extraction, full blood count (FBC)
98, 20 32, 44–​45 alcoholism, 79, 87
extractable nuclear Fisher’s exact test, 293 vitamin C deficiency,
antibodies fissure sealants, 44 59, 72
(ENAs), 66 fixation screws, dislodged full gold crowns, 84, 92
extra-​oral drainage, 39, 47 fixation screws, 47 full mouth rehabilitation,
eye ptosis, 24, 220 fixed appliances, amelogenesis
orthodontics, 47, imperfecta,
F 50, 55, 57
fixed treatment
4, 29
functional appliances,
face discontinuation, orthodontics, 47
asymmetry in McCune–​ orthodontics, functional (supporting)
Albright syndrome, 49, 55 cusps, 23
258, 262 flaps fusidic acid cream, 65
blood supply, f,  buccal advancement flap, Fusobacterium infection, 73
external carotid artery 09, 44f, 44
branch, 3, 
fibrous dysplasia
palatal flap, 227
palatal rotation flaps, G
(cherubisms), 263 44f, 44 gag reflexes, 225, 242–​43
Parkinson’s disease, 84 flat anterior bite plane, galvanic corrosion,
pituitary tumours, orthodontic 237, 249
77, 83 removable gamma cells, islets of
right-​side pain, 4, appliance, 55 Langerhans, 8
3–​4 flexible splints Gardner’s syndrome
swelling in anaphylactic paediatric (familial
shock, 96, 205 dentistry, 29, 4 adenomatous
trauma, 39, 46 tooth fracture, 29, 4 polyposis),
unilateral paralysis, 7, flumazenil, 26, 223 258, 26
2–​22, 22f fluoride gaseous porosity, dentures,
facial nerve, 2, 8, 48, 49 dental caries , 24–​25
cervical branch, 47 prevention, 37 gastro-​oesophageal disease
mandibular branch, mouth rinse, 40 (GORD), 94
39, 47 varnish, 40 Gates–​Glidden burs, 9
falls, Parkinson’s FNA (fine-​needle GCS (Glasgow coma
disease, 84 aspiration), 70 score), 80,
false negatives, 290–​9 foramen ovale, 4, 3 89, 89t
false positives, statistics, 45, XO (Turner’s general anaesthetic,
287, 290–​9 syndrome), 260 28, 226
familial adenomatous Fox’s occlusal plane guidelines, 227
polyposis (FAP: guide, 9–​20 paediatric dentistry,
Gardner’s FPMs (first permanent 32, 44–​45
syndrome), molars), extraction, General Dental Council
258, 26 32, 44–​45 (GDC), 20

https://t.me/DentalBooksWorld
302 Index

generalized tooth wear, glass-​fibre posts, 233, 242 halitosis, 98, 208
5, 3 glass-​ionomer cement hard palate, 66, 76–​77
general medicine, 75 (GIC) restoration, Hashimoto’s
acute myeloblastic 232, 240 thyroiditis, 85
leukaemia, glomerular filtration rate Hawthorne effect,
77, 83–​84 (GFR), 89 285, 29
adrenal crisis, glossitis, atrophic, 70 hay fever, 94
78, 85–​86 glossodynia, 60 headaches
AIDS, 80, 88 glossopharyngeal cluster headaches, 68
alcoholism, 79, 87 nerve, 7, 2 sudden-​onset headache,
chronic kidney disease, glossopharyngeal 98, 20
80, 89 neuralgia, 67 unilateral headaches,
chronic obstructive glucagon 56, 67
pulmonary disease, diabetes mellitus type , head and neck anatomy, 
76, 82 93, 200 head and neck
controlled bleeding, production of, 8 syndromes, 255
77, 82 glucose, diabetes mellitus cleidocranial synostosis,
diabetes mellitus type , type , 20 256, 260
76, 8 glyceryl trinitrate (GTN), Gardner’s syndrome,
diabetes mellitus type 2, 95, 204 258, 26
76, 8, 82 goitre, 85 Gorlin–​Goltz syndrome,
Glasgow coma score, gold onlays, 234, 244 257, 26
80, 89, 89t gold shell crown, 3, 28 McCune–​Albright
Graves’ disease, GORD (gastro-​ syndrome, 258, 262
78, 85 oesophageal Treacher–​Collins
Pancoast tumour, disease), 94 syndrome, 258,
79, 88 Gorlin–​Goltz syndrome 259f, 262
Parkinson’s disease, (naevoid basal headgear, orthodontics, 47
78, 84 cell carcinoma), head tilt, blurred
pituitary tumours, 257, 26 vision, 45
77, 83 GP (gutta percha), 95 Health and safety at work
transient ischaemic attack Gracey curettes, Act etc (974), 48
(TIA), 79, 86 62f, 62, 69 Health and Safety
warfarin, 77, 82 granular IgA deposits, 7 Executive (UK),
geographic tongue, Plate 2, Graves’ disease, 67, 78, 85 59, 73
52, 60 greater palatine needle-​stick injuries, 22
GFR (glomerular filtration nerve, 3, 2 health promotion, Ottowa
rate), 89 greyscale images, 246 charter, 283, 289
GIC (glass-​ionomer GTN (glyceryl trinitrate), heart palpitations, Graves’
cement) 95, 204 disease, 78
restoration, gumma (tertiary) syphilitic HEBP
232, 240 lesions, 69 (-​hydroxyethylidene-​
gingival tissue gutta percha (GP), 95 , -​bisphosphonate:
alveolar mucosa etidronate), 24
attachment, 67, 77
anaesthesia, 2 H Heerfordt syndrome, 26
heparin, 90
hyperplasia, ciclosporin haematology, reference hepatitis B, 22
adverse effects, intervals, xxi hepatitis C, 22
96, 205 haemodialysis, 90 herpetiform recurrent
recession, 79 haemophilia, 227 aphthous stomatitis
swelling in acute haemorrhages (RAS), 7,
myeloblastic controlled bleeding, 53, 63
leukaemia, 77 77, 82 high-​flow oxygen
gingivectomy, 76 needle-​stick injuries, anaphylactic shock, 205
gingivitis, 68, 78–​79 25, 22 asthma, 203
biofilm-​induced gingivitis, retrobulbar chronic obstructive
63, 69–​70 haemorrhage, pulmonary disease,
necrotizing 39, 46–​47 76, 82
gingivitis, 64, 73 subarachnoid high voltage radiography,
periodontitis vs., haemorrhages, 266, 27
63, 69–​70 86–​87, 98, 20 histograms, statistics,
Glasgow coma score (GCS), subconjunctival 285, 292
80, 89, 89t haemorrhages, 47 histology, dental caries, 93

https://t.me/DentalBooksWorld
Index 303

HIV infection immunocompromised hepatitis C, 22


needle-​stick injuries, 22 individuals, 73 HIV infection, 22
see also AIDS immunofluorescence, leukaemia, 83–​84
Horner’s syndrome, 66, 88 pemphigus Mycobacterium
hydrocolloids, 252 vulgaris, 7 tuberculosis
hydrocortisone, 205 immunoglobulin A infection, 85
hydrogen peroxide, 02 (IgA), granular Prevotella intermedia
-​hydroxyethylidene-​, deposits, 7 infection see
-​bisphosphonate immunosuppressants, drug-​ Prevotella intermedia
(HEBP) induced gingival infection
(etidronate), 24 overgrowth, 73 Staphylococcus aureus
hypertension implant-​retained infection, Plate 7,
anaesthesia, 225 restorations, 32 55, 65
idiopathic intracranial impression materials, 235, Streptococcus mutans
hypertension, 20 239, 245–​46, 252 infection, 97
hyperthyroidism, 85 hydrocolloids, 252 Treponema infection, 73
hypocalcified defects, 45 impression plaster, 252 upper respiratory tract
hypodontia, trisomy incisal guidance table, infections, 225
2 (Down’s 2, 25–​26 inferior alveolar artery, 
syndrome), 260 incisional biopsy, squamous inferior alveolar nerve
hypoglossal nerve, 3, cell carcinoma, 62 (IAN), 7, 22–​23
9, 2, 23 incisors block, 3, 0, 25, 22
hypoglycaemia, 76, British Standards infrabony defects, 67, 78
8, 82 Institute Incisor inhalation sedation,
hypoglycaemic attack, 200 classification, 5, 57 26, 224
hypogonadism, delayed eruption, 54 claustrophobia, 226
iatrogenic, 83 lateral incisor INR (international
hypomaturation, paediatric replacement, normalized ratio),
dentistry, 45 09, 20 36, 4
hypophosphatasia, 74–​75 mandibular incisors, intercuspal position
hypoplastic defects, single root (ICP), 7
paediatric canals, 96–​97 internal root
dentistry, 45 relationship, 5, 57 resorption, 20
hypoplastic zygomas, root canal treatment, international caries
Treacher–​Collins 7, 22–​23 detection and
syndrome, 262–​63 upper central incisor assessment system
hypothyroidism, 85 pain, 87, 98, 20 (ICDAS II), 28
hypoxia, diffusion upper incisor International Commission
hypoxia, 22 obturation, 85, 95 on Radiological
upper permanent incisor Protection (IRCPs),
I trauma, 30, 42
increased overbites, 55
267, 272
international normalized
IAN see inferior alveolar Independent Mental ratio (INR),
nerve (IAN) Capacity Advocate 36, 4
iatrogenic (IMCA), 228 International Workshop
hypogonadism, 83 index of orthodontic for Classification of
ibuprofen, 80, 89 treatment need Periodontal Disease
ICDAS II (international (IOTN), 48, and Conditions
caries detection 54, 28 (999), 70
and assessment infections, 86, 97, 00, 8 inter-​occlusal rest, 24
system), 28 Aggregatibacter interquartile range,
ICP (intercuspal actinomycetem­ histograms, 285, 292
position), 7 comitans infection, intracellular
idiopathic intracranial 64, 72, 75 immunoglobulins,
hypertension, 20 Candida albicans pemphigus vulgaris,
IHD (ischaemic heart infection see Plate 4, 58, 7
disease), 225 Candida albicans intramembranous
IMCA (Independent infection ossification, 2, 8–​9
Mental Capacity Enterococcus faecalis intraoral films,
Advocate), 228 infection, 86, 97 radiography, 279
immediate maxillary Fusobacterium intraoral swelling, 5, 6
complete denture, infection, 73 intravenous
07, 8 hepatitis B, 22 anaesthesia, 222

https://t.me/DentalBooksWorld
304 Index

intravenous drug use,


AIDS, 80
K anaesthetics, 28, 227
analgesia, 27, 225, 226t
intravenous sedation, 25, Kaposi’s sarcoma, Citanest (prilocaine),
26, 27, 222, AIDS, 88–​89 28, 227
223, 224 Kelly’s syndrome, 30 localized recession
conscious sedation, Kennedy classification, defects, 68, 79
25, 222 0, 22 longitudinal studies,
Parkinson’s disease, keratoconjunctivitis, 66 epidemiology,
28, 226 keratocysts, odontognic 284, 290
iodine see odontogenic loose teeth, 56, 63
allergy, 57, 70 keratocysts periodontitis, 64, 72
Enterococcus faecalis keratosis, frictional, lorazepam, 200
infection, 97 65, 72 lower first permanent
iodoform-​based intracanal Klinefelter’s syndrome incisor, root canal
medicaments, (XXY), 260 treatment, 7, 22–​23
Enterococcus lower motor neuron
faecalis
infection, 97
L (LMN)
lesion, 2–​22
Ionising Radiation (Medical labyrinthitis, 84–​85 lower occlusal 45°
Exposure) LAD (leucocyte adhesion radiograph, 279
Regulations, deficiency), 74–​75 lower third molar
267, 272 lansoprazole, 94, 202 extraction, 2, 9
Ionising Radiation lateral incisor replacement, low-​viscosity silicone, 246
Regulations (IRR) 09, 20 LPS (lipopolysaccharide),
207, 268, 275 lateral luxation, paediatric 67, 78
ionization, 270 dentistry, 27, 36 lymphadenopathy, 69–​70
IOTN (index of lateral periodontal bilateral cervical, 77
orthodontic cysts, 73 leukaemia, 83–​84
treatment need), lateral skull
radiographs, 277
48, 54, 28
IRCPs (International lathe-​cut amalgam, M
Commission 234, 244 MAC (membrane attack
on Radiological left inferior alveolar nerve complex), 78
Protection), block, 24, 220 macroglossia, 60
267, 272 leucocyte adhesion MAF (master apical file),
iron deficiency deficiency 85, 95–​96
anaemia, 94 (LAD), 74–​75 magnetic resonance
irreversible pulpitis, 82, leukaemia, 83–​84 imaging (MRI), 280
90, 67 leukoedema, 68 malignant melanoma, 6
analgesia, 27, 225, lichenoid reactions, 62 malocclusions
226t ramipril, 209 definition, 47
irrigants, 94 lichen planus, Plate 4, 58, orthodontics, 48, 54
Enterococcus faecalis 68, 72 mandible (jaw), 0f
infection, 97 Light index, 28 Gorlin–​Goltz syndrome,
ischaemic heart disease linea alba, 65 257, 26
(IHD), 225 linear IgA deposits, 7 magnetic resonance
islets of Langerhans, lingual nerve, 2, 47 imaging, 57, 70
pancreas, 76, 8 damage to, 2, 9 opening/​closing, 7
isosorbide dinitrate, lipopolysaccharide osteonecrosis, 42
halitosis, 98, (LPS), 67, 78 pain, 56, 67
208 lips, pigmented, Plate 5, registration, , 24
Iwanson gauge, 9–​20 52, 6 trauma radiographs,
lithium disilicate, 92 3, 37, 43
live modelling, 43
J liver function tests,
mandibular incisors, root
canals, 96–​97
jaw see mandible (jaw) alcoholism, mandibular nerve, 2
joint statistic needs 79, 87 mannitol, 46–​47
assessment LMN (lower motor Mann–​Whitney U test,
( JSNA), 293–​94 neuron) statistics, 286, 292
junctional epithelium, lesion, 2–​22 Marfan’s syndrome, 263
periodontium local anaesthesia, 25, 28, master (working)
attachment 222, 227 impression,
structures, 74 alveolar osteitis, 4 dentures, 7

https://t.me/DentalBooksWorld
Index 305

master models, 60 melatonin, 73 Missing teeth, Overjets,


matched-​point single cone Melkersson–​Rosenthal Crossbites,
obturation, 95 syndrome, 66, Displacement of
maxillary arch, 49, 56 260, 262 contact points,
maxillary nerve, 2 membrane attack complex Overbites
maxillary prominence, cleft (MAC), 78 (MOCDO), 54
lip, 6, 7 Mental Capacity Act Mixture of Tetracycline
maxillary tuberosity (2005), 227 (doxycycline),
fracture, 43 mental nerve, 47 citric Acid, and a
MB (mesiobuccal) root mesenchymal cells, tooth Detergent (MTAD),
canal, 87, 99 development, 9 Enterococcus faecalis
McCune–​Albright mesial tipping, paediatric infection, 97
syndrome, 258, 262 dentistry, 39 ML (mesio-​lingual) root
MDMA (ecstasy), 97, 207 mesiobuccal (MB) root canal, 87, 99
mealtime syndrome, 70 canal, 87, 99 MMP (mucous
mean, statistics, 282, mesio-​lingual (ML) root membrane
282t, 287 canal, 87, 99 pemphigoid), 7
mechanical debridement mesio-​occlusal composite, MOCDO (Missing
and obturation, 94 234, 245 teeth, Overjets,
medial nasal mesothelioma, 88 Crossbites,
prominence, 6, 7 metallo-​ceramic crown Displacement of
median, statistics, 282, with metal contact points,
282t, 287 palatal surface, Overbites), 54
histograms, 285, 292 5, 3 mode, statistics, 282,
median rhomboid metastatic effects, 282t, 287
glossitis, 60 radiographs, modified double-​flare
Medical Devices regulations 27, 272f techniques, master
2002, 48 methaemoglobinaemia, apical file, 95–​96
medical emergencies, 9 anaesthetics, moisture contamination,
ABCDE approach, 9 28, 227 amalgam, 245
acute coronary metronidazole, adverse molars
syndrome, 95, effects, 87 permanent molar
203, 204 miconazole cream, growth, 29, 39f,
adrenal crisis, 78, 85–​86 55, 65 39, 40f
airway patency loss, 45 microabrasion, 253 primary
anaphylactic shock, microhybrid extraction, 29, 40
96, 205 composites, 250–​5 restoration, 88, 00
anticoagulant drugs, microscopes, operating morphine, 204
98, 209 microscopes, 8 mouth ulcers see oral
asthma, 95, 202 midazolam ulcerations
diabetes mellitus type , disposal, 99, 2 movement of teeth block,
93, 200 intravenous sedation, orthodontic
drug disposal, 99, 2 25, 26, 27, 222, removable
halitosis, 98, 208 223, 224 appliance, 55
recurrent aphthous tonic–​clonic seizures, MRI see magnetic
stomatitis (RAS), 93, 200 resonance
97, 207 middle superior alveolar imaging (MRI)
Steven–​Johnson nerve, 3, 2 MRONJ (medication-​
syndrome, 97, 208 midline expansion related
subarachnoid screws, 49, 56 osteonecrosis
haemorrhages, midline supernumerary of the jaw), 42,
98, 20 tooth, 48, 54 96, 206
tonic–​clonic seizures, migraines, 20 MTA (mineral trioxide
93, 200 Miller’s classification, tooth aggregate), 94, 236,
unconsciousness, mobility, 286, 247, 250
94, 20 292, 293 MTAD (Mixture of
medication-​related Millers recession defect, Tetracycline
osteonecrosis of 66, 76–​77 (doxycycline),
the jaw (MRONJ), mineral trioxide aggregate citric Acid, and
42, 96, 206 (MTA), 94, 236, a Detergent),
melanoma, AIDS, 88–​89 247, 250 Enterococcus faecalis
melanotic macule, Plate 5, missed canal, endodontics, infection, 97
52, 6 84, 96–​97 mucosal pigmentation, 6

https://t.me/DentalBooksWorld
306 Index

mucous membrane non-​accidental injury odontogenic


pemphigoid (NAI), paediatric myxoma, 42–​43
(MMP), 7 dentistry, 28, 38 odontomes, 58
multiple myeloma, non-​functional (balancing) oesophageal spasm, 204
58, 7 cusp, 23 OFG (orofacial
Mycobacterium tuberculosis non-​Hodgkin’s lymphoma granulomatosis),
infection see (NHL), 84 Plate 7, 54, 65
tuberculosis (TB) non-​parametric OHNA (oral health needs
mylohyoid ridge, 4, 5f, 5 distribution, 293 assessment),
myofascial pain, 60, 68 non-​setting calcium 286, 293
hydroxide, 92, 94 O’Leary plaque index, 69
N non-​steroidal anti-​
inflammatory drugs
OMFS see oral and
maxillofacial
naevi, pigmented, 6 (NSAIDs), 92 surgery (OMFS)
naevoid basal cell non-​ST segment elevation OPG see orthopantomo­
carcinoma myocardial gram (OPG)
(Gorlin–​Goltz infarction ophthalmological
syndrome), 257, (NSTEMI), 203 assessment, orbital
26 non-​surgical periodontal floor fracture, 46
NAI (non-​accidental therapy, 62, opioid analgesics, 223
injury), paediatric 65, 69, 74 optic nerve trauma, 45
dentistry, 28, 38 novel oral anticoagulants oral and maxillofacial
naloxone, 89, 223 (NOACs), 63 surgery (OMFS), 6
nanohybrid composite, NSAIDs (non-​steroidal alveolar osteitis,
238, 250–​5 anti-​inflammatory 36, 4
nanoparticles, 250–​5 drugs), 92 anti-​clotting medication,
naproxen, 207 NSTEMI (non-​ST 36, 4
National Institute for segment elevation bisphosphonate use,
Health and Care myocardial 36, 42
Excellence (NICE), infarction), 203 buccal advancement flap,
pain management, null hypothesis, 290–​9 09, 44f, 44
2–​22 nystatin, 09, 63, 65 cranial nerve damage,
Nayar core, 88, 00 38, 45
necrotizing gingivitis, 64, 73
needle phobia, 222 O dentigerous cysts,
37, 42–​43
needle-​stick injuries, objectivity, statistics, disposable item signage,
25, 22 284, 289 40f, 40, 48
neglect, paediatric observational studies, double vision, 39, 46
dentistry, 46 285, 29 extra-​oral drainage,
nervous system, occipitomental (OM) 39, 47
enteric nervous radiographs, 269, facial trauma, 39, 46
system, 2 276, 277f, 278f inability to swallow,
neutral zone, impressions occlusal caries, 30, 33, 42, 46 38, 45
for dentures, 8 occlusal radiographs, 70 jaw trauma radiographs,
new oral anticoagulants occlusal splints, TMDs, 60 3, 37, 43
(NOACs), 4 occlusal vertical dimension orbital floor fracture,
NHL (non-​Hodgkin’s (OVD), 76, 0, 39, 46–​47
lymphoma), 84 , 23, 24, 236, oroantral communication,
nicorandil, 207 248, 250–​5 09, 44
nifedipine, 207 retruded axis position, pain, 36, 4
drug-​induced gingival 4, 29 retrobulbar
overgrowth, 73 oculocardiac reflex, 47 haemorrhage,
Nikolsky’s sign, 7 oculomotor nerve 39, 46–​47
NiTi rotary file systems, 98 trauma, 45 submandibular abscess,
nitrous oxide, 24, oculovagal reflex, 47 39, 47
220, 22 odontoblasts, 9 tuberosity fracture,
contraindications, odontogenic keratocysts, 37, 43
27, 225 42–​43, 73 oral cavity, 75
pregnant workers, Gorlin–​Goltz syndrome, oral disease, 75
29, 228 257, 26 oral dysaesthesia, 69
NOACs (novel oral oral medicine/​ oral epithelial cells, 9
anticoagulants), pathology, Plate 5, oral glucose, diabetes
4, 63 59, 73 mellitus type , 20

https://t.me/DentalBooksWorld
Index 307

oral health needs orthopantomogram osteotomy, bimaxillary


assessment (OPG), 3 osteotomy, 2, 8–​9
(OHNA), 286, 293 oral ulcerations, Plate 4, otic ganglion, 2
oral health promotion 58, 7 Ottowa charter, health
projects, 284, 290 Steven–​Johnson promotion,
oral hygiene, Plate 4, syndrome, 97, 208 283, 289
09, 63 trauma, 7, 70 OVD see occlusal vertical
paediatric dentistry, 25 orbital floor fracture, dimension (OVD)
periodontics, 65, 73–​74 39, 46–​47 overbite, 49, 53, 55, 59
oral lichen planus, Plate 3, ordinal data, statistics, overjets, orthodontics,
53, 62 283, 287 52, 59
oral medicine/​ oroantral communication, over-​sedation, 223, 224
pathology, 5 09, 44 oxygen administration
atypical odontalgia orofacial granulomatosis acute coronary
(phantom tooth (OFG), Plate 7, syndrome, 95,
pain), 57, 68 54, 65 204
Behçet’s disease, orthodontics, 30, 4, 47 high-​flow see
59, 73 appliances uses, 47 high-​flow oxygen
denture hygiene, Plate 6, cephalometrics, 5, 58 inhalation sedation
09, 63 skeletal recovery, 26,
differential diagnosis, 5 discrepancy, 50, 56 224
Frey’s syndrome, conical supernumerary nitrous oxide sedation,
55, 66 teeth, 52, 58 24, 22
geographic tongue, fixed appliances,
Plate 2, 52, 60
herpetiform recurrent
50, 55, 57
functional P
aphthous stomatitis, appliance, 50, 56 paediatric dentistry, 25
7, 53, 63 impacted canines, 50, 57 amelogenesis
odontogenic incisal imperfecta, 32, 45
keratocysts, Plate 5, relationship, 5, 57 behavioural issues, 25
59, 73 malocclusion, 48, 54 consent issues, 27, 36
oral ulceration, 54, 64 maxillary arch crowding, 30, 4
orofacial granulomatosis, expansion, 49, 56 crown fractures, trauma,
Plate 7, 54, 65 midline expansion 3, 43–​44
painless white lesion, screws, 49, 56 dens invaginatus, 3, 44
Plate 3, 53, 62 midline supernumerary dental caries, 27, 37
pigmented lip, Plate 5, tooth, 48, 54 diabetes mellitus, 32, 45
52, 6 model storage, 53, 59 diet, 25
red patches, 09, 63 overbite, 49, 53, 55, 59 flexible splints,
Sjögren’s syndrome, overjet, 52, 59 trauma 29, 4
55, 66 removable general anaesthesia,
skin lesions, Plate 7, appliance, 49, 55 32, 44–​45
55, 65 retainers, 50, 56 lateral luxation,
sore tongue, 7, 53, unerupted teeth, 5, 57 trauma, 27, 36
58, 63, 7 orthopantomogram non-​accidental
temporomandibular joint (OPG), 3 injury, 28, 38
click, 52, 60 ossification, 2, 8–​9 occlusal caries, 30,
tongue lesion, Plate 3, ectopic ossification, 8–​9 33, 42, 46
57, 69 endochondral oral hygiene, 25
tongue swelling, ossification, 8–​9 permanent molar
57, 70 intramembranous growth, 29, 39f,
tongue ulcer, 53, 6 ossification, 2, 8–​9 39, 40f
trigeminal neuralgia, osteitis, alveolar, 36, 4 plaque-​induced gingivitis,
56, 67 osteoblasts, 8–​9 28, 37–​38
ulcerations, Plate 4, osteolytic lesions, multiple pulpotomy restoration,
58, 7 myeloma, 7 28, 38–​39
unilateral headaches, osteomas, Gardner’s tooth development, 26,
56, 67 syndrome, 26 34, 34t, 35, 35t
vitamin C deficiency, osteomyelitis, 4 tooth extraction, 29, 32,
59, 72 osteonecrosis 40, 44–​45
white lesions, Plate 6, denosumab, 206 trauma, 26, 35
Plate 7, 56, 58, jaw, 42 Paget’s disease, Plate ,
68, 72 radiotherapy, 206 4, 3

https://t.me/DentalBooksWorld
308 Index

pain, 67 PD (peritoneal PHG (primary herpetic


atypical facial pain, 90, dialysis), 90 gingivostomatitis),
67, 68 PDL see periodontal Plate 6, 54, 64
atypical odontalgia ligament (PDL) photodynamic
(phantom tooth PE (pulmonary therapy, 75–​76
pain), 57, 68 embolism), 88 photoelectric effect,
bone pain in multiple peak expiratory flow radiography,
myeloma, 7 rate (PEFR), 266, 270
cold stimuli, 82, 90 asthma, 202 photostimulable phosphor
dental pain, 67 Pearson’s correlation, 293 (PSP) plates,
facial pain, 4, 3–​4 pedicle grafts, 77 268, 274
jaw pain, 56, 67 PEFR (peak expiratory Pierre–​Robin syndrome,
myofascial pain, 60, 68 flow rate), 260, 263
NICE management asthma, 202 pigmented naevi, 6
guidelines, 2–​22 pemphigus vulgaris, Plate 4, Pin Index Safety
oral and maxillofacial 58, 7 System, 220
surgery, 36, 4 periapical periodontitis, pituitary tumours, 77, 83
prosthodontics, acute, 09, plaque, 73–​74
2, 25 2–​22 antibiotics, 66, 75
root canal, 88, 00 periapical radiography, induced gingivitis,
sodium hypochlorite 269, 275 28, 37–​38
accident, 92 pericoronitis, 37 retention, 69
palatal flap, general periodontal disease plasma cells, multiple
anaesthetic, 227 classification, 70 myeloma, 7–​72
palatally impacted smoking, 284, 290 plasticized GP obturation
canines, 50, 57 periodontal landmarks, 77f techniques, 95
palatal maxillary cusp, 23 periodontal ligament pleomorphic adenoma, 55
palatal rotation flaps, (PDL), 74, 32 PMC (preformed metal
44f, 44 compression, 6, 32 crown), 38–​39
palate, nerve supply, periodontics, 6 PMR (polymyalgia
2, 3f examination, 63, 7, 72t rheumatica),
PA (posterioanterior) non-​surgical periodontal 78, 85–​86
mandible therapy, 62, Pneumocystis jiroveci
radiograph, 3, 65, 69, 74 pneumonia (PJP), 88
37, 43 root surface pneumothorax,
Panavia, 244 debridement, spontaneous, 88
Pancoast tumour, 79, 88 62, 69 pneumothorax index, 28
pancreas, islets of systemic disease, 6 polarized images, 246
Langerhans, periodontitis poliglecaprone sutures, 253
76, 8 aggressive see aggressive polyacrylic acid sutures,
pancreatic polypeptide periodontitis (AP) 232, 240
(PP), 8 basic examination, 63, polyester sutures, 253
Papillon–​Lefreve syndrome, 7, 72t polyether impression
65, 74–​75 diagnosis, 56, 63 materials, 239, 252
paracetamol antidotes, 223 gingivitis vs., 63, 69–​70 polyglycolic acid (Vicryl)
paradental cysts, 42–​43 loose teeth, 64, 72 sutures, 253
parallax, 57 periodontium, attachment polymyalgia rheumatica
parasympathetic nervous structures, 74 (PMR),
system, 2 peritoneal dialysis 78, 85–​86
Parkinson’s disease, (PD), 90 polyostotic fibrous
78, 84 permanent molar growth, dysplasia, 262
anaesthesia, 28, 226 29, 39f, 39 polyposis of the colon, 26
intravenous sedation, Peutz–​Jeghers polypropylene (ProleneTM)
28, 226 syndrome, 262 sutures, 239,
parotid salivary glands, 6 Pfeiffer syndrome, 260 252, 253
Patau syndrome (trisomy PG (pyogenic porosity, dentures,
3), 260 granuloma), 72 , 24–​25
Paterson–​Brown–​Kelly phantom tooth pain post crown, prosthodontics,
syndrome, 26 (atypical 09, 2
path of insertion, dental odontalgia), posterioanterior (PA)
impressions, 8 57, 68 mandible
patient records, pharyngeal arches, 2, 8, 8t radiograph, 3,
40, 48–​49 phenytoin, 73 37, 43

https://t.me/DentalBooksWorld
Index 309

posterior circulation ProleneTM (polypropylene) pterygopalatine


infarct, 85 sutures, 239, ganglion, 2
posterior teeth eruption, 252, 253 public health see dental
3, 26–​27 proprioception, 32 public health (DPH)
post-​operative prosthodontics, 05 public transport,
radiotherapy, cobalt–​chrome denture intravenous
squamous cell framework, sedation, 27, 224
carcinoma, , 23–​24 pulmonary
266, 270 cobalt-​chrome partial embolism, 203–​4
postural (proximate) denture, 08, 4, pulmonary embolism
muscle weakness, 20, 28–​29 (PE), 88
Parkinson’s combination syndrome, pulmonary
disease, 84 4, 30 tuberculosis, 88
power, statistics, 29 composite restoration, AIDS, 88
PP (pancreatic 0, 22 pulpitis
polypeptide), 8 copy techniques, 08, irreversible see
prednisolone 8, 9f, 9–​20 irreversible pulpitis
removal from, double abutement reversible, 67
78, 85–​86 bridge, Plate , pulpotomy restoration,
temporal 2, 26 28, 38–​39
arteritis, 67–​68 faults, , 24–​25 p value (significance
preformed metal crown generalized tooth wear, level), 29
(PMC), 38–​39 5, 3 pyogenic granuloma
pregnant workers, gold shell crown, (PG), 72
nitrous oxide 3, 28
use, 29, 228
premolar spacing, 39
immediate maxillary
complete denture, Q
preventative dentistry, 25 07, 8 quad helix,
Prevotella intermedia incisal guidance table, orthodontics, 47
infection, 97 2, 25–​26 qualitative data,
necrotizing gingivitis, 73 jaw registration process, statistics, 288
prilocaine (Citanest), , 24 quantitative data,
local anaesthetic, Kennedy classification, statistics, 288
28, 227 0, 22
primary herpetic
gingivostomatitis
pain from, 2, 25
periodontal ligament R
(PHG), Plate 6, compression, radiation protection
54, 64 6, 32 advisor (RPA),
primary post crown, 09, 2 268, 275
hyperaldosteronism posterior teeth radicular cysts, 73, 26
(Conn’s eruption, 3, radiography, 265
syndrome), 83 26–​27 aluminum filtration,
primary impressions, primary/​secondary 266, 270
prosthodontics, impressions, bitewing
07, 7 07, 7 radiograph, 30, 42
primary molar resin-​bonded bridge bone level assessment,
extraction, 29, 40 failure, 3, 27 267, 267t, 27
primary root canal retruded contact collimator horizontal
treatment, 82, position (RCP), angulation,
83f, 9 07, 7 268, 273
primary syphilitic chancre, screw-​retained columnator angulation
Plate 3, 57, 69 implant crown, 268, 273, 274f
primers, 245 6, 32–​33 digital radiography, 274
Principles of swing-​lock dentures, lateral skull
Application, 272 5, 30 radiographs, 277
Principles of treatment planning, 05 lower occlusal 45°
Justification, 272 worn dentures, 08, 8 radiograph, 279
Principles of see also adhesive milliamperage, 27
Optimization, 272 dentistry occipitomental
probiotic therapy, 76 protamine sulfate, 209 radiographs, 269,
progression, periodontal PSP (photostimulable 276, 277f, 278f
disease phosphor) plates, occlusal
classification, 70, 7 268, 274 radiographs, 70

https://t.me/DentalBooksWorld
310 Index

radiography (cont.) recreational drugs, root apices, radiological


PA mandible 97, 207 assessment,
radiograph, 3 rectangular collimators, 4, 5f, 5
periapical radiography, radiography, root canals
269, 275 267, 272 pain, 88, 00
photoelectric effect, recurrent aphthous preparation, 9
266, 270 stomatitis (RAS), retreatment, 88, 00
photostimulable 7, 53, 63, symmetry, 99f, 99–​00
phosphor plates, 69–​70, 97, 207 treatment, 7, 22–​23,
268, 274 red patches, 09, 63 82, 83f, 83, 84, 85,
posterioanterior referrer, radiography, 86, 87, 9, 92, 94,
mandible 267, 272 95–​96, 99, 8,
radiograph, 3, refractive index, dental 233, 242
37, 43 caries, 3, 27–​28 root cause analysis
Principles of regenerative (RCA), 294
Application, 272 endodontics, 94–​95 root development, 42
Principles of remineralization, 253 crown fractures, 43–​44
Justification, 272 removable appliances, 49, root fractures, trauma, 43
Principles of 50, 55, 56 radiography, 269,
Optimization, 272 maxillary arch 275, 276f
radiation protection expansion, 49, 56 roots
advisor, 268, 275 removable partial resorption, 87, 98, 20
rectangular collimators, denture (RPD), retained roots, 4
267, 272 28–​29, 30 root surface debridement
root apices assessment, repeat-​root canal (RSD), 62, 65,
4, 5f, 5 treatment 69, 73–​74
root fractures, trauma, (ReRCT), 233, 24 rotational movements,
269, 275, 276f ReRCT (repeat-​root orthodontic fixed
salivary gland calculus, canal treatment), appliances, 55
269, 278f, 278 233, 24 RPA (radiation protection
split tooth, 0 resin-​bonded bridges advisor), 268, 275
standard occipitomental (RBBs), 20 RPD (removable partial
radiograph, 3 failure, 3, 27 denture),
stick-​on-​tab bitewing resin cement, 234, 244 28–​29, 30
holding technique, resin infiltration, 239, 253 RSD (root surface
268, 273 respiratory drives, 82 debridement),
upper permanent canine response bias, 65, 73–​74
eruption, 34 statistics, 29–​92
voltage, 266, 27
zygomatic complex
resting face height (RFH),
, 9–​20, 24 S
fracture, 269, 276, restoration, 253 sagittal plane, cone
277f, 278f Resuscitation Council (UK) beam computed
radiotherapy, guidelines, 20 tomography
osteonecrosis, 206 retainers, 47 (CBCT), 5
ramipril, 208 distal retainer, 2, 26 salbutamol
ranitidine, halitosis, 209 orthodontics, 50, 56 anaphylactic shock, 205
RAP (retruded axis prosthodontics, 2, asthma, 95, 202
position), occlusal 3, 26, 27 salivary flow tests, 66
vertical dimension, rete processes, 72 salivary glands, 6
4, 29 retrobulbar haemorrhage, calculus, 269, 278f, 278
RAS (recurrent aphthous 39, 46–​47 sublingual, 6
stomatitis), 7, retruded axis position submandibular, 6
53, 63, 69–​70, (RAP), occlusal salivary surface tension,
97, 207 vertical dimension, dentures, 7
rash, urticarial rash, 4, 29 sarcoidosis, 66
96, 205 retruded contact position SCC see squamous cell
RBBs see resin-​bonded (RCP), 07, carcinoma (SCC)
bridges (RBBs) 7, 29 Schirmer’s test, 66
RCA (root cause reversible pulpitis, 67 screw-​retained
analysis), 294 RFH (resting face height), implant crown,
RCP (retruded contact , 9–​20, 24 6, 32–​33
position), 07, right zygomatic arch sebaceous cysts, Gardner’s
7, 29 fracture, 6, 6–​7 syndrome, 26

https://t.me/DentalBooksWorld
Index 311

secondary impressions, silicones, somatostatin (growth


prosthodontics, addition-​cured, 24 hormone inhibitory
07, 7 silk sutures, 253 hormone
secondary (snail track single file systems, 99 (GHIH)), 8
ulcers) syphilitic SIRS (systemic sore tongue, 7, 53,
lesions, 69 inflammatory 58, 63, 7
sedation response spasms
inhalation see inhalation syndrome), 86 oesophageal spasm, 204
sedation Sjögren’s syndrome, superior rectus
inhalation sedation, 55, 66 myospasm, 46
26, 224 skeletal discrepancy class Spearman’s coefficient,
intravenous see II, 50, 56 statistics, 293
intravenous skewed distribution, specificity, statistics, 287
sedation continuous speech, slurred, 79, 86
intravenous sedation, data, 292 sphenomandibular
25, 26, 27, 222, skin lesions, Plate 7, ligament, 0
223, 224 55, 65 spherical amalgam, 244
see also anaesthesia skin pigmentation, split tooth, 88, 0
seizures, tonic–​clonic McCune–​Albright spontaneous
seizures, 93, 200 syndrome, 262 pneumothorax, 88
selective laser melting SLE (systemic lupus squamous cell carcinoma
(SLM), 237, 249 erythematosus), (SCC), 53, 6,
selective pressure 66 62–​63, 69
techniques, 246 SLM (selective laser AIDS, 88–​89
selective serotonin melting), 237, 249 post-​operative
reuptake inhibitors SLOB rule, 57–​58 radiotherapy,
(SSRIs), 209 slurred speech, transient 266, 270
self-​etch adhesive bonding ischaemic attack, SSRIs (selective serotonin
system, 234, 243 79, 86 reuptake
sella turcica, nasion, and small retained roots, 4 inhibitors), 209
A point (SNA) smoking stainless steel crowns
angle, 58 alveolar osteitis, 4 (SSCs), 38–​39
sella turcica, nasion, and medication-​related stainless steel hand files,
B point (SNB) osteonecrosis 85, 95–​96
angle, 58 of the jaw standard assessment,
semi-​adjustable (MRONJ), 206 dental public health,
articulators, periodontal disease and, 286, 294
2, 25–​26 284, 290 standard deviation,
sensitivity, statistics, 282, pregnancy in, and cleft statistics, 292
287, 29 lip, 283, 288 standard occipitomental
sensory feedback, root SNA (sella turcica, nasion, radiograph, 3
canal treatment, 92 and A point) stannous fluoride, 238, 25
septic shock, 86 angle, 58 staphylococcal scalded skin
severity, periodontal snail track ulcers syndrome, 208
disease (secondary) Staphylococcus aureus
classification, 70, 7 syphilitic infection, Plate 7,
shade guide, 235, 246 lesions, 69 55, 65
shade matching, composite SNB (sella turcica, nasion, statins, halitosis, 209
restoration, 22 and B point) statistics, 28
shuffling gait, Parkinson’s angle, 58 Bradford Hill criteria,
disease, 78, 84 sodium hypochlorite 285, 29
sialadenitis, bacterial, 66 (NaOCl), 84, 94, causation, 285, 29
sialography, 70, 280 24, 247 false positives, 287
Sjögren’s syndrome, 66 Enterococcus faecalis histograms, 285, 292
sialoliths, 269, 278 infection, 97 Mann–​Whitney U test,
Sicca syndrome, 67 ethylenediam­ 286, 292
significance level (p value), inetetraacetic acid mean, 282, 282t, 287
statistics, 29 (EDTA) cycling, median, 282, 282t, 287
silane coupling agent, 89, 02 mode, 282, 282t, 287
237, 248 root canal objectivity, 284, 289
silent (asymptomatic) treatment, 83, 9 ordinal data, 283, 287
myocardial sodium lauryl sulfate, 25 sensitivity, 282, 287
infarctions, 20 solitary bone cysts, 26 specificity, 287

https://t.me/DentalBooksWorld
312 Index

statistics (cont.) superior rectus myospasm, brushing, 37


summary measures orbital floor conical supernumerary
of, 292 fracture, 46 teeth, 52, 58
true positives, 282, 287 supernummary teeth, crowding, 30, 4
type II errors, 284, 290 256, 260 decayed, missing filled
see also dental public supporting (functional) teeth, 28
health (DPH); cusps, 23 development, 9, 20f,
epidemiology supracrestal attached 26, 34t, 35, 35t
status epilepticus, 200 tissues, 74 discoloration,
STEMI (ST segment surface root resorption, 20 chlorhexidine
elevation surfactants, 24 digluconate,
myocardial sutures, 239, 252 94, 202
infarction), 203 poliglecaprone eruption, maxillary
stereolithography, 249 sutures, 253 arch, 26, 35
steroids, 46–​47 polyacrylic acid sutures, extraction, paediatric
airway patency loss, 45 232, 240 dentistry, 32, 44–​45
asthma, 203 polyester sutures, 253 fracture from
iatrogenic polyglycolic acid (Vicryl) trauma, flexible
hypogonadism, 83 sutures, 253 splints, 29, 4
immunosuppressive polypropylene generalized tooth wear,
effects, 86 (ProleneTM) sutures, 5, 3
Steven–​Johnson syndrome 239, 252 loose teeth see
(toxic epidermal silk sutures, 253 loose teeth
necrolysis (TEN)), vertical mattress midline supernumerary
97, 208 sutures, 44 tooth, 48, 54
stochastic effects, swallowing, inability to, Missing teeth, Overjet,
radiography, 267, 38, 45 Crossbite,
267t, 27 sweating, Graves’ Displacement of
stomatis nicotina, 68 disease, 78 contact points,
StR (speciality registrar), swelling, sodium Overbite, 54
267, 272 hypochlorite mobility, Miller’s
Streptococcus mutans extrusion, 92 classification,
infection, 97 swing-​lock dentures, 286, 292
stroke, 7, 2–​22 5, 30 movement of teeth
ST segment elevation sympathetic nervous block, 55
myocardial system, 2 posterior teeth eruption,
infarction synthetic monofilament 3, 26–​27
(STEMI), 203 fibres, 252 split tooth, 88, 0
studies syphilis, Plate 3, 57, 69 supernummary teeth,
design, 283, 288 necrotizing gingivitis, 73 256, 260
power, 29 primary chancre, Plate 3, unerupted teeth, 5, 57
subarachnoid 57, 69 wear, 66, 76
haemorrhages, systematic desensitization, whitening, 89, 02
86–​87, 98, 20 crown fractures, 43 see also canines;
subconjunctival systemic disease, incisors; molars
haemorrhages, 47 periodontics, 6 TEGMA (tri-​ethylene glycol
sublingual salivary systemic inflammatory dimethylacrylate),
glands, 6 response syndrome 248–​49
submandibular abscess, (SIRS), 86 temporal arteritis, 56, 67
oral and systemic lupus temporalis muscle, 6–​7
maxillofacial erythematosus temporomandibular
surgery, 39, 47 (SLE), 66 disorder (TMD),
submandibular ganglion, 2 68, 69
submandibular salivary
glands, 6 T temporomandibular joint
(TMJ), 3–​4, 4f
sudden-​onset headache, Taper, endodontics, 9 click, 52, 60
98, 20 TCOF-​ gene, Treacher–​ TEN (toxic epidermal
sugar intake, dental Collins syndrome, necrolysis: Steven–​
caries, 37 262–​63 Johnson syndrome),
summary measures of teeth 97, 208
statistics, 292 anterior maxillary tertiary (gumma) syphilitic
superior oblique muscle teeth see anterior lesions, 69
innervation, 45 maxillary teeth tetracyclines, 202

https://t.me/DentalBooksWorld
Index 313

beta-​thalassaemia, 84 alveolar osteitis, 4 twin block


thirst, Graves’ disease, 78 brain injury, 46 appliances, 52, 59
thoracic aortic oculomotor nerve type I errors,
aneurysm, 204 trauma, 45 statistics, 290–​9
three-​walled infrabony optic nerve trauma, 45 type II errors, statistics,
defects, 67, 78 paediatric dentistry, 26, 284, 290
thrombocytopenia, 83–​84 30, 35, 42
thyroid cancer, 85
thyroid-​stimulating hormone
repositioning, lateral
luxation, 36 U
(TSH), 85 traumatic optic neuropathy UA (unstable angina), 203
TIA (transient ischaemic (TON), 47 ulcerations, 54, 64
attack), 79, 86 traumatic ulcers, 7, 70 oral medicine/​
ticagrelor, 98, 209 Treacher–​Collins pathology, Plate 4,
tipping, orthodontic syndrome, 2, 8, 58, 7
removable 258, 259f, 262 ulcerative colitis (UC), 65
appliance, 55 treatment response, ultrasound, 280
titanium fluoride, 25 periodontal disease Sjögren’s syndrome, 66
titanium-​reinforced classification, 7 UMN (upper motor
membranes, 78 Treponema pallidum see neuron) lesion,
TMD (temporomandibular syphilis 2–​22, 22f
disorder), 68, 69 Treponema pallidum unconsciousness
TMJ see temporomandi­ haemagglutination medical emergencies,
bular joint (TMJ) assay (TPHA), 69 94, 20
TMN staging, 283, 287 tricalcium silicate-​based paediatric
O-​toludine, 227 restorative dentistry, 26, 35
TON (traumatic optic materials, 240 unerupted teeth, 5, 57
neuropathy), 47 tri-​ethylene glycol unilateral facial paralysis,
tongue dimethylacrylate 7, 2–​22
lesions, Plate 3, 57, 69 (TEGMA), 248–​49 unilateral headaches,
sore, 7, 53, 58, trigeminal nerve (cranial 56, 67
63, 7 nerve V), 2 unmodified scatter, 270
swelling, 57, 70 trigeminal neuralgia, 68, unstable angina (UA), 203
ulcer, 53, 6 69, 20 upper complete denture,
tonic–​clonic seizures, oral medicine/​ 235, 245–​46
93, 200 pathology, 56, upper midline occlusal
topical antifungals, 63 67 radiography,
torsional fatigue, 98 Paget’s disease, Plate , 269, 275
toxic epidermal necrolysis 4, 3 upper motor neuron
(TEN) (Steven–​ trisomy 3 (Patau/​ (UMN) lesion,
Johnson syndrome), D-​syndrome), 260 2–​22, 22f
97, 208 trisomy 8 (Edward’s upper partial cobalt-​
TPHA (Treponema pallidum syndrome), 260 chromium
haemagglutination trisomy 2 (Down’s (CoCr) denture,
assay), 69 syndrome), 74–​75, 233, 242–​43
tranexamic acid 256, 257f, 260 upper permanent
mouthwash, trochlear nerve (CN IV) canine eruption, 26,
98, 209 damage, 38, 45 34
transient ischaemic attack true positives, statistics, upper removable appliance
(TIA), 79, 86 282, 287 with flat anterior
translational movement, TSH (thyroid-​stimulating bite plane, 59
condylar head, 4, hormone), 85 upper respiratory tract
3–​4, 4f tuberculosis (TB), 85 infections, 225
translucent zone, dental pulmonary see urticarial rash, 96, 205
caries, 84, 93 pulmonary
transpalatal arch, 47
transverse cervical
tuberculosis
tuberosity fracture, oral V
plexus, 48 and maxillofacial vacuum-​formed
transverse plane, cone surgery, 37, 43 retainers, 50, 56
beam computed tubular sclerosis, 84, 93 vasodilatation,
tomography, 5 tumours, fibrous, Gardner’s periodontitis vs.
trauma syndrome, 26 gingivitis, 69–​70
abducens nerve Turner’s syndrome (45, vertebrobasilar
trauma, 45 XO), 260 insufficiency, 87

https://t.me/DentalBooksWorld
314 Index

vertical angulation of wet-​bonding, mesio-​ wisdom tooth


collimator, 273f, occlusal composite, extraction, 37
273, 274f 234, 245 working (master)
vertical mattress wheezing impression,
sutures, 44 anaphylactic shock, dentures, 7
vertical root fracture, 96, 205 working models, 60
88, 0 asthma, 95 worn dentures,
Vicryl™ (polyglycolic Whip-​Springs, prosthodontics,
acid) sutures, maxillary arch 08, 8
253 expansion, 56 WSN (white sponge
vinylsiloxane white blood cell (WBC) naevus), Plate 8,
polymers, 24 count, 59, 72 56, 62, 68
Vita™ 3D shade guide, white-​eyed blowout,
235, 246
vitamin C deficiency,
46, 47
white lesions, Plate 3, X
59, 72 53, 62 xerostomia, 66
vitamin K, 209 white sponge naevus amitriptylene, 209
warfarin, 83 (WSN), Plate 8, XXY (Klinefelter’s
vocal problems, 25 56, 62, 68 syndrome), 260
white spot lesions, Plate 4,
W 58, 72, 239,
253 Z
warfarin, 36, 4, AIDS, 80 zinc oxide eugenol (ZOE),
77, 82 dental caries, 3, 232, 240
warm vertical 9f, 27–​28 zirconia, 3
compaction, 85, oral medicine/​ zoledronic acid, 36
95 pathology, Plate 8, zygomatic complex
weight loss, Graves’ 56, 68 fracture, 269, 276,
disease, 78 Willis bite gauge, 9–​20 277f, 278f

https://t.me/DentalBooksWorld
https://t.me/DentalBooksWorld
https://t.me/DentalBooksWorld

You might also like